Вы находитесь на странице: 1из 116

CONSTITUTIONAL

LAW 2 Case Digest


[Pick the date]
In the case of a taxpayer, he is allowed to sue where there is a claim that public funds are illegally
INTRODUCTION TO CONSTITUTIONAL LAW 2 disbursed, or that public money is being deflected to any improper purpose, or that there is a wastage
of public funds through the enforcement of an invalid or unconstitutional law. Before he can invoke the
power of judicial review, however, he must specifically prove that he has sufficient interest in preventing
THE NATURE OF THE CONSTITUTION AND ITS RELATION WITH THE COURTS the illegal expenditure of money raised by taxation and that he would sustain a direct injury as a result
of the enforcement of the questioned statute or contract. It is not sufficient that he has merely a general
interest common to all members of the public.
FRANCISCO VS. HOUSE OF REPRESENTATIVES
[415 SCRA 44; G.R. No. 160261; 10 Nov 2003] At all events, courts are vested with discretion as to whether or not a taxpayer's suit should be
entertained. This Court opts to grant standing to most of the petitioners, given their allegation that any
Facts: impending transmittal to the Senate of the Articles of Impeachment and the ensuing trial of the Chief
Justice will necessarily involve the expenditure of public funds.
Impeachment proceedings were filed against Supreme Court Chief Justice Hilario Davide. The
justiciable controversy poised in front of the Court was the constitutionality of the subsequent filing of a As for a legislator, he is allowed to sue to question the validity of any official action which he claims
second complaint to controvert the rules of impeachment provided for by law infringes his prerogatives as a legislator. Indeed, a member of the House of Representatives has
standing to maintain inviolate the prerogatives, powers and privileges vested by the Constitution in his
Issue: office.83

Whether or Not the filing of the second impeachment complaint against Chief Justice Hilario G. Davide, The framers of the Constitution also understood initiation in its ordinary meaning. Thus when a proposal
Jr. with the House of Representatives falls within the one year bar provided in the Constitution and reached the floor proposing that "A vote of at least one-third of all the Members of the House shall be
whether the resolution thereof is a political question – has resulted in a political crisis. necessary… to initiate impeachment proceedings," this was met by a proposal to delete the line on the
ground that the vote of the House does not initiate impeachment proceeding but rather the filing of a
Held: complaint does.

In any event, it is with the absolute certainty that our Constitution is sufficient to address all the issues To the argument that only the House of Representatives as a body can initiate impeachment
which this controversy spawns that this Court unequivocally pronounces, at the first instance, that the proceedings because Section 3 (1) says "The House of Representatives shall have the exclusive power
feared resort to extra-constitutional methods of resolving it is neither necessary nor legally permissible. to initiate all cases of impeachment," This is a misreading of said provision and is contrary to the
Both its resolution and protection of the public interest lie in adherence to, not departure from, the principle of reddendo singula singulis by equating "impeachment cases" with "impeachment
Constitution. proceeding."
In passing over the complex issues arising from the controversy, this Court is ever mindful of the
essential truth that the inviolate doctrine of separation of powers among the legislative, executive or Having concluded that the initiation takes place by the act of filing and referral or endorsement of the
judicial branches of government by no means prescribes for absolute autonomy in the discharge by impeachment complaint to the House Committee on Justice or, by the filing by at least one-third of the
each of that part of the governmental power assigned to it by the sovereign people. members of the House of Representatives with the Secretary General of the House, the meaning of
Section 3 (5) of Article XI becomes clear. Once an impeachment complaint has been initiated, another
At the same time, the corollary doctrine of checks and balances which has been carefully calibrated by impeachment complaint may not be filed against the same official within a one year period.
the Constitution to temper the official acts of each of these three branches must be given effect without
destroying their indispensable co-equality. There exists no constitutional basis for the contention that The Court in the present petitions subjected to judicial scrutiny and resolved on the merits only the main
the exercise of judicial review over impeachment proceedings would upset the system of checks and issue of whether the impeachment proceedings initiated against the Chief Justice transgressed the
balances. Verily, the Constitution is to be interpreted as a whole and "one section is not to be allowed to constitutionally imposed one-year time bar rule. Beyond this, it did not go about assuming jurisdiction
defeat another." Both are integral components of the calibrated system of independence and where it had none, nor indiscriminately turn justiciable issues out of decidedly political questions.
interdependence that insures that no branch of government act beyond the powers assigned to it by the Because it is not at all the business of this Court to assert judicial dominance over the other two great
Constitution. branches of the government.

When suing as a citizen, the interest of the petitioner assailing the constitutionality of a statute must be No one is above the law or the Constitution. This is a basic precept in any legal system which
direct and personal. He must be able to show, not only that the law or any government act is invalid, but recognizes equality of all men before the law as essential to the law's moral authority and that of its
also that he sustained or is in imminent danger of sustaining some direct injury as a result of its agents to secure respect for and obedience to its commands. Perhaps, there is no other government
enforcement, and not merely that he suffers thereby in some indefinite way. It must appear that the branch or instrumentality that is most zealous in protecting that principle of legal equality other than the
person complaining has been or is about to be denied some right or privilege to which he is lawfully Supreme Court which has discerned its real meaning and ramifications through its application to
entitled or that he is about to be subjected to some burdens or penalties by reason of the statute or act numerous cases especially of the high-profile kind in the annals of jurisprudence. The Chief Justice is
complained of. In fine, when the proceeding involves the assertion of a public right, the mere fact that not above the law and neither is any other member of this Court. But just because he is the Chief
he is a citizen satisfies the requirement of personal interest. Justice does not imply that he gets to have less in law than anybody else. The law is solicitous of every
individual's rights irrespective of his station in life.
Page 2
thereafter to execute the necessary clearances and to do such other acts and deeds as may be
Thus, the Rules of Procedure in Impeachment Proceedings which were approved by the House of necessary for purpose.
Representatives on November 28, 2001 are unconstitutional. Consequently, the second impeachment
complaint against Chief Justice Hilario G. Davide, Jr is barred under paragraph 5, section 3 of Article XI PEOPLE VS. POMAR
of the Constitution. [46 Phil 126; G.R. No. L-22008; 3 Nov 1924]

MANILA PRINCE HOTEL VS. GSIS Facts:


[267 SCRA 408; G.R. No. 122156; 3 Feb 1997]
Macaria Fajardo was an employee of La Flor de la Isabela, a Tobacco factory. She was granted a
Facts: vacation leave, by reason of her pregnancy, which commenced on the 16 th of July 1923. According to
Fajardo, during that time, she was not given the salary due her in violation of the provisions of Act No.
The controversy arose when respondent Government Service Insurance System (GSIS), pursuant to 3071. Fajardo filed a criminal complaint based on Section 13 and 15 of said Act against the manager
the privatization program of the Philippine Government under Proclamation No. 50 dated 8 December of the tobacco Factory, Julio Pomar, herein defendant. The latter, on the other hand, claims that the
1986, decided to sell through public bidding 30% to 51% of the issued and outstanding shares of facts in the complaint did not constitute an offense and further alleges that the aforementioned
respondent Manila Hotel Corporation. In a close bidding held on 18 September 1995 only two (2) provisions of Act No. 3071 was unconstitutional. Section 13, Act No. 3071 provides that, “Every person,
bidders participated: petitioner Manila Prince Hotel Corporation, a Filipino corporation, which offered to firm or corporation owning or managing a factory, shop or place of labor of any description shall be
buy 51% of the MHC or 15,300,000 shares at P41.58 per share, and Renong Berhad, a Malaysian firm, obliged to grant to any woman employed by it as laborer who may be pregnant, thirty days vacation
with ITT-Sheraton as its hotel operator, which bid for the same number of shares at P44.00 per share, with pay before and another thirty days after confinement: Provided, That the employer shall not
or P2.42 more than the bid of petitioner. discharge such laborer without just cause, under the penalty of being required to pay to her wages
equivalent to the total of two months counting from the day of her discharge.” Section 15 of the same
Pending the declaration of Renong Berhad as the winning bidder/strategic partner and the execution of Act provides for the penalty of any violation of section 13. The latter was enacted by the legislature in
the necessary contracts, matched the bid price of P44.00 per share tendered by Renong Berhad. the exercise of its supposed Police Power with the purpose of safeguarding the health of pregnant
women laborers in "factory, shop or place of labor of any description," and of insuring to them, to a
On 17 October 1995, perhaps apprehensive that respondent GSIS has disregarded the tender of the certain extent, reasonable support for one month before and one month after their delivery. The trial
matching bid and that the sale of 51% of the MHC may be hastened by respondent GSIS and court rendered a decision in favor of plaintiff, sentencing the defendant to pay the fine of fifty pesos and
consummated with Renong Berhad, petitioner came to this Court on prohibition and mandamus. in case of insolvency, to suffer subsidiary imprisonment. Hence, the case was raised to the Court of
Appeals which affirmed the former decision.
In the main, petitioner invokes Sec. 10, second par., Art. XII, of the 1987 Constitution and submits that
the Manila Hotel has been identified with the Filipino nation and has practically become a historical Issue:
monument which reflects the vibrancy of Philippine heritage and culture. It is a proud legacy of an
earlier generation of Filipinos who believed in the nobility and sacredness of independence and its Whether or not Section 13 of Act No. 3071 is unconstitutional.
power and capacity to release the full potential of the Filipino people. To all intents and purposes, it has
become a part of the national patrimony. 6 Petitioner also argues that since 51% of the shares of the Whether or not the promulgation of the questioned provision was a valid exercise of Police Power.
MHC carries with it the ownership of the business of the hotel which is owned by respondent GSIS, a
government-owned and controlled corporation, the hotel business of respondent GSIS being a part of Held:
the tourism industry is unquestionably a part of the national economy.
The Supreme Court declared Section 13 of Act No. 3071 to be unconstitutional for being violative or
Issue: restrictive of the right of the people to freely enter into contracts for their affairs. It has been decided
several times, that the right to contract about one's affairs is a part of the liberty of the individual,
Whether or Not the sale of Manila Hotel to Renong Berhad is violative of the Constitutional provision of protected by the "due process of law" clause of the constitution. The contracting parties may establish
Filipino First policy and is therefore null and void. any agreements, terms, and conditions they may deem advisable, provided they are not contrary to
law, morals or public policy
Held:
The police power of the state is a very broad and expanding power. The police power may encompass
The Manila Hotel or, for that matter, 51% of the MHC, is not just any commodity to be sold to the every law for the restraint and punishment of crimes, for the preservation of the public peace, health,
highest bidder solely for the sake of privatization. The Manila Hotel has played and continues to play a and morals. But that power cannot grow faster than the fundamental law of the state, nor transcend or
significant role as an authentic repository of twentieth century Philippine history and culture. This is the violate the express inhibition of the constitution. The Police Power is subject to and is controlled by the
plain and simple meaning of the Filipino First Policy provision of the Philippine Constitution. And this paramount authority of the constitution of the state, and will not be permitted to violate rights secured or
Court, heeding the clarion call of the Constitution and accepting the duty of being the elderly watchman guaranteed by the latter.
of the nation, will continue to respect and protect the sanctity of the Constitution. It was thus ordered
that GSIS accepts the matching bid of petitioner MANILA PRINCE HOTEL CORPORATION to LAMBINO VS. COMELEC
purchase the subject 51% of the shares of the Manila Hotel Corporation at P44.00 per share and [G.R. No. 174153; 25 Oct 2006]
Page 3
[270 SCRA 106; G.R. No.127325; 19 Mar 1997]
Facts:
Facts:
Petitioners (Lambino group) commenced gathering signatures for an initiative petition to change the
1987 constitution, they filed a petition with the COMELEC to hold a plebiscite that will ratify their Private respondent Atty. Jesus Delfin, president of People’s Initiative for Reforms, Modernization and
initiative petition under RA 6735. Lambino group alleged that the petition had the support of 6M Action (PIRMA), filed with COMELEC a petition to amend the constitution to lift the term limits of
individuals fulfilling what was provided by art 17 of the constitution. Their petition changes the 1987 elective officials, through People’s Initiative. He based this petition on Article XVII, Sec. 2 of the 1987
constitution by modifying sections 1-7 of Art 6 and sections 1-4 of Art 7 and by adding Art 18. the Constitution, which provides for the right of the people to exercise the power to directly propose
proposed changes will shift the present bicameral- presidential form of government to unicameral- amendments to the Constitution. Subsequently the COMELEC issued an order directing the publication
parliamentary. COMELEC denied the petition due to lack of enabling law governing initiative petitions of the petition and of the notice of hearing and thereafter set the case for hearing. At the hearing,
and invoked the Santiago Vs. Comelec ruling that RA 6735 is inadequate to implement the initiative Senator Roco, the IBP, Demokrasya-Ipagtanggol ang Konstitusyon, Public Interest Law Center, and
petitions. Laban ng Demokratikong Pilipino appeared as intervenors-oppositors. Senator Roco filed a motion to
dismiss the Delfin petition on the ground that one which is cognizable by the COMELEC. The
Issue: petitioners herein Senator Santiago, Alexander Padilla, and Isabel Ongpin filed this civil action for
prohibition under Rule 65 of the Rules of Court against COMELEC and the Delfin petition rising the
Whether or Not the Lambino Group’s initiative petition complies with Section 2, Article XVII of the several arguments, such as the following: (1) The constitutional provision on people’s initiative to
Constitution on amendments to the Constitution through a people’s initiative. amend the constitution can only be implemented by law to be passed by Congress. No such law has
been passed; (2) The people’s initiative is limited to amendments to the Constitution, not to revision
Whether or Not this Court should revisit its ruling in Santiago declaring RA 6735 “incomplete, thereof. Lifting of the term limits constitutes a revision, therefore it is outside the power of people’s
inadequate or wanting in essential terms and conditions” to implement the initiative clause on initiative. The Supreme Court granted the Motions for Intervention.
proposals to amend the Constitution.
Issue:
Whether or Not the COMELEC committed grave abuse of discretion in denying due course to the
Lambino Group’s petition. Whether or not Sec. 2, Art. XVII of the 1987 Constitution is a self-executing provision.

Held: Whether or not COMELEC Resolution No. 2300 regarding the conduct of initiative on amendments to
the Constitution is valid, considering the absence in the law of specific provisions on the conduct of
According to the SC the Lambino group failed to comply with the basic requirements for conducting a such initiative.
people’s initiative. The Court held that the COMELEC did not grave abuse of discretion on dismissing
the Lambino petition. Whether the lifting of term limits of elective officials would constitute a revision or an amendment of the
Constitution.
1. The Initiative Petition Does Not Comply with Section 2, Article XVII of the Constitution on Direct
Proposal by the People Held:
The petitioners failed to show the court that the initiative signer must be informed at the time of the
signing of the nature and effect, failure to do so is “deceptive and misleading” which renders the Sec. 2, Art XVII of the Constitution is not self executory, thus, without implementing legislation the same
initiative void. cannot operate. Although the Constitution has recognized or granted the right, the people cannot
exercise it if Congress does not provide for its implementation.
2. The Initiative Violates Section 2, Article XVII of the Constitution Disallowing Revision through
Initiatives The portion of COMELEC Resolution No. 2300 which prescribes rules and regulations on the conduct
The framers of the constitution intended a clear distinction between “amendment” and “revision, it of initiative on amendments to the Constitution, is void. It has been an established rule that what has
is intended that the third mode of stated in sec 2 art 17 of the constitution may propose only been delegated, cannot be delegated (potestas delegata non delegari potest). The delegation of the
amendments to the constitution. Merging of the legislative and the executive is a radical change, power to the COMELEC being invalid, the latter cannot validly promulgate rules and regulations to
therefore a constitutes a revision. implement the exercise of the right to people’s initiative.

3. A Revisit of Santiago v. COMELEC is Not Necessary The lifting of the term limits was held to be that of a revision, as it would affect other provisions of the
Even assuming that RA 6735 is valid, it will not change the result because the present petition Constitution such as the synchronization of elections, the constitutional guarantee of equal access to
violated Sec 2 Art 17 to be a valid initiative, must first comply with the constitution before opportunities for public service, and prohibiting political dynasties. A revision cannot be done by
complying with RA 6735 initiative. However, considering the Court’s decision in the above Issue, the issue of whether or not the
petition is a revision or amendment has become academic.
Petition is dismissed.
GONZALES VS. COMELEC
SANTIAGO VS. COMELEC [21 SCRA 774; G.R. No. L-28196; 9 Nov 1967]
Page 4
may propose amendments to this Constitution or call a contention for that purpose.
Facts: Such amendments shall be valid as part of this Constitution when approved by a
majority of the votes cast at an election at which the amendments are submitted to
The case is an original action for prohibition, with preliminary injunction. the people for their ratification.

The main facts are not disputed. On March 16, 1967, the Senate and the House of Representatives From our viewpoint, the provisions of Article XV of the Constitution are satisfied so long as the
passed the following resolutions: electorate knows that R. B. H. No. 3 permits Congressmen to retain their seats as legislators, even if
they should run for and assume the functions of delegates to the Convention.
1. R. B. H. (Resolution of Both Houses) No. 1, proposing that Section 5, Article VI, of the Constitution of
the Philippines, be amended so as to increase the membership of the House of Representatives from a SANIDAD VS. COMELEC
maximum of 120, as provided in the present Constitution, to a maximum of 180, to be apportioned [78 SCRA 333; G.R. No. 90878; 29 Jan 1990]
among the several provinces as nearly as may be according to the number of their respective
inhabitants, although each province shall have, at least, one (1) member; Facts:

2. R. B. H. No. 2, calling a convention to propose amendments to said Constitution, the convention to This is a petition for certiorari assailing the constitutionality of Section 19 of Comelec Resolution No.
be composed of two (2) elective delegates from each representative district, to be "elected in the 2167 on the ground that it violates the constitutional guarantees of the freedom of expression and of the
general elections to be held on the second Tuesday of November, 1971;" and press. On October 23, 1989, Republic Act No. 6766, entitled "AN ACT PROVIDING FOR AN ORGANIC
ACT FOR THE CORDILLERA AUTONOMOUS REGION" was enacted into law. Pursuant to said law,
3. R. B. H. No. 3, proposing that Section 16, Article VI, of the same Constitution, be amended so as to the City of Baguio and the Cordilleras which consist of the provinces of Benguet, Mountain Province,
authorize Senators and members of the House of Representatives to become delegates to the Ifugao, Abra and Kalinga-Apayao, all comprising the Cordillera Autonomous Region, shall take part in a
aforementioned constitutional convention, without forfeiting their respective seats in Congress. plebiscite for the ratification of said Organic Act originally scheduled last December 27, 1989 which
was, however, reset to January 30, 1990 by virtue of Comelec Resolution No. 2226 dated December
Subsequently, Congress passed a bill, which, upon approval by the President, on June 17, 1967, 27, 1989. The Commission on Elections, by virtue of the power vested by the 1987 Constitution, the
became Republic Act No. 4913, providing that the amendments to the Constitution proposed in the Omnibus Election Code (BP 881), said R.A. 6766 and other pertinent election laws, promulgated
aforementioned Resolutions No. 1 and 3 be submitted, for approval by the people, at the general Resolution No. 2167, to govern the conduct of the plebiscite on the said Organic Act for the Cordillera
elections which shall be held on November 14, 1967. Autonomous Region. In a petition dated November 20, 1989, herein petitioner Pablito V. Sanidad, who
claims to be a newspaper columnist of the "OVERVIEW" for the BAGUIO MIDLAND COURIER, a
Issue: weekly newspaper circulated in the City of Baguio and the Cordilleras, assailed the constitutionality of
Section 19 of Comelec Resolution No. 2167, which provides:
Whether or Not a Resolution of Congress, acting as a constituent assembly, violates the Constitution.
Section 19. Prohibition on columnists, commentators or announcers. — During the
Held: plebiscite campaign period, on the day before and on the plebiscite day, no mass
media columnist, commentator, announcer or personality shall use his column or
Inasmuch as there are less than eight (8) votes in favor of declaring Republic Act 4913 and R. B. H. radio or television time to campaign for or against the plebiscite Issue.
Nos. 1 and 3 unconstitutional and invalid, the petitions in these two (2) cases must be, as they are
hereby, dismiss and the writs therein prayed for denied, without special pronouncement as to costs. It is It is alleged by petitioner that said provision is void and unconstitutional because it violates
so ordered. the constitutional guarantees of the freedom of expression and of the press enshrined in the
Constitution. Unlike a regular news reporter or news correspondent who merely reports the
As a consequence, the title of a de facto officer cannot be assailed collaterally. It may not be contested news, petitioner maintains that as a columnist, his column obviously and necessarily contains
except directly, by quo warranto proceedings. Neither may the validity of his acts be questioned upon and reflects his opinions, views and beliefs on any issue or subject about which he writes.
the ground that he is merely a de facto officer. And the reasons are obvious: (1) it would be an indirect Petitioner likewise maintains that if media practitioners were to express their views, beliefs
inquiry into the title to the office; and (2) the acts of a de facto officer, if within the competence of his and opinions on the issue submitted to a plebiscite, it would in fact help in the government
office, are valid, insofar as the public is concerned. drive and desire to disseminate information, and hear, as well as ventilate, all sides of the
issue.
"The judicial department is the only constitutional organ which can be called upon to determine the
proper allocation of powers between the several departments and among the integral or constituent Issue:
units thereof."
Whether or not Section 19 of Comelec Resolution No. 2167 is unconstitutional.
Article XV of the Constitution provides:
Held:
. . . The Congress in joint session assembled, by a vote of three-fourths of all the
Members of the Senate and of the House of Representatives voting separately,
Page 5
The Supreme Court ruled that Section 19 of Comelec Resolution No. 2167 is unconstitutional. It is clear Representatives decided to withdraw the nomination and rescind the election of Congressman
from Art. IX-C of the 1987 Constitution that what was granted to the Comelec was the power to Camasura to the HRET.
supervise and regulate the use and enjoyment of franchises, permits or other grants issued for the
operation of transportation or other public utilities, media of communication or information to the end Issue:
that equal opportunity, time and space, and the right to reply, including reasonable, equal rates
therefor, for public information campaigns and forums among candidates are ensured. The evil sought Whether or not the House of Representatives, at the request of the dominant political party therein, may
to be prevented by this provision is the possibility that a franchise holder may favor or give any undue change that party’s representation in the HRET to thwart the promulgation of a decision freely reached
advantage to a candidate in terms of advertising space or radio or television time. This is also the by the tribunal in an election contest pending therein
reason why a "columnist, commentator, announcer or personality, who is a candidate for any elective
office is required to take a leave of absence from his work during the campaign period (2nd par. Section Held:
11(b) R.A. 6646). It cannot be gainsaid that a columnist or commentator who is also a candidate would
be more exposed to the voters to the prejudice of other candidates unless required to take a leave of The purpose of the constitutional convention creating the Electoral Commission was to provide an
absence. independent and impartial tribunal for the determination of contests to legislative office, devoid of
partisan consideration.
However, neither Article IX-C of the Constitution nor Section 11 (b), 2nd par. of R.A. 6646 can be
construed to mean that the Comelec has also been granted the right to supervise and regulate the As judges, the members of the tribunal must be non-partisan. They must discharge their functions with
exercise by media practitioners themselves of their right to expression during plebiscite periods. Media complete detachment, impartiality and independence even independence from the political party to
practitioners exercising their freedom of expression during plebiscite periods are neither the franchise which they belong. Hence, disloyalty to party and breach of party discipline are not valid grounds for
holders nor the candidates. In fact, there are no candidates involved in a plebiscite. Therefore, Section the expulsion of a member of the tribunal. In expelling Congressman Camasura from the HRET for
19 of Comelec Resolution No. 2167 has no statutory basis. having cast a “conscience vote” in favor of Bondoc, based strictly on the result of the examination and
appreciation of the ballots and the recount of the votes by the tribunal, the House of Representatives
Plebiscite Issue are matters of public concern and importance. The people's right to be informed and to committed a grave abuse of discretion, an injustice and a violation of the Constitution. Its resolution of
be able to freely and intelligently make a decision would be better served by access to an unabridged expulsion against Congressman Camasura is, therefore, null and void.
discussion of the Issue, including the forum. The people affected by the Issue presented in a plebiscite
should not be unduly burdened by restrictions on the forum where the right to expression may be Another reason for the nullity of the expulsion resolution of the House of Representatives is that it
exercised. Comelec spaces and Comelec radio time may provide a forum for expression but they do violates Congressman Camasura’s right to security of tenure. Members of the HRET, as sole judge of
not guarantee full dissemination of information to the public concerned because they are limited to congressional election contests, are entitled to security of tenure just as members of the Judiciary enjoy
either specific portions in newspapers or to specific radio or television times. security of tenure under the Constitution. Therefore, membership in the HRET may not be terminated
except for a just cause, such as, the expiration of the member’s congressional term of office, his death,
The instant petition is GRANTED. Section 19 of Comelec Resolution No. 2167 is declared null and void permanent disability, resignation from the political party he represents in the tribunal, formal affiliation
and unconstitutional. with another political party or removal for other valid cause. A member may not be expelled by the
House of Representatives for party disloyalty, short of proof that he has formally affiliated with another
BONDOC VS. PINEDA
[201 SCRA 792; G.R. No. 97710; 26 Sep 1991] MIRASOL VS CA
[351 SCRA 44; G.R. No. 128448; 1 Feb 2001]
Facts:
Facts:
In the elections held on May 11, 1987, Marciano Pineda of the LDP and Emigdio Bondoc of the NP
were candidates for the position of Representative for the Fourth District of Pampanga. Pineda was The Mirasols are sugarland owners and planters. Philippine National Bank (PNB) financed the Mirasols'
proclaimed winner. Bondoc filed a protest in the House of Representatives Electoral Tribunal (HRET), sugar production venture FROM 1973-1975 under a crop loan financing scheme. The Mirasols signed
which is composed of 9 members, 3 of whom are Justices of the SC and the remaining 6 are members Credit Agreements, a Chattel Mortgage on Standing Crops, and a Real Estate Mortgage in favor of
of the House of Representatives (5 members belong to the LDP and 1 member is from the NP). PNB. The Chattel Mortgage empowered PNB to negotiate and sell the latter's sugar and to apply the
Thereafter, a decision had been reached in which Bondoc won over Pineda. Congressman Camasura proceeds to the payment of their obligations to it.
of the LDP voted with the SC Justices and Congressman Cerilles of the NP to proclaim Bondoc the
winner of the contest. President Marcos issued PD 579 in November, 1974 authorizing Philippine Exchange Co., Inc.
(PHILEX) to purchase sugar allocated for export and authorized PNB to finance PHILEX's purchases.
On the eve of the promulgation of the Bondoc decision, Congressman Camasura The decree directed that whatever profit PHILEX might realize was to be remitted to the government.
received a letter informing him that he was already expelled from the LDP for allegedly helping to Believing that the proceeds were more than enough to pay their obligations, petitioners asked PNB for
organize the Partido Pilipino of Eduardo Cojuangco and for allegedly inviting LDP members in Davao an accounting of the proceeds which it ignored. Petitioners continued to avail of other loans from PNB
Del Sur to join said political party. On the day of the promulgation of the decision, the Chairman of and to make unfunded withdrawals from their accounts with said bank. PNB asked petitioners to settle
HRET received a letter informing the Tribunal that on the basis of the letter from the LDP, the House of their due and demandable accounts. As a result, petitioners, conveyed to PNB real properties by way of

Page 6
dacion en pago still leaving an unpaid amount. PNB proceeded to extrajudicially foreclose the Petitioner Dumlao questions the constitutionality of Sec. 4 of Batas Pambansa Blg 52 as discriminatory
mortgaged properties. PNB still had a deficiency claim. and contrary to equal protection and due process guarantees of the Constitution. Sec. 4 provides that
any retired elective provincial or municipal official who has received payments of retirement benefits
Petitioners continued to ask PNB to account for the proceeds, insisting that said proceeds, if properly and shall have been 65 years of age at the commencement of the term of office to which he seeks to
liquidated, could offset their outstanding obligations. PNB remained adamant in its stance that under be elected, shall not be qualified to run for the same elective local office from which he has retired.
P.D. No. 579, there was nothing to account since under said law, all earnings from the export sales of According to Dumlao, the provision amounts to class legislation. Petitioners Igot and Salapantan Jr.
sugar pertained to the National Government. also assail the validity of Sec. 4 of Batas Pambansa Blg 52, which states that any person who has
committed any act of disloyalty to the State, including those amounting to subversion, insurrection,
On August 9, 1979, the Mirasols filed a suit for accounting, specific performance, and damages against rebellion, or other similar crimes, shall not be qualified for any of the offices covered by the act, or to
PNB. participate in any partisan activity therein: provided that a judgment of conviction of those crimes shall
be conclusive evidence of such fact and the filing of charges for the commission of such crimes before
Issue: a civil court or military tribunal after preliminary investigation shall be prima facie evidence of such fact.

Whether or not the Trial Court has jurisdiction to declare a statute unconstitutional without notice to the Issue:
Solicitor General where the parties have agreed to submit such issue for the resolution of the Trial
Court. Whether or Not the aforementioned statutory provisions violate the Constitution and thus, should be
declared null and void
Whether PD 579 and subsequent issuances thereof are unconstitutional.
Whether or not the requisites of judicial review are complied with
Whether or not said PD is subject to judicial review.
Held: Held:

It is settled that Regional Trial Courts have the authority and jurisdiction to consider the constitutionality No constitutional question will be heard and decided by the Court unless there is compliance with the
of a statute, presidential decree, or executive order. The Constitution vests the power of judicial review requisites of a judicial inquiry, which are: 1) There must be an actual case or controversy; 2) The
or the power to declare a law, treaty, international or executive agreement, presidential decree, order, question of constitutionality must be raised by the proper party; 3) The constitutional question must be
instruction, ordinance, or regulation not only in this Court, but in all Regional Trial Courts. raised at the earliest possible opportunity; and 4) The decision of the constitutional question must be
necessary to the determination of the case itself.
The purpose of the mandatory notice in Rule 64, Section 3 is to enable the Solicitor General to decide
whether or not his intervention in the action assailing the validity of a law or treaty is necessary. To As to (1), Dumlao has not been adversely affected by the application of the provision. His question is
deny the Solicitor General such notice would be tantamount to depriving him of his day in court. We posed merely in the abstract, and without the benefit of a detailed factual record. As to (2), neither Igot
must stress that, contrary to petitioners' stand, the mandatory notice requirement is not limited to nor Salapantan has been charged with acts of loyalty to the State, nor disqualified from being
actions involving declaratory relief and similar remedies. The rule itself provides that such notice is candidates for local elective positions. They have no personal nor substantial interest at stake. Igot and
required in "any action" and not just actions involving declaratory relief. Where there is no ambiguity in Salapantan have institute the case as a taxpayer’s suit, but the institution of a taxpayer’s suit per se is
the words used in the rule, there is no room for construction. 15 In all actions assailing the validity of a no assurance of judicial review. As to (4), there is no cause of action in this particular case. Therefore,
statute, treaty, presidential decree, order, or proclamation, notice to the Solicitor General is mandatory. the necessity for resolving the issue of constitutionality is absent.

Petitioners contend that P.D. No. 579 and its implementing issuances are void for violating the due In regards to the unconstitutionality of the provisions, Sec. 4 of BP Blg 52 remains constitutional and
process clause and the prohibition against the taking of private property without just compensation. valid. The constitutional guarantee of equal protection of the laws is subject to rational classification.
Petitioners now ask this Court to exercise its power of judicial review. One class can be treated differently from another class. In this case, employees 65 years of age are
classified differently from younger employees. The purpose of the provision is to satisfy the “need for
Jurisprudence has laid down the following requisites for the exercise of this power: First, there must be new blood” in the workplace. In regards to the second paragraph of Sec. 4, it should be declared null
before the Court an actual case calling for the exercise of judicial review. Second, the question before and void for being violative of the constitutional presumption of innocence guaranteed to an accused.
the Court must be ripe for adjudication. Third, the person challenging the validity of the act must have
standing to challenge. Fourth, the question of constitutionality must have been raised at the earliest
opportunity, and lastly, the issue of constitutionality must be the very lis mota of the case. LACSON VS. PEREZ
[357 SCRA 756; G.R. No. 147780 ;10 May 2001]
DUMLAO VS. COMELEC
[95 SCRA 392; G.R. No.L-52245; 22 Jan 1980] Facts:

Facts: President Macapagal-Arroyo declared a State of Rebellion (Proclamation No. 38) on May 1, 2001 as
well as General Order No. 1 ordering the AFP and the PNP to suppress the rebellion in the NCR.
Warrantless arrests of several alleged leaders and promoters of the “rebellion” were thereafter effected.
Page 7
Petitioner filed for prohibition, injunction, mandamus and habeas corpus with an application for the State of Rebellion. Negotiations took place and the officers went back to their barracks in the evening of
issuance of temporary restraining order and/or writ of preliminary injunction. Petitioners assail the the same day. On August 1, 2003, both the Proclamation and General Orders were lifted, and
declaration of Proc. No. 38 and the warrantless arrests allegedly effected by virtue thereof. Petitioners Proclamation No. 435, declaring the Cessation of the State of Rebellion was issued.
furthermore pray that the appropriate court, wherein the information against them were filed, would
desist arraignment and trial until this instant petition is resolved. They also contend that they are In the interim, however, the following petitions were filed: (1) SANLAKAS AND PARTIDO NG
allegedly faced with impending warrantless arrests and unlawful restraint being that hold departure MANGGAGAWA VS. EXECUTIVE SECRETARY, petitioners contending that Sec. 18 Article VII of the
orders were issued against them. Constitution does not require the declaration of a state of rebellion to call out the AFP, and that there is
no factual basis for such proclamation. (2)SJS Officers/Members v. Hon. Executive Secretary, et al,
Issue: petitioners contending that the proclamation is a circumvention of the report requirement under the
same Section 18, Article VII, commanding the President to submit a report to Congress within 48 hours
Whether or Not Proclamation No. 38 is valid, along with the warrantless arrests and hold departure from the proclamation of martial law. Finally, they contend that the presidential issuances cannot be
orders allegedly effected by the same. construed as an exercise of emergency powers as Congress has not delegated any such power to the
President. (3) Rep. Suplico et al. v. President Macapagal-Arroyo and Executive Secretary Romulo,
Held: petitioners contending that there was usurpation of the power of Congress granted by Section 23 (2),
Article VI of the Constitution. (4) Pimentel v. Romulo, et al, petitioner fears that the declaration of a
President Macapagal-Arroyo ordered the lifting of Proc. No. 38 on May 6, 2006, accordingly the instant state of rebellion "opens the door to the unconstitutional implementation of warrantless arrests" for the
petition has been rendered moot and academic. Respondents have declared that the Justice crime of rebellion.
Department and the police authorities intend to obtain regular warrants of arrests from the courts for all
acts committed prior to and until May 1, 2001. Under Section 5, Rule 113 of the Rules of Court, Issue:
authorities may only resort to warrantless arrests of persons suspected of rebellion in suppressing the
rebellion if the circumstances so warrant, thus the warrantless arrests are not based on Proc. No. 38. Whether or Not Proclamation No. 427 and General Order No. 4 are constitutional?
Petitioner’s prayer for mandamus and prohibition is improper at this time because an individual
warrantlessly arrested has adequate remedies in law: Rule 112 of the Rules of Court, providing for Whether or Not the petitioners have a legal standing or locus standi to bring suit?
preliminary investigation, Article 125 of the Revised Penal Code, providing for the period in which a
warrantlessly arrested person must be delivered to the proper judicial authorities, otherwise the officer Held:
responsible for such may be penalized for the delay of the same. If the detention should have no legal
ground, the arresting officer can be charged with arbitrary detention, not prejudicial to claim of damages The Court rendered that the both the Proclamation No. 427 and General Order No. 4 are constitutional.
under Article 32 of the Civil Code. Petitioners were neither assailing the validity of the subject hold Section 18, Article VII does not expressly prohibit declaring state or rebellion. The President in addition
departure orders, nor were they expressing any intention to leave the country in the near future. To to its Commander-in-Chief Powers is conferred by the Constitution executive powers. It is not disputed
declare the hold departure orders null and void ab initio must be made in the proper proceedings that the President has full discretionary power to call out the armed forces and to determine the
initiated for that purpose. Petitioners’ prayer for relief regarding their alleged impending warrantless necessity for the exercise of such power. While the Court may examine whether the power was
arrests is premature being that no complaints have been filed against them for any crime, furthermore, exercised within constitutional limits or in a manner constituting grave abuse of discretion, none of the
the writ of habeas corpus is uncalled for since its purpose is to relieve unlawful restraint which petitioners here have, by way of proof, supported their assertion that the President acted without factual
Petitioners are not subjected to. basis. The issue of the circumvention of the report is of no merit as there was no indication that military
tribunals have replaced civil courts or that military authorities have taken over the functions of Civil
Petition is dismissed. Respondents, consistent and congruent with their undertaking earlier adverted Courts. The issue of usurpation of the legislative power of the Congress is of no moment since the
to, together with their agents, representatives, and all persons acting in their behalf, are hereby President, in declaring a state of rebellion and in calling out the armed forces, was merely exercising a
enjoined from arresting Petitioners without the required judicial warrants for all acts committed in wedding of her Chief Executive and Commander-in-Chief powers. These are purely executive powers,
relation to or in connection with the May 1, 2001 siege of Malacañang. vested on the President by Sections 1 and 18, Article VII, as opposed to the delegated legislative
powers contemplated by Section 23 (2), Article VI. The fear on warrantless arrest is unreasonable,
since any person may be subject to this whether there is rebellion or not as this is a crime punishable
SANLAKAS VS. EXECUTIVE SECRETARY under the Revised Penal Code, and as long as a valid warrantless arrest is present.
[421 SCRA 656; G.R. No. 159085; 3 Feb 2004]

Facts: Legal standing or locus standi has been defined as a personal and substantial interest in the case such
that the party has sustained or will sustain direct injury as a result of the governmental act that is being
During the wee hours of July 27, 2003, some three-hundred junior officers and enlisted men of the AFP, challenged. The gist of the question of standing is whether a party alleges "such personal stake in the
acting upon instigation, command and direction of known and unknown leaders have seized the outcome of the controversy as to assure that concrete adverseness which sharpens the presentation of
Oakwood Building in Makati. Publicly, they complained of the corruption in the AFP and declared their Issue upon which the court depends for illumination of difficult constitutional questions. Based on the
withdrawal of support for the government, demanding the resignation of the President, Secretary of foregoing, petitioners Sanlakas and PM, and SJS Officers/Members have no legal standing to sue. Only
Defense and the PNP Chief. These acts constitute a violation of Article 134 of the Revised Penal Code, petitioners Rep. Suplico et al. and Sen. Pimentel, as Members of Congress, have standing to challenge
and by virtue of Proclamation No. 427 and General Order No. 4, the Philippines was declared under the the subject issuances. It sustained its decision in Philippine Constitution Association v. Enriquez, that

Page 8
the extent the powers of Congress are impaired, so is the power of each member thereof, since his Whether or not PCGG has complied with the due process clause and other statutory requirements for
office confers a right to participate in the exercise of the powers of that institution. the exportation and sale of the subject items.

Whether or not the petition has become moot and academic, and if so, whether the above Issue
JOYA VS. PCGG warrant resolution from this Court.
[225 SCRA 568; G.R. No. 96541; 24 Aug 1993]
Held:
Facts:
This is premised on Sec. 2, Rule 3, of the Rules of Court which provides that every action must be
On 9 August 1990, Mateo A.T. Caparas, then Chairman of PCGG, wrote then President Corazon C. prosecuted and defended in the name of the real party-in-interest, and that all persons having interest
Aquino, requesting her for authority to sign the proposed Consignment Agreement between the in the subject of the action and in obtaining the relief demanded shall be joined as plaintiffs. The Court
Republic of the Philippines through PCGG and Christie, Manson and Woods International, Inc will exercise its power of judicial review only if the case is brought before it by a party who has the legal
concerning the scheduled sale on 11 January 1991 of eighty-two) Old Masters Paintings and antique standing to raise the constitutional or legal question. "Legal standing" means a personal and substantial
silverware seized from Malacañang and the Metropolitan Museum of Manila alleged to be part of the ill- interest in the case such that the party has sustained or will sustain direct injury as a result of the
gotten wealth of the late President Marcos, his relatives and cronies. On 14 August 1990, then governmental act that is being challenged. The term "interest" is material interest, an interest in issue
President Aquino, through former Executive Secretary Catalino Macaraig, Jr., authorized Chairman and to be affected by the decree, as distinguished from mere interest in the question involved, or a
Caparas to sign the Consignment Agreement allowing Christie's of New York to auction off the subject mere incidental interest. Moreover, the interest of the party plaintiff must be personal and not one
art pieces for and in behalf of the Republic of the Philippines. On 15 August 1990, PCGG, through based on a desire to vindicate the constitutional right of some third and related party.
Chairman Caparas, representing the Government of the Republic of the Philippines, signed the
Consignment Agreement with Christie's of New York. According to the agreement, PCGG shall consign There are certain instances however when this Court has allowed exceptions to the rule on legal
to CHRISTIE'S for sale at public auction the eighty-two Old Masters Paintings then found at the standing, as when a citizen brings a case for mandamus to procure the enforcement of a public duty for
Metropolitan Museum of Manila as well as the silverware contained in seventy-one cartons in the the fulfillment of a public right recognized by the Constitution, and when a taxpayer questions the
custody of the Central Bank of the Philippines, and such other property as may subsequently be validity of a governmental act authorizing the disbursement of public funds.
identified by PCGG and accepted by CHRISTIE'S to be subject to the provisions of the agreement.
Petitioners' arguments are devoid of merit. They lack basis in fact and in law. The ownership of these
On 26 October 1990, the Commission on Audit through then Chairman Eufemio C. Domingo submitted paintings legally belongs to the foundation or corporation or the members thereof, although the public
to President Aquino the audit findings and observations of COA on the Consignment Agreement of 15 has been given the opportunity to view and appreciate these paintings when they were placed on
August 1990 to the effect that: the authority of former PCGG Chairman Caparas to enter into the exhibit.
Consignment Agreement was of doubtful legality; the contract was highly disadvantageous to the
government; PCGG had a poor track record in asset disposal by auction in the U.S.; and, the assets
subject of auction were historical relics and had cultural significance, hence, their disposal was The confiscation of these properties by the Aquino administration however should not be understood to
prohibited by law. mean that the ownership of these paintings has automatically passed on the government without
complying with constitutional and statutory requirements of due process and just compensation. If these
After the oral arguments of the parties on 9 January 1991, we issued immediately our resolution properties were already acquired by the government, any constitutional or statutory defect in their
denying the application for preliminary injunction to restrain the scheduled sale of the artworks on the acquisition and their subsequent disposition must be raised only by the proper parties the true owners
ground that petitioners had not presented a clear legal right to a restraining order and that proper thereof whose authority to recover emanates from their proprietary rights which are protected by
parties had not been impleaded. statutes and the Constitution. Having failed to show that they are the legal owners of the artworks or
that the valued pieces have become publicly owned, petitioners do not possess any clear legal right
On 11 January 1991, the sale at public auction proceeded as scheduled and the proceeds of whatsoever to question their alleged unauthorized disposition.
$13,302,604.86 were turned over to the Bureau of Treasury.
Neither can this petition be allowed as a taxpayer's suit. Obviously, petitioners are not challenging any
Issue: expenditure involving public funds but the disposition of what they allege to be public properties. It is
worthy to note that petitioners admit that the paintings and antique silverware were acquired from
Whether or not petitioners have legal standing. private sources and not with public money.
Anent the second requisite of actual controversy, petitioners argue that this case should be resolved by
Whether or not the Old Masters Paintings and antique silverware are embraced in the phrase "cultural this Court as an exception to the rule on moot and academic cases; that although the sale of the
treasure of the nation". paintings and silver has long been consummated and the possibility of retrieving the treasure trove is
nil, yet the novelty and importance of the Issue raised by the petition deserve this Court's attention.
Whether or not the paintings and silverware are properties of public dominion on which can be They submit that the resolution by the Court of the Issue in this case will establish future guiding
disposed of through the joint concurrence of the President and Congress. principles and doctrines on the preservation of the nation's priceless artistic and cultural possessions
for the benefit of the public as a whole.

Page 9
For a court to exercise its power of adjudication, there must be an actual case of controversy — one Plaintiffs thus filed the instant special civil action for certiorari under Rule 65 of the Revised Rules of
which involves a conflict of legal rights, an assertion of opposite legal claims susceptible of judicial Court and ask this Court to rescind and set aside the dismissal order on the ground that the respondent
resolution; the case must not be moot or academic or based on extra-legal or other similar Judge gravely abused his discretion in dismissing the action. Again, the parents of the plaintiffs-minors
considerations not cognizable by a court of justice. A case becomes moot and academic when its not only represent their children, but have also joined the latter in this case.
purpose has become stale, such as the case before us. Since the purpose of this petition for prohibition
is to enjoin respondent public officials from holding the auction sale of the artworks on a particular date Petitioners contend that the complaint clearly and unmistakably states a cause of action as it contains
— 11 January 1991 — which is long past, the Issue raised in the petition have become moot and sufficient allegations concerning their right to a sound environment based on Articles 19, 20 and 21 of
academic. the Civil Code (Human Relations), Section 4 of Executive Order (E.O.) No. 192 creating the DENR,
Section 3 of Presidential Decree (P.D.) No. 1151 (Philippine Environmental Policy), Section 16, Article II
The cultural properties of the nation which shall be under the protection of the state are classified as the of the 1987 Constitution recognizing the right of the people to a balanced and healthful ecology, the
"important cultural properties" and the "national cultural treasures." On the other hand, a "national concept of generational genocide in Criminal Law and the concept of man's inalienable right to self-
cultural treasures" is a unique object found locally, possessing outstanding historical, cultural, artistic preservation and self-perpetuation embodied in natural law. Petitioners likewise rely on the
and/or scientific value which is highly significant and important to this country and nation. This Court respondent's correlative obligation per Section 4 of E.O. No. 192, to safeguard the people's right to a
takes note of the certification issued by the Director of the Museum that the Italian paintings and healthful environment.
silverware subject of this petition do not constitute protected cultural properties and are not among
those listed in the Cultural Properties Register of the National Museum. It is further claimed that the issue of the respondent Secretary's alleged grave abuse of discretion in
granting Timber License Agreements (TLAs) to cover more areas for logging than what is available
WHEREFORE, for lack of merit, the petition for prohibition and mandamus is DISMISSED. involves a judicial question.

Anent the invocation by the respondent Judge of the Constitution's non-impairment clause, petitioners
maintain that the same does not apply in this case because TLAs are not contracts. They likewise
OPOSA VS. FACTORAN, JR. submit that even if TLAs may be considered protected by the said clause, it is well settled that they may
[224 SCRA 792; G.R. No. 101083; 30 Jul 1993] still be revoked by the State when the public interest so requires.

Facts: Issue:

Principal petitioners, are all minors duly represented and joined by their respective parents. Impleaded Whether or not the petitioners have locus standi.
as an additional plaintiff is the Philippine Ecological Network, Inc. (PENI), a domestic, non-stock and
non-profit corporation organized for the purpose of, inter alia, engaging in concerted action geared for Whether or not the petiton is in a form of a class suit.
the protection of our environment and natural resources. The original defendant was the Honorable
Fulgencio S. Factoran, Jr., then Secretary of the Department of Environment and Natural Resources Whether or not the TLA’s can be out rightly cancelled.
(DENR). His substitution in this petition by the new Secretary, the Honorable Angel C. Alcala, was
subsequently ordered upon proper motion by the petitioners. The complaint was instituted as a Whether or not the petition should be dismissed.
taxpayers' class suit and alleges that the plaintiffs "are all citizens of the Republic of the Philippines,
taxpayers, and entitled to the full benefit, use and enjoyment of the natural resource treasure that is the Held:
country's virgin tropical forests." The same was filed for themselves and others who are equally
concerned about the preservation of said resource but are "so numerous that it is impracticable to bring As to the matter of the cancellation of the TLAs, respondents submit that the same cannot be done by
them all before the Court." the State without due process of law. Once issued, a TLA remains effective for a certain period of time
— usually for twenty-five (25) years. During its effectivity, the same can neither be revised nor
On 22 June 1990, the original defendant, Secretary Factoran, Jr., filed a Motion to Dismiss the cancelled unless the holder has been found, after due notice and hearing, to have violated the terms of
complaint based on two grounds, namely: the plaintiffs have no cause of action against him and, the the agreement or other forestry laws and regulations. Petitioners' proposition to have all the TLAs
issue raised by the plaintiffs is a political question which properly pertains to the legislative or executive indiscriminately cancelled without the requisite hearing would be violative of the requirements of due
branches of Government. In their 12 July 1990 Opposition to the Motion, the petitioners maintain that, process.
the complaint shows a clear and unmistakable cause of action, the motion is dilatory and the action
presents a justiciable question as it involves the defendant's abuse of discretion. The subject matter of the complaint is of common and general interest not just to several, but to all
citizens of the Philippines. Consequently, since the parties are so numerous, it, becomes impracticable,
On 18 July 1991, respondent Judge issued an order granting the aforementioned motion to dismiss. In if not totally impossible, to bring all of them before the court. The plaintiffs therein are numerous and
the said order, not only was the defendant's claim that the complaint states no cause of action against representative enough to ensure the full protection of all concerned interests. Hence, all the requisites
him and that it raises a political question sustained, the respondent Judge further ruled that the granting for the filing of a valid class suit under Section 12, Rule 3 of the Revised Rules of Court are present
of the relief prayed for would result in the impairment of contracts which is prohibited by the both in the said civil case and in the instant petition, the latter being but an incident to the former.
fundamental law of the land. Petitioners minors assert that they represent their generation as well as generations yet unborn. Their
personality to sue in behalf of the succeeding generations can only be based on the concept of
Page 10
intergenerational responsibility insofar as the right to a balanced and healthful ecology is concerned. healthful ecology; hence, the full protection thereof requires that no further TLAs should be renewed or
Nature means the created world in its entirety. Every generation has a responsibility to the next to granted.
preserve that rhythm and harmony for the full enjoyment of a balanced and healthful ecology. The
minors' assertion of their right to a sound environment constitutes, at the same time, the performance of It is settled in this jurisdiction that in a motion to dismiss based on the ground that the complaint fails to
their obligation to ensure the protection of that right for the generations to come. state a cause of action; the question submitted to the court for resolution involves the sufficiency of the
facts alleged in the complaint itself. No other matter should be considered; furthermore, the truth of
The complaint focuses on one specific fundamental legal right the right to a balanced and healthful falsity of the said allegations is beside the point for the truth thereof is deemed hypothetically admitted.
ecology which, for the first time in our nation's constitutional history, is solemnly incorporated in the Policy formulation or determination by the executive or legislative branches of Government is not
fundamental law. Section 16, Article II of the 1987 Constitution. squarely put in issue. What is principally involved is the enforcement of a right vis-a-vis policies already
formulated and expressed in legislation. It must, nonetheless, be emphasized that the political question
While the right to a balanced and healthful ecology is to be found under the Declaration of Principles doctrine is no longer, the insurmountable obstacle to the exercise of judicial power or the impenetrable
and State Policies and not under the Bill of Rights, it does not follow that it is less important than any of shield that protects executive and legislative actions from judicial inquiry or review.
the civil and political rights enumerated in the latter. Such a right belongs to a different category of
rights altogether for it concerns nothing less than self-preservation and self-perpetuation — aptly and In the second place, even if it is to be assumed that the same are contracts, the instant case does not
fittingly stressed by the petitioners the advancement of which may even be said to predate all involve a law or even an executive issuance declaring the cancellation or modification of existing timber
governments and constitutions. As a matter of fact, these basic rights need not even be written in the licenses. Hence, the non-impairment clause cannot as yet be invoked. Nevertheless, granting further
Constitution for they are assumed to exist from the inception of humankind. If they are now explicitly that a law has actually been passed mandating cancellations or modifications, the same cannot still be
mentioned in the fundamental charter, it is because of the well-founded fear of its framers that unless stigmatized as a violation of the non-impairment clause. This is because by its very nature and purpose,
the rights to a balanced and healthful ecology and to health are mandated as state policies by the such as law could have only been passed in the exercise of the police power of the state for the
Constitution itself, thereby highlighting their continuing importance and imposing upon the state a purpose of advancing the right of the people to a balanced and healthful ecology, promoting their health
solemn obligation to preserve the first and protect and advance the second, the day would not be too and enhancing the general welfare.
far when all else would be lost not only for the present generation, but also for those to come
generations which stand to inherit nothing but parched earth incapable of sustaining life. Finally, it is difficult to imagine, as the trial court did, how the non-impairment clause could apply with
respect to the prayer to enjoin the respondent Secretary from receiving, accepting, processing,
Conformably with the enunciated right to a balanced and healthful ecology and the right to health, as renewing or approving new timber licenses for, save in cases of renewal, no contract would have as of
well as the other related provisions of the Constitution concerning the conservation, development and yet existed in the other instances. Moreover, with respect to renewal, the holder is not entitled to it as a
utilization of the country's natural resources, then President Corazon C. Aquino promulgated on 10 matter of right.
June 1987 E.O. No. 192, Section 4 of which expressly mandates that the Department of Environment
and Natural Resources "shall be the primary government agency responsible for the conservation, Petition is hereby GRANTED, and the challenged Order of respondent Judge of 18 July 1991
management, development and proper use of the country's environment and natural resources, dismissing Civil Case No. 90-777 is hereby set aside. The petitioners may therefore amend their
specifically forest and grazing lands, mineral, resources, including those in reservation and watershed complaint to implead as defendants the holders or grantees of the questioned timber license
areas, and lands of the public domain, as well as the licensing and regulation of all natural resources as agreements.
may be provided for by law in order to ensure equitable sharing of the benefits derived therefrom for the
welfare of the present and future generations of Filipinos." Section 3 thereof makes the following
statement of policy:

The above provision stresses "the necessity of maintaining a sound ecological balance and protecting
and enhancing the quality of the environment." Section 2 of the same Title, on the other hand,
specifically speaks of the mandate of the DENR; however, it makes particular reference to the fact of
the agency's being subject to law and higher authority.
AGAN JR. VS. PIATCO
It may, however, be recalled that even before the ratification of the 1987 Constitution, specific statutes [402 SCRA 612; G.R. No. 155001; 5 May 2003]
already paid special attention to the "environmental right" of the present and future generations. On 6
June 1977, P.D. No. 1151 and P.D. No. 1152 were issued. Thus, the right of the petitioners to a Facts:
balanced and healthful ecology is as clear as the DENR's duty under its mandate and by virtue of its
powers and functions under E.O. No. 192 and the Administrative Code of 1987 to protect and advance Some time in 1993, six business leaders, explored the possibility of investing in the new NAIA airport
the said right. terminal, so they formed Asians Emerging Dragon Corp. They submitted proposals to the government
for the development of NAIA Intl. Passenger Terminal III (NAIA IPT III). The NEDA approved the NAIA
A denial or violation of that right by the other who has the correlative duty or obligation to respect or IPT III project. Bidders were invited, and among the proposal Peoples Air Cargo (Paircargo) was
protect the same gives rise to a cause of action. Petitioners maintain that the granting of the TLAs, chosen. AEDC protested alleging that preference was given to Paircargo, but still the project was
which they claim was done with grave abuse of discretion, violated their right to a balanced and awarded to Paircargo. Because of that, it incorporated into, Phil. Intl. Airport Terminals Co. (PIATCO).
The DOTC and PIATCO entered into a concession agreement in 1997 to franchise and operate the
Page 11
said terminal for 21years. In Nov. 1998 it was amended in the matters of pertaining to the definition of
the obligations given to the concessionaire, development of facilities and proceeds, fees and charges, Whether or Not Petitioner was denied due process of law
and the termination of contract. Since MIAA is charged with the maintenance and operations of NAIA
terminals I and II, it has a contract with several service providers. The workers filed the petition for Whether or Not the PCAGC is a validly Constituted government agency and whether the petitioner can
prohibition claiming that they would lose their job, and the service providers joined them, filed a motion raise the issue of constitutionality belatedly in its motion for reconsideration of the trial courts decision.
for intervention. Likewise several employees of the MIAA filed a petition assailing the legality of
arrangements. A group of congressmen filed similar petitions. Pres. Arroyo declared in her speech that Whether or Not the ombudsman's resolution dismissing the charges against the petitioner is still basis
she will not honor PIATCO contracts which the Exec. Branch's legal office concluded null and void. for the petitioner's dismissal with forfeiture of benefits as ruled in AO No. 152

Issue: Held:

Whether or Not the 1997 concession agreement is void, together with its amendments for being Petitioner maintains that as a career executive service officer, he can only be removed for cause and
contrary to the constitution. under the Administrative Code of 1987, 6 loss of confidence is not one of the legal causes or grounds
for removal. Consequently, his dismissal from office on the ground of loss confidence violated his right
Held: to security of tenure, petitioner theorized. After a careful study, we are of the irresistible conclusion that
the Court of Appeals ruled correctly on the first three Issue. To be sure, petitioner was not denied the
The 1997 concession agreement is void for being contrary to public policy. The amendments have the right to due process before the PCAGC. Records show that the petitioner filed his answer and other
effect of changing it into and entirely different agreement from the contract bidded upon. The pleadings with respect to his alleged violation of internal revenue laws and regulations, and he attended
amendments present new terms and conditions which provide financial benefit to PIATCO which may the hearings before the investigatory body. It is thus decisively clear that his protestation of non-
have the altered the technical and financial parameters of other bidders had they know that such terms observance of due process is devoid of any factual or legal basis. Neither can it be said that there was
were available. The 1997 concession agreement, the amendments and supplements thereto are set a violation of what petitioner asserts as his security of tenure. According to petitioner, as a Regional
aside for being null and void. Director of Bureau of Internal Revenue, he is CESO eligible entitled to security of tenure. However,
petitioner's claim of CESO eligibility is anemic of evidentiary support. It was incumbent upon him to
prove that he is a CESO eligible but unfortunately, he failed to adduce sufficient evidence on the
The petitioners have local standi. They are prejudiced by the concession agreement as their livelihood
matter. His failure to do so is fatal. As regards the issue of constitutionality of the PCAGC, it was only
is to be taken away from them.
posed by petitioner in his motion for reconsideration before the Regional Trial Court of Makati. It was
certainly too late to raise for the first time at such late stage of the proceedings. As to last issue, It is
worthy to note that in the case under consideration, the administrative action against the petitioner was
UMALI VS. GUINGONA
taken prior to the institution of the criminal case. The charges included in Administrative Order No. 152
[305 SCRA 533; G.R. No. 131124; 21 Mar 1999]
were based on the results of investigation conducted by the PCAGC and not on the criminal charges
before the Ombudsman. In sum, the petition is dismissable on the ground that the Issue posited by the
Facts:
petitioner do not constitute a valid legal basis for overturning the finding and conclusion arrived at by
the Court of Appeals. However, taking into account the antecedent facts and circumstances
Osmundo Umali the petitioner was appointed Regional Director of the Bureau of Internal Revenue by
aforementioned, the Court, in the exercise of its equity powers, has decided to consider the dismissal of
Pres Fidel V. Ramos. He assigned him in Manila, November 29, 1993 to March 15, 1994 and Makati,
the charges against petitioner before the Ombudsman, the succinct and unmistakable manifestation by
March 16, 1994 to August 4, 1994. On August 1, 1994, President Ramos received a confidential
the Commissioner of the Bureau of Internal Revenue that his office is no longer interested in pursuing
memorandum against the petitioner for alleged violations of internal revenue laws, rules and regulations
the case, and the position taken by the Solicitor General, that there is no more basis for Administrative
during his incumbency as Regional Director, more particularly the following malfeasance, misfeasance
Order No. 152, as effective and substantive supervening events that cannot be overlooked.
and nonfeasance. upon receipt of the said confidential memorandum, former President authorized the
issuance of an Order for the preventive suspension of the petitioner and immediately referred the
Complaint against the latter to the Presidential Commission on Anti-Graft and Corruption (PCAGC), for
IN RE CUNANAN
investigation. Petitioner was duly informed of the charges against him. And was directed him to send in
[94 Phil 534; Resolution; 18 Mar 1954]
his answer, copies of his Statement of Assets, and Liabilities for the past three years (3), and Personal
Data Sheet. Initial hearing was set on August 25, 1994, at 2:00 p.m., at the PCAGC Office. On August
Facts:
23, the petitioner filed his required answer. After evaluating the evidence on record, the PCAGC issued
its Resolution of September 23, 1994, finding a prima facie evidence to support six (6) of the twelve
Congress passed Republic Act Number 972, commonly known as the “Bar Flunkers’ Act of 1953.” In
(12) charges against petitioner. On October 6, 1994, acting upon the recommendation of the PCAGC,
accordance with the said law, the Supreme Court then passed and admitted to the bar those
then President Ramos issued Administrative Order No. 152 dismissing petitioner from the service, with
candidates who had obtained an average of 72 per cent by raising it to 75 percent.
forfeiture of retirement and all benefits under the law.
After its approval, many of the unsuccessful postwar candidates filed petitions for admission to the bar
Issue:
invoking its provisions, while other motions for the revision of their examination papers were still
pending also invoked the aforesaid law as an additional ground for admission. There are also others
Whether or Not AO No. 152 violated petitioner's Right to Security of Tenure.
Page 12
who have sought simply the reconsideration of their grades without, however, invoking the law in
question. To avoid injustice to individual petitioners, the court first reviewed the motions for
reconsideration, irrespective of whether or not they had invoked Republic Act No. 972.

Issue:

Whether or Not RA No. 972 is constitutional and valid.

Held:

RA No. 972 has for its object, according to its author, to admit to the Bar, those candidates who
suffered from insufficiency of reading materials and inadequate preparation.

In the judicial system from which ours has been evolved, the admission, suspension, disbarment and
reinstatement of attorneys at law in the practice of the profession and their supervision have been
indisputably a judicial function and responsibility. We have said that in the judicial system from which
ours has been derived, the admission, suspension, disbarment or reinstatement of attorneys at law in
the practice of the profession is concededly judicial.

On this matter, there is certainly a clear distinction between the functions of the judicial and legislative
departments of the government.

It is obvious, therefore, that the ultimate power to grant license for the practice of law belongs
exclusively to this Court, and the law passed by Congress on the matter is of permissive character, or
as other authorities may say, merely to fix the minimum conditions for the license.

Republic Act Number 972 is held to be unconstitutional.

REPUBLIC ACT 6735, INITIATIVE AND REFERENDUM ACT

R.A. No. 6735 was, as its history reveals, intended to cover initiative to propose amendments to the
Constitution. The Act is a consolidation of House Bill No. 21505 and Senate Bill No. 17. The former was
prepared by the committee on Suffrage and Electoral Reforms of Representatives on the basis of two
House Bills referred to it, viz., (a) House Bill No. 497, which dealt with the initiative and referendum
mentioned in Sections 1 and 32 of Article VI of the Constitution; and (b) House Bill No. 988, which dealt
with the subject matter of House Bill No. 497, as well as with initiative and referendum under Section 3
of Article XVII of the Constitution. Senate Bill No. 17 solely, dealt with initiative and referendum
concerning ordinances or resolutions of local government units. The Bicameral Conference Committee
consolidated Senate Bill No. 17 and House Bill No. 21505 into a draft bill, which was subsequently
approved on 8 June 1989 by the Senate and by the House of Representatives. This approved bill is
now R.A. No. 6735.

Page 13
THE FUNDAMENTAL POWERS OF Facts:
THE STATE
Republic Act 1180 or commonly known as “An Act to Regulate the Retail Business” was passed. The
THE POLICE POWER said law provides for a prohibition against foreigners as well as corporations owned by foreigners from
engaging from retail trade in our country. This was protested by the petitioner in this case. According to
him, the said law violates the international and treaty of the Philippines therefore it is unconstitutional.
AGUSTIN VS. EDU Specifically, the Treaty of Amity between the Philippines and China was violated according to him.
[88 SCRA 195; G.R. No. L-49112; 2 Feb 1979]
Issue:
Facts:
Whether or Not Republic Act 1180 is a valid exercise of police power.
President Marcos issued the Letter of Instruction No. 229 which states that all owners, users or drivers
Held:
shall have at all times one pair of early warning devise (EWD) in their cars acquire from any source
depending on the owner’s choice. The Letter of Instruction was assailed by petitioner Leovillo Agustin
to have violated the constitution guarantee of due process against Hon Edu, Land Transportation According to the Court, RA 1180 is a valid exercise of police power. It was also then provided that
Commissioner, Hon. Juan Ponce Enrile, Minister of national Defense, Hon. Juinio, Minister of Public police power can not be bargained away through the medium of a treaty or a contract. The Court also
Works, Transportation and Communication and Hon. Aquino, Minister of Public Highways. Because of provided that RA 1180 was enacted to remedy a real and actual danger to national economy posed by
such contentions, the Implementing Rules and Regulation was ordered to be suspended for a period of alien dominance and control. If ever the law infringes upon the said treaty, the latter is always subject to
6 months. Petitioner alleges that EWD are not necessary because vehicles already have hazard lights qualification or amendment by a subsequent law and the same may never curtain or restrict the scope
(blinking lights) that can be use as a warning device. Also petitioner contest that the letter of instruction of the police power of the state.
violates the delegation of police power because it is deemed harsh, oppressive and unreasonable for
the motorists and those dealers of EWD will become instant millionaires because of such law.
LUTZ VS. ARANETA
Issue: [98 Phil 148; G.R. No. L-7859; 22 Dec 1955]

Facts:
Whether or not Petitioner’s contentions possess merit.
Walter Lutz, as the Judicial Administrator of the Intestate Estate of Antonio Jayme Ledesma, seeks to
Held:
recover from J. Antonio Araneta, the Collector of Internal Revenue, the sum of money paid by the
estate as taxes, pursuant to the Sugar Adjustment Act. Under Section 3 of said Act, taxes are levied on
Petitioner’s contentions are without merit because the exercise of police power may interfere with the owners or persons in control of the lands devoted to the cultivation of sugar cane. Furthermore,
personal liberty or property to ensure and promote the safety, health and prosperity of the State. Also, Section 6 states all the collections made under said Act shall be for aid and support of the sugar
such letter of instruction is intended to promote public safety and it is indeed a rare occurrence that industry exclusively. Lutz contends that such purpose is not a matter of public concern hence making
such contention was alleged in a instruction with such noble purpose. Petitioner also failed to present the tax levied for that cause unconstitutional and void. The Court of First Instance dismissed his
the factual foundation that is necessary to invalidate the said letter of instruction. In cases where there petition, thus this appeal before the Supreme Court.
is absence in the factual foundation, it should be presumed that constitutionality shall prevail. Pres.
Marcos on the other hand possesses vital statistics that will justify the need for the implementation of Issue:
this instruction. As signatory to the 1968 Vienna Conventions on Road Signs and Signals, our country
must abide with the standards given as stated in our Constitution that “the Philippines adopts the Whether or Not the tax levied under the Sugar Adjustment Act ( Commonwealth Act 567) is
generally accepted principles of International Law as part of the law of the land. In the case at bar, the unconstitutional.
Vienna Convention also requires the use of EWD. Vehicle owners are not obliged to buy an EDW. They
can personally create a EWD provided that it is in accordance to the specifications provided by law.
Petitioner’s allegation against the manufacturers of EDW being millionaires is deemed to be an
unfounded speculation. Wherefore, the petition is dismissed. The restraining order regarding the Held:
implementation of the Reflector Law is lifted making the said law immediately executory.
The tax levied under the Sugar Adjustment Act is constitutional. The tax under said Act is levied with a
regulatory purpose, to provide means for the rehabilitation and stabilization of the threatened sugar
industry. Since sugar production is one of the great industries of our nation, its promotion, protection,
and advancement, therefore redounds greatly to the general welfare. Hence, said objectives of the Act
ICHONG VS. HERNANDEZ is a public concern and is therefore constitutional. It follows that the Legislature may determine within
[101 Phil 1117; G.R. No. L-7995; 31 May 1957] reasonable bounds what is necessary for its protection and expedient for its promotion. If objectives
Page 14
and methods are alike constitutionally valid, no reason is seen why the state may not levy taxes to raise
funds for their prosecution and attainment. Taxation may be made with the implement of the state’s Whether or nor the DECREE is constitutional .
police power. In addition, it is only rational that the taxes be obtained from those that will directly benefit
from it. Therefore, the tax levied under the Sugar Adjustment Act is held to be constitutional. Held:

Taxation has been made the implement of the state's police power. The levy of the 30% tax is for a
TIO VS. VIDEOGRAM REGULATORY BOARD public purpose. It was imposed primarily to answer the need for regulating the video industry,
[151 SCRA 208; G.R. No. L-75697; 18 Jun 1987] particularly because of the rampant film piracy, the flagrant violation of intellectual property rights, and
the proliferation of pornographic video tapes. And while it was also an objective of the DECREE to
Facts: protect the movie industry, the tax remains a valid imposition.

The case is a petition filed by petitioner on behalf of videogram operators adversely affected by We find no clear violation of the Constitution which would justify us in pronouncing Presidential Decree
Presidential Decree No. 1987, “An Act Creating the Videogram Regulatory Board" with broad powers to No. 1987 as unconstitutional and void. While the underlying objective of the DECREE is to protect the
regulate and supervise the videogram industry. moribund movie industry, there is no question that public welfare is at bottom of its enactment,
considering "the unfair competition posed by rampant film piracy; the erosion of the moral fiber of the
A month after the promulgation of the said Presidential Decree, the amended the National Internal viewing public brought about by the availability of unclassified and unreviewed video tapes containing
Revenue Code provided that: pornographic films and films with brutally violent sequences; and losses in government revenues due to
the drop in theatrical attendance, not to mention the fact that the activities of video establishments are
"SEC. 134. Video Tapes. — There shall be collected on each processed virtually untaxed since mere payment of Mayor's permit and municipal license fees are required to
video-tape cassette, ready for playback, regardless of length, an annual tax of five engage in business."
pesos; Provided, That locally manufactured or imported blank video tapes shall be
subject to sales tax." WHEREFORE, the instant Petition is hereby dismissed. No costs.

"Section 10. Tax on Sale, Lease or Disposition of Videograms. —


Notwithstanding any provision of law to the contrary, the province shall collect a tax ASSO. OF SMALL LANDOWNERS VS. SEC. OF DAR
of thirty percent (30%) of the purchase price or rental rate, as the case may be, for [175 SCRA 343; G.R. NO. L-78742; 14 JUL 1989]
every sale, lease or disposition of a videogram containing a reproduction of any
motion picture or audiovisual program.” Facts:

“Fifty percent (50%) of the proceeds of the tax collected shall accrue to the Several petitions are the root of the case:
province, and the other fifty percent (50%) shall accrue to the municipality where
the tax is collected; PROVIDED, That in Metropolitan Manila, the tax shall be a. A petition alleging the constitutionality of PD No. 27, EO 228 and 229 and RA 6657.
shared equally by the City/Municipality and the Metropolitan Manila Commission.” Subjects of the petition are a 9-hectare and 5 hectare Riceland worked by four tenants.
Tenants were declared full owners by EO 228 as qualified farmers under PD 27. The
The rationale behind the tax provision is to curb the proliferation and unregulated circulation of petitioners now contend that President Aquino usurped the legislature’s power.
videograms including, among others, videotapes, discs, cassettes or any technical improvement or b. A petition by landowners and sugarplanters in Victoria’s Mill Negros Occidental against
variation thereof, have greatly prejudiced the operations of movie houses and theaters. Such Proclamation 131 and EO 229. Proclamation 131 is the creation of Agrarian Reform
unregulated circulation have caused a sharp decline in theatrical attendance by at least forty percent Fund with initial fund of P50Billion.
(40%) and a tremendous drop in the collection of sales, contractor's specific, amusement and other c. A petition by owners of land which was placed by the DAR under the coverage of
taxes, thereby resulting in substantial losses estimated at P450 Million annually in government Operation Land Transfer.
revenues. d. A petition invoking the right of retention under PD 27 to owners of rice and corn lands
not exceeding seven hectares.
Videogram(s) establishments collectively earn around P600 Million per annum from rentals, sales and
disposition of videograms, and these earnings have not been subjected to tax, thereby depriving the Issue:
Government of approximately P180 Million in taxes each year.
Whether or Not the aforementioned EO’s, PD, and RA were constitutional.
The unregulated activities of videogram establishments have also affected the viability of the movie
industry. Held:

Issue: The promulgation of PD 27 by President Marcos was valid in exercise of Police power and eminent
domain.
Whether or not tax imposed by the DECREE is a valid exercise of police power.
Page 15
The power of President Aquino to promulgate Proc. 131 and EO 228 and 229 was authorized under
Sec. 6 of the Transitory Provisions of the 1987 Constitution. Therefore it is a valid exercise of Police Facts:
Power and Eminent Domain.
Kwong Sing, in his own behalf and of other Chinese laundrymen who has general and the same
RA 6657 is likewise valid. The carrying out of the regulation under CARP becomes necessary to interest, filed a complaint for a preliminary injunction. The Plaintiffs also questioned the validity of
deprive owners of whatever lands they may own in excess of the maximum area allowed, there is enforcing Ordinance No. 532 by the city of Manila. Ordinance No. 532 requires that the receipt be in
definitely a taking under the power of eminent domain for which payment of just compensation is duplicate in English and Spanish duly signed showing the kind and number of articles delivered by
imperative. The taking contemplated is not a mere limitation of the use of the land. What is required is laundries and dyeing and cleaning establishments. The permanent injunction was denied by the trial
the surrender of the title and the physical possession of said excess and all beneficial rights accruing to court. The appellants claim is that Ordinance No. 532 savors of class legislation; putting in mind that
the owner in favour of the farmer. they are Chinese nationals. It unjustly discriminates between persons in similar circumstances; and that
it constitutes an arbitrary infringement of property rights. They also contest that the enforcement of the
A statute may be sustained under the police power only if there is concurrence of the lawful subject and legislation is an act beyond the scope of their police power. In view of the foregoing, this is an appeal
the method. with the Supreme Court.

Subject and purpose of the Agrarian Reform Law is valid, however what is to be determined is the Issue:
method employed to achieve it.
Whether or Not the enforcement of Ordinance no, 532 is an act beyond the scope of police power

LOZANO VS. MARTINEZ Whether or Not the enforcement of the same is a class legislation that infringes property rights.
[146 SCRA 323; G.R. No. L-63419; 18 Dec 1986]

Facts: Held:

A motion to quash the charge against the petitioners for violation of the BP 22 was made, contending Reasonable restraints of a lawful business for such purposes are permissible under the police power.
that no offense was committed, as the statute is unconstitutional. Such motion was denied by the RTC. The police power of the City of Manila to enact Ordinance No. 532 is based on Section 2444,
The petitioners thus elevate the case to the Supreme Court for relief. The Solicitor General, commented paragraphs (l) and (ee) of the Administrative Code, as amended by Act No. 2744, authorizes the
that it was premature for the accused to elevate to the Supreme Court the orders denying their motions municipal board of the city of Manila, with the approval of the mayor of the city:
to quash. However, the Supreme Court finds it justifiable to intervene for the review of lower court's
denial of a motion to quash. (l) To regulate and fix the amount of the license fees for the following: xxxx
xxxxxlaundries xxxx.
Issue:
(ee) To enact all ordinances it may deem necessary and proper for the sanitation
Whether or not BP 22 is constitutional as it is a proper exercise of police power of the State. and safety, the furtherance of the prosperity, and the promotion of the morality,
peace, good order, comfort, convenience, and general welfare of the city and its
Held: inhabitants.

The enactment of BP 22 a valid exercise of the police power and is not repugnant to the constitutional The court held that the obvious purpose of Ordinance No. 532 was to avoid disputes between
inhibition against imprisonment for debt. laundrymen and their patrons and to protect customers of laundries who are not able to decipher
Chinese characters from being defrauded. (Considering that in the year 1920s, people of Manila are
The offense punished by BP 22 is the act of making and issuing a worthless check or a check that is more familiar with Spanish and maybe English.)
dishonored upon its presentation for payment. It is not the non-payment of an obligation which the law
punishes. The law is not intended or designed to coerce a debtor to pay his debt. In whether the ordinance is class legislation, the court held that the ordinance invades no fundamental
right, and impairs no personal privilege. Under the guise of police regulation, an attempt is not made to
The law punishes the act not as an offense against property, but an offense against public order. The violate personal property rights. The ordinance is neither discriminatory nor unreasonable in its
thrust of the law is to prohibit, under pain of penal sanctions, the making of worthless checks and operation. It applies to all public laundries without distinction, whether they belong to Americans,
putting them in circulation. An act may not be considered by society as inherently wrong, hence, not Filipinos, Chinese, or any other nationality. All, without exception, and each every one of them without
malum in se but because of the harm that it inflicts on the community, it can be outlawed and criminally distinction, must comply with the ordinance. The obvious objection for the implementation of the
punished as malum prohibitum. The state can do this in the exercise of its police power. ordinance is based in sec2444 (ee) of the Administrative Code. Although, an additional burden will be
imposed on the business and occupation affected by the ordinance such as that of the appellant by
learning even a few words in Spanish or English, but mostly Arabic numbers in order to properly issue a
receipt, it seems that the same burdens are cast upon the them. Yet, even if private rights of person or
KWONG SING VS. CITY OF MANILA
property are subjected to restraint, and even if loss will result to individuals from the enforcement of the
[41 Phil 103; G.R. No. 15972; 11 Oct 1920]
Page 16
ordinance, this is not sufficient ground for failing to uphold the power of the legislative body. The very MECS Order No. 52, s. 1985, issued by the then Minister of Education, Culture and Sports and dated
foundation of the police power is the control of private interests for the public welfare. 23 August 1985, established a uniform admission test called the National Medical Admission Test
(NMAT) as an additional requirement for issuance of a certificate of eligibility for admission into medical
Finding that the ordinance is valid, judgment is affirmed, and the petition for a preliminary injunction is schools of the Philippines, beginning with the school year 1986-1987. This Order goes on to state that:
denied, with costs against the appellants. "2. The NMAT, an aptitude test, is considered as an instrument toward upgrading the selection of
applicants for admission into the medical schools and its calculated to improve the quality of medical
education in the country. The cutoff score for the successful applicants, based on the scores on the
NMAT, shall be determined every year by the Board of Medical Education after consultation with the
TABLARIN VS. GUTIERREZ Association of Philippine Medical Colleges. The NMAT rating of each applicant, together with the other
[152 SCRA 730; G.R. No. 78164; 31 July 1987] admission requirements as presently called for under existing rules, shall serve as a basis for the
issuance of the prescribed certificate of eligibility for admission into the medical colleges.
Facts:
Issue:
The petitioners sought to enjoin the Secretary of Education, Culture and Sports, the Board of Medical
Education and the Center for Educational Measurement from enforcing Section 5 (a) and (f) of Republic Whether or not Section 5 (a) and (f) of Republic Act No. 2382, as amended, and MECS Order No. 52,
Act No. 2382, as amended, and MECS Order No. 52, series of 1985, dated 23 August 1985 and from s. 1985 are constitutional.
requiring the taking and passing of the NMAT as a condition for securing certificates of eligibility for
admission, from proceeding with accepting applications for taking the NMAT and from administering the Held:
NMAT as scheduled on 26 April 1987 and in the future. The trial court denied said petition on 20 April
1987. The NMAT was conducted and administered as previously scheduled. Yes. We conclude that prescribing the NMAT and requiring certain minimum scores therein as a
condition for admission to medical schools in the Philippines, do not constitute an unconstitutional
Republic Act 2382, as amended by Republic Acts Nos. 4224 and 5946, known as the "Medical Act of imposition.
1959" defines its basic objectives in the following manner:
The police power, it is commonplace learning, is the pervasive and non-waivable power and authority of
"SECTION 1. Objectives. — This Act provides for and shall govern (a) the the sovereign to secure and promote all the important interests and needs — in a word, the public order
standardization and regulation of medical education; (b) the examination for — of the general community. An important component of that public order is the health and physical
registration of physicians; and (c) the supervision, control and regulation of the safety and well being of the population, the securing of which no one can deny is a legitimate objective
practice of medicine in the Philippines." of governmental effort and regulation. Perhaps the only issue that needs some consideration is whether
there is some reasonable relation between the prescribing of passing the NMAT as a condition for
The statute, among other things, created a Board of Medical Education. Its functions as specified in admission to medical school on the one hand, and the securing of the health and safety of the general
Section 5 of the statute include the following: community, on the other hand. This question is perhaps most usefully approached by recalling that the
regulation of the practice of medicine in all its branches has long been recognized as a reasonable
"(a) To determine and prescribe requirements for admission into a method of protecting the health and safety of the public.
recognized college of medicine;
x x x MECS Order No. 52, s. 1985 articulates the rationale of regulation of this type: the improvement of the
(f) To accept applications for certification for admission to a medical school professional and technical quality of the graduates of medical schools, by upgrading the quality of those
and keep a register of those issued said certificate; and to collect from said admitted to the student body of the medical schools. That upgrading is sought by selectivity in the
applicants the amount of twenty-five pesos each which shall accrue to the process of admission, selectivity consisting, among other things, of limiting admission to those who
operating fund of the Board of Medical Education;” exhibit in the required degree the aptitude for medical studies and eventually for medical practice. The
need to maintain, and the difficulties of maintaining, high standards in our professional schools in
Section 7 prescribes certain minimum requirements for applicants to medical schools: general, and medical schools in particular, in the current stage of our social and economic
development, are widely known. We believe that the government is entitled to prescribe an admission
"Admission requirements. — The medical college may admit any student who has test like the NMAT as a means for achieving its stated objective of "upgrading the selection of
not been convicted by any court of competent jurisdiction of any offense involving applicants into [our] medical schools" and of "improv[ing] the quality of medical education in the country.
moral turpitude and who presents (a) a record of completion of a bachelor's degree We are entitled to hold that the NMAT is reasonably related to the securing of the ultimate end of
in science or arts; (b) a certificate of eligibility for entrance to a medical school from legislation and regulation in this area. That end, it is useful to recall, is the protection of the public from
the Board of Medical Education; (c) a certificate of good moral character issued by the potentially deadly effects of incompetence and ignorance in those who would undertake to treat our
two former professors in the college of liberal arts; and (d) birth certificate. Nothing bodies and minds for disease or trauma.
in this act shall be construed to inhibit any college of medicine from establishing, in
addition to the preceding, other entrance requirements that may be deemed WHEREFORE, the Petition for Certiorari is DISMISSED and the Order of the respondent trial court
admissible.” denying the petition for a writ of preliminary injunction is AFFIRMED. Costs against petitioners.

Page 17
and safety, …, and for the protection of property therein; and enforce obedience
thereto with such lawful fines or penalties as the City Council may prescribe under
CITY GOVERNMENT OF QUEZON CITY VS. ERICTA the provisions of subsection (jj) of this section.”
[122 SCRA 759; G.R. No. L-34915; 24 Jun 1983]
The power to regulate does not include the power to prohibit. The power to regulate does not include
Facts: the power to confiscate. The ordinance in question not only confiscates but also prohibits the operation
of a memorial park cemetery, because under Section 13 of said ordinance, 'Violation of the provision
Section 9 of Ordinance No. 6118, S-64, entitled "Ordinance Regulating The Establishment, thereof is punishable with a fine and/or imprisonment and that upon conviction thereof the permit to
Maintenance And Operation Of Private Memorial Type Cemetery Or Burial Ground Within The operate and maintain a private cemetery shall be revoked or cancelled’. The confiscatory clause and
Jurisdiction Of Quezon City And Providing Penalties For The Violation Thereof" provides: the penal provision in effect deter one from operating a memorial park cemetery.

Sec. 9. At least six (6) percent of the total area of the memorial park cemetery shall Moreover, police power is defined by Freund as 'the power of promoting the public welfare by
be set aside for charity burial of deceased persons who are paupers and have restraining and regulating the use of liberty and property'. It is usually exerted in order to merely
been residents of Quezon City for at least 5 years prior to their death, to be regulate the use and enjoyment of property of the owner. If he is deprived of his property outright, it is
determined by competent City Authorities. The area so designated shall not taken for public use but rather to destroy in order to promote the general welfare.
immediately be developed and should be open for operation not later than six
months from the date of approval of the application. It seems to the court that Section 9 of Ordinance No. 6118, Series of 1964 of Quezon City is not a mere
police regulation but an outright confiscation. It deprives a person of his private property without due
For several years, the aforequoted section of the Ordinance was not enforced but seven years after the process of law, nay, even without compensation.
enactment of the ordinance, the Quezon City Council passed a resolution to request the City Engineer,
Quezon City, to stop any further selling and/or transaction of memorial park lots in Quezon City where
the owners thereof have failed to donate the required 6% space intended for paupers burial. MMDA Vs. Bel-Air Village
[328 SCRA 836; G.R. No. 135962; 27 Mar 2000]
The Quezon City Engineer then notified respondent Himlayang Pilipino, Inc. in writing that Section 9 of
the ordinance would be enforced. Facts:

Respondent Himlayang Pilipino reacted by filing a petition for declaratory relief, prohibition and Metropolitan Manila Development Authority (MMDA), petitioner herein, is a Government Agency tasked
mandamus with preliminary injunction seeking to annul Section 9 of the Ordinance in question. with the delivery of basic services in Metro Manila. Bel-Air Village Association (BAVA), respondent
Respondent alleged that the same is contrary to the Constitution, the Quezon City Charter, the Local herein, received a letter of request from the petitioner to open Neptune Street of Bel-Air Village for the
Autonomy Act, and the Revised Administrative Code. use of the public. The said opening of Neptune Street will be for the safe and convenient movement of
persons and to regulate the flow of traffic in Makati City. This was pursuant to MMDA law or Republic
Issue: Act No. 7924. On the same day, the respondent was appraised that the perimeter wall separating the
subdivision and Kalayaan Avenue would be demolished.
Whether or Not Section 9 of the ordinance in question is a valid exercise of police power.
The respondent, to stop the opening of the said street and demolition of the wall, filed a preliminary
Held: injunction and a temporary restraining order. Respondent claimed that the MMDA had no authority to
do so and the lower court decided in favor of the Respondent. Petitioner appealed the decision of the
Section 9 of the City ordinance in question is not a valid exercise of police power. Section 9 cannot be lower courts and claimed that it has the authority to open Neptune Street to public traffic because it is
justified under the power granted to Quezon City to tax, fix the license fee, and regulate such other an agent of the State that can practice police power in the delivery of basic services in Metro Manila.
business, trades, and occupation as may be established or practiced in the City.

Bill of rights states that 'no person shall be deprived of life, liberty or property without due process of Issue:
law' (Art. Ill, Section 1 subparagraph 1, Constitution). On the other hand, there are three inherent
powers of government by which the state interferes with the property rights, namely-. (1) police power, Whether or not the MMDA has the mandate to open Neptune Street to public traffic pursuant to its
(2) eminent domain, (3) taxation. regulatory and police powers.

The police power of Quezon City is defined in sub-section 00, Sec. 12, Rep. Act 537 that reads as Held:
follows:
The Court held that the MMDA does not have the capacity to exercise police power. Police power is
“To make such further ordinance and regulations not repugnant to law as may be primarily lodged in the National Legislature. However, police power may be delegated to government
necessary to carry into effect and discharge the powers and duties conferred by units. Petitioner herein is a development authority and not a political government unit. Therefore, the
this act and such as it shall deem necessary and proper to provide for the health MMDA cannot exercise police power because it cannot be delegated to them. It is not a legislative unit
Page 18
of the government. Republic Act No. 7924 does not empower the MMDA to enact ordinances, approve
resolutions and appropriate funds for the general welfare of the inhabitants of Manila. There is no
syllable in the said act that grants MMDA police power.

It is an agency created for the purpose of laying down policies and coordinating with various national
government agencies, people’s organizations, non-governmental organizations and the private sector
for the efficient and expeditious delivery of basic services in the vast metropolitan area.

TATEL VS. MUNICIPALITY OF VIRAC


[207 SCRA 157; G.R. No. 40243; 11 Mar 1992]

Facts:

Petitioner Celestino Tatel owns a warehouse in barrio Sta. Elena, Municipality of Virac. Complaints
were received by the municipality concerning the disturbance caused by the operation of the abaca
bailing machine inside petitioner’s warehouse. A committee was then appointed by the municipal
council, and it noted from its investigation on the matter that an accidental fire within the warehouse of
the petitioner created a danger to the lives and properties of the people in the neighborhood. Resolution
No. 29 was then passed by the Municipal council declaring said warehouse as a public nuisance within
a purview of Article 694 of the New Civil Code. According to respondent municipal officials, petitioner’s
warehouse was constructed in violation of Ordinance No. 13, series of 1952, prohibiting the
construction of warehouses near a block of houses either in the poblacion or barrios without
maintaining the necessary distance of 200 meters from said block of houses to avoid loss of lives and
properties by accidental fire. On the other hand, petitioner contends that Ordinance No. 13 is
unconstitutional.

Issue:

Whether or not petitioner’s warehouse is a nuisance within the meaning Article 694 of the Civil Code

Whether or not Ordinance No. 13, series of 1952 of the Municipality of Virac is unconstitutional and
void.

Held:

The storage of abaca and copra in petitioner’s warehouse is a nuisance under the provisions of Article
694 of the Civil Code. At the same time, Ordinance No. 13 was passed by the Municipal Council of
Virac in the exercise of its police power. It is valid because it meets the criteria for a valid municipal
ordinance: 1) must not contravene the Constitution or any statute, 2) must not be unfair or oppressive,
3) must not be partial or discriminatory, 4) must not prohibit but may regulate trade, 5) must be general
and consistent with public policy, and 6) must not be unreasonable. The purpose of the said ordinance
is to avoid the loss of property and life in case of fire which is one of the primordial obligation of
government. The lower court did not err in its decision.

Page 19
THE POWER OF EMINENT DOMAIN In the instant case, it is manifest that the petitioner, in pursuit of an objective beneficial to public
interest, seeks to realize the same through its power of eminent domain. In exercising this power,
petitioner intended to acquire not only physical possession but also the legal right to possess and
REPUBLIC VS. TAGLE ultimately to own the subject property. Hence, its mere physical entry and occupation of the property
[299 SCRA 549; G.R. No. 129079; 2 Dec 1998] fall short of the taking of title, which includes all the rights that may be exercised by an owner over the
subject property.
Facts:

Private respondent Helena Z. Benitez is the registered owner of two (2) parcels of land located in
Barangay Salawag, Dasmariñas, Cavite containing an area of 483,331 square meters more or less.

The Philippine Government, through the Philippine Human Resources Development Center (PHRDC),
negotiated with the Japanese International Cooperation Agency (JICA) Survey Team on the CITY OF MANILA VS. CHINESE COMMUNITY
technicalities of the establishment of the ASEAN Human Resources Development Project in the [40 Phil 349; No. 14355; 31 Oct 1919]
Philippines. Among the five (5) main programs of the proposed project was Program III (Construction
Manpower Development) which involved the establishment of a Construction Manpower Development Facts: The City of Manila, plaintiff herein, prayed for the expropriation of a portion private cemetery for
Center (CMDC). PHRDC and private respondent Helena Z. Benitez, signed a Memorandum of the conversion into an extension of Rizal Avenue. Plaintiff claims that it is necessary that such public
Agreement which provides, among others, that Benitez undertakes to lease within the period of twenty improvement be made in the said portion of the private cemetery and that the said lands are within their
(20) years and/or sell a portion of that property (which is no less than ten-hectares) in favor of PHRDC jurisdiction.
which likewise agrees to lease within a period of twenty (20) years and/or buy said property site.
Defendants herein answered that the said expropriation was not necessary because other routes were
The Philippine Women’s University (PWU) and Benitez granted a permit to PHRDC to occupy and use available. They further claimed that the expropriation of the cemetery would create irreparable loss and
the land in question and to undertake land development, electrical and road network installations and injury to them and to all those persons owing and interested in the graves and monuments that would
other related works necessary to attain its objectives. Pursuant thereto, the CMDC took possession of have to be destroyed.
the property and erected buildings and other related facilities necessary for its operations. A deposit
made by the plaintiff with the Philippine National Bank (PNB) in the amount of P708,490.00 which is The lower court ruled that the said public improvement was not necessary on the particular-strip of land
equivalent to the assessed value of the property subject matter hereof based on defendant’s 1990 tax in question. Plaintiff herein assailed that they have the right to exercise the power of eminent domain
declaration, was made. and that the courts have no right to inquire and determine the necessity of the expropriation. Thus, the
same filed an appeal.
In view of the agreement on the sale of the land in question, PHRDC prepared a Deed of Absolute Sale
with Benitez, as vendor, and PHRDC and CMDC, as vendees, duly represented by then Issue:
Undersecretary Gloria M. Arroyo, for the signature of Benitez. Benitez in her own capacity did not sign
the deed of absolute sale. Whether or not the courts may inquire into, and hear proof of the necessity of the expropriation.
Failing to acquire the property involved through negotiated sale, petitioner, through the Department of
Trade and Industry, to which CMDC is attached, instituted a complaint for Eminent Domain, pursuant to Held:
the provisions of Executive Order No. 1035, dated June 25, 1985.
The courts have the power of restricting the exercise of eminent domain to the actual reasonable
A Motion for Issuance of Writ of Possession was granted by the court but quashed it subsequently. necessities of the case and for the purposes designated by the law. The moment the municipal
corporation or entity attempts to exercise the authority conferred, it must comply with the conditions
Issue: accompanying the authority. The necessity for conferring the authority upon a municipal corporation to
exercise the right of eminent domain is admittedly within the power of the legislature. But whether or not
Whether or Not the respondent judge may quash a writ of possession on the ground that the the municipal corporation or entity is exercising the right in a particular case under the conditions
expropriating government agency is already occupying the property sought to be expropriated. imposed by the general authority, is a question that the courts have the right to inquire to.

Held:
REPUBLIC VS. PLDT
No. Under Section 7 of EO 1035, when the government or its authorized agent makes the required [26 SCRA 320; G.R. No. L-18841; 27 Jan 1969]
deposit, the trial court has a ministerial duty to issue a writ of possession. The expropriation of real  
property does not include mere physical entry or occupation of land. Although eminent domain usually Facts:
involves a taking of title, there may also be compensable taking of only some, not all, of the property  
interests in the bundle of rights that constitute ownership. The plaintiff Republic of the Philippines is a political entity exercising government powers through one
of its branches, the Bureau of Telecommunication. Herein defendant, PLDT is a public service
Page 20
corporation holding a franchise to install operates and maintains a telephone system. After its creation, We do not overlook that the modern tendency is to regard the beautification of neighborhoods as
the BOT set up its own government telephone system by utilizing its own appropriations and other conducive to the comfort and happiness of residents.
equipment and by renting trunk lines of the PLDT to enable the govt offices to call privately. BOT
entered into an agreement with the RCA communications for joint overseas telephone service whereby As the case now stands, every structure that may be erected on appellants' land, regardless of its own
BOT would convey overseas calls received by RCA to local residents. PLDT complained to the BOT beauty, stands condemned under the ordinance in question, because it would interfere with the view of
that it was a violation of the condition of their agreement since the BOT had used trunk lines only for the the public plaza from the highway. The appellants would, in effect, be constrained to let their land
use of government offices but even to serve private persons or the general public in competition with remain idle and unused for the obvious purpose for which it is best suited, being urban in character. To
the business of PLDT. Subsequently, the plaintiff commenced suit against PLDT asking the court legally achieve that result, the municipality must give appellants just compensation and an opportunity
judgment be rendered ordering the PLDT to execute a contract with the plaintiff, through the BOT for to be heard.
the use of the facilities of PLDT's telephone system throughout the country under such conditions as
the court may consider reasonable. The CFI rendered judgment stating that it could not compel PLDT
to enter into such agreement. Hence this petition. CITY OF BAGUIO V. NAWASA
[106 Phil; G.R. No. L-12032; 31 Aug 1959]
Issue:
Facts:
Whether or Not PLDT may be compelled to enter into such agreement.
  Plaintiff a municipal corporation filed a complaint against defendant a public corporation, created under
Held: Act.1383. It contends that the said act does not include within its purview the Baguio Water Works
system, assuming that it does, is unconstitutional because it deprives the plaintiff ownership, control
Yes, the state, may, in the interest of national welfare transfer utilities to public ownership upon and operation of said water works without just compensation and due process of law. The defendant
payment of just compensation, there is no reason why the state ma not require a public utility to render filed a motion to dismiss ion the ground that it is not a proper exercise of police power and eminent
services in the general interest provided just compensation is paid. domain. The court denied the motion and ordered the defendants to file an answer. The court holds that
the water works system of Baguio belongs to private property and cannot be expropriated without just
compensation. Sec. 8 of R.A.1383 provides for the exchange of the NAWASA assets for the value of
the water works system of Baguio is unconstitutional for this is not just compensation. Defendants
PEOPLE VS. FAJARDO motion for reconsideration was denied hence this appeal.
[104 Phil 443; G.R. No. L-12172; 29 Aug 1958]
Issue:
Facts:
Whether or Not there is a valid exercise of police power of eminent domain.
The municipal council of baao, camarines sur stating among others that construction of a building,
which will destroy the view of the plaza, shall not be allowed and therefore be destroyed at the expense Held:
of the owner, enacted an ordinance. Herein appellant filed a written request with the incumbent
municipal mayor for a permit to construct a building adjacent to their gasoline station on a parcel of land R.A. 1383 does not constitute a valid exercise of police power. The act does not confiscate, destroy or
registered in Fajardo's name, located along the national highway and separated from the public plaza appropriate property belonging to a municipal corporation. It merely directs that all water works
by a creek. The request was denied, for the reason among others that the proposed building would belonging to cities, municipalities and municipal districts in the Philippines to be transferred to the
destroy the view or beauty of the public plaza. Defendants reiterated their request for a building permit, NAWASA. The purpose is placing them under the control and supervision of an agency with a view to
but again the mayor turned down the request. Whereupon, appellants proceeded with the construction promoting their efficient management, but in so doing does not confiscate them because it directs that
of the building without a permit, because they needed a place of residence very badly, their former they be paid with equal value of the assets of NAWASA.
house having been destroyed by a typhoon and hitherto they had been living on leased property.
Thereafter, defendants were charged in violation of the ordinance and subsequently convicted. Hence The Baguio water works system is not like a public road, the park, street other public property held in
this appeal. trust by a municipal corporation for the benefit of the public. But it is a property of a municipal
  corporation, water works cannot be taken away except for public use and upon payment of just
Issue: compensation. Judgment affirmed.

Whether or Not the ordinance is a valid exercise of police power.


  NATIONAL POWER CORP. VS. GUTIERREZ
Held: [193 SCRA 1; G.R. No. 60077; 18 Jan 1991]

No. It is not a valid exercise of police power. The ordinance is unreasonable and oppressive, in that it Facts:
operates to permanently deprive appellants of the right to use their own property; hence, it oversteps
the bounds of police power, and amounts to a taking of appellant’s property without just compensation. Petitioner filed an action to acquire a right of way over the land of Respondents for the construction of
Page 21
transmission lines. Petitioner was adjudged to pay the full market value of land traversed by the deflation because in eminent domain cases the obligation to pay arises from law independent of
transmission lines. Petitioner argued that it was only asking for a right of way. contract.

Issue:
EPZA VS. DULAY
Whether or Not the acquisition of the right of way constitutes "taking" and such the case will be entitled [148 SCRA 305; G.R. No. L-59603; 29 Apr 1987]
just compensation.
Facts:
Held:
The four parcels of land which are the subject of this case is where the Mactan Export Processing Zone
The acquisition of the right of way constitutes taking. It perpetually deprives Respondents of their Authority in Cebu (EPZA) is to be constructed. Private respondent San Antonio Development
proprietary rights. No plant higher than three meters is allowed below the transmission lines. Because Corporation (San Antonio, for brevity), in which these lands are registered under, claimed that the lands
of high tension current conveyed through the transmission lines, danger to life and limbs cannot be were expropriated to the government without them reaching the agreement as to the compensation.
discounted. The owner of the property is entitled to just compensation. Respondent Judge Dulay then issued an order for the appointment of the commissioners to determine
the just compensation. It was later found out that the payment of the government to San Antonio would
be P15 per square meter, which was objected to by the latter contending that under PD 1533, the basis
REPUBLIC VS. CASTELVI of just compensation shall be fair and according to the fair market value declared by the owner of the
[58 SCRA 336; G.R. No. L-20620; 15 Aug 1974] property sought to be expropriated, or by the assessor, whichever is lower. Such objection and the
subsequent Motion for Reconsideration were denied and hearing was set for the reception of the
Facts: commissioner’s report. EPZA then filed this petition for certiorari and mandamus enjoining the
respondent from further hearing the case.
In 1947, the republic, through the Armed Forces of the Philippines (AFP), entered into a lease
agreement with Castelvi on a year-to-year basis. When Castelvi gave notice to terminate the lease in Issue:
1956, the AFP refused. She then instituted an ejectment proceeding against the AFP. In 1959,
however, the republic commenced the expropriation proceedings for the land in question. Whether or Not the exclusive and mandatory mode of determining just compensation in PD 1533 is
unconstitutional.
Issue:
Held:
Whether or Not the compensation should be determined as of 1947 or 1959.
The Supreme Court ruled that the mode of determination of just compensation in PD 1533 is
Held: unconstitutional.

The Supreme Court ruled that the “taking” should not be reckoned as of 1947, and that just The method of ascertaining just compensation constitutes impermissible encroachment to judicial
compensation should not be determined on the basis of the value of the property as of that year. prerogatives. It tends to render the courts inutile in a matter in which under the Constitution is reserved
to it for financial determination. The valuation in the decree may only serve as guiding principle or one
The requisites for taking are: 1) the expropriator must enter a private property, 2) the entry must be for of the factors in determining just compensation, but it may not substitute the court’s own judgment as to
more than a momentary period, 3) it must be under warrant or color of authorities, 4) the property must what amount should be awarded and how to arrive at such amount. The determination of just
be devoted for public use or otherwise informally appropriated or injuriously affected, and 5) the compensation is a judicial function. The executive department or the legislature may make the initial
utilization of the property for public use must be such a way as to oust the owner and deprive him of determination but when a party claims a violation of the guarantee in the Bill of Rights that the private
beneficial enjoyment of the property. Under Sec. 4 Rule 67 of the Rules of Court, “just compensation” is party may not be taken for public use without just compensation, no statute, decree, or executive order
to be determined as of the date of the filing of the complaint. The Supreme Court has ruled that when can mandate that its own determination shall prevail over the court’s findings. Much less can the courts
the taking of the property sought to be expropriated coincides with the commencement of the be precluded from looking into the justness of the decreed compensation.
expropriation proceedings, or takes place subsequent to the filing of the complaint for eminent domain,
the just compensation should be determined as of the date of the filing of the complaint. In the instant
case, it is undisputed that the Republic was placed in possession of the Castelvi property, by authority AMIGABLE VS. CUENCA
of court, on August 10, 1959. The “taking” of the Castelvi property for the purposes of determining the [43 SCRA 360; G.R. No. L-26400; 29 Feb. 1972]
just compensation to be paid must, therefore, be reckoned as of June 26, 1959 when the complaint for
eminent domain was filed. There is no basis to the contention of the Republic that a lease on a year-to- Facts:
year basis can give rise to permanent right to occupy since by express provision a lease made for a
determinate time, as was the lease of Castelvi land in the instant case, ceases upon the day fixed, Victoria Amigable is the registered owner of a particular lot. At the back of her Transfer Certificate of
without need of a demand (Art. 1669, New Civil Code). The Supreme Court, however, did not apply Art. Title (1924), there was no annotation in favor of the government of any right or interest in the property.
1250 of the New Civil Code for the adjustment of the peso rate in times of extraordinary inflation or Without prior expropriation or negotiated sale, the government used a portion of the lot for the
Page 22
construction of the Mango and Gorordo Avenues. On 1958, Amigable’s counsel wrote the President of
the Philippines, requesting payment of the portion of the said lot. It was disallowed by the Auditor
General in his 9th Endorsement. Petitioner then filed in the court a quo a complaint against the Republic
of the Philippines and Nicolas Cuenca, in his capacity as Commissioner of Public Highways for the Held:
recovery of ownership and possession of the lot. According to the defendants, the action was
premature because it was not filed first at the Office of the Auditor General. According to them, the right The Supreme Court declared the Resolution as unconstitutional. It held that to compel print media
of action for the recovery of any amount had already prescribed, that the Government had not given its companies to donate “Comelec space” amounts to “taking” of private personal property without
consent to be sued, and that plaintiff had no cause of action against the defendants. payment of the just compensation required in expropriation cases. Moreover, the element of necessity
for the taking has not been established by respondent Comelec, considering that the newspapers were
not unwilling to sell advertising space. The taking of private property for public use is authorized by the
Issue: constitution, but not without payment of just compensation. Also Resolution No. 2772 does not
constitute a valid exercise of the police power of the state. In the case at bench, there is no showing of
Whether or Not, under the facts of the case, appellant may properly sue the government. existence of a national emergency to take private property of newspaper or magazine publishers.

Held:
REYES VS. NATIONAL HOUSING AUTHORITY
In the case of Ministerio v. Court of First Instance of Cebu, it was held that when the government takes [395 SCRA 494; GR NO. 147511; 20 JAN 2003]
away property from a private landowner for public use without going through the legal process of
expropriation or negotiated sale, the aggrieved party may properly maintain a suit against the
government without violating the doctrine of governmental immunity from suit without its consent. In the Facts:
case at bar, since no annotation in favor of the government appears at the back of the certificate of title
and plaintiff has not executed any deed of conveyance of any portion of the lot to the government, then Respondent National Housing Authority (NHA) filed complaints for the expropriation of sugarcane lands
she remains the owner of the lot. She could then bring an action to recover possession of the land belonging to the petitioners. The stated public purpose of the expropriation was the expansion of the
anytime, because possession is one of the attributes of ownership. However, since such action is not Dasmariñas Resettlement Project to accommodate the squatters who were relocated from the
feasible at this time since the lot has been used for other purposes, the only relief left is for the Metropolitan Manila area. The trial court rendered judgment ordering the expropriation of these lots and
government to make due compensation—price or value of the lot at the time of the taking. the payment of just compensation. The Supreme Court affirmed the judgment of the lower court.

A few years later, petitioners contended that respondent NHA violated the stated public purpose for the
PHILIPPINE PRESS INSTITUTE VS. COMELEC expansion of the Dasmariñas Resettlement Project when it failed to relocate the squatters from the
[244 SCRA 272; G.R. No. 119694; 22 May 1995] Metro Manila area, as borne out by the ocular inspection conducted by the trial court which showed that
most of the expropriated properties remain unoccupied. Petitioners likewise question the public nature
of the use by respondent NHA when it entered into a contract for the construction of low cost housing
Facts: units, which is allegedly different from the stated public purpose in the expropriation proceedings.
Hence, it is claimed that respondent NHA has forfeited its rights and interests by virtue of the
Respondent Comelec promulgated Resolution No. 2772 directing newspapers to provide free Comelec expropriation judgment and the expropriated properties should now be returned to herein petitioners.
space of not less than one-half page for the common use of political parties and candidates. The
Comelec space shall be allocated by the Commission, free of charge, among all candidates to enable Issue:
them to make known their qualifications, their stand on public Issue and their platforms of government.
The Comelec space shall also be used by the Commission for dissemination of vital election Whether or not the judgment of expropriation was forfeited in the light of the failure of respondent NHA
information. to use the expropriated property for the intended purpose but for a totally different purpose.

Held:
Petitioner Philippine Press Institute, Inc. (PPI), a non-profit organization of newspaper and magazine
publishers, asks the Supreme Court to declare Comelec Resolution No. 2772 unconstitutional and void
The Supreme Court held in favor of the respondent NHA. Accordingly, petitioners cannot insist on a
on the ground that it violates the prohibition imposed by the Constitution upon the government against
restrictive view of the eminent domain provision of the Constitution by contending that the contract for
the taking of private property for public use without just compensation. On behalf of the respondent
low cost housing is a deviation from the stated public use. It is now settled doctrine that the concept of
Comelec, the Solicitor General claimed that the Resolution is a permissible exercise of the power of
public use is no longer limited to traditional purposes. The term "public use" has now been held to be
supervision (police power) of the Comelec over the information operations of print media enterprises
synonymous with "public interest," "public benefit," "public welfare," and "public convenience." Thus,
during the election period to safeguard and ensure a fair, impartial and credible election.
whatever may be beneficially employed for the general welfare satisfies the requirement of public use."
Issue:
In addition, the expropriation of private land for slum clearance and urban development is for a public
purpose even if the developed area is later sold to private homeowners, commercials firms,
Whether or not Comelec Resolution No. 2772 is unconstitutional.
Page 23
entertainment and service companies, and other private concerns. Moreover, the Constitution itself g. A petition by owners of land which was placed by the DAR under the coverage of
allows the State to undertake, for the common good and in cooperation with the private sector, a Operation Land Transfer.
continuing program of urban land reform and housing which will make at affordable cost decent housing h. A petition invoking the right of retention under PD 27 to owners of rice and corn lands
and basic services to underprivileged and homeless citizens in urban centers and resettlement areas. not exceeding seven hectares.
The expropriation of private property for the purpose of socialized housing for the marginalized sector is
in furtherance of social justice. Issue:

Whether or Not the aforementioned EO’s, PD, and RA were constitutional.


MUNICIPALITY OF PARAÑAQUE VS. VM REALTY CORPORATION
[292 SCRA 676; G. R. NO. 127820; 20 JUL 1998] Held:

Facts: The promulgation of PD 27 by President Marcos was valid in exercise of Police power and eminent
domain.
Petitioner sought to exercise its power of eminent domain based on a resolution by the municipal
council. Petitioner cites a previous case wherein a resolution gave authority to exercise eminent The power of President Aquino to promulgate Proc. 131 and EO 228 and 229 was authorized under
domain. Petitioner also relies on the Implementing Rules, which provides that a resolution authorizes a Sec. 6 of the Transitory Provisions of the 1987 Constitution. Therefore it is a valid exercise of Police
Local Government Unit to exercise eminent domain. Power and Eminent Domain.

Issue: RA 6657 is likewise valid. The carrying out of the regulation under CARP becomes necessary to
deprive owners of whatever lands they may own in excess of the maximum area allowed, there is
Whether or Not an LGU can exercise its power of eminent domain pursuant to a resolution by its law- definitely a taking under the power of eminent domain for which payment of just compensation is
making body. imperative. The taking contemplated is not a mere limitation of the use of the land. What is required is
the surrender of the title and the physical possession of said excess and all beneficial rights accruing to
Held: the owner in favour of the farmer.

Under Section 19, of the present Local Government Code (RA 7160), it is stated as the first requisite A statute may be sustained under the police power only if there is concurrence of the lawful subject and
that LGUs can exercise its power of eminent domain if there is an ordinance enacted by its legislative the method.
body enabling the municipal chief executive. A resolution is not an ordinance, the former is only an
opinion of a law-making body, the latter is a law. The case cited by Petitioner involves BP 337, which Subject and purpose of the Agrarian Reform Law is valid, however what is to be determined is the
was the previous Local Government Code, which is obviously no longer in effect. RA 7160 prevails method employed to achieve it.
over the Implementing Rules, the former being the law itself and the latter only an administrative rule
which cannot amend the former.
ESLABAN VS. ONORIO
[360 SCRA 230; G.R. NO. 146062; 28 JUN 2001]

Facts:

ASLP VS. SEC. OF AGRARIAN REFORM Clarita Vda. De Onorio is the owner of the land in Barangay M. Roxas, Sto. Nino, South Cotabato. Such
[175 SCRA 343; G.R. NO. 78742; 14 JUL 1989] land is the subject for the construction of an irrigation canal of the National Irrigation Administration
(NIA). Mr. Santiago Eslaban Jr. is the project manager of NIA. The parties agreed to the construction of
Facts: the canal provided that the government will pay for the area that has been taken. A right-of-way
agreement was entered into by the parties in which respondent was paid the amount of P4, 180.00 as
Several petitions are the root of the case: right of way damages. Subsequently, respondent executed an Affidavit of Waiver of Rights and Fees
which waives her rights for the damage to the crops due to construction of the right of way. After which,
e. A petition alleging the constitutionality of PD No. 27, EO 228 and 229 and RA 6657. respondent demands that petitioner pay P111, 299.55 for taking her property but the petitioner refused.
Subjects of the petition are a 9-hectare and 5 hectare Riceland worked by four tenants. Petitioner states that the government had not consented to be sued and that the respondent is not
Tenants were declared full owners by EO 228 as qualified farmers under PD 27. The entitled for compensation by virtue of the homestead patent under CA no. 141. The RTC held that the
petitioners now contend that President Aquino usurped the legislature’s power. NIA should pay respondent the amount of P107, 517.60 as just compensation for the 24,660 sq meters
f. A petition by landowners and sugarplanters in Victoria’s Mill Negros Occidental against that have been used for the construction of the canal. The Court of Appeals also affirmed the decision
Proclamation 131 and EO 229. Proclamation 131 is the creation of Agrarian Reform of the RTC.
Fund with initial fund of P50Billion.
Issue:
Page 24
passed B.P. Blg. 340 authorizing the national government to expropriate certain properties in Pasay
City for the EDSA Extension. The property of the Knechts was part of those expropriated under B.P.
Whether or Not the CA erred in affirming the decision of the RTC.
Blg. 340. The government gave out just compensation for the lands expropriated under B.P. Blg. 340.
Salem was included and received partial payment. Seven of the eight houses of the Knechts were
Held:
demolished and the government took possession of the portion of land on which the houses stood.
Since the Knechts refused to vacate their one remaining house, Salem filed a case against them for
The CA is correct in affirming the decision of the RTC but modifications shall be made regarding the
unlawful detainer. As defense, the Knechts claimed ownership of the land and building. The Municipal
value of the just compensation. The following are the points to be considered in arriving in this decision.
Trial Court however ordered the Knechts' ejectment thus their residence was demolished.
First, Rule 7 par 5 of the Rule of Civil Procedure provides that the certification against forum shopping
The Knechts continuously claimed ownership of the property and allege that they must be given just
should only be executed by the plaintiff or the principal. The petition for review was filed by Mr. Eslaban
compensation.
jr. while the verification or certification were signed by Mr. Cesar Gonzales, an administrator of the
agency. Neither of the two has the authority to sign such certificate for they are not the plaintiff or
Issue:
principal. Such case is a sufficient ground for dismissing this petition.
Whether or not Knechts are the lawful owners of the land at subject.
Second, PD NO. 1529 provides that the owner is required to recognize in favor of the government the
easement of a “public highway, way, private way established by law, or any government canal where
Held:
the certificate of title does not state that the boundaries thereof have been pre-determined. In the case
at bar, the irrigation canal was constructed on Oct 1981 after the property had been registered in May
The Supreme Court held that the Knechts were not the owners anymore of the said land. The Knechts'
of 1976. In this case, prior expropriation proceedings must be filed and just compensation shall be paid
right to the land had been foreclosed after they failed to redeem it one year after the sale at public
to the owner before the land could be taken for public use.
auction. Since the petitions questioning the order of dismissal were likewise dismissed by the Court of
Appeals and this Court, the order of dismissal became final and res judicata on the issue of ownership
Third, In this case, just compensation is defined as not only the correct amount to be paid but the
of the land. Petitioners contended that they did not receive notice of their tax delinquency. Neither did
reasonable time for the Government to pay the owner. The CA erred in this point by stating that the
they receive notice of the auction sale. However, this question has been previously raised in the cases
market value (just compensation) of the land is determined in the filing of the complaint in 1991.The
which have been already set aside. The court is not a trier of facts. Res judicata has already set it. The
determination of such value should be from the time of its taking by the NIA in 1981.
Knechts therefore are not the lawful owners of the land and are not any longer accountable for just
compensation given by the government.
Lastly, the petitioner cannot argue that the Affidavit of waiver of rights and fees executed by the
respondent pertains to the payment of the value of the land therefore exempting NIA to pay the value of
Note: Res judicata is a ground for dismissal of an action. It is a rule that precludes parties from
the land taken. Such waiver pertains only to the crops and improvements that were damage due to the
relitigating Issue actually litigated and determined by a prior and final judgment. It pervades every well-
construction of the right-of-way not the value of the land.
regulated system of jurisprudence, and is based upon two grounds embodied in various maxims of the
common law — one, public policy and necessity, that there should be a limit to litigation; and another,
Wherefore, decision of CA affirmed with modification regarding the just compensation in the amount of
the individual should not be vexed twice for the same cause. When a right of fact has been judicially
P16, 047.61 per hectare.
tried and determined by a court of competent jurisdiction, or an opportunity for such trial has been
given, the judgment of the court, so long as it remains unreversed, should be conclusive upon the
parties and those in privity with them in law or estate. To follow a contrary doctrine would subject the
KNECHT VS. COURT OF APPEALS
public peace and quiet to the will and neglect of individuals and prefer the gratification of the litigious
[290 SCRA 223; G.R. NO. 108015, 20 MAY 1998]
disposition of the parties to the preservation of the public tranquility.
Facts:
Res judicata applies when: (1) the former judgment or order is final; (2) the judgment or order is one on
the merits; (3) it was rendered by a court having jurisdiction over the subject matter and the parties; (4)
The instant case is an unending sequel to several suits commenced almost twenty years ago involving
there is between the first and second actions, identity of parties, of subject matter and of cause of
a parcel of land located at the corner of the south end of EDSA and F.B. Harrison in Pasay City. The
action.
land was owned by petitioners Cristina de Knecht and her son, Rene Knecht. On the land, the Knechts
constructed eight houses, leased out the seven and occupied one of them as their residence. In 1979,
the government filed for the expropriation of Knechts’ property. The government wanted to use the land
REPUBLIC VS. KER
for the completion of the Manila Flood Control and Drainage Project and the extension of the EDSA
[383 SCRA 584; G.R. NO. 136171, 2 JULY 2002]
towards Roxas Boulevard. In 1982, the City Treasurer of Pasay discovered that the Knechts failed to
pay real estate taxes on the property from 1980 to 1982. As a consequence of this deficiency, the City
Facts:
Treasurer sold the property at public auction for the same amount of their deficiency taxes. The highest
bidders were respondent Spouses Anastacio and Felisa Babiera (the Babieras) and respondent
Petitioner filed before the Regional Trial Court of Davao City a petition for expropriation of portions of
Spouses Alejandro and Flor Sangalang (the Sangalangs). Subsequently, Sangalang and Babiera sold
two parcels of land owned by respondent. Petitioner needed the parcels of land for the widening of the
the land to respondent Salem Investment Corporation. On February 17, 1983, the Batasang Pambansa
Page 25
road component of J.P. Laurel-Buhangin Interchange in Davao City. The Regional trial court rendered
decision of a fair just compensation for defendant Ker Corporation. However, it was challenged by
Petitioner Republic of the Philippines, represented by the Department of Public Works and Highways
alleging that just compensation for site must be reduced. Petitioner alleged that when the petition for
expropriation was filed, the tax declaration of the property indicated its assessed value at a lower price.

Issue:

Whether or not respondent Ker Company was given a decision for fair just compensation.

Held:

The Supreme Court held that the valuation for the lot Sites are excessive and unreasonable. Just
compensation cannot be measured by the assessed value of the property as stated in the tax
declaration and schedule of market values. For the purpose of appraisal, the fair market value of the
property is taken into account and such value refers to the highest price in terms of money which a
property will bring if exposed for sale in the public market.

In computing just compensation for expropriation proceedings, it is the value of the land at the time of
the taking or at the time of the filing of the complaint not at the time of the rendition of judgment which
should be taken into consideration. 4 Section 4, Rule 67 of the 1997 Rules of Civil Procedure provides
that just compensation is to be determined as of the date of the taking or the filing of the complaint
whichever came first. On this matter, the appellate court is correct in disregarding petitioner's claim.

MANOSCA VS. COURT OF APPEALS


[252 SCRA 412; G.R. NO. 106440, 29 JAN. 1996]

Facts:

The National Historical Institute declared the parcel of land owned by Petitioners as a national historical
landmark, because it was the site of the birth of Felix Manalo, the founder of Iglesia ni Cristo. The
Republic of the Philippines filed an action to appropriate the land. Petitioners argued that the
expropriation was not for a public purpose.

Issue:

Whether or Not the taking or exercise of eminent domain may be granted.

Held:

Public use should not be restricted to the traditional uses. The taking is for a public use because of the
contribution of Felix Manalo to the culture and history of the Philippines.

Page 26
Petitioners, who are professionals in the city, assail Ordinance No. 3398 together with the law
THE POWER OF TAXATION authorizing it (Section 18 of the Revised Charter of the City of Manila). The ordinance imposes a
municipal occupation tax on persons exercising various professions in the city and penalizes non-
payment of the same. The law authorizing said ordinance empowers the Municipal Board of the city to
PASCUAL VS. SEC. OF PUBLIC WORKS impose a municipal occupation tax on persons engaged in various professions. Petitioners, having
[110 PHIL 331; G.R. NO.L-10405; 29 DEC 1960] already paid their occupation tax under section 201 of the National Internal Revenue Code, paid the tax
under protest as imposed by Ordinance No. 3398. The lower court declared the ordinance invalid and
Facts: affirmed the validity of the law authorizing it.

Petitioner, the governor of the Province of Rizal, filed an action for declaratory relief with injunction on Issue:
the ground that RA 920, Act appropriating funds for public works, providing P85,000 for the
construction, reconstruction, repair, extension and improvement of Pasig feeder road terminals, were Whether or Not the ordinance and law authorizing it constitute class legislation, and authorize what
nothing but projected and planned subdivision roads within Antonio Subdivision. Antonio Subdivision is amounts to double taxation.
owned by the respondent, Jose Zulueta, a member of the Senate of the Philippines. Respondent
offered to donate the said feeder roads to the municipality of Pasig and the offer was accepted by the Held:
council, subject to a condition that the donor would submit plan of the roads and an agreement to
change the names of two of the street. However, the donation was not executed, which prompted The Legislature may, in its discretion, select what occupations shall be taxed, and in its discretion may
Zuleta to write a letter to the district engineer calling attention the approval of RA 920. The district tax all, or select classes of occupation for taxation, and leave others untaxed. It is not for the courts to
engineer, on the other hand, did not endorse the letter that inasmuch the feeder roads in question were judge which cities or municipalities should be empowered to impose occupation taxes aside from that
private property at the time of passage and approval of RA 920, the appropriation for the construction imposed by the National Government. That matter is within the domain of political departments. The
was illegal and therefore, void ab initio. Petitioner, prayed for RA 920 be declared null and void and the argument against double taxation may not be invoked if one tax is imposed by the state and the other is
alleged deed of donation be declared unconstitutional. Lower court dismissed the case and dissolved imposed by the city. It is widely recognized that there is nothing inherently terrible in the requirement
the writ of preliminary injunction. that taxes be exacted with respect to the same occupation by both the state and the political
subdivisions thereof. Judgment of the lower court is reversed with regards to the ordinance and
Issue: affirmed as to the law authorizing it.

Whether or Not the deed of donation and the appropriation of funds stipulated in RA 920 are
constitutional. OSMEÑA VS. ORBOS
[220 SCRA 703; G.R. NO. 99886; 31 MAR 1993]
Held:
Facts:
The ruling case law rules that the legislature is without power to appropriate public revenue for anything
but public purpose. The taxing power must be exercised for public purposes only and the money raised On October 10, 1984, Pres. Marcos issued P.D. 1956 creating a Special Account in the General Fund,
by taxation can be expended only for public purposes and not for the advantage of private individuals. designated as the Oil Price Stabilization Fund (OPSF). The OPSF was designed to reimburse oil
companies for cost increases in crude oil and imported petroleum products resulting from exchange
In the case at bar, the legality of the appropriation of the feeder roads depend upon whether the said rate adjustments and from increases in the world market prices of crude oil.
roads were public or private property when the bill was passed by congress or when it became
effective. The land which was owned by Zulueta, the appropriation sought a private purpose and hence, Subsequently, the OPSF was reclassified into a "trust liability account," in virtue of E.O. 1024, and
null and void. The donation did not cure the nullity of the appropriation; therefore a judicial nullification ordered released from the National Treasury to the Ministry of Energy.
of a said donation need not precede the declaration of unconstitutionality of the said appropriation.
Pres. Aquino, amended P.D. 1956. She promulgated Executive Order No. 137 on February 27, 1987,
expanding the grounds for reimbursement to oil companies for possible cost underrecovery incurred as
The decision appealed from is reversed.
a result of the reduction of domestic prices of petroleum products, the amount of the underrecovery
being left for determination by the Ministry of Finance.
The petition avers that the creation of the trust fund violates
29(3), Article VI of the Constitution, reading as follows:
PUNSALAN VS. MUNICIPAL BOARD OF MANILA
(3) All money collected on any tax levied for a special purpose shall be treated as a
[95 PHIL 46; NO.L-4817; 26 MAY 1954]
special fund and paid out for such purposes only. If the purpose for which a special
fund was created has been fulfilled or abandoned, the balance, if any, shall be
Facts:
transferred to the general funds of the Government.

Page 27
The petitioner argues that "the monies collected pursuant to . . P.D. 1956, as amended, must be treated d) Any resulting peso cost differentials in case the actual peso costs paid by oil companies in
as a 'SPECIAL FUND,' not as a 'trust account' or a 'trust fund,' and that "if a special tax is collected for a the importation of crude oil and petroleum products is less than the peso costs computed
specific purpose, the revenue generated therefrom shall 'be treated as a special fund' to be used only using the reference foreign exchange rate as fixed by the Board of Energy.
for the purpose indicated, and not channeled to another government objective." Petitioner further points
out that since "a 'special fund' consists of monies collected through the taxing power of a State, such Hence, it seems clear that while the funds collected may be referred to as taxes, they are exacted in
amounts belong to the State, although the use thereof is limited to the special purpose/objective for the exercise of the police power of the State. Moreover, that the OPSF is a special fund is plain from
which it was created." the special treatment given it by E.O. 137. It is segregated from the general fund; and while it is placed
in what the law refers to as a "trust liability account," the fund nonetheless remains subject to the
He also contends that the "delegation of legislative authority" to the ERB violates 28 (2). Article VI of the scrutiny and review of the COA. The Court is satisfied that these measures comply with the
Constitution, viz.: constitutional description of a "special fund." Indeed, the practice is not without precedent.

(2) The Congress may, by law, authorize the President to fix, within specified limits, With regard to the alleged undue delegation of legislative power, the Court finds that the provision
and subject to such limitations and restrictions as it may impose, tariff rates, import conferring the authority upon the ERB to impose additional amounts on petroleum products provides a
and export quotas, tonnage and wharfage dues, and other duties or imposts within sufficient standard by which the authority must be exercised. In addition to the general policy of the law
the framework of the national development program of the Government; to protect the local consumer by stabilizing and subsidizing domestic pump rates, § 8(c) of P.D. 1956
expressly authorizes the ERB to impose additional amounts to augment the resources of the Fund.
and, inasmuch as the delegation relates to the exercise of the power of taxation, "the limits, limitations
and restrictions must be quantitative, that is, the law must not only specify how to tax, who (shall) be What petitioner would wish is the fixing of some definite, quantitative restriction, or "a specific limit on
taxed (and) what the tax is for, but also impose a specific limit on how much to tax." 12 how much to tax." The Court is cited to this requirement by the petitioner on the premise that what is
involved here is the power of taxation; but as already discussed, this is not the case. What is here
Issue: involved is not so much the power of taxation as police power. Although the provision authorizing the
ERB to impose additional amounts could be construed to refer to the power of taxation, it cannot be
Whether or Not the invalidity of the "TRUST ACCOUNT" in the books of account of the Ministry of overlooked that the overriding consideration is to enable the delegate to act with expediency in carrying
Energy (now, the Office of Energy Affairs), created pursuant to § 8, paragraph 1, of P.D. No. 1956, as out the objectives of the law which are embraced by the police power of the State.
amended, "said creation of a trust fund being contrary to Section 29 (3), Article VI of the Constitution.
The interplay and constant fluctuation of the various factors involved in the determination of the price of
Whether or Not the unconstitutionality of 8, paragraph 1 (c) of P.D. No. 1956, as amended by Executive oil and petroleum products, and the frequently shifting need to either augment or exhaust the Fund, do
Order No. 137, for "being an undue and invalid delegation of legislative power to the Energy Regulatory not conveniently permit the setting of fixed or rigid parameters in the law as proposed by the petitioner.
Board. To do so would render the ERB unable to respond effectively so as to mitigate or avoid the undesirable
consequences of such fluidity. As such, the standard as it is expressed suffices to guide the delegate in
the exercise of the delegated power, taking account of the circumstances under which it is to be
exercised.
Held:

The OPSF is a "Trust Account" which was established "for the purpose of minimizing the frequent price
changes brought about by exchange rate adjustment and/or changes in world market prices of crude oil
and imported petroleum products." Under P.D. No. 1956, as amended by Executive Order No. 137 LLADOC VS. COMMISSIONER OF INTERNAL REVENUE
dated 27 February 1987, this Trust Account may be funded from any of the following sources: [14 SCRA 292; NO.L-19201; 16 JUN 1965]

a) Any increase in the tax collection from ad valorem tax or customs duty imposed on Facts:
petroleum products subject to tax under this Decree arising from exchange rate adjustment,
as may be determined by the Minister of Finance in consultation with the Board of Energy; Sometime in 1957, M.B. Estate Inc., of Bacolod City, donated 10,000.00 pesos in cash to Fr. Crispin
Ruiz, the parish priest of Victorias, Negros Occidental, and predecessor of Fr. Lladoc, for the
b) Any increase in the tax collection as a result of the lifting of tax exemptions of government construction of a new Catholic church in the locality. The donated amount was spent for such purpose.
corporations, as may be determined by the Minister of Finance in consultation with the Board
of Energy; On March 3, 1958, the donor M.B. Estate filed the donor's gift tax return. Under date of April 29, 1960.
Commissioner of Internal Revenue issued an assessment for the donee's gift tax against the Catholic
c) Any additional amount to be imposed on petroleum products to augment the resources of Parish of Victorias of which petitioner was the parish priest.
the Fund through an appropriate Order that may be issued by the Board of Energy requiring
payment of persons or companies engaged in the business of importing, manufacturing Issue:
and/or marketing petroleum products;

Page 28
Whether or not the imposition of gift tax despite the fact the Fr. Lladoc was not the Parish priest at the The deed constituted a contract between the Spanish Government and the plaintiff. The obligation of
time of donation, Catholic Parish priest of Victorias did not have juridical personality as the which contract was impaired by the enactment of sec. 134 of the Internal Revenue Law infringing sec. 5
constitutional exemption for religious purpose is valid. of the Act of Congress which provides that “no law impairing the obligation of contracts shall be
enacted”. Sec. 134 of the Internal Revenue Law of 1904 is void because it impairs the obligation of
Held: contracts contained in the concessions of mine made by the Spanish Government. Judgment reversed.

Yes, imposition of the gift tax was valid, under Section 22(3) Article VI of the Constitution contemplates
exemption only from payment of taxes assessed on such properties as Property taxes contra
distinguished from Excise taxes The imposition of the gift tax on the property used for religious purpose
is not a violation of the Constitution. A gift tax is not a property by way of gift inter vivos.

The head of the Diocese and not the parish priest is the real party in interest in the imposition of the
donee's tax on the property donated to the church for religious purpose.

CASSANOVAS VS. HORD


[8 Phil 125; No. 3473; 22 Mar 1907]

Facts:

The Spanish Govt. by virtue of a royal decree granted the plaintiff certain mines. The plaintiff is now the
owner of those mines. The Collector of Internal Revenue imposed tax on the properties, contending
that they were valid perfected mine concessions and it falls within the provisions of sec.134 of Act No.
1189 known as Internal Revenue Act. The plaintiff paid under protest. He brought an action against the
defendant Collector of Internal Revenue to recover the sum of Php. 9, 600 paid by him as taxes.
Judgment was rendered in favor of the defendant, so the plaintiff appealed.

Issue:

Whether or Not Sec. 164 is void or valid.

Held:

Page 29
THE BILL
OF RIGHTS

Page 30
DUE PROCESS OF LAW It admits of no doubt therefore that there being a presumption of validity, the necessity for evidence to
rebut it is unavoidable, unless the statute or ordinance is void on its face which is not the case here.
The principle has been nowhere better expressed than in the leading case of O'Gorman & Young v.
Art 3, Sec. 1. “No person shall be deprived of life, liberty, or property without due process of Hartford Fire Insurance Co. where the American Supreme Court through Justice Brandeis tersely and
law…” succinctly summed up the matter thus: The statute here questioned deals with a subject clearly within
the scope of the police power. We are asked to declare it void on the ground that the specific method of
regulation prescribed is unreasonable and hence deprives the plaintiff of due process of law. As
ERMITA-MALATE HOTEL AND MOTEL OPERATORS ASSO. VS. MAYOR OF MANILA underlying questions of fact may condition the constitutionality of legislation of this character, the
[20 SCRA 849; G.R. NO.L-24693; 31 JULY 1967] resumption of constitutionality must prevail in the absence of some factual foundation of record for
overthrowing the statute." No such factual foundation being laid in the present case, the lower court
Facts: deciding the matter on the pleadings and the stipulation of facts, the presumption of validity must
prevail and the judgment against the ordinance set aside.
Petitioners Ermita-Malate Hotel and Motel Operators Association with one of its members, Hotel del
Mar Inc., and Go Chiu, the president and general manager of the second petitioner, filed a petition for
prohibition against Ordinance No. 4760 against the respondent Mayor of the City of Manila who was VILLEGAS VS. HIU CHIONG
sued in his capacity as such charged with the general power and duty to enforce ordinances of the City [86 SCRA 270; NO.L-29646; 10 NOV 1978]
of Manila and to give the necessary orders for the execution and enforcement of such ordinances. It
was alleged that the petitioner non-stock corporation is dedicated to the promotion and protection of the Facts:
interest of its eighteen members operating hotels and motels, characterized as legitimate businesses
duly licensed by both national and city authorities and regularly paying taxes. It was alleged that on The controverted Ordinance no. 6537 was passed by the Municipal Board of Manila on February 22,
June 13, 1963, the Municipal Board of the City of Manila enacted Ordinance No. 4760, approved on 1968 and signed by Mayor Villegas. It is an ordinance making it unlawful for any person not a citizen of
June 14, 1963 by the then acting City Mayor, Vice-Mayor Herminio Astorga. After which the alleged the Philippines to be employed in any place of employment or to be engaged in any kind of trade
grievances against the ordinance were set forth in detail. There was the assertion of its being beyond business or occupation within the city of Manila without securing an employment permit from the Mayor
the powers of the Municipal Board of the City of Manila to enact insofar as it regulate motels, on the of Manila and for other purposes.
ground that in the revised charter of the City of Manila or in any other law, no reference is made to
motels. it also being provided that the premises and facilities of such hotels, motels and lodging houses Hiu Chiong Tsai Pao Ho, who was employed in Manila filed a petition praying for the writ of preliminary
would be open for inspection either by the City Mayor, or the Chief of Police, or their duly authorized injunction and restraining order to stop the enforcement of said ordinance.
representatives. The lower court on July 6, 1963 issued a writ of preliminary injunction ordering
respondent Mayor to refrain from enforcing said Ordinance No. 4760 from and after July 8, 1963. Issue:

Issue: Whether or Not Ordinance no.6537 violates the due process and equal protection clauses of the
Constitution.
Whether or Not Ordinance No. 4760 of the City of Manila is unconstitutional, therefore, null and void.
Held:
Held:
It is a revenue measure. The city ordinance which imposes a fee of 50.00 pesos to enable aliens
A decent regard for constitutional doctrines of a fundamental character ought to have admonished the generally to be employed in the city of Manila is not only for the purpose of regulation.
lower court against such a sweeping condemnation of the challenged ordinance. Its decision cannot be
allowed to stand, consistently with what has been the accepted standards of constitutional adjudication, While it is true that the first part which requires the alien to secure an employment permit from
in both procedural and substantive aspects. the Mayor involves the exercise of discretion and judgment in processing and approval or
disapproval of application is regulatory in character, the second part which requires the payment of
Primarily what calls for a reversal of such a decision is the absence of any evidence to offset the a sum of 50.00 pesos is not a regulatory but a revenue measure.
presumption of validity that attaches to a challenged statute or ordinance. As was expressed
categorically by Justice Malcolm: "The presumption is all in favor of validity x x x . The action of the Ordinance no. 6537 is void and unconstitutional. This is tantamount to denial of the basic human right
elected representatives of the people cannot be lightly set aside. The councilors must, in the very of the people in the Philippines to engaged in a means of livelihood. While it is true that the Philippines
nature of things, be familiar with the necessities of their particular municipality and with all the facts and as a state is not obliged to admit aliens within it's territory, once an alien is admitted he cannot be
circumstances which surround the subject and necessitate action. The local legislative body, by deprived of life without due process of law. This guarantee includes the means of livelihood. Also it
enacting the ordinance, has in effect given notice that the regulations are essential to the well being of does not lay down any standard to guide the City Mayor in the issuance or denial of an alien
the people x x x . The Judiciary should not lightly set aside legislative action when there is not a clear employment permit fee.
invasion of personal or property rights under the guise of police regulation.

NAMIL VS. COMELEC


Page 31
[414 SCRA 553; G.R. NO. 150540; 28 OCT 2003] alien dominance and control. If ever the law infringes upon the said treaty, the latter is always subject to
qualification or amendment by a subsequent law and the same may never curtain or restrict the scope
Facts: of the police power of the state.

On May 20, 2001, the Municipal Board of Canvassers of Palimbang, Sultan Kudarat proclaimed the
petitioners as winning candidates for their Sangguniang Bayan. The following day, herein private PHIL. PHOSPHATE FERTILIZER CORP. VS. TORRES
respondents were proclaimed winners as well. Private respondents claimed that they should be [231 SCRA 335; G.R. NO.98050; 17 MAR 1994]
recognized as the winners, and not the petitioners. Upon receipt of such letter, the Commissioner-in-
charge for Region XII asked the Law Department, the Regional Election Registrar and the Provincial Facts:
Elections Supervisor to submit their reports on the matter. All of them found the second proclamation
valid. Hence, the COMELEC issued a Resolution ordering the immediate installation of the private Philphos Movement for Progress, Inc. (PMPI for brevity), filed with the Department of Labor and
respondents as the newly elected members of the Sangguniang Bayan, even though petitioners herein Employment a petition for certification election among the supervisory employees of petitioner, alleging
have already taken their oath and have assumed office. Petitioners contend that such Resolution is that as a supervisory union duly registered with the Department of Labor and Employment it was
null and void because they were not accorded due notice and hearing, hence constituting a violation of seeking to represent the supervisory employees of Philippine Phosphate Fertilizer Corporation.
the due process principle. Mediator-Arbiter Rodolfo S. Milado issued an order directing the holding of a certification election
among the supervisory employees of petitioner, excluding therefrom the superintendents and the
Issue: professional and technical employees. However, the PMPI filed an amended petition with the Mediator-
Arbiter wherein it sought to represent not only the supervisory employees of petitioner but also its
Whether or Not due the COMELEC has the power to suspend a proclamation or the effects thereof professional/technical and confidential employees. The parties therein agreed to submit their respective
without notice and hearing. position papers and to consider the amended petition submitted for decision on the basis thereof and
related documents. Mediator-Arbiter Milado issued an order granting the petition and directing the
Held: holding of a certification election among the "supervisory, professional (engineers, analysts, mechanics,
accountants, nurses, midwives, etc.), technical, and confidential employees. PHILPHOS appealed the
No. The COMELEC is without power to partially or totally annul a proclamation or suspend the effects order to the Secretary of Labor and Employment who rendered a decision through Undersecretary
of a proclamation without notice and hearing. The proclamation on May 20, 2001 enjoys the Bienvenido Laguesma dismissing the appeal. PHILPHOS moved for reconsideration but the same was
presumption of regularity and validity since no contest or protest was even filed assailing the same. denied; hence, the instant petition alleging denial of due process on the part of the DOLE to which the
The petitioners cannot be removed from office without due process of law. Due process in quasi- mediator-arbiter was under.
judicial proceedings before the COMELEC requires due notice and hearing. Furthermore, the
proclamation of a winning candidate cannot be annulled if he has not been notified of any motion to set Issue:
aside his proclamation. Hence, as ruled in Fariñas vs. COMELEC, Reyes vs. COMELEC and Gallardo
vs. COMELEC, the COMELEC is without power to partially or totally annul a proclamation or suspend Whether or Not there was denial of due process.
the effects of a proclamation without notice and hearing.
ICHONG VS. HERNANDEZ Held:
[101 PHIL 1155; L-7995; 31 MAY 1957]
There was no denial of due process. The essence of due process is simply an opportunity to be heard
Facts: or, as applied to administrative proceedings, an opportunity to explain one's side or an opportunity to
seek a reconsideration of the action or ruling complained of petitioner PHILPHOS agreed to file its
Republic Act 1180 or commonly known as “An Act to Regulate the Retail Business” was passed. The position paper with the Mediator-Arbiter and to consider the case submitted for decision on the basis of
said law provides for a prohibition against foreigners as well as corporations owned by foreigners from the position papers filed by the parties, there was sufficient compliance with the requirement of due
engaging from retail trade in our country. This was protested by the petitioner in this case. According to process, as petitioner was afforded reasonable opportunity to present its side. Moreover, petitioner
him, the said law violates the international and treaty of the Philippines therefore it is unconstitutional. could have, if it so desired, insisted on a hearing to confront and examine the witnesses of the other
Specifically, the Treaty of Amity between the Philippines and China was violated according to him. party. But it did not; instead it opted to submit its position paper with the Mediator-Arbiter. Besides,
petitioner had all the opportunity to ventilate its arguments in its appeal to the Secretary of Labor.
Issue:

Whether or Not Republic Act 1180 is a valid exercise of police power. RUBI VS. PROVINCIAL BOARD OF MINDORO
[39 PHIL 660; G.R. NO. 14078; 7 MAR 1919]
Held:
Facts:
According to the Court, RA 1180 is a valid exercise of police power. It was also then provided that
This is an application for habeas corpus in favor of Rubi and other Manguianes of the Province of
police power can not be bargained away through the medium of a treaty or a contract. The Court also
Mindoro.
provided that RA 1180 was enacted to remedy a real and actual danger to national economy posed by
Page 32
public forests in which they roam; (5) the necessity of introducing civilized customs among the
The provincial board of Mindoro adopted resolution No. 25 which states that “provincial governor of any Manguianes.
province in which non-Christian inhabitants (uncivilized tribes) are found is authorized, when such a
course is deemed necessary in the interest of law and order, to direct such inhabitants to take up their Considered purely as an exercise of the police power, the courts cannot fairly say that the Legislature
habitation on sites on unoccupied public lands to be selected by him and approved by the provincial has exceeded its rightful authority. It is, indeed, an unusual exercise of that power. But a great malady
board”. It is resolved that under section 2077 of the Administrative Code, 800 hectares of public land in requires an equally drastic remedy. One cannot hold that the liberty of the citizen is unduly interfered
the sitio of Tigbao on Naujan Lake be selected as a site for the permanent settlement of Mangyanes in without when the degree of civilization of the Manguianes is considered. They are restrained for their
Mindoro. Further, Mangyans may only solicit homesteads on this reservation providing that said own good and the general good of the Philippines. Nor can one say that due process of law has not
homestead applications are previously recommended by the provincial governor. been followed.

Thereafter, the provincial governor of Mindoro issued executive order No. 2, which says that the None of the rights of the citizen can be taken away except by due process of law. To constitute "due
provincial governor has selected a site in the sitio of Tigbao on Naujan Lake for the permanent process of law," as has been often held, a judicial proceeding is not always necessary. In some
settlement of Mangyanes in Mindoro. In that case, pursuant to Section 2145 of the Revised instances, even a hearing and notice are not requisite a rule which is especially true where much must
Administrative Code, all the Mangyans in the townships of Naujan and Pola and the Mangyans east of be left to the discretion of the administrative officers in applying a law to particular cases.
the Baco River including those in the districts of Dulangan and Rubi's place in Calapan, were ordered to
take up their habitation on the site of Tigbao, Naujan Lake. Also, that any Mangyan who shall refuse to The idea of the provision in question is to unify the people of the Philippines so that they may approach
comply with this order shall upon conviction be imprisoned not exceed in sixty days, in accordance with the highest conception of nationality. The public policy of the Government of the Philippine Islands is
section 2759 of the revised Administrative Code. shaped with a view to benefit the Filipino people as a whole. The Manguianes, in order to fulfill this
governmental policy, must be confined for a time, as we have said, for their own good and the good of
Said resolution of the provincial board of Mindoro were claimed as necessary measures for the the country.
protection of the Mangyanes of Mindoro as well as the protection of public forests in which they roam,
and to introduce civilized customs among them. Therefore, petitioners are not unlawfully imprisoned or restrained of their liberty. Habeas corpus can,
therefore, not issue.
It appeared that Rubi and those living in his rancheria have not fixed their dwelling within the
reservation of Tigbao and are liable to be punished.
KWONG SING VS. CITY OF MANILA
It is alleged that the Manguianes are being illegally deprived of their liberty by the provincial officials of [41 PHIL 103; G.R. NO. 15972; 11 OCT 1920]
that province. Rubi and his companions are said to be held on the reservation established at Tigbao,
Mindoro, against their will, and one Dabalos is said to be held under the custody of the provincial sheriff Facts:
in the prison at Calapan for having run away form the reservation.
Kwong Sing, in his own behalf and of other Chinese laundrymen who has general and the same
Issue: interest, filed a complaint for a preliminary injunction. The Plaintiffs also questioned the validity of
enforcing Ordinance No. 532 by the city of Manila. Ordinance No. 532 requires that the receipt be in
Whether or Not Section 2145 of the Administrative Code deprive a person of his liberty without due duplicate in English and Spanish duly signed showing the kind and number of articles delivered by
process of law. laundries and dyeing and cleaning establishments. The permanent injunction was denied by the trial
court. The appellants claim is that Ordinance No. 532 savors of class legislation; putting in mind that
Whether or Not Section 2145 of the Administrative Code of 1917 is constitutional. they are Chinese nationals. It unjustly discriminates between persons in similar circumstances; and that
it constitutes an arbitrary infringement of property rights. They also contest that the enforcement of the
Held: legislation is an act beyond the scope of their police power. In view of the foregoing, this is an appeal
with the Supreme Court.
The Court held that section 2145 of the Administrative Code does not deprive a person of his liberty
without due process of law and does not deny to him the equal protection of the laws, and that Issue:
confinement in reservations in accordance with said section does not constitute slavery and involuntary
servitude. The Court is further of the opinion that section 2145 of the Administrative Code is a legitimate Whether or Not the enforcement of Ordinance no, 532 is an act beyond the scope of police power
exertion of the police power, somewhat analogous to the Indian policy of the United States. Section
2145 of the Administrative Code of 1917 is constitutional. Whether or not the enforcement of the same is a class legislation that infringes property rights.

The preamble of the resolution of the provincial board of Mindoro which set apart the Tigbao Held:
reservation, it will be read, assigned as reasons fort the action, the following: (1) The failure of former
attempts for the advancement of the non-Christian people of the province; and (2) the only successfully Reasonable restraints of a lawful business for such purposes are permissible under the police power.
method for educating the Manguianes was to oblige them to live in a permanent settlement. The The police power of the City of Manila to enact Ordinance No. 532 is based on Section 2444,
Solicitor-General adds the following; (3) The protection of the Manguianes; (4) the protection of the
Page 33
paragraphs (l) and (ee) of the Administrative Code, as amended by Act No. 2744, authorizes the Held:
municipal board of the city of Manila, with the approval of the mayor of the city:
Yes. The Philippine government may make every reasonable requirement of its taxpayers to keep
(l) To regulate and fix the amount of the license fees for the following: xxxx xxxxxlaundries proper records of their business transactions in English or Spanish or Filipino dialect by which an
xxxx. adequate measure of what is due from them in meeting the cost of government can be had. But we are
(ee) To enact all ordinances it may deem necessary and proper for the sanitation and safety, clearly of opinion that it is not within the police power of the Philippine Legislature, because it would be
the furtherance of the prosperity, and the promotion of the morality, peace, good order, oppressive and arbitrary, to prohibit all Chinese merchants from maintaining a set of books in the
comfort, convenience, and general welfare of the city and its inhabitants. Chinese language, and in the Chinese characters, and thus prevent them from keeping advised of the
status of their business and directing its conduct.
The court held that the obvious purpose of Ordinance No. 532 was to avoid disputes between
laundrymen and their patrons and to protect customers of laundries who are not able to decipher
Chinese characters from being defrauded. (Considering that in the year 1920s, people of Manila are ANIAG VS. COMELEC
more familiar with Spanish and maybe English.) [237 SCRA 194; G.R. NO. 104961; 7 OCT 1994]

In whether the ordinance is class legislation, the court held that the ordinance invades no fundamental Facts:
right, and impairs no personal privilege. Under the guise of police regulation, an attempt is not made to
violate personal property rights. The ordinance is neither discriminatory nor unreasonable in its In preparation for the synchronized national and local elections, the COMELEC issued Resolution No.
operation. It applies to all public laundries without distinction, whether they belong to Americans, 2323, “Gun Ban”, promulgating rules and regulations on bearing, carrying and transporting of firearm or
Filipinos, Chinese, or any other nationality. All, without exception, and each every one of them without other deadly weapons on security personnel or bodyguards, on bearing arms by members of security
distinction, must comply with the ordinance. The obvious objection for the implementation of the agencies or police organizations, and organization or maintenance of reaction forces during the election
ordinance is based in sec2444 (ee) of the Administrative Code. Although, an additional burden will be period. COMELEC also issued Resolution No. 2327 providing for the summary disqualification of
imposed on the business and occupation affected by the ordinance such as that of the appellant by candidates engaged in gunrunning, using and transporting of firearms, organizing special strike forces,
learning even a few words in Spanish or English, but mostly Arabic numbers in order to properly issue a and establishing spot checkpoints. Pursuant to the “Gun Ban”, Mr. Serrapio Taccad, Sergeant at Arms
receipt, it seems that the same burdens are cast upon the them. Yet, even if private rights of person or of the House of Representatives, wrote petitioner for the return of the two firearms issued to him by the
property are subjected to restraint, and even if loss will result to individuals from the enforcement of the House of Representatives. Petitioner then instructed his driver, Arellano, to pick up the firearms from
ordinance, this is not sufficient ground for failing to uphold the power of the legislative body. The very petitioner’s house and return them to Congress. The PNP set up a checkpoint. When the car driven by
foundation of the police power is the control of private interests for the public welfare. Arellano approached the checkpoint, the PNP searched the car and found the firearms. Arellano was
apprehended and detained. He then explained the order of petitioner. Petitioner also explained that
Finding that the ordinance is valid, judgment is affirmed, and the petition for a preliminary injunction is Arellano was only complying with the firearms ban, and that he was not a security officer or a
denied, with costs against the appellants. bodyguard. Later, COMELEC issued Resolution No.92-0829 directing the filing of information against
petitioner and Arellano for violation of the Omnibus Election Code, and for petitioner to show cause why
he should not be disqualified from running for an elective position. Petitioner then questions the
YU CONG ENG VS. TRINIDAD constitutionality of Resolution No. 2327. He argues that “gunrunning, using or transporting firearms or
[47 PHIL 385; G.R. NO. 20479; 6 FEB 1925] similar weapons” and other acts mentioned in the resolution are not within the provisions of the
Omnibus Election Code. Thus, according to petitioner, Resolution No. 2327 is unconstitutional. The
Facts: issue on the disqualification of petitioner from running in the elections was rendered moot when he lost
his bid for a seat in Congress in the elections.
The petitioner, Yu Cong Eng, was charged by information in the court of first instance of Manila, with a
violation of Act 2972, which provides that (Section 1) it shall be unlawful for any person, company, or Issue:
partnership or corporation engaged in commerce, industry or any other activity for the purpose of profit
in the Philippine Islands, in accordance with existing law, to keep its account books in any language Whether or Not petitioner can be validly prosecuted for instructing his driver to return the firearms
other than English, Spanish or any local dialect. He was arrested, his books were seized, and the trial issued to him on the basis of the evidence gathered from the warrant less search of his car
was about to proceed, when he and the other petitioner, Co Liam, on their own behalf, and on behalf of
all the other Chinese merchants in the Philippines, filed the petition against the fiscal, or prosecuting
attorney of Manila, and the collector of internal revenue engaged in the prosecution, and against the
judge presiding.
Held:
Issue:
A valid search must be authorized by a search warrant issued by an appropriate authority. However, a
Whether or Not Act 2972 is unconstitutional. warrantless search is not violative of the Constitution for as long as the vehicle is neither searched nor
its occupants subjected to a body search, and the inspection of the vehicle is merely limited to a visual
search. In the case at bar, the guns were not tucked in Arellano’s waist nor placed within his reach, as
Page 34
they were neatly packed in gun cases and placed inside a bag at the back of the car. Given these Due process is intended to insure that confidence by requiring compliance with what Justice Frankfurter
circumstances, the PNP could not have thoroughly searched the car lawfully as well as the package calls the rudiments of fair play. Fair play cans for equal justice. There cannot be equal justice where a
without violating the constitutional injunction. Absent any justifying circumstance specifically pointing to suitor approaches a court already committed to the other party and with a judgment already made and
the culpability of petitioner and Arellano, the search could not have been valid. Consequently, the waiting only to be formalized after the litigants shall have undergone the charade of a formal hearing.
firearms obtained from the warrantless search cannot be admitted for any purpose in any proceeding. It Judicial (and also extra-judicial) proceedings are not orchestrated plays in which the parties are
was also shown in the facts that the PNP had not informed the public of the purpose of setting up the supposed to make the motions and reach the denouement according to a prepared script. There is no
checkpoint. Petitioner was also not among those charged by the PNP with violation of the Omnibus writer to foreordain the ending. The judge will reach his conclusions only after all the evidence is in and
Election Code. He was not informed by the City Prosecutor that he was a respondent in the preliminary all the arguments are filed, on the basis of the established facts and the pertinent law.
investigation. Such constituted a violation of his right to due process. Hence, it cannot be contended
that petitioner was fully given the opportunity to meet the accusation against him as he was not
informed that he was himself a respondent in the case. Thus, the warrantless search conducted by the YNOT VS. IAC
PNP is declared illegal and the firearms seized during the search cannot be used as evidence in any [148 SCRA 659; G.R. NO. 74457; 20 MAR 1987]
proceeding against the petitioner. Resolution No. 92-0829 is unconstitutional, and therefore, set aside.
Facts:

JAVIER VS. COMELEC Executive Order No. 626-A prohibited the transportation of carabaos and carabeef from one province to
[144 SCRA 194; G.R. NOS. L-68379-81; 22 SEPT 1986] another. The carabaos of petitioner were confiscated for violation of Executive Order No 626-A while he
was transporting them from Masbate to Iloilo. Petitioner challenged the constitutionality of Executive
Facts: Order No. 626-A. The government argued that Executive Order No. 626-A was issued in the exercise of
police power to conserve the carabaos that were still fit for farm work or breeding.
The petitioner and the private respondent were candidates in Antique for the Batasang Pambansa in
the May 1984 elections. The former appeared to enjoy more popular support but the latter had the Issue:
advantage of being the nominee of the KBL with all its perquisites of power. On May 13, 1984, the eve
of the elections, the bitter contest between the two came to a head when several followers of the Whether or Not EO No. 626-A is a violation of Substantive Due Process.
petitioner were ambushed and killed, allegedly by the latter's men. Seven suspects, including
respondent Pacificador, are now facing trial for these murders. Owing to what he claimed were attempts Held:
to railroad the private respondent's proclamation, the petitioner went to the Commission on Elections to
question the canvass of the election returns. His complaints were dismissed and the private respondent The challenged measure is an invalid exercise of police power, because it is not reasonably necessary
was proclaimed winner by the Second Division of the said body. The petitioner thereupon came to this for the purpose of the law and is unduly oppressive. It is difficult to see how prohibiting the transfer of
Court, arguing that the proclamation was void because made only by a division and not by the carabaos from one province to another can prevent their indiscriminate killing. Retaining the carabaos
Commission on Elections en banc as required by the Constitution. Meanwhile, on the strength of his in one province will not prevent their slaughter there. Prohibiting the transfer of carabeef, after the
proclamation, the private respondent took his oath as a member of the Batasang Pambansa. slaughter of the carabaos, will not prevent the slaughter either.

PHILCOMSAT VS. ALCUAZ


Issue: [180 SCRA 218; G.R. NO.84818; 18 DEC 1989]

Whether or Not the Second Division of the Commission on Elections authorized to promulgate its Facts:
decision of July 23, 1984, proclaiming the private respondent the winner in the election.
Herein petitioner is engaged in providing for services involving telecommunications. Charging rates for
Held:
certain specified lines that were reduced by order of herein respondent Jose Alcuaz Commissioner of
the National Telecommunications Commission. The rates were ordered to be reduced by fifteen percent
This Court has repeatedly and consistently demanded "the cold neutrality of an impartial judge" as the
(15%) due to Executive Order No. 546 which granted the NTC the power to fix rates. Said order was
indispensable imperative of due process. To bolster that requirement, we have held that the judge
issued without prior notice and hearing.
must not only be impartial but must also appear to be impartial as an added assurance to the parties
that his decision will be just. The litigants are entitled to no less than that. They should be sure that
Issue:
when their rights are violated they can go to a judge who shall give them justice. They must trust the
judge, otherwise they will not go to him at all. They must believe in his sense of fairness, otherwise they
Whether or Not E.O. 546 is unconstitutional.
will not seek his judgment. Without such confidence, there would be no point in invoking his action for
the justice they expect.
Held:

Page 35
Yes. Respondents admitted that the application of a policy like the fixing of rates as exercised by (5) Decision must be based on the evidence presented at hearing
administrative bodies is quasi-judicial rather than quasi-legislative. But respondent’s contention that (6) The tribunal body must act on its own independent consideration of law and
notice and hearing are not required since the assailed order is merely incidental to the entire facts and not simply accept subordinate’s views
proceedings and temporary in nature is erroneous. Section 16(c) of the Public Service Act, providing for (7) Court must render decision in such a manner that the proceeding can know
the proceedings of the Commission, upon notice and hearing, dictates that a Commission has power to the various issued involved and reasons for decisions rendered.
fix rates, upon proper notice and hearing, and, if not subject to the exceptions, limitations or saving
provisions. The court stresses that while there is no controlling and precise definition of Due Process, it gives an
unavoidable standard that government actions must conform in order that deprivation of life, liberty and
It is thus clear that with regard to rate-fixing, respondent has no authority to make such order without property is valid.
first giving petitioner a hearing, whether the order be temporary or permanent, and it is immaterial
whether the same is made upon a complaint, a summary investigation, or upon the commission's own The closure of the radio station is like wise a violation of the constitutional right of freedom of speech
motion as in the present case. and expression. The court stresses that all forms of media, whether print or broadcast are entitled to
this constitutional right. Although the government still has the right to be protected against broadcasts
WHEREFORE, the writ prayed for is GRANTED and the order of respondents is hereby SET ASIDE. which incite the listeners to violently overthrow it. The test for the limitation of freedom of expression is
the “clear and present danger” rule. If in the circumstances that the media is used in such nature as to
create this danger that will bring in such evils, then the law has the right to prevent it. However, Radio
EASTERN BROADCASTING CORP (DYRE) V. DANS JR. and television may not be used to organize a rebellion or signal a start of widespread uprising. The
[137 SCRA 628; L-59329; 19 JUL 1985] freedom to comment on public affairs is essential to the vitality of a representative democracy. The
people continues to have the right to be informed on public affairs and broadcast media continues to
Facts: have the pervasive influence to the people being the most accessible form of media. Therefore,
broadcast stations deserve the the special protection given to all forms of media by the due process
A petition was filed to reopen the Radio Station DYRE. DYRE was “summarily closed” on grounds of and freedom of expression clauses of the Constitution.
national security. The radio station was allegedly used to incite people to sedition. Petitioner, DYRE
contends that they were denied due process. There was no hearing to establish factual evidence for
the closure. Furthermore, the closure of the radio station violates freedom of expression. Before the ANG TIBAY VS. COURT OF INDUSTRIAL RELATIONS (CIR)
court could even promulgate a decision upon the Issue raised, Petitioner, through its president Mr. [69 PHIL 635; G.R. NO. 46496; 27 FEB 1940]
Rene Espina, filed a motion to withdraw the petition. The rights of the station were sold to a new
owner, Manuel Pastrana; who is no longer interested in pursuing the case. Despite the case becoming Facts:
moot and academic, (because there are no longer interested parties, thus the dismissal of the case) the
Supreme Court still finds that there is need to pass a “RESOLUTION” for the guidance of inferior courts There was agreement between Ang Tibay and the National Labor Union, Inc (NLU). The NLU alleged
and administrative tribunals in matters as this case. that the supposed lack of leather material claimed by Toribio Teodoro was but a scheme adopted to
systematically discharge all the members of the NLU, from work. And this averment is desired to be
Issue: proved by the petitioner with the records of the Bureau of Customs and Books of Accounts of native
dealers in leather. That National Worker's Brotherhood Union of Ang Tibay is a company or employer
Whether or not due process was exercised in the case of DYRE. union dominated by Toribio Teodoro, which was alleged by the NLU as an illegal one. The CIR, decided
the case and elevated it to the Supreme Court, but a motion for new trial was raised by the NLU. But
Whether or not the closure of DYRE is a violation of the Constitutional Right of Freedom of Expression. the Ang Tibay filed a motion for opposing the said motion.

Held: Issue:

The court finds that the closure of the Radio Station in 1980 as null and void. The absence of a hearing Whether or Not, the motion for new trial is meritorious to be granted.
is a violation of Constitutional Rights. The primary requirements in administrative proceedings are laid
down in the case of Ang Tibay v. Court of Industrial Relation (69 Phil.635). The Ang Tibay Doctrine Held:
should be followed before any broadcast station may be closed. The Ang Tibay Doctrine provides the
following requirements: To begin with the issue before us is to realize the functions of the CIR. The CIR is a special court
whose functions are specifically stated in the law of its creation which is the Commonwealth Act No.
(1) The right to hearing, includes the right to present one’s case and submit 103). It is more an administrative board than a part of the integrated judicial system of the nation. It is
evidence presented. not intended to be a mere receptive organ of the government. Unlike a court of justice which is
(2) The tribunal must consider the evidence presented essentially passive, acting only when its jurisdiction is invoked and deciding only cases that are
(3) The decision must have something to support itself. presented to it by the parties litigant, the function of the CIR, as will appear from perusal of its organic
(4) Evidence must be substantial (reasonable evidence that is adequate to law is more active, affirmative and dynamic. It not only exercises judicial or quasi-judicial functions in
support conclusion) the determination of disputes between employers and employees but its functions are far more
Page 36
comprehensive and extensive. It has jurisdiction over the entire Philippines, to consider, investigate, Investigating Committee, after receiving the written statements and hearing the testimonies of several
decide, and settle any question, matter controversy or disputes arising between, and/ or affecting witness, found a prima facie case against respondent students for violation of Rule 3 of the Law School
employers and employees or laborers, and landlords and tenants or farm-laborers, and regulates the Catalogue entitled "Discipline." Respondent students were then required to file their written answers to
relations between them, subject to, and in accordance with, the provisions of CA 103. the formal charge. Petitioner Dean created a Disciplinary Board to hear the charges against respondent
students. The Board found respondent students guilty of violating Rule No. 3 of the Ateneo Law School
As laid down in the case of Goseco v. CIR, the SC had the occasion to point out that the CIR is not Rules on Discipline which prohibits participation in hazing activities. However, in view of the lack of
narrowly constrained by technical rules of procedure, and equity and substantial merits of the case, unanimity among the members of the Board on the penalty of dismissal, the Board left the imposition of
without regard to technicalities or legal forms and shall not be bound by any technical rules of legal the penalty to the University Administration. Accordingly, Fr. Bernas imposed the penalty of dismissal
evidence but may inform its mind in such manner as it may deem just and equitable. on all respondent students. Respondent students filed with RTC Makati a TRO since they are currently
enrolled. This was granted. A TRO was also issued enjoining petitioners from dismissing the
respondents. A day after the expiration of the temporary restraining order, Dean del Castillo created a
Special Board to investigate the charges of hazing against respondent students Abas and Mendoza.
The fact, however, that the CIR may be said to be free from rigidity of certain procedural requirements This was requested to be stricken out by the respondents and argued that the creation of the Special
does not mean that it can in justiciable cases coming before it, entirely ignore or disregard the Board was totally unrelated to the original petition which alleged lack of due process. This was granted
fundamental and essential requirements of due process in trials and investigations of an administrative and reinstatement of the students was ordered.
character. There cardinal primary rights which must be respected even in proceedings of this character:
Issue:
(1) the right to a hearing, which includes the right to present one's cause and
submit evidence in support thereof; Was there denial of due process against the respondent students.
(2) The tribunal must consider the evidence presented;
(3) The decision must have something to support itself; Held:
(4) The evidence must be substantial;
(5) The decision must be based on the evidence presented at the hearing; or at There was no denial of due process, more particularly procedural due process. Dean of the Ateneo Law
least contained in the record and disclosed to the parties affected; School, notified and required respondent students to submit their written statement on the incident.
(6) The tribunal or body or any of its judges must act on its own independent Instead of filing a reply, respondent students requested through their counsel, copies of the charges.
consideration of the law and facts of the controversy, and not simply accept The nature and cause of the accusation were adequately spelled out in petitioners' notices. Present is
the views of a subordinate; the twin elements of notice and hearing.
(7) The Board or body should, in all controversial questions, render its decision in
such manner that the parties to the proceeding can know the various Issue Respondent students argue that petitioners are not in a position to file the instant petition under Rule 65
involved, and the reason for the decision rendered. considering that they failed to file a motion for reconsideration first before the trial court, thereby by
passing the latter and the Court of Appeals. It is accepted legal doctrine that an exception to the
The failure to grasp the fundamental issue involved is not entirely attributable to the parties adversely doctrine of exhaustion of remedies is when the case involves a question of law, as in this case, where
affected by the result. Accordingly, the motion for a new trial should be, and the same is hereby the issue is whether or not respondent students have been afforded procedural due process prior to
granted, and the entire record of this case shall be remanded to the CIR, with instruction that it reopen their dismissal from Petitioner University.
the case receive all such evidence as may be relevant, and otherwise proceed in accordance with the
requirements set forth. So ordered. Minimum standards to be satisfied in the imposition of disciplinary sanctions in academic institutions,
such as petitioner university herein, thus:

ATENEO DE MANILA UNIVERSITY VS. HON. JUDGE IGNACIO CAPULONG (1) the students must be informed in writing of the nature and cause of any
[222 SCRA 644; G.R. 99327; 27 MAY 1993] accusation against them;
(2) that they shall have the right to answer the charges against them with the
Facts: assistance of counsel, if desired:
(3) they shall be informed of the evidence against them
Leonardo H. Villa, a first year law student of Petitioner University, died of serious physical injuries at (4) they shall have the right to adduce evidence in their own behalf; and
Chinese General Hospital after the initiation rites of Aquila Legis. Bienvenido Marquez was also (5) the evidence must be duly considered by the investigating committee or
hospitalized at the Capitol Medical Center for acute renal failure occasioned by the serious physical official designated by the school authorities to hear and decide the case.
injuries inflicted upon him on the same occasion. Petitioner Dean Cynthia del Castillo created a Joint
Administration-Faculty-Student Investigating Committee which was tasked to investigate and submit a
report within 72 hours on the circumstances surrounding the death of Lennie Villa. Said notice also
required respondent students to submit their written statements within twenty-four (24) hours from US GOVERNMENT VS. JUDGE PURUNGAN
receipt. Although respondent students received a copy of the written notice, they failed to file a reply. In [389 SCRA 623; G.R. NO. 148571, 24 SEPT 2002]
the meantime, they were placed on preventive suspension. The Joint Administration-Faculty-Student
Page 37
Facts:

The United States of America, pursuant to the existing RP-US extradition treaty, requested the
extradition of Mark B. Jimenez. Upon receipt of the request, the secretary of foreign affairs (SFA)
transmitted them to the secretary of justice (SOJ) for appropriate action. In such event, the RTC held
that Jimenez shell be deprived of the right to notice and hearing during the evaluation stage of the
extradition process. Thereafter the US government, through DOJ, filed Petition for Extradition and
Jimenez’s immediate arrest, to avoid flight. Before the RTC could render its decision, Jimenez filed an
"Urgent Manifestation/Ex-Parte Motion," praying that his application for an arrest warrant be set for
hearing, which was granted. During which, the lower court issued its questioned July 3, 2001 Order,
directing the issuance of a warrant for his arrest and fixing bail for his temporary liberty at one million
pesos in cash. After Jimenez had surrendered his passport and posted the required cash bond, he was
granted provisional liberty via the challenged Order dated July 4, 2001. Thus, Petition prays for the
lifting of the bail Order, the cancellation of the bond, and the taking of Jimenez into legal custody.

Issue:

Whether or not Jimenez is entitled to notice and hearing before a warrant for his arrest can be issued.

Whether or not he is entitled to bail and to provisional liberty while the extradition proceedings are
pending.

Held:

By nature, extradition proceedings are not equivalent to a criminal case in which guilt or innocence is
determined. Consequently, an extradition case is not one in which the constitutional rights of the
accused are necessarily available. Having once escaped the jurisdiction of the requesting state, the
reasonable prima facie presumption is that the person would escape again if given the opportunity.
Hence, if the judge is convinced that a prima facie case exists, he immediately Issue a warrant for the
arrest of the potential extraditee and summons him or her to answer and to appear at scheduled
hearings on the petition. Potential extraditees are entitled to the rights to due process and to
fundamental fairness. Due process does not always call for a prior opportunity to be heard. A
subsequent opportunity is sufficient due to the flight risk involved. Indeed, available during the hearings
on the petition and the answer is the full chance to be heard and to enjoy fundamental fairness that is
compatible with the summary nature of extradition.

After being taken into custody, potential extraditees may apply for bail. Since the applicants have a
history of absconding, they have the burden of showing that (a) there is no flight risk and no danger to
the community; and (b) there exist special, humanitarian or compelling circumstances. In extradition
cases, bail is not a matter of right; it is subject to judicial discretion in the context of the peculiar facts of
each case.

Page 38
EQUAL PROTECTION Facts:

Petitioner, Phil association of Service Exporters, Inc., is engaged principally in the recruitment of
Art 3, Sec. 1. “…nor shall any person be denied the equal protection of the laws.” Filipino workers, male and female of overseas employment. It challenges the constitutional validity of
Dept. Order No. 1 (1998) of DOLE entitled “Guidelines Governing the Temporary Suspension of
Deployment of Filipino Domestic and Household Workers.” It claims that such order is a discrimination
PEOPLE VS. CAYAT against males and females. The Order does not apply to all Filipino workers but only to domestic
[68 PHIL 12; G.R. NO. 45987; 5 MAY 1939] helpers and females with similar skills, and that it is in violation of the right to travel, it also being an
invalid exercise of the lawmaking power. Further, PASEI invokes Sec 3 of Art 13 of the Constitution,
Facts: providing for worker participation in policy and decision-making processes affecting their rights and
benefits as may be provided by law. Thereafter the Solicitor General on behalf of DOLE submitting to
“Law prohibits any member of a non-Christian tribe to buy, receive, have in his possession, or drink, the validity of the challenged guidelines involving the police power of the State and informed the court
any intoxicating liquors of any kind.” The law, Act No. 1639, exempts only the so-called native wines or that the respondent have lifted the deployment ban in some states where there exists bilateral
liquors which the members of such tribes have been accustomed to take. agreement with the Philippines and existing mechanism providing for sufficient safeguards to ensure
the welfare and protection of the Filipino workers.
Issue:
Issue:
Whether or Not the law denies equal protection to one prosecuted and sentenced for violation of said
law.
Whether or not there has been a valid classification in the challenged Department Order No. 1.
Held:
Held:
No. It satisfies the requirements of a valid classification, one of which is that the classification under the
SC in dismissing the petition ruled that there has been valid classification, the Filipino female domestics
law must rest on real or substantial distinctions.
working abroad were in a class by themselves, because of the special risk to which their class was
exposed. There is no question that Order No.1 applies only to female contract workers but it does not
The distinction is reasonable. The classification between the members of the non- Christian and the
thereby make an undue discrimination between sexes. It is well settled hat equality before the law
members of the Christian tribes is not based upon accident of birth or parentage but upon the degree of
under the constitution does not import a perfect identity of rights among all men and women. It admits
civilization and culture. The term ‘non-Christian tribes’ refers to a geographical area and more directly to
of classification, provided that:
natives of the Philippines of a low grade civilization usually living in tribal relationship apart from settled
communities. The distinction is reasonable for the Act was intended to meet the peculiar conditions
1. Such classification rests on substantial distinctions
existing in the non- Christian tribes”
2. That they are germane to the purpose of the law
3. They are not confined to existing conditions
The prohibition is germane to the purposes of the law. It is designed to insure peace and order in and
4. They apply equally to al members of the same class
among the non- Christian tribes has often resulted in lawlessness and crime thereby hampering the
efforts of the government to raise their standards of life and civilization. This law is not limited in its
In the case at bar, the classifications made, rest on substantial distinctions.
application to conditions existing at the time of the enactment. It is intended to apply for all times as
long as those conditions exists. The Act applies equally to all members of the class. That it may be
Dept. Order No. 1 does not impair the right to travel. The consequence of the deployment ban has on
unfair in its operation against a certain number of non- Christians by reason of their degree of culture is
the right to travel does not impair the right, as the right to travel is subjects among other things, to the
not an argument against the equality of its operation nor affect the reasonableness of the classification
requirements of “public safety” as may be provided by law. Deployment ban of female domestic helper
thus established.
is a valid exercise of police power. Police power as been defined as the state authority to enact
legislation that may interfere with personal liberty or property in order to promote general welfare.
Neither is there merit in the contention that Department Order No. 1 constitutes an invalid exercise of
legislative power as the labor code vest the DOLE with rule making powers.

DUMLAO VS. COMELEC


[95 SCRA 392; L-52245; 22 JAN 1980]
PASEI VS. DRILON
[163 SCRA 386; L-81958; 30 JUN 1988]
Facts:
Page 39
Petitioner questions the constitutionality of section 4 of Batas Pambansa Blg. 52 as discriminatory and Petitioner claims that it suffered losses running to several million pesos in providing COMELEC Time in
contrary to the equal protection and due process guarantees of the Constitution. connection with the 1992 presidential election and 1995 senatorial election and that it stands to suffer
even more should it be required to do so again this year. Petitioners claim that the primary source of
Section 4 provided that any retired municipal or provincial city official that already received retirement revenue of the radio and television stations is the sale of air time to advertisers and to require these
benefits and is 65 years of age shall not be qualified to run for the same local elective office from which stations to provide free air time is to authorize unjust taking of private property. According to
he has retired. petitioners, in 1992 it lost P22,498,560.00 in providing free air time for one hour each day and, in this
year’s elections, it stands to lost P58,980,850.00 in view of COMELEC’s requirement that it provide at
Issue: least 30 minutes of prime time daily for such.

Whether or Not Sec. 4 of BP.52 is unconstitutional being contrary to the equal protection and due Issue:
process rights.
Whether of not Section 92 of B.P. No. 881 denies radio and television broadcast companies the equal
Held: protection of the laws.

No. The guarantee of equal protection is subject to rational classification based on reasonable and real Whether or not Section 92 of B.P. No. 881 constitutes taking of property without due process of law and
differentiations. In the present case, employees 65 years of age have been classified differently from without just compensation.
younger employees. The former are subject to compulsory retirement while the latter are not.

Retirement is not a reasonable disqualification for elective local officials because there can be retirees Held:
who are even younger and a 65 year old retiree could be as good as a 65 year old official who is not a
retiree. But there is reason to disqualify a 65 year old elective official who is trying to run for office Petitioner’s argument is without merit. All broadcasting, whether radio or by television stations, is
because there is the “need for new blood to assume relevance”. When an official has retired he has licensed by the government. Airwave frequencies have to be allocated as there are more individuals
already declared himself tired and unavailable for the same government work. who want to broadcast that there are frequencies to assign. Radio and television broadcasting
companies, which are given franchises, do not own the airwaves and frequencies through which they
WHEREFORE, the first paragraph of section 4 of Batas pambansa Bilang 52 is hereby declared valid. transmit broadcast signals and images. They are merely given the temporary privilege to use them.
Thus, such exercise of the privilege may reasonably be burdened with the performance by the grantee
of some form of public service. In granting the privilege to operate broadcast stations and supervising
TELECOMMUNICATIONS AND BROADCAST ATTORNEYS OF THE PHILS. VS. COMELEC radio and television stations, the state spends considerable public funds in licensing and supervising
[289 SCRA 337; G.R. NO. 132922; 21 APR 1998] them.

Facts: The argument that the subject law singles out radio and television stations to provide free air time as
against newspapers and magazines which require payment of just compensation for the print space
Petitioner Telecommunications and Broadcast Attorneys of the Philippines, Inc. (TELEBAP) is an they may provide is likewise without merit. Regulation of the broadcast industry requires spending of
organization of lawyers of radio and television broadcasting companies. It was declared to be without public funds which it does not do in the case of print media. To require the broadcast industry to
legal standing to sue in this case as, among other reasons, it was not able to show that it was to suffer provide free air time for COMELEC is a fair exchange for what the industry gets.
from actual or threatened injury as a result of the subject law. Petitioner GMA Network, on the other
hand, had the requisite standing to bring the constitutional challenge. Petitioner operates radio and As radio and television broadcast stations do not own the airwaves, no private property is taken by the
television broadcast stations in the Philippines affected by the enforcement of Section 92, B.P. No. 881. requirement that they provide air time to the COMELEC.

Petitioners challenge the validity of Section 92, B.P. No. 881 which provides:
“Comelec Time- The Commission shall procure radio and television time to be LACSON VS. EXECUTIVE SECRETARY
known as the “Comelec Time” which shall be allocated equally and impartially [301 SCRA 298; G.R. NO. 128096; 20 JAN 1999]
among the candidates within the area of coverage of all radio and television
stations. For this purpose, the franchise of all radio broadcasting and television Facts:
stations are hereby amended so as to provide radio or television time, free of
charge, during the period of campaign.” Eleven persons believed to be members of the Kuratong Baleleng gang, an organized crime syndicate
involved in bank robberies, were slain by elements of the Anti-Bank Robbery and Intelligence Task
Petitioner contends that while Section 90 of the same law requires COMELEC to procure print space in Group (ABRITG). Among those included in the ABRITG were petitioners and petitioner-intervenors.
newspapers and magazines with payment, Section 92 provides that air time shall be procured by
COMELEC free of charge. Thus it contends that Section 92 singles out radio and television stations to Acting on a media expose of SPO2 Eduardo delos Reyes, a member of the Criminal Investigation
provide free air time. Command, that what actually transpired was a summary execution and not a shoot-out between the
Page 40
Kuratong Baleleng gang members and the ABRITG, Ombudsman Aniano Desierto formed a panel of declared by the Court as not a penal law, but clearly a procedural statute, one which prescribes rules of
investigators to investigate the said incident. Said panel found the incident as a legitimate police procedure by which courts applying laws of all kinds can properly administer justice. Not being a penal
operation. However, a review board modified the panel’s finding and recommended the indictment for law, the retroactive application of R.A. 8249 cannot be challenged as unconstitutional.
multiple murder against twenty-six respondents including herein petitioner, charged as principal, and
herein petitioner-intervenors, charged as accessories. After a reinvestigation, the Ombudsman filed In People vs. Montejo, it was held that an offense is said to have been committed in relation to the
amended informations before the Sandiganbayan, where petitioner was charged only as an accessory. office if it is intimately connected with the office of the offender and perpetrated while he was in the
performance of his official functions. Such intimate relation must be alleged in the information which is
The accused filed separate motions questioning the jurisdiction of the Sandiganbayan, asserting that essential in determining the jurisdiction of the Sandiganbayan. However, upon examination of the
under the amended informations, the cases fall within the jurisdiction of the Regional Trial Court amended information, there was no specific allegation of facts that the shooting of the victim by the said
pursuant to Section 2 of R.A. 7975. They contend that the said law limited the jurisdiction of the principal accused was intimately related to the discharge of their official duties as police officers.
Sandiganbayan to cases where one or ore of the “principal accused” are government officals with Likewise, the amended information does not indicate that the said accused arrested and investigated
Salary Grade 27 or higher, or PNP officials with rank of Chief Superintendent or higher. Thus, they did the victim and then killed the latter while in their custody. The stringent requirement that the charge set
not qualify under said requisites. However, pending resolution of their motions, R.A. 8249 was forth with such particularity as will reasonably indicate the exact offense which the accused is alleged to
approved amending the jurisdiction of the Sandiganbayan by deleting the word “principal” from the have committed in relation to his office was not established.
phrase “principal accused” in Section 2 of R.A. 7975.
Consequently, for failure to show in the amended informations that the charge of murder was intimately
Petitioner questions the constitutionality of Section 4 of R.A. 8249, including Section 7 which provides connected with the discharge of official functions of the accused PNP officers, the offense charged in
that the said law shall apply to all cases pending in any court over which trial has not begun as of the the subject criminal cases is plain murder and, therefore, within the exclusive original jurisdiction of the
approval hereof. Regional Trial Court and not the Sandiganbayan.

Issue:
INT'L. SCHOOL ALLIANCE VS. QUISUMBING
Whether or not Sections 4 and 7 of R.A. 8249 violate the petitioners’ right to due process and the equal [333 SCRA 13; G.R. NO. 128845; 1 JUN 2000]
protection clause of the Constitution as the provisions seemed to have been introduced for the
Sandiganbayan to continue to acquire jurisdiction over the Kuratong Baleleng case. Facts:

Whether or not said statute may be considered as an ex-post facto statute.


Receiving salaries less than their counterparts hired abroad, the local-hires of private respondent
School, mostly Filipinos, cry discrimination. We agree. That the local-hires are paid more than their
Whether or not the multiple murder of the alleged members of the Kuratong Baleleng was committed in
colleagues in other schools is, of course, beside the point. The point is that employees should be given
relation to the office of the accused PNP officers which is essential to the determination whether the
equal pay for work of equal value.
case falls within the Sandiganbayan’s or Regional Trial Court’s jurisdiction.
Private respondent International School, Inc. (the School, for short), pursuant to Presidential Decree
Held:
732, is a domestic educational institution established primarily for dependents of foreign diplomatic
personnel and other temporary residents. To enable the School to continue carrying out its educational
Petitioner and intervenors’ posture that Sections 4 and 7 of R.A. 8249 violate their right to equal
program and improve its standard of instruction, Section 2(c) of the same decree authorizes the School
protection of the law is too shallow to deserve merit. No concrete evidence and convincing argument
to
were presented to warrant such a declaration. Every classification made by the law is presumed
employ its own teaching and management personnel selected by it either locally or abroad, from
reasonable and the party who challenges the law must present proof of arbitrariness. The classification
Philippine or other nationalities, such personnel being exempt from otherwise applicable laws and
is reasonable and not arbitrary when the following concur: (1) it must rest on substantial distinction; (2)
regulations attending their employment, except laws that have been or will be enacted for the protection
it must be germane to the purpose of the law; (3) must not be limited to existing conditions only, and (4)
of employees.
must apply equally to all members of the same class; all of which are present in this case.
Accordingly, the School hires both foreign and local teachers as members of its faculty, classifying the
same into two: (1) foreign-hires and (2) local-hires.
Paragraph a of Section 4 provides that it shall apply “to all cases involving” certain public officials and
under the transitory provision in Section 7, to “all cases pending in any court.” Contrary to petitioner
The School grants foreign-hires certain benefits not accorded local-hires. These include housing,
and intervenors’ argument, the law is not particularly directed only to the Kuratong Baleleng cases. The
transportation, shipping costs, taxes, and home leave travel allowance. Foreign-hires are also paid a
transitory provision does not only cover cases which are in the Sandiganbayan but also in “any court.”
salary rate twenty-five percent (25%) more than local-hires. The School justifies the difference on two
"significant economic disadvantages" foreign-hires have to endure, namely: (a) the "dislocation factor"
There is nothing ex post facto in R.A. 8249. Ex post facto law, generally, provides retroactive effect of
and (b) limited tenure.
penal laws. R.A. 8249 is not a penal law. It is a substantive law on jurisdiction which is not penal in
character. Penal laws are those acts of the Legislature which prohibit certain acts and establish
Issue:
penalties for their violations or those that define crimes and provide for their punishment. R.A. 7975, as
regards the Sandiganbayan’s jurisdiction, its mode of appeal and other procedural matters, has been
Page 41
Whether or Not the grants provided by the school to foreign hires and not to local hires discriminative of On June 1, 1964, Ormoc Sugar Company, Inc. filed before the Court of First Instance of Leyte, with
their constitutional right to the equal protection clause. service of a copy upon the Solicitor General, a complaint against the City of Ormoc as well as its
Treasurer, Municipal Board and Mayor, alleging that the afore-stated ordinance is unconstitutional for
being violative of the equal protection clause (Sec. 1[1], Art. III, Constitution) and the rule of uniformity
Held:
of taxation (Sec. 22[1]), Art. VI, Constitution).
The foregoing provisions impregnably institutionalize in this jurisdiction the long honored legal truism of
Answering, the defendants asserted that the tax ordinance was within defendant city's power to enact
"equal pay for equal work." Persons who work with substantially equal qualifications, skill, effort and
under the Local Autonomy Act and that the same did not violate the afore-cited constitutional
responsibility, under similar conditions, should be paid similar salaries. This rule applies to the School,
limitations. After pre-trial and submission of the case on memoranda, the Court of First Instance, on
its "international character" notwithstanding.
August 6, 1964, rendered a decision that upheld the constitutionality of the ordinance and declared the
taxing power of defendant chartered city broadened by the Local Autonomy Act to include all other
The School contends that petitioner has not adduced evidence that local-hires perform work equal to
forms of taxes, licenses or fees not excluded in its charter.
that of foreign-hires. The Court finds this argument a little cavalier. If an employer accords employees
the same position and rank, the presumption is that these employees perform equal work. This
Issue:
presumption is borne by logic and human experience. If the employer pays one employee less than the
rest, it is not for that employee to explain why he receives less or why the others receive more. That
Whether or Not the ordinance is unconstitutional for being violative of the equal protection clause under
would be adding insult to injury. The employer has discriminated against that employee; it is for the
Sec. 1[1], Art. III, Constitution.
employer to explain why the employee is treated unfairly.
Whether or not it was violative of the rule of uniformity of taxation under the Bill of Rights, Sec. 22[1],
While we recognize the need of the School to attract foreign-hires, salaries should not be used as an
Art. VI, Constitution.
enticement to the prejudice of local-hires. The local-hires perform the same services as foreign-hires
and they ought to be paid the same salaries as the latter. For the same reason, the "dislocation factor"
Held:
and the foreign-hires' limited tenure also cannot serve as valid bases for the distinction in salary rates.
The Constitution in the bill of rights provides: ". . . nor shall any person be denied the equal protection of
The Constitution enjoins the State to "protect the rights of workers and promote their welfare," "to afford
the laws." (Sec. 1 [1], Art. III) In Felwa vs. Salas, We ruled that the equal protection clause applies only
labor full protection." The State, therefore, has the right and duty to regulate the relations between labor
to persons or things identically situated and does not bar a reasonable classification of the subject of
and capital. These relations are not merely contractual but are so impressed with public interest that
legislation, and a classification is reasonable where (1) it is based on substantial distinctions which
labor contracts, collective bargaining agreements included, must yield to the common good. Should
make real differences; (2) these are germane to the purpose of the law; (3) the classification applies not
such contracts contain stipulations that are contrary to public policy, courts will not hesitate to strike
only to present conditions but also to future conditions which are substantially identical to those of the
down these stipulations.
present; (4) the classification applies only to those who belong to the same class.
In this case, we find the point-of-hire classification employed by respondent School to justify the
A perusal of the requisites instantly shows that the questioned ordinance does not meet them, for it
distinction in the salary rates of foreign-hires and local hires to be an invalid classification. There is no
taxes only centrifugal sugar produced and exported by the Ormoc Sugar Company, Inc. and none
reasonable distinction between the services rendered by foreign-hires and local-hires.
other. At the time of the taxing ordinance's enactment, Ormoc Sugar Company, Inc., it is true, was the
only sugar central in the city of Ormoc. Still, the classification, to be reasonable, should be in terms
Wherefore, the petition is given due course. The petition is hereby granted in part. The orders of the
applicable to future conditions as well. The taxing ordinance should not be singular and exclusive as to
secretary of labor and employment dated June 10, 1996 and march 19, 1997, are hereby reversed and
exclude any subsequently established sugar central, of the same class as plaintiff, for the coverage of
set aside insofar as they uphold the practice of respondent school of according foreign-hires higher
the tax. As it is now, even if later a similar company is set up, it cannot be subject to the tax because
salaries than local-hires.
the ordinance expressly points only to Ormoc City Sugar Company, Inc. as the entity to be levied upon.

Appellant, however, is not entitled to interest; on the refund because the taxes were not arbitrarily
ORMOC SUGAR COMPANY VS. TREASURER OF ORMOC CITY
collected (Collector of Internal Revenue v. Binalbagan). 6 At the time of collection, the ordinance
[22 SCRA 603; L-23794; 17 FEB 1968]
provided a sufficient basis to preclude arbitrariness, the same being then presumed constitutional until
declared otherwise.
Facts:
Wherefore, the decision appealed from is hereby reversed, the challenged ordinance is declared
On January 29, 1964, the Municipal Board of Ormoc City passed Ordinance No. 4, Series of 1964,
unconstitutional and the defendants-appellees are hereby ordered to refund the P12,087.50 plaintiff-
imposing "on any and all productions of centrifugal sugar milled at the Ormoc Sugar Company, Inc., in
appellant paid under protest. No costs. So ordered.
Ormoc City a municipal tax equivalent to one per centum (1%) per export sale to the United States of
PHILIPPINE JUDGES ASSO. VS. PRADO
America and other foreign countries." Payments for said tax were made, under protest, by Ormoc
[227 SCRA 703; G.R. NO. 105371; 11 NOV 1993]
Sugar Company, Inc. on March 20, 1964 for P7, 087.50 and on April 20, 1964 for P5, 000, or a total of
P12, 087.50.
Facts:
Page 42
The Philippine Postal Corporation issued circular No. 92-28 to implement Section 35 of RA 7354
withdrawing the franking privilege from the SC, CA, RTCs, MeTCs, MTCs and Land Registration
Commission and with certain other government offices. It is alleged that RA 7354 is discriminatory
becasue while withdrawing the franking privilege from judiciary, it retains the same for the President &
Vice-President of the Philippines, Senator & members of the House of Representatives, COMELEC,
National Census & Statistics Office and the general public. The respondents counter that there is no
discrimination because the law is based on a valid classification in accordance with the equal protection
clause.

Issue:

Whether or Not Section 35 of RA 7354 is constitutional.

Held:

The equal protection of the laws is embraced in the concept of due process, as every unfair
discrimination offends the requirements of justice and fair play. It has nonetheless been embodied in a
separate clause in Article III Section 1 of the Constitution to provide for amore specific guarantee
against any form of undue favoritism or hostility from the government. Arbitrariness in general may be
challenged on the basis of the due process clause. But if the particular act assailed partakes of an
unwarranted partiality or prejudice, the sharper weapon to cut it down is the equal protection clause.
Equal protection simply requires that all persons or things similarly situated should be treated alike,
both as to rights conferred and responsibilities imposed. What the clause requires is equality among
equals as determined according to a valid classification. Section 35 of RA 7354 is declared
unconstitutional. Circular No. 92-28 is set aside insofar

Page 43
SEARCHES AND SEIZURES Stated otherwise, may an act of a private individual, allegedly in violation of appellant's constitutional
rights, be invoked against the State. In the absence of governmental interference, the liberties
guaranteed by the Constitution cannot be invoked against the State. It was Mr. Job Reyes, the
Art 3, Sec. 2. “The right of the people to be secure in their persons, houses, papers, and effects proprietor of the forwarding agency, who made search/inspection of the packages. Said inspection was
against unreasonable searches and seizures of whatever nature and for any purpose shall be reasonable and a standard operating procedure on the part of Mr. Reyes as a precautionary measure
inviolable, and no search warrant or warrant of arrest shall issue except upon probable cause to be before delivery of packages to the Bureau of Customs or the Bureau of Posts. Second, the mere
determined personally by the judge after examination under oath or affirmation of the complainant and presence of the NBI agents did not convert the reasonable search effected by Reyes into a warrantless
the witnesses he may produce, and particularly describing the place to be searched and the persons or search and seizure proscribed by the Constitution. Merely to observe and look at that which is in plain
things to be seized.” sight is not a search. Having observed that which is open, where no trespass has been committed in
aid thereof, is not search.”
Art 3, Sec. 3. “(1) The privacy of communication and correspondence shall be inviolable except No. “The law enforcers testified that accused/appellant was informed of his constitutional rights. It is
upon lawful order of the court, or when public safety or order requires otherwise as prescribed by law. presumed that they have regularly performed their duties (See. 5(m), Rule 131) and their testimonies
(2) Any evidence obtained in violation of this or the preceding section shall be inadmissible for should be given full faith and credence, there being no evidence to the contrary.”
any purpose in any proceeding.” No. “Appellant signed the contract as the owner and shipper thereof giving more weight to the
presumption that things which a person possesses, or exercises acts of ownership over, are owned by
him (Sec. 5 [j], Rule 131). At this point, appellant is therefore estopped to claim otherwise.”
PEOPLE VS. MARTI
[193 SCRA 57; G.R. NO. 81561; 18 JAN 1991]
WATEROUS DRUG VS. NLRC
Facts: [280 SCRA 735 ; G.R.NO. 113271; 16 OCT 1997]

Accused-appellant went to a forwarding agency to send four packages to a friend in Zurich. Initially, the Facts:
accused was asked by the proprietress if the packages can be examined. However, he refused. Before
delivering said packages to the Bureau of Customs and the Bureau of Posts, the husband of the Catolico was hired as a pharmacist by petitioner Waterous Drug Corporation on 15 August 1988. On 31
proprietress opened said boxes for final inspection. From that inspection, included in the standard July 1989, Catolico received a memorandum from WATEROUS Vice President-General Manager
operating procedure and out of curiosity, he took several grams of its contents. Emma R. Co warning her not to dispense medicine to employees chargeable to the latter's accounts
because the same was a prohibited practice. On the same date, Co issued another memorandum to
He brought a letter and the said sample to the National Bureau of Investigation. When the NBI was Catolico warning her not to negotiate with suppliers of medicine without consulting the Purchasing
informed that the rest of the shipment was still in his office, three agents went back with him. In their Department, as this would impair the company's control of purchases and, besides she was not
presence, the husband totally opened the packages. Afterwards, the NBI took custody of said authorized to deal directly with the suppliers.
packages. The contents , after examination by forensic chemists, were found to be marijuana flowering
tops. As regards the first memorandum, Catolico did not deny her responsibility but explained that her act
was "due to negligence," since fellow employee Irene Soliven "obtained the medicines in bad faith and
The appellant, while claiming his mail at the Central Post Office, was invited by the agents for through misrepresentation when she claimed that she was given a charge slip by the Admitting Dept."
questioning. Later on, the trial court found him guilty of violation of the Dangerous Drugs Act. Catolico then asked the company to look into the fraudulent activities of Soliven.

Issue: In a memorandum dated 21 November 1989, WATEROUS Supervisor Luzviminda E. Bautro warned
Catolico against the "rush delivery of medicines without the proper documents." On 29 January 1990,
Whether or Not the items admitted in the searched illegally searched and seized. WATEROUS Control Clerk Eugenio Valdez informed Co that he noticed an irregularity involving
Catolico and Yung Shin Pharmaceuticals, Inc.
Whether or Not custodial investigation properly applied. Forthwith, in her memorandum dated 37 January 1990, Co asked Catolico to explain, within twenty-four
hours, her side of the reported irregularity. Catolico asked for additional time to give her explanation,
Whether or Not the trial court not give credence to the explanation of the appellant on how said and she was granted a 48-hour extension from 1 to 3 February 1990. However, on 2 February 1990,
packages came to his possession. she was informed that effective 6 February 1990 to 7 March 1990, she would be placed on preventive
suspension to protect the interests of the company.

Held: In a letter dated 2 February 1990, Catolico requested access to the file containing Sales Invoice No.
266 for her to be able to make a satisfactory explanation. In said letter she protested Saldaña's invasion
No. “The case at bar assumes a peculiar character since the evidence sought to be excluded was of her privacy when Saldaña opened an envelope addressed to Catolico.
primarily discovered and obtained by a private person, acting in a private capacity and without the
intervention and participation of State authorities. Under the circumstances, can accused/appellant In a letter to Co dated 10 February 1990, Catolico, through her counsel, explained that the check she
validly claim that his constitutional right against unreasonable searches and seizure has been violated. received from YSP was a Christmas gift and not a "refund of overprice." She also averred that the
Page 44
preventive suspension was ill-motivated, as it sprang from an earlier incident between her and Co's
secretary, Irene Soliven. Catolico was denied due process. Procedural due process requires that an employee be apprised of
the charge against him, given reasonable time to answer the charge, allowed ample opportunity to be
On 5 March 1990, WATEROUS Supervisor Luzviminda Bautro, issued a memorandum notifying heard and defend himself, and assisted by a representative if the employee so desires. Ample
Catolico of her termination. On 5 May 1990, Catolico filed before the Office of the Labor Arbiter a opportunity connotes every kind of assistance that management must accord the employee to enable
complaint for unfair labor practice, illegal dismissal, and illegal suspension. In his decision of 10 May him to prepare adequately for his defense, including legal representation. In the case at bar, although
1993, Labor Arbiter Alex Arcadio Lopez found no proof of unfair labor practice against petitioners. Catolico was given an opportunity to explain her side, she was dismissed from the service in the
Nevertheless, he decided in favor of Catolico because petitioners failed to "prove what alleged as memorandum of 5 March 1990 issued by her Supervisor after receipt of her letter and that of her
complainant's dishonesty," and to show that any investigation was conducted. Hence, the dismissal counsel. No hearing was ever conducted after the issues were joined through said letters.
was without just cause and due process. He thus declared the dismissal and suspension illegal but
disallowed reinstatement. Catolico was also unjustly dismissed. It is settled that the burden is on the employer to prove just and
valid cause for dismissing an employee, and its failure to discharge that burden would result in a finding
Petitioners seasonably appealed from the decision and urged the NLRC to set it aside because the that the dismissal is unjustified. It clearly appears then that Catolico's dismissal was based on hearsay
Labor Arbiter erred in finding that Catolico was denied due process and that there was no just cause to information. Catolico's dismissal then was obviously grounded on mere suspicion, which in no case can
terminate her services. justify an employee's dismissal. Suspicion is not among the valid causes provided by the Labor Code
In its decision of 30 September 1993, the NLRC affirmed the findings of the Labor Arbiter on the ground for the termination of employment; and even the dismissal of an employee for loss of trust and
that petitioners were not able to prove a just cause for Catolico's dismissal from her employment. It confidence must rest on substantial grounds and not on the employer's arbitrariness, whims, caprices,
found that petitioner's evidence consisted only of the check of P640.00 drawn by YSP in favor of or suspicion. Besides, Catolico was not shown to be a managerial employee, to which class of
complainant, which her co-employee saw when the latter opened the envelope. But, it declared that the employees the term "trust and confidence" is restricted.
check was inadmissible in evidence pursuant to Sections 2 and 3(1 and 2) of Article III of the As regards the constitutional violation upon which the NLRC anchored its decision, that the Bill of
Constitution. It concluded: Rights does not protect citizens from unreasonable searches and seizures perpetrated by private
individuals. It is not true, as counsel for Catolico claims, that the citizens have no recourse against such
With the smoking gun evidence of respondents being rendered inadmissible, by assaults. On the contrary, and as said counsel admits, such an invasion gives rise to both criminal and
virtue of the constitutional right invoked by complainants, respondents' case falls civil liabilities.
apart as it is bereft of evidence which cannot be used as a legal basis for
complainant's dismissal. Finally, since it has been determined by the Labor Arbiter that Catolico's reinstatement would not be to
the best interest of the parties, he correctly awarded separation pay to Catolico. Separation pay in lieu
The NLRC then dismissed the appeal for lack of merit, but modified the dispositive portion of the of reinstatement is computed at one month's salary for every year of service. In this case, however,
appealed decision by deleting the award for illegal suspension as the same was already included in the Labor Arbiter Lopez computed the separation pay at one-half month's salary for every year of service.
computation of the aggregate of the awards in the amount of P35,401.86. Catolico did not oppose or raise an objection. As such, we will uphold the award of separation pay as
fixed by the Labor Arbiter.

Issue: WHEREFORE, the instant petition is hereby DISMISSED and the challenged decision and resolution of
the National Labor Relations Commission dated 30 September 1993 and 2 December 1993,
Whether or Not the dismissal of the private respondent is in violation of the Constitution, under the Bill respectively, in NLRC-NCR CA No. 005160-93 are AFFIRMED, except as to its reason for upholding
of Rights. the Labor Arbiter's decision, viz., that the evidence against private respondent was inadmissible for
having been obtained in violation of her constitutional rights of privacy of communication and against
Held: unreasonable searches and seizures which is hereby set aside.

As to the first and second grounds, petitioners insist that Catolico had been receiving "commissions"
from YSP, or probably from other suppliers, and that the check issued to her on 9 November 1989 was STONEHILL VS. DIOKNO
not the first or the last. They also maintained that Catolico occupied a confidential position and that [20 SCRA 383; L-19550; 19 JUN 1967]
Catolico's receipt of YSP's check, aggravated by her "propensity to violate company rules," constituted
breach of confidence. And contrary to the findings of NLRC, Catolico was given ample opportunity to Facts:
explain her side of the controversy.
Upon application of the officers of the government named on the margin 1 — hereinafter referred to as
In her Comment, Catolico asserts that petitioners' evidence is too "flimsy" to justify her dismissal. The Respondents-Prosecutors — several judges 2 — hereinafter referred to as Respondents-Judges —
check in issue was given to her, and she had no duty to turn it over to her employer. Company rules do issued, on different dates, 3 a total of 42 search warrants against petitioners herein4 and/or the
not prohibit an employee from accepting gifts from clients, and there is no indication in the contentious corporations of which they were officers, 5 directed to the any peace officer, to search the persons
check that it was meant as a refund for overpriced medicines. Besides, the check was discovered in above-named and/or the premises of their offices, warehouses and/or residences, and to seize and
violation of the constitutional provision on the right to privacy and communication; hence, as correctly take possession of the following personal property to wit:
held by the NLRC, it was inadmissible in evidence.
Page 45
Books of accounts, financial records, vouchers, correspondence, receipts, ledgers, journals, Two points must be stressed in connection with this constitutional mandate, namely: (1) that no warrant
portfolios, credit journals, typewriters, and other documents and/or papers showing all shall issue but upon probable cause, to be determined by the judge in the manner set forth in said
business transactions including disbursements receipts, balance sheets and profit and loss provision; and (2) that the warrant shall particularly describe the things to be seized.
statements and Bobbins (cigarette wrappers).
None of these requirements has been complied with in the contested warrants. Indeed, the same were
as "the subject of the offense; stolen or embezzled and proceeds or fruits of the offense," or "used or issued upon applications stating that the natural and juridical person therein named had committed a
intended to be used as the means of committing the offense," which is described in the applications "violation of Central Ban Laws, Tariff and Customs Laws, Internal Revenue (Code) and Revised Penal
adverted to above as "violation of Central Bank Laws, Tariff and Customs Laws, Internal Revenue Code." In other words, no specific offense had been alleged in said applications. The averments thereof
(Code) and the Revised Penal Code." with respect to the offense committed were abstract. As a consequence, it was impossible for the
judges who issued the warrants to have found the existence of probable cause, for the same
Petitioners contentions are: presupposes the introduction of competent proof that the party against whom it is sought has performed
(1) they do not describe with particularity the documents, books and things to be seized; particular acts, or committed specific omissions, violating a given provision of our criminal laws. As a
(2) cash money, not mentioned in the warrants, were actually seized; matter of fact, the applications involved in this case do not allege any specific acts performed by herein
(3) the warrants were issued to fish evidence against the aforementioned petitioners in deportation petitioners. It would be the legal heresy, of the highest order, to convict anybody of a "violation of
cases filed against them; Central Bank Laws, Tariff and Customs Laws, Internal Revenue (Code) and Revised Penal Code," —
(4) the searches and seizures were made in an illegal manner; and as alleged in the aforementioned applications — without reference to any determinate provision of said
(5) the documents, papers and cash money seized were not delivered to the courts that issued the laws or
__________________________
warrants, to be disposed of in accordance with law —
1
Hon. Jose W. Diokno, in his capacity as Secretary of Justice, Jose Lukban, in his capacity as Acting
Respondents-prosecutors contentions Director, National Bureau of Investigation, Special Prosecutors Pedro D. Cenzon, Efren I. Plana and
(1) that the contested search warrants are valid and have been issued in accordance with law; Manuel Villareal, Jr. and Assistant Fiscal Maneses G. Reyes, City of Manila.
2
(2) that the defects of said warrants, if any, were cured by petitioners' consent; and Hon. Amado Roan, Judge of the Municipal (now City) Court of Manila, Hon. Roman Cansino, Judge of
(3) that, in any event, the effects seized are admissible in evidence against herein petitioners, the Municipal (now City) Court of Manila, Hon. Hermogenes Caluag, Judge of the Court of First
regardless of the alleged illegality of the aforementioned searches and seizures. Instance of Rizal, Quezon City Branch, Hon. Eulogio Mencias, Judge of the Court of First Instance of
Rizal, Pasig Branch, and Hon. Damian Jimenez, Judge of the Municipal (now City) Court of Quezon
The documents, papers, and things seized under the alleged authority of the warrants in question may City.
3
be split into two (2) major groups, namely: (a) those found and seized in the offices of the Covering the period from March 3 to March 9, 1962.
4
aforementioned corporations, and (b) those found and seized in the residences of petitioners herein. Harry S. Stonehill, Robert P. Brooks, John J. Brooks and Karl Beck.
5
U.S. Tobacco Corporation, Atlas Cement Corporation, Atlas Development Corporation, Far East
Issue: Publishing Corporation (Evening News), Investment Inc., Industrial Business Management Corporation,
General Agricultural Corporation, American Asiatic Oil Corporation, Investment Management
Whether or not those found and seized in the offices of the aforementioned corporations are obtained Corporation, Holiday Hills, Inc., Republic Glass Corporation, Industrial and Business Management
legally. Corporation, United Housing Corporation, The Philippine Tobacco-Flue-Curing and Redrying
Corporation, Republic Real Estate Corporation and Merconsel Corporation.
Whether or not those found and seized in the residences of petitioners herein are obtained legally.

Held: BURGOS, SR. V. CHIEF OF STAFF, AFP


[133 SCRA 800; G.R. NO. 64261; 26 DEC 1984]
The petitioners have no cause of action to assail the legality of the contested warrants and of the
seizures made in pursuance thereof, for the simple reason that said corporations have their respective Facts:
personalities, separate and distinct from the personality of herein petitioners, regardless of the amount
of shares of stock or of the interest of each of them in said corporations, and whatever the offices they Petitioners assail the validity of 2 search warrants issued on December 7, 1982 by respondent Judge
hold therein may be. Indeed, it is well settled that the legality of a seizure can be contested only by the Cruz-Pano of the then Court of First Instance of Rizal, under which the premises known as No. 19,
party whose rights have been impaired thereby, and that the objection to an unlawful search and Road 3, Project 6, Quezon City, and 784 Units C & D, RMS Building, Quezon Avenue, Quezon City,
seizure is purely personal and cannot be availed of by third parties. business addresses of the "Metropolitan Mail" and "We Forum" newspapers, respectively, were
searched, and office and printing machines, equipment, paraphernalia, motor vehicles and other
With respect to the documents, papers and things seized in the residences of petitioners herein, the articles used in the printing, publication and distribution of the said newspapers, as well as numerous
aforementioned resolution of June 29, 1962, lifted the writ of preliminary injunction previously issued by papers, documents, books and other written literature alleged to be in the possession and control of
this Court, thereby, in effect, restraining herein Respondents-Prosecutors from using them in evidence petitioner Jose Burgos, Jr. publisher-editor of the "We Forum" newspaper, were seized. As a
against petitioners herein. consequence of the search and seizure, these premises were padlocked and sealed, with the further
result that the printing and publication of said newspapers were discontinued. Respondents contend
that petitioners should have filed a motion to quash said warrants in the court that issued them before
Page 46
impugning the validity of the same before this Court. Respondents also assail the petition on ground of 3. Section 2, Rule 126, of the Rules of Court, does not require that the property to be seized
laches (Failure or negligence for an unreasonable and unexplained length of time to do that which, by should be owned by the person against whom the search warrant is directed. It may or may
exercising due diligence, could or should have been done earlier. It is negligence or omission to assert not be owned by him.
a right within a reasonable time, warranting a presumption that the party entitled to assert it either has 4. Petitioners do not claim to be the owners of the land and/or building on which the
abandoned it or declined to assert it). Respondents further state that since petitioner had already used machineries were placed. This being the case, the machineries in question, while in fact
as evidence some of the documents seized in a prior criminal case, he is stopped from challenging the bolted to the ground, remain movable property susceptible to seizure under a search warrant.
validity of the search warrants. 5. The broad statements in the application and joint affidavit are mere conclusions of law and
does not satisfy the requirements of probable cause. Deficient of such particulars as would
Petitioners submit the following reasons to nullify the questioned warrants: justify a finding of the existence of probable cause, said allegation cannot serve as basis for
1. Respondent Judge failed to conduct an examination under oath or affirmation of the applicant the issuance of a search warrant and it was a grave error for respondent judge to have done
and his witnesses, as mandated by the above-quoted constitutional provision as well as Sec. so. In Alvarez v. Court of First Instance, this Court ruled that "the oath required must refer to
4, Rule 126 of the Rules of Court. the truth of the facts within the personal knowledge of the petitioner or his witnesses,
2. The search warrants pinpointed only one address which would be the former because the purpose thereof is to convince the committing magistrate, not the individual
abovementioned address. making the affidavit and seeking the issuance of the warrant, of the existence of probable
3. Articles belonging to his co-petitioners were also seized although the warrants were only cause." Another factor which makes the search warrants under consideration constitutionally
directed against Jose Burgos, Jr. objectionable is that they are in the nature of general warrants. The description of the articles
4. Real properties were seized. sought to be seized under the search warrants in question are too general.
5. The application along with a joint affidavit, upon which the warrants were issued, from the
Metrocom Intelligence and Security Group could not have provided sufficient basis for the With regard to the respondents invoking PD 885, there is an absence of any implementing rules and
finding of a probable cause upon which a warrant may be validly issued in accordance with regulations promulgated by the Minister of National Defense. Furthermore, President Marcos himself
Section 3, Article IV of the 1973 Constitution. denies the request of military authorities to sequester the property seized from petitioners. The closure
of the premises subjected to search and seizure is contrary to the freedom of the press as guaranteed
Respondents justify the continued sealing of the printing machines on the ground that they have been in our fundamental law. The search warrants are declared null and void.
sequestered under Section 8 of Presidential Decree No. 885, as amended, which authorizes
sequestration of the property of any person engaged in subversive activities against the government in
accordance with implementing rules and regulations as may be issued by the Secretary of National
Defense.

Issue:
TAMBASEN VS. PEOPLE
Whether or Not the 2 search warrants were validly issued and executed. [246 SCRA 184; G.R. NO. 89103; 14 JUL 1995]

Held: Facts:

In regard to the quashal of warrants that petitioners should have initially filed to the lower court, this In August 1988, P/Sgt. Natuel applied for issuance of search warrant alleging that he received
Court takes cognizance of this petition in view of the seriousness and urgency of the constitutional information that Petitioner had in his possession at his house “M-16 Armalite rifles, hand grenades, .45
Issue raised, not to mention the public interest generated by the search of the "We Forum" offices Cal. pistols, dynamite sticks and subversive documents”, which were “used or intended to be used” for
which was televised in Channel 7 and widely publicized in all metropolitan dailies. The existence of this illegal purposes. The application was granted.
special circumstance justifies this Court to exercise its inherent power to suspend its rules. With the
contention pertaining to laches, the petitioners gave an explanation evidencing that they have In September, a police team, searched the house of petitioner and seized “2 envelopes containing
exhausted other extra-judicial efforts to remedy the situation, negating the presumption that they have P14000, handset with antennae, transceiver with antennae, regulator supply, academy notebook and
abandoned their right to the possession of the seized property. assorted papers and handset battery pack”. In October, petitioner moved that the search and seizure
be declared illegal and that the seized articles be returned to him. In December, MTCC, in its order,
On the enumerated reasons: directed Lt. Col. Torres to return the money seized to petitioner ruling that any seizure should be limited
1. This objection may properly be considered moot and academic, as petitioners themselves to the specified items covered thereby. SolGen petitioned with the RTC for the annulment of the order
conceded during the hearing on August 9, 1983, that an examination had indeed been of MTCC citing that pending the determination of legality of seizure of the articles, they should remain in
conducted by respondent judge of Col. Abadilla and his witnesses. custogia legis. RTC granted the petition.
2. The defect pointed out is obviously a typographical error. Precisely, two search warrants
were applied for and issued because the purpose and intent were to search two distinct Issue:
premises. It would be quite absurd and illogical for respondent judge to have issued two
warrants intended for one and the same place. Whether or Not the seizure of the articles which were not mentioned in the search warrant was legal.

Page 47
Held: In these consolidated cases, three principal issues were raised: (1) whether or not petitioners were
denied due process when informations for libel were filed against them although the finding of the
Section 2 Article III of the 1987 Constitution requires that a search warrant should particularly describe existence of a prima facie case was still under review by the Secretary of Justice and, subsequently, by
the things to be seized. The police acts beyond the parameters of their authority if they seize articles the President; and (2) whether or not the constitutional rights of Beltran were violated when respondent
not described in the search warrants. The evident purpose and intent of the requirement is to limit the RTC judge issued a warrant for his arrest without personally examining the complainant and the
things to be seized, to leave the officers of the law with no discretion; that unreasonable search and witnesses, if any, to determine probable cause. Subsequent events have rendered the first issue moot
seizure may not be made and that abuses may not be committed. and academic. On March 30, 1988, the Secretary of Justice denied petitioners' motion for
reconsideration and upheld the resolution of the Undersecretary of Justice sustaining the City Fiscal's
Petition granted. People of the Philippines is ordered to return the money seized. finding of a prima facie case against petitioners. A second motion for reconsideration filed by petitioner
Beltran was denied by the Secretary of Justice on April 7, 1988. On appeal, the President, through the
Executive Secretary, affirmed the resolution of the Secretary of Justice on May 2, 1988. The motion for
PLACER VS. JUDGE VILLANUEVA reconsideration was denied by the Executive Secretary on May 16, 1988. With these developments,
[126 SCRA 463; G.R. NOS. L-60349-62; 29 DEC 1983] petitioners' contention that they have been denied the administrative remedies available under the law
has lost factual support.
Facts:
Issue:
Petitioners filed informations in the city court and they certified that Preliminary Investigation and
Examination had been conducted and that prima facie cases have been found. Upon receipt of said Whether or Not petitioners were denied due process when informations for libel were filed against them
informations, respondent judge set the hearing of the criminal cases to determine propriety of issuance although the finding of the existence of a prima facie case was still under review by the Secretary of
of warrants of arrest. After the hearing, respondent issued an order requiring petitioners to submit to Justice and, subsequently, by the President.
the court affidavits of prosecution witnesses and other documentary evidence in support of the
informations to aid him in the exercise of his power of judicial review of the findings of probable cause Whether or Not the constitutional rights of Beltran were violated when respondent RTC judge issued a
by petitioners. Petitioners petitioned for certiorari and mandamus to compel respondent to issue warrant for his arrest without personally examining the complainant and the witnesses, if any, to
warrants of arrest. They contended that the fiscal’s certification in the informations of the existence of determine probable cause
probable cause constitutes sufficient justification for the judge to issue warrants of arrest.
Held:
Issue:
With respect to petitioner Beltran, the allegation of denial of due process of law in the preliminary
Whether or Not respondent city judge may, for the purpose of issuing warrants of arrest, compel the investigation is negated by the fact that instead of submitting his counter- affidavits, he filed a "Motion to
fiscal to submit to the court the supporting affidavits and other documentary evidence presented during Declare Proceedings Closed," in effect waiving his right to refute the complaint by filing counter-
the preliminary investigation. affidavits. Due process of law does not require that the respondent in a criminal case actually file his
counter-affidavits before the preliminary investigation is deemed completed. All that is required is that
Held: the respondent be given the opportunity to submit counter-affidavits if he is so minded.

Judge may rely upon the fiscal’s certification for the existence of probable cause and on the basis
thereof, issue a warrant of arrest. But, such certification does not bind the judge to come out with the
warrant. The issuance of a warrant is not a mere ministerial function; it calls for the exercise of judicial The second issue, raised by petitioner Beltran, calls for an interpretation of the constitutional provision
discretion on the part of issuing magistrate. Under Section 6 Rule 112 of the Rules of Court, the judge on the issuance of warrants of arrest. The pertinent provision reads:
must satisfy himself of the existence of probable cause before issuing a warrant of arrest. If on the face
of the information, the judge finds no probable cause, he may disregard the fiscal’s certification and Art. III, Sec. 2. The right of the people to be secure in their persons, houses,
require submission of the affidavits of witnesses to aid him in arriving at the conclusion as to existence papers and effects against unreasonable searches and seizures of whatever
of probable cause. nature and for any purpose shall be inviolable, and no search warrant or warrant of
arrest shall issue except upon probable cause to be determined personally by the
Petition dismissed. judge after examination nder oath or affirmation of the complainant and the
witnesses he may produce, and particularly describing the place to be searched
and the persons or things to be seized.
SOLIVEN VS. MAKASIAR
[167 SCRA 393; G.R. NO. 82585; 14 NOV 1988] The addition of the word "personally" after the word "determined" and the deletion of the grant of
authority by the 1973 Constitution to issue warrants to "other responsible officers as may be authorized
Facts: by law," has apparently convinced petitioner Beltran that the Constitution now requires the judge to
personally examine the complainant and his witnesses in his determination of probable cause for the
issuance of warrants of arrest. This is not an accurate interpretation.
Page 48
Under the new Constitution, “. . . no search warrant or warrant of arrest shall issue except upon
What the Constitution underscores is the exclusive and personal responsibility of the issuing judge to probable cause to be determined personally by the judge after examination under oath or affirmation of
satisfy himself of the existence of probable cause. In satisfying himself of the existence of probable the complainant and the witnesses he may produce, and particularly describing the place to be
cause for the issuance of a warrant of arrest, the judge is not required to personally examine the searched and the persons or things to be seized”. Mayors and prosecuting officers cannot issue
complainant and his witnesses. Following established doctrine and procedure, he shall: (1) personally warrants of seizure or arrest. The Closure and Seizure Order was based on Article 38 of the Labor
evaluate the report and the supporting documents submitted by the fiscal regarding the existence of Code. The Supreme Court held, “We reiterate that the Secretary of Labor, not being a judge, may no
probable cause and, on the basis thereof, issue a warrant of arrest; or (2) if on the basis thereof he longer issue search or arrest warrants. Hence, the authorities must go through the judicial process. To
finds no probable cause, he may disregard the fiscal's report and require the submission of supporting that extent, we declare Article 38, paragraph (c), of the Labor Code, unconstitutional and of no force
affidavits of witnesses to aid him in arriving at a conclusion as to the existence of probable cause. and effect… The power of the President to order the arrest of aliens for deportation is, obviously,
exceptional. It (the power to order arrests) cannot be made to extend to other cases, like the one at bar.
Sound policy dictates this procedure, otherwise judges would be unduly laden with the preliminary Under the Constitution, it is the sole domain of the courts.” Furthermore, the search and seizure order
examination and investigation of criminal complaints instead of concentrating on hearing and deciding was in the nature of a general warrant. The court held that the warrant is null and void, because it must
cases filed before their courts. It has not been shown that respondent judge has deviated from the identify specifically the things to be seized.
prescribed procedure. Thus, with regard to the issuance of the warrants of arrest, a finding of grave
abuse of discretion amounting to lack or excess of jurisdiction cannot be sustained. The petitions fail to
establish that public respondents, through their separate acts, gravely abused their discretion as to
amount to lack of jurisdiction. Hence, the writs of certiorari and prohibition prayed for cannot issue. WHEREFORE, the petition is GRANTED. Article 38, paragraph (c) of the Labor Code is declared
UNCONSTITUTIONAL and null and void. The respondents are ORDERED to return all materials seized
WHEREFORE, finding no grave abuse of discretion amounting to excess or lack of jurisdiction on the as a result of the implementation of Search and Seizure Order No. 1205.
part of the public respondents, the Court Resolved to DISMISS the petitions in G. R. Nos. 82585,
82827 and 83979. The Order to maintain the status quo contained in the Resolution of the Court en
banc dated April 7, 1988 and reiterated in the Resolution dated April 26, 1988 is LIFTED. MORANO VS. VIVO
[20 SCRA 562; G.R. L-22196; 30 JUN 1967]
SALAZAR VS. ACHACOSO
[183 SCRA 145; G.R. NO. 81510; 14 MAR 1990] Facts:

Facts: Chan Sau Wah, a Chinese citizen born in Fukien, China arrived in the Philippines on November 1961 to
visit her cousin, Samuel Lee Malaps. She left China and her children by a first marriage: Fu Tse Haw
Rosalie Tesoro of Pasay City in a sworn statement filed with the POEA, charged petitioner with illegal and Fu Yan Kai both minors, in the care of neighbors in Fukien, China. Chan Sau wah arrived in the
recruitment. Public respondent Atty. Ferdinand Marquez sent petitioner a telegram directing him to Philippines with Fu Yan Fun, her minor son also by the first marriage. Chan Sau Wah and her minor
appear to the POEA regarding the complaint against him. On the same day, after knowing that son Fu Yan Fun were permitted only into the Philippines under a temporary visitor's visa for two months
petitioner had no license to operate a recruitment agency, public respondent Administrator Tomas and after they posted a cash bond of 4,000 pesos. On January 1962, Chan Sau Wah married Esteban
Achacoso issued a Closure and Seizure Order No. 1205 to petitioner. It stated that there will a seizure Morano, a native-born Filipino citizen. Born to this union on September 1962 was Esteban Morano, Jr.
of the documents and paraphernalia being used or intended to be used as the means of committing To prolong their stay in the Philippines, Chan Sau Wah and Fu Yan Fun obtained several extensions.
illegal recruitment, it having verified that petitioner has— (1) No valid license or authority from the The last extension expired on September 10, 1962. In a letter dated August 31, 1962, the
Department of Labor and Employment to recruit and deploy workers for overseas employment; (2) Commissioner of Immigration ordered Chan Sau Wah and her son, Fu Yan Fun, to leave the country on
Committed/are committing acts prohibited under Article 34 of the New Labor Code in relation to Article or before September 10, 1962 with a warning that upon failure so to do, he will issue a warrant for their
38 of the same code. A team was then tasked to implement the said Order. The group, accompanied arrest and will cause the confiscation of their bond.
by mediamen and Mandaluyong policemen, went to petitioner’s residence. They served the order to a
certain Mrs. For a Salazar, who let them in. The team confiscated assorted costumes. Petitioner filed Issue:
with POEA a letter requesting for the return of the seized properties, because she was not given prior
notice and hearing. The said Order violated due process. She also alleged that it violated sec 2 of the Whether or Not the issuance of the warrant of arrest is unconstitutional.
Bill of Rights, and the properties were confiscated against her will and were done with unreasonable
force and intimidation. Held:

Issue: Chan Sau Wah entered the Philippines on a tourist-temporary visitor's visa. She is a non-immigrant.
Under Section 13 just quoted, she may therefore be admitted if she were a qualified and desirable alien
Whether or Not the Philippine Overseas Employment Administration (or the Secretary of Labor) can and subject to the provisions of the last paragraph of Section 9. Therefore, first, she must depart
validly issue warrants of search and seizure (or arrest) under Article 38 of the Labor Code voluntarily to some foreign country; second, she must procure from the appropriate consul the proper
visa; and third, she must thereafter undergo examination by the officials of the Bureau of Immigration at
Held: the port of entry for determination of her admissibility in accordance with the requirements of the
immigration Act. This Court in a number of cases has ruled, and consistently too, that an alien admitted
Page 49
as a temporary visitor cannot change his or her status without first departing from the country and
complying with the requirements of Section 9 of the Immigration Act. The gravamen of petitioners' Issue:
argument is that Chan Sau Wah has, since her entry, married in Manila a native-born Filipino, Esteban
Morano. It will not particularly help analysis for petitioners to appeal to family solidarity in an effort to Whether or Not the Commissioner has the power to arrest and detain petitioners pending determination
thwart her deportation. Chan Sau Wah, seemingly is not one who has a high regard for such solidarity. of existence of probable cause.
Proof: She left two of her children by the first marriage, both minors, in the care of neighbors in Fukien,
China.Then, the wording of the statute heretofore adverted to is a forbidding obstacle which will prevent Whether or Not there was unreasonable searches and seizures by CID agents.
this Court from writing into the law an additional provision that marriage of a temporary alien visitor to a
Filipino would ipso facto make her a permanent resident in his country. This is a field closed to judicial
Whether or Not the writ of Habeas Corpus may be granted to petitioners.
action. No breadth of discretion is allowed. We cannot insulate her from the State's power of
deportation. it would be an easy matter for an alien woman to enter the Philippines as a temporary
Held:
visitor, go through a mock marriage, but actually live with another man as husband and wife, and
thereby skirt the provisions of our immigration law. Also, a woman of undesirable character may enter
While pedophilia is not a crime under the Revised Penal Code, it violates the declared policy of the
this country, ply a pernicious trade, marry a Filipino, and again throw overboard Sections 9 and 13 of
state to promote and protect the physical, moral, spiritual and social well being of the youth. The arrest
the Act. Such a flanking movement, we are confident, is impermissible.Recently we confirmed the rule
of petitioners was based on the probable cause determined after close surveillance of 3 months. The
that an alien wife of a Filipino may not stay permanently without first departing from the Philippines.
existence of probable cause justified the arrest and seizure of articles linked to the offense. The
Reason: Discourage entry under false pretenses.
articles were seized as an incident to a lawful arrest; therefore the articles are admissible evidences
(Rule 126, Section12 of Rules on Criminal Procedure).
HARVEY V. DEFENSOR-SANTIAGO
The rule that search and seizures must be supported by a valid warrant of arrest is not an absolute rule.
[162 SCRA 840; G.R. NO. 82544; 28 JUN 1988]
There are at least three exceptions to this rule. 1.) Search is incidental to the arrest. 2.) Search in a
moving vehicle. 3.) Seizure of evidence in plain view. In view of the foregoing, the search done was
Facts:
incidental to the arrest.
This is a petition for Habeas Corpus. Petitioners are the following: American nationals Andrew Harvey,
The filing of the petitioners for bail is considered as a waiver of any irregularity attending their arrest and
52 and Jonh Sherman 72. Dutch Citizen Adriaan Van Den Elshout, 58. All reside at Pagsanjan
estops them from questioning its validity. Furthermore, the deportation charges and the hearing
Laguna respondent Commissioner Miriam Defensor Santiago issued Mission Orders to the Commission
presently conducted by the Board of Special Inquiry made their detention legal. It is a fundamental rule
of Immigration and Deportation (CID) to apprehended petitioners at their residences. The “Operation
that habeas corpus will not be granted when confinement is or has become legal, although such
Report” read that Andrew Harvey was found together with two young boys. Richard Sherman was
confinement was illegal at the beginning.
found with two naked boys inside his room. While Van Den Elshout in the “after Mission Report” read
that two children of ages 14 and 16 has been under his care and subjects confirmed being live-in for
The deportation charges instituted by the Commissioner of Immigration are in accordance with Sec37
sometime now.
(a) of the Philippine Immigration Act of 1940 in relation to sec69 of the Revised Administrative code.
Section 37 (a) provides that aliens shall be arrested and deported upon warrant of the Commissioner of
Seized during the petitioner’s apprehension were rolls of photo negatives and photos of suspected child
Immigration and Deportation after a determination by the Board of Commissioners of the existence of a
prostitutes shown in scandalous poses as well as boys and girls engaged in sex. Posters and other
ground for deportation against them. Deportation proceedings are administrative in character and
literature advertising the child prostitutes were also found.
never construed as a punishment but a preventive measure. Therefore, it need not be conducted
strictly in accordance with ordinary Court proceedings. What is essential is that there should be a
Petitioners were among the 22 suspected alien pedophiles. They were apprehended 17 February1988
specific charge against the alien intended to be arrested and deported. A fair hearing must also be
after close surveillance for 3 month of the CID in Pagsanjan, Laguna. 17 of the arrested aliens opted
conducted with assistance of a counsel if desired.
for self-deportation. One released for lack of evidence, another charged not for pedophile but working
with NO VISA, the 3 petitioners chose to face deportation proceedings. On 4 March1988, deportation
Lastly, the power to deport aliens is an act of the State and done under the authority of the sovereign
proceedings were instituted against aliens for being undesirable aliens under Sec.69 of Revised
power. It a police measure against the undesirable aliens whose continued presence in the country is
Administrative Code.
found to be injurious to the public good and tranquility of the people.
Warrants of Arrest were issued 7March1988 against petitioners for violation of Sec37, 45 and 46 of
Immigration Act and sec69 of Revised Administrative Code. Trial by the Board of Special Inquiry III
commenced the same date. Petition for bail was filed 11March 1988 but was not granted by the
SALES VS. SANDIGANBAYAN
Commissioner of Immigration. 4 April1988 Petitioners filed a petition for Writ of Habeas Corpus. The
[369 SCRA 293 G.R. NO. 143802; 16 NOV 2001]
court heard the case on oral argument on 20 April 1988.
Facts:

Page 50
The petitioner, the incumbent mayor of Pagudpud Ilocos Norte, shot the former mayor and his political Judgment is rendered setting aside the resolution of the Sandiganbayan, ordering the Sandiganbayan
rival Atty. Benemerito. After the shooting, he surrendered himself and hence the police inspector and to quash the warrant of arrest and remanding the OMB for completion of the preliminary investigation.
wife of the victim filed a criminal complaint for murder against him. The judge after conducting the
preliminary examination (p.e. for brevity) found probable cause and issued a warrant of arrest. Also
after conducting the preliminary investigation (p.i. for brevity), he issued a resolution forwarding the SILVA VS. PRESIDING JUDGE
case to the prosecutor for appropriate action. Petitioner received a subpoena directing him to file his [203 SCRA 140; G.R. No. 81756; 21 Oct 1991]
counter affidavit, affidavit of witnesses and other supporting documents. He did it the following day.
While proceedings are ongoing, he filed a petition for habeas corpus with the C.A alleging that: the Facts:
warrant was null and void because the judge who issued it was a relative by affinity of the private
respondent and the p.e. and the p.i. were illegal and irregular as the judge doesn’t have jurisdiction on Sgt. Villamor, chief of the PC Narcom Detachment in Dumaguete City filed an "application for search
the case. The C.A. granted the petition holding that the judge was a relative by affinity by 3 rd degree to warrant" and "Deposition of witness" against petitioner Nicomedes Silva and Martin Silva. Judge
the private respondent and the p.i. he conducted has 2 stages, the p.e. and the p.i. proper. The Nickarter Ontal, then the presiding judge of RTC of Dumaguete issued Search Warrant No.1 pursuant
proceeding now consists only of one stage. He conducted the requisite investigation prior to the to the said applications for violation of RA 6425 Dangerous Drugs ACT of 1972. Such warrant states
issuance of warrant of arrest. Moreover he did not complete it. He only examined the witness of the that there is a probable cause to believe that Mr. Tama Silva has the possession and control of
complainant. But the prosecution instead of conducting p.i. of his own forwarded the records to the marijuana dried leaves, cigarette and joint. The warrant authorizes Sgt. Villamor to make an immediate
Ombudsman (OMB for brevity) for the latter to conduct the same. The OMB directed the petitioner to search at any time of the room of Mr. Tama Silva at the residence of his father Comedes Silva and to
submit his counter affidavit, but he did not comply with it finding the same superfluous. The graft open aparadors, lockers, cabinets, cartons and containers to look for said illegal drugs. In the course of
investigator recommended the filing of information for murder which the OMB approved. Petitioner the search, the officers seized money belonging to Antonieta Silva in the amount of P1,231.40.
received a copy of the resolution but prevented seeking reconsideration thereof he filed a motion to Petitioner filed a motion to quash Search Warrant No.1 on the ground that 1) it was issued on the sole
defer issuance of warrant of arrest pending the determination of probable cause. The Sandiganbayan basis of mimeographed 2) the judge failed to personally examine the complainant and witness by
denied the motion. This is now a petition for review on the decision of the Sandiganbayan, searching questions and answers.

Issue: Issue:

Whether or Not the OMB followed the procedure in conducting preliminary investigation. Whether or Not Search Warrant No.1 is invalid. WON the officers abused their authority in seizing the
money of Antonieta Silva.
Whether or Not petitioner was afforded an opportunity to be heard and to submit controverting
evidence. Held:

Held: Search Warrant No. 1 is invalid due to the failure of the judge to examine the witness in the form of
searching questions and answers. The questions asked were leading as they are answerable by mere
The proper procedure in the conduct of preliminary investigation was not followed because of the yes or no. Such questions are not sufficiently searching to establish probable cause. The questions
following reasons. Firstly, the preliminary investigation was conducted by 3 different investigators, none were already mimeographed and all the witness had to do was fill in their answers on the blanks
of whom completed the preliminary investigation There was not one continuous proceeding but rather, provided. Judge Ontal is guilty of grave abuse of discretion when he rejected the motion of Antonieta
cases of passing the buck, the last one being the OMB throwing the buck to the Sandiganbayan. Silva seeking the return of her money.
Secondly, the charge of murder is a non bailable offense. The gravity of the offense alone should have
merited a deeper and more thorough preliminary investigation. The OMB did nothing of the sort but The officers who implemented the search warrant clearly abused their authority when they seized the
wallowed the resolution of the graft investigator. He did a worse job than the judge, by actually adopting money of Antonieta Silva. The warrant did not indicate the seizure of money but only for marijuana
the resolution of the graft investigator without doing anything and threw everything to the leaves, cigarettes..etc. Search Warrant No. 1 is declared null and void.
Sandiganbayan for evaluation. Thirdly, a person under preliminary investigation by the OMB is entitled
to a motion for reconsideration, as maintained by the Rules of Procedure by the OMB. The filing of the *** Sec 4 Rule 126 Rules of Court
motion for reconsideration is an integral part of the preliminary investigation proper. The denial thereof Examination of the complainant, record -the judge before issuing the warrant, personally examine in the
is tantamount to the denial of the right itself to a preliminary investigation. This fact alone renders form of searching questions and answers, in writing and under oath the complainant and any witness
preliminary investigation conducted in this case incomplete. And lastly, it was patent error for the he may produce the facts personally known to them and attach to the record their sworn statements
Sandiganbayan to have relied purely on the OMB’s certification of probable cause given the prevailing together with their affidavits.
facts of the case much more so in the face of the latter’s flawed report and one side factual findings.

The court cannot accept the Sandiganbayan’s assertion of having found probable cause on its own,
considering the OMB’s defective report and findings, which merely rekied on the testimonies of the
witnesses for the prosecution and disregarded the evidence for the defense.

Page 51
Whether or Not Presidential Decree No. 1866, or at least the third paragraph of Section 1 thereof, is
VEROY VS. LAYAGUE unconstitutional for being violative of the due process and equal protection clauses of the Constitution.
[210 SCRA 97; G.R. No. 95630; 18 Jun 1992]
Held:
Facts:
The issue of constitutionality of Presidential Decree No. 1866 has been laid to rest in the case of
Petitioners are husband and wife who owned and formerly resided at No. 13 Isidro St., Skyline Village. Misolas v. Panga, G.R. No. 83341, January 30, 1990 (181 SCRA 648), where this Court held that the
Catalunan Grande, Davao City. When petitioner Leopoldo Veroy was promoted to the position of declaration of unconstitutionality of the third paragraph of Section 1 of Presidential Decree No. 1866 is
Assistant Administrator of the Social Security System sometime in June, 1988, he and his family wanting in legal basis since it is neither a bill of attainder nor does it provide a possibility of a double
transferred to 130 K-8th St., East Kamias, Quezon City, where they are presently residing. The care jeopardy.
and upkeep of their residence in Davao City was left to two (2) houseboys, Jimmy Favia and Eric
Burgos, who had their assigned quarters at a portion of the premises. The Veroys would occasionally Petitioners' contention that Republic Act 6968 has repealed Presidential Decree No. 1866 is bereft of
send money to Edna Soguilon for the salary of the said houseboys and other expenses for the upkeep merit. It is a cardinal rule of statutory construction that where the words and phrases of a statute are not
of their house. While the Veroys had the keys to the interior of the house, only the key to the kitchen, obscure or ambiguous. its meaning and the intention of the legislature must be determined from the
where the circuit breakers were located, was entrusted to Edna Soguilon to give her access in case of language employed, and where there is no ambiguity in the words, there is no room for construction.
an emergency. Hence, since 1988, the key to the master's bedroom as well as the keys to the Petitioners contend that Section 1 of Presidential Decree No. 1866 is couched in general or vague
children's rooms were retained by herein Petitioners so that neither Edna Soguilon nor the caretakers terms. The terms "deal in", "acquire", "dispose" or "possess" are capable of various interpretations such
could enter the house. that there is no definiteness as to whether or not the definition includes "constructive possession" or
how the concept of constructive possession should be applied. Petitioners were not found in actual
Police Officers had an information that the petitioner’s residence was being used as a safehouse of possession of the firearm and ammunitions. They were in Quezon City while the prohibited articles
rebel soldiers. They were able to enter the yard with the help of the caretakers but did not enter the were found in Davao City. Yet they were being charged under Presidential Decree No. 1866 upon the
house since the owner was not present and they did not have a search warrant. Petitioner Ma. Luisa sole circumstance that the house wherein the items were found belongs to them.
was contacted by telephone in her Quezon City residence by Capt. Obrero to ask permission to search
the house in Davao City as it was reportedly being used as a hideout and recruitment center of rebel
soldiers. Petitioner Ma. Luisa Veroy responded that she is flying to Davao City to witness the search but
relented if the search would not be conducted in the presence of Major Ernesto Macasaet, an officer of Petitioners question the admissibility in evidence of the articles seized in violation of their constitutional
the PC/INP, Davao City and a long time family friend of the Veroys. right against unreasonable search and seizure. Petitioners aver that while they concede that Capt.
Obrero had permission from Ma. Luisa Veroy to break open the door of their residence, it was merely
The following day, Capt. Obrero and Major Macasaet met at the house of herein petitioners in Skyline for the purpose of ascertaining thereat the presence of the alleged "rebel" soldiers. The permission did
Village to conduct the search pursuant to the authority granted by petitioner Ma. Luisa Veroy. The not include any authority to conduct a room to room search once inside the house. The items taken
caretakers facilitated their entry into the yard, and using the key entrusted to Edna Soguilon, they were were, therefore, products of an illegal search, violative of their constitutional rights As such, they are
able to gain entrance into the kitchen. However, a locksmith by the name of George Badiang had to be inadmissible in evidence against them.
employed to open the padlock of the door leading to the children's room. Capt. Obrero and Major
Macasaet then entered the children's room and conducted the search. Capt. Obrero recovered a .45 The Constitution guarantees the right of the people to be secure in their persons, houses, papers and
cal. handgun with a magazine containing seven (7) live bullets in a black clutch bag inside an unlocked effects against unreasonable searches and seizures (Article III, Section 2 of the 1987 Constitution).
drawer. Three (3) half-full jute sacks containing printed materials of RAM-SFP were also found in the However, the rule that searches and seizures must be supported by a valid warrant is not an absolute
children's room. A search of the children's recreation and study area revealed a big travelling bag one. Among the recognized exceptions thereto are: (1) a search incidental to an arrest; (2) a search of
containing assorted polo shirts, men's brief, two (2) pieces polo barong and short sleeve striped gray a moving vehicle; and (3) seizure of evidence in plain view (People v. Lo Ho Wing, G.R. No. 88017,
polo. sweat shirt, two (2) pairs men's socks, a towel made in U.S.A., one blanket, a small black bag, January 21, 1991 [193 SCRA 122]).
Gandhi brand, containing a book entitled "Islamic Revolution Future Path of the Nation", a road map of
the Philippines, a telescope, a plastic bag containing assorted medicines and religious pamphlets was None of these exceptions pertains to the case at bar. The reason for searching the house of herein
found in the master's bedroom. Sgt. Leo Justalero was instructed by Capt. Obrero to make an inventory petitioners is that it was reportedly being used as a hideout and recruitment center for rebel soldiers.
and receipt of the articles seized, in the house. While Capt. Obrero was able to enter the compound, he did not enter the house because he did not
have a search warrant and the owners were not present. This shows that he himself recognized the
The case was referred for preliminary investigation to Quezon City Assistant Prosecutor Rodolfo need for a search warrant, hence, he did not persist in entering the house but rather contacted the
Ponferrada who was designated Acting Provincial Prosecutor for Davao City. In a resolution dated Veroys to seek permission to enter the same. Permission was indeed granted by Ma. Luisa Veroy to
August 6, 1990, Fiscal Ponferrada recommended the filing of an information against herein petitioners enter the house but only to ascertain the presence of rebel soldiers. Under the circumstances it is
for Violation of Presidential Decree No. 1866 (Illegal Possession of Firearms and Ammunitions in undeniable that the police officers had ample time to procure a search warrant but did not.
Furtherance of Rebellion). No bail was recommended.
Undeniably, the offense of illegal possession of firearms is malum prohibitum but it does not follow that
Issue: the subject thereof is necessarily illegal per se. Motive is immaterial in mala prohibita but the subjects of
this kind of offense may not be summarily seized simply because they are prohibited. A search warrant
Page 52
is still necessary. Hence, the rule having been violated and no exception being applicable, the articles About 2pm that day, a police raiding team armed with a search warrant went to the Brgy captain for
seized were confiscated illegally and are therefore protected by the exclusionary principle. They cannot them to be accompanied in serving the said warrant at the residence of the accused. The police was
be used as evidence against the petitioners in the criminal action against them for illegal possession of allowed to enter the house upon the strength of the warrant shown to the accused. The accused
firearms. (Roan v. Gonzales, 145 SCRA 689-690 [1986]). Besides, assuming that there was indeed a begged the police not to search and to leave the house. The police still searched the house and was
search warrant, still in mala prohibita, while there is no need of criminal intent, there must be knowledge led to the kitchen. She pointed a metal basin on top of a table as the hiding place of died marijuana
that the same existed. Without the knowledge or voluntariness there is no crime. flowering tops contained in a plastic bag marked ISETANN. The police also recovered from a native
“uway” cabinet dried marijuana flowering tops wrapped in 3 pieces of komiks paper.
PREMISES CONSIDERED, the petition as granted and the criminal case against the petitioners for
illegal possession of firearms is DISMISSED. According to the accused, when the police arrived at her house, she saw Sgt. Yte and PFC Jose
Luciano. She invited Sgt. Yte to enter her house while Luciano was left in the jeep that was parked near
the house. While inside the house Yte showed the accused something he claimed as a search warrant,
when someone coming from the kitchen uttered “eto na” They proceeded to the kitchen and saw
PEOPLE VS. DEL ROSARIO Luciano holding a plastic bag with four other companions. They confronted the accused and insisted
[234 SCRA 246; G.R. NO. 109633; 20 JUL 1994] that the bags belonged to her. Accused denied the accusation and told them that she doesn’t know
anything about it. She was made to sign a prepared document. She was brought to the police station
Facts: and was detained.

Accused was charged and convicted by the trial court of illegal possession of firearms and illegal The court renders judgment finding the accused guilty.
possession and sale of drugs, particularly methamphetamine or shabu. After the issuance of the search
warrant, which authorized the search and seizure of an undetermined quantity of methamphetamine Issue:
and its paraphernalia’s, an entrapment was planned that led to the arrest of del Rosario and to the
seizure of the shabu, its paraphernalia’s and of a .22 caliber pistol with 3 live ammunition. Whether or Not the evidence was properly obtained by the police.

Issue: Held:

Whether or Not the seizure of the firearms was proper. In the investigation report prepared by Luciano stated that during the search they discovered a hole at
the backyard of the house of the suspect, there was a big biscuit can inside the hole and on top of the
Held: cover a flower pot was placed wherein the marijuana was kept. However, there was no mention of any
marijuana obtained from a flower pot in any of their testimonies. There were inconsistencies insofar the
No. Sec 2 art. III of the constitution specifically provides that a search warrant must particularly describe prosecution is concerned, as to what was recovered and where, the trial court concluded that these
the things to be seized. In herein case, the only objects to be seized that the warrant determined was inconsistencies are trivial. There must sufficient evidence that the marijuana was actually surrendered
the methamphetamine and the paraphernalia’s therein. The seizure of the firearms was by the accused. As held in PP vs. Remorosa, Irreconcilable and unexplained contradictions in the
unconstitutional. testimonies of the prosecution witnesses cast doubt on the guilt of appellant and his culpability to the
crime charged.
Wherefore the decision is reversed and the accused is acquitted.
The claim that the marijuana was planted was strengthen as the police violated sec 7, rule 126 rules of
the court provides no search of a house, room or any other premise shall be made except in the
presence of the lawful occupant thereof or any member of his family or in the absence of the latter, in
the presence of two (2) witnesses of sufficient age and discretion residing in the same locality. This
requirement is mandatory to ensure regularity in the execution of the search warrant. Violation of said
rule is in fact punishable under Article 130 of the Revised Penal Code.
PEOPLE VS. GESMUNDO
[219 SCRA 743; G.R. NO. 89373; 19 MAR 1993]

Facts: The document (PAGPAPATUNAY) was inadmissible to the court as the accused was not informed of
her right not to sign the document neither was she informed that she has the right to the assistance of a
According to the prosecution, in the morning of Nov. 17, 1986, PO Jose Luciano gave money and counsel and the fact that it may be used as evidence against her. It was not proved that the marijuana
instructed his civilian informer to buy marijuana from the accused at the Cocoland Hotel. He actually belonged to her. Not only does the law require the presence of witnesses when the search is
saw the accused selling marijuana to his civilian informer and that same day Luciano applied for a conducted, but it also imposes upon the person making the search the duty to issue a detailed receipt
search warrant. for the property seized. He is likewise required to deliver the property seized to the judge who issued
the warrant, together with a true and accurate inventory thereof duly verified under oath. Again, these
duties are mandatory and are required to preclude substitution of the items seized by interested parties.
Page 53
Issue:
The guilt of the accused was has not been established. Judgment is reversed.
Whether or Not arrest without warrant is lawful.
UMIL VS. RAMOS
[187 SCRA 311; G.R. NO. 81567; 3 OCT 1991] Whether or Not evidence from such arrest is admissible.

Facts: Held:

On 1 February 1988, military agents were dispatched to the St. Agnes Hospital, Roosevelt Avenue, Search and seizures supported by a valid warrant of arrest is not an absolute rule. Rule 126, Sec 12 of
Quezon City, to verify a confidential information which was received by their office, about a "sparrow Rules of Criminal Procedure provides that a person lawfully arrested may be searched for dangerous
man" (NPA member) who had been admitted to the said hospital with a gunshot wound. That the weapons or anything, which may be used as proff of the commission of an offense, without a search
wounded man in the said hospital was among the five (5) male "sparrows" who murdered two (2) warrant.(People v. Castiller) The failure of the police officers to secure a warrant stems from the fact
Capcom mobile patrols the day before, or on 31 January 1988 at about 12:00 o'clock noon, before a that their knowledge required from the surveillance was insufficient to fulfill requirements for its
road hump along Macanining St., Bagong Barrio, Caloocan City. The wounded man's name was listed issuance. However, warantless search and seizures are legal as long as PROBABLE CAUSE existed.
by the hospital management as "Ronnie Javellon," twenty-two (22) years old of Block 10, Lot 4, South The police officers have personal knowledge of the actual commission of the crime from the
City Homes, Biñan, Laguna however it was disclosed later that the true name of the wounded man was surveillance of the activities of the accused. As police officers were the ones conducting the
Rolando Dural. In view of this verification, Rolando Dural was transferred to the Regional Medical surveillance, it is presumed that they are regularly in performance of their duties.
Servicesof the CAPCOM, for security reasons. While confined thereat, he was positively identified by
the eyewitnesses as the one who murdered the 2 CAPCOM mobile patrols.
PEOPLE V. RODRIGUEZA
Issue: [205 SCRA 791; G.R. No. 95902; 4 Feb 1992]

Whether or Not Rolando was lawfully arrested. Facts:

Held: NARCOM agents staged a buy-bust operation, after gaining information that there was an ongoing
illegal traffic of prohibited drugs in Tagas, Albay. The participating agents were given money treated
Rolando Dural was arrested for being a member of the NPA, an outlawed subversive organization. with ultraviolet powder. One of the agents went to said location, asked for a certain Don. Thereafter,
Subversion being a continuing offense, the arrest without warrant is justified as it can be said that he the Don, herein accused, met with him and “a certain object wrapped in a plastic” later identified as
was committing as offense when arrested. The crimes rebellion, subversion, conspiracy or proposal to marijuana was given in exchange for P200. The agent went back to headquarters and made a report,
commit such crimes, and crimes or offenses committed in furtherance therefore in connection therewith based on which, a team was subsequently organized and a raid was conducted in the house of the
constitute direct assaults against the state and are in the nature of continuing crimes. father of the accused. During the raid, the NARCOM agents were able to confiscate dried marijuana
leaves and a plastic syringe among others. There was no authorization by any search warrant. The
accused was found positive of ultraviolet powder. The lower court, considering the evidences obtained
PEOPLE VS. SUCRO and testimonies from the prosecution, found him guilty of violating the Dangerous Drugs Act of 1972
[195 SCRA 388; G.R. No. 93239; 18 Mar 1991] and sentenced him to reclusion perpetua.

Facts: Issue:

Pat. Fulgencio went to Arlie Regalado’s house at C. Quimpo to monitor activities of Edison SUCRO Whether or Not the lower court was correct in its judgment.
(accused). Sucro was reported to be selling marijuana at a chapel 2 meters away from Regalado’s
house. Sucro was monitored to have talked and exchanged things three times. These activities are
reported through radio to P/Lt. Seraspi. A third buyer was transacting with appellant and was reported
and later identified as Ronnie Macabante. From that moment, P/Lt.Seraspi proceeded to the area.
While the police officers were at the Youth Hostel in Maagama St. Fulgencio told Lt. Seraspi to Held:
intercept. Macabante was intercepted at Mabini and Maagama crossing in front of Aklan Medical
center. Macabante saw the police and threw a tea bag of marijuana on the ground. Macabante The NARCOM agents’ procedure in the entrapment of the accused failed to meet the qualification that
admitted buying the marijuana from Sucro in front of the chapel. the suspected drug dealer must be caught red-handed in the act of selling marijuana to a person posing
as a buyer, since the operation was conducted after the actual exchange. Said raid also violated
The police team intercepted and arrested SUCRO at the corner of C. Quimpo and Veterans. accused’ right against unreasonable search and seizure, as the situation did not fall in the
Recovered were 19 sticks and 4 teabags of marijuana from a cart inside the chapel and another teabag circumstances wherein a search may be validly made even without a search warrant, i.e. when the
from Macabante. search is incidental to a lawful arrest; when it involves prohibited articles in plain view. The NARCOM
agents could not have justified their act by invoking the urgency and necessity of the situation because
Page 54
the testimonies of the prosecution witnesses reveal that the place had already been put under which yielded twenty (20) pieces of live .22 caliber firearm bullets from his left back pocket. When
surveillance for quite some time. Had it been their intention to conduct the raid, then they should, SPO2 Nunag peeked into the contents of the Zest-O box, he saw that it contained a crystalline
because they easily could, have first secured a search warrant during that time. The Court further substance. SPO2 Nulud instantly confiscated the small transparent plastic bag, the Zest-O juice box,
notes the confusion and ambiguity in the identification of the confiscated marijuana leaves and other the twenty (20) pieces of .22 caliber firearm bullets and the car used by accused-appellant. SPO2
prohibited drug paraphernalia presented as evidence against appellant: Nulud and the other police operatives who arrived at the scene brought the confiscated items to the
office of Col. Guttierez at the PNP Headquarters in Camp Pepito, Angeles City.
CIC Taduran, who acted as the poseur buyer, testified that appellant sold him 100 grams of dried
marijuana leaves wrapped in a plastic bag. Surprisingly, and no plausible explanation has been Accused-appellant vehemently denied the accusation against him and narrated a different version of
advanced therefor, what were submitted to and examined by the PCCL and thereafter utilized as the incident.
evidence against the appellant were the following items:
Accused-appellant alleged that he was driving the car of his wife to follow her and his son to Manila.
One (1) red and white colored plastic bag containing the following: He felt sleepy, so he decided to take the old route along McArthur Highway. He stopped in front of a
small store near Thunder Inn Hotel to buy cigarettes and candies. While at the store, he noticed a man
Exh. "A"—Thirty (30) grams of suspected dried marijuana fruiting tops contained approaches and examines the inside of his car. When he called the attention of the onlooker, the man
inside a transparent plastic bag. immediately pulled out a .45 caliber gun and made him face his car with raised hands. The man later on
Exh. "B"— Fifty (50) grams of suspected dried marijuana leaves and seeds identified himself as a policeman. During the course of the arrest, the policeman took out his wallet and
contained inside a white colored plastic labelled "Robertson". instructed him to open his car. He refused, so the policeman took his car keys and proceeded to
Exh. "C"— Four (4) aluminum foils each containing suspected dried marijuana search his car. At this time, the police officer’s companions arrived at the scene in two cars. PO2 Nulud,
fruiting tops having a total weight of seven grams then further wrapped who just arrived at the scene, pulled him away from his car in a nearby bank, while the others searched
with a piece of aluminum foil. his car.
Exh. "D"— Five (5) small transparent plastic bags each containing suspected dried
marijuana fruiting tops having a total weight of seventeen grams. Thereafter, he was brought to a police station and was held inside a bathroom for about fifteen minutes
Exh. "E"— One plastic syringe. until Col. Guttierez arrived, who ordered his men to call the media. In the presence of reporters, Col.
Guttierez opened the box and accused-appellant was made to hold the box while pictures were being
Evidently, these prohibited articles were among those confiscated during the so-called follow-up raid in taken.
the house of Rodrigueza’s father. The unanswered question then arises as to the identity of the
marijuana leaves that became the basis of appellant's conviction. In People vs. Rubio, this Court had The lower court acquitted Sy Chua for the Illegal Possession of Ammunitions, yet convicted him for
the occasion to rule that the plastic bag and the dried marijuana leaves contained therein constitute the Illegal Possession of 1,955.815 grams of shabu. Hence, this appeal to the Court.
corpus delicti of the crime. As such, the existence thereof must be proved with certainty and
conclusiveness. Failure to do so would be fatal to the cause of the prosecution. Conviction is reversed Issue:
and set aside and accused is acquitted.
Whether or Not the arrest of accused-appellant was lawful; and (2) WON the search of his person and
the subsequent confiscation of shabu allegedly found on him were conducted in a lawful and valid
PEOPLE VS. SY CHUA manner.
[396 SCRA 657; G.R. No.136066-67; 4 Feb 2003]
Held:
Facts:
The lower court believed that since the police received information that the accused will distribute illegal
Accused-appellant Binad Sy Chua was charged with violation of Section 16, Article III of R.A. 6425, as drugs that evening at the Thunder Inn Hotel and its vicinities. The police officer had to act quickly and
amended by R.A. 7659, and for Illegal Possession of Ammunitions and Illegal Possession of Drugs in there was no more time to secure a search warrant. The search is valid being akin to a “stop and frisk”.
two separate Informations.
The trial court confused the concepts of a “stop-and-frisk” and of a search incidental to a lawful arrest.
SPO2 Nulud and PO2 Nunag received a report from their confidential informant that accused-appellant These two types of warrantless searches differ in terms of the requisite quantum of proof before they
was about to deliver drugs that night at the Thunder Inn Hotel in Balibago, Angeles City. So, the PNP may be validly effected and in their allowable scope.
Chief formed a team of operatives. The group positioned themselves across McArthur Highway near
Bali Hai Restaurant, fronting the hotel. The other group acted as their back up. In a search incidental to a lawful arrest, as the precedent arrest determines the validity of the incidental
search, the legality of the arrest is questioned, e.g., whether an arrest was merely used as a pretext for
Afterwards, their informer pointed to a car driven by accused-appellant which just arrived and parked conducting a search. In this instance, the law requires that there first be arrest before a search can be
near the entrance of the hotel. After accused-appellant alighted from the car carrying a sealed Zest-O made—the process cannot be reversed. Accordingly, for this exception to apply, two elements must
juice box, SPO2 Nulud and PO2 Nunag hurriedly accosted him and introduced themselves as police concur: (1) the person to be arrested must execute an overt act indicating that he has just committed,
officers. As accused-appellant pulled out his wallet, a small transparent plastic bag with a crystalline is actually committing, or is attempting to commit a crime; and (2) such overt act is done in the
substance protruded from his right back pocket. Forthwith, SPO2 Nulud subjected him to a body search presence or within the view of the arresting officer.
Page 55
We find the two aforementioned elements lacking in the case at bar. Accused-appellant did not act in a Whether or Not warrantless arrest of petitioner was lawful.
suspicious manner. For all intents and purposes, there was no overt manifestation that accused-
appellant has just committed, is actually committing, or is attempting to commit a crime. “Reliable Whether or Not petitioner effectively waived his right to preliminary investigation.
information” alone, absent any overt act indicative of a felonious enterprise in the presence and within
the view of the arresting officers, is not sufficient to constitute probable cause that would justify an in Held:
flagrante delicto arrest.
Petitioner and prosecutor err in relying on Umil v. Ramos, wherein the Court upheld the warrantless
With regard to the concept of “stop-and frisk”: mere suspicion or a hunch will not validate a “stop-and- arrest as valid effected 1 to 14 days from actual commission of the offenses, which however constituted
frisk”. A genuine reason must exist, in light of the police officer’s experience and surrounding “continuing crimes,” i.e. subversion, membership in an outlawed organization, etc. There was no lawful
conditions, to warrant the belief that the person detained has weapons concealed about him. Finally, a warrantless arrest under Section 5, Rule 113. This is because the arresting officers were not actually
“stop-and-frisk” serves a two-fold interest: (1) the general interest of effective crime prevention and there during the incident, thus they had no personal knowledge and their information regarding
detection for purposes of investigating possible criminal behavior even without probable cause; and (2) petitioner were derived from other sources. Further, Section 7, Rule 112, does not apply.
the interest of safety and self-preservation which permit the police officer to take steps to assure himself
that the person with whom he deals is not armed with a deadly weapon that could unexpectedly and
fatally be used against the police officer. Petitioner was not arrested at all, as when he walked in the police station, he neither expressed
surrender nor any statement that he was or was not guilty of any crime. When a complaint was filed to
A stop-and-frisk was defined as the act of a police officer to stop a citizen on the street, interrogate him, the prosecutor, preliminary investigation should have been scheduled to determine probable cause.
and pat him for weapon(s) or contraband. It should also be emphasized that a search and seizure Prosecutor made a substantive error, petitioner is entitled to preliminary investigation, necessarily in a
should precede the arrest for this principle to apply. The foregoing circumstances do not obtain in the criminal charge, where the same is required appear thereat. Petition granted, prosecutor is ordered to
case at bar. To reiterate, accused-appellant was first arrested before the search and seizure of the conduct preliminary investigation, trial for the criminal case is suspended pending result from
alleged illegal items found in his possession. The apprehending police operative failed to make any preliminary investigation, petitioner is ordered released upon posting a bail bond.
initial inquiry into accused-appellant’s business in the vicinity or the contents of the Zest-O juice box he
was carrying. The apprehending police officers only introduced themselves when they already had
custody of accused-appellant. CALLANTA VS. VILLANUEVA
[77 SCRA 377; G.R. NOS. 24646 & L-24674; 20 JUN 1977]
In the case at bar, neither the in flagrante delicto nor the “stop and frisk” principles is applicable to
justify the warrantless arrest and consequent search and seizure made by the police operatives on Facts:
accused-appellant.
Two complaints for grave oral defamation were filed against Faustina Callanta. The City Judge of
Wherefore, accused-appellant Binad Sy Chua is hereby Acquitted. Dagupan City, Felipe Villanueva, denied the motions to quash the complaints. Thus, petitioner Callanta
brought the suits for certiorari in the Supreme Court. Petitioner questions the validity of the issuance of
warrant of arrest by respondent, arguing that the City Fiscal should have conducted the preliminary
GO VS. COURT OF APPEALS investigation. According to petitioner’s counsel, there was jurisdictional infirmity. After the issuance of
[206 SCRA 138; G.R. NO. 101837; 11 FEB 1992] the warrants of arrest and the bail fixed at P600, petitioner posted the bail bond, thus obtaining her
provisional liberty. The City Fiscal in this case did not disagree with the judge’s investigation, and
Facts: agreed with the complaints filed.

Petitioner, while traveling in the wrong direction on a one-way street, almost had a collision with another Issue:
vehicle. Petitioner thereafter got out of his car, shot the driver of the other vehicle, and drove off. An
eyewitness of the incident was able to take down petitioner’s plate number and reported the same to Whether or Not petitioner’s contentions are to be given merit.
the police, who subsequently ordered a manhunt for petitioner. 6 days after the shooting, petitioner
presented himself in the police station, accompanied by 2 lawyers, the police detained him. Held:
Subsequently a criminal charge was brought against him. Petitioner posted bail, the prosecutor filed
the case to the lower court, setting and commencing trial without preliminary investigation. Prosecutor Based on many precedent cases of the Supreme Court, “where the accused has filed bail and waived
reasons that the petitioner has waived his right to preliminary investigation as bail has been posted and the preliminary investigation proper, he has waived whatever defect, if any, in the preliminary
that such situation, that petitioner has been arrested without a warrant lawfully, falls under Section 5, examination conducted prior to the issuance of the warrant of arrest”. In the case at bar, it is futile for
Rule 113 and Section 7, Rule 112 of The 1985 Rules of Criminal Procedure which provides for the rules the petitioner to question the validity of the issuance of the warrant of arrest, because she posted the
and procedure pertaining to situations of lawful warrantless arrests. Petitioner in his petition for bail bond. Petitioner also erred in arguing that only the City Fiscal can conduct a preliminary
certiorari assails such procedure and actions undertaken and files for a preliminary investigation. investigation. According to the Charter of the City of Dagupan, “the City Court of Dagupan City may
also conduct preliminary investigation for any offense, without regard to the limits of punishment, and
Issue: may release, or commit and bind over any person charged with such offense to secure his appearance
Page 56
before the proper court”. Petition for certiorari is denied. Restraining order issued by the Court is lifted Clearly, the search in the case at bar can be sustained under the exceptions heretofore discussed, and
and set aside. hence, the constitutional guarantee against unreasonable searches and seizures has not been violated.

PEOPLE V. MENGOTE
[210 SCRA 174; G.R. NO. 87059; 22 JUN 1992]

Facts:
POSADAS VS. COURT OF APPEALS
[188 SCRA 288; G.R. NO. 89139; 2 AUG 1990] The Western Police District received a telephone call from an informer that there were three suspicious
looking persons at the corner of Juan Luna and North Bay Boulevard in Tondo, Manila. A surveillance
Facts: team of plainclothesmen was forthwith dispatched to the place. The patrolmen saw two men looking
from side to side, one of whom holding his abdomen. They approached the persons and identified
themselves as policemen, whereupon the two tried to run but unable to escape because the other
Members of the Integrated National Police (INP) of the Davao Metrodiscom assigned with the
lawmen surrounded them. The suspects were then searched. One of them the accused-appellant was
Intelligence Task Force, Pat. Ursicio Ungab and Pat. Umbra Umpar conducted surveillance along
found with a .38 caliber with live ammunitions in it, while his companion had a fan knife. The weapons
Magallanes Street, Davao City. While in the vicinity of Rizal Memorial Colleges they spotted petitioner
were taken from them and they were turned over to the police headquarters for investigation. An
carrying a "buri" bag and they noticed him to be acting suspiciously. They approached the petitioner
information was filed before the RTC convicting the accused of illegal possession of firearm arm. A
and identified themselves as members of the INP. Petitioner attempted to flee but his attempt to get
witness testified that the weapon was among the articles stolen at his shop, which he reported to the
away was unsuccessful. They then checked the "buri" bag of the petitioner where they found one (1)
police including the revolver. For his part, Mengote made no effort to prove that he owned the fire arm
caliber .38 Smith & Wesson revolver with Serial No. 770196, two (2) rounds of live ammunition for a .38
or that he was licensed to possess it but instead, he claimed that the weapon was planted on him at the
caliber gun, a smoke (tear gas) grenade, and two (2) live ammunitions for a .22 caliber gun. They
time of his arrest. He was convicted for violation of P.D.1866 and was sentenced to reclusion perpetua.
brought the petitioner to the police station for further investigation. In the course of the same, the
In his appeal he pleads that the weapon was not admissible as evidence against him because it had
petitioner was asked to show the necessary license or authority to possess firearms and ammunitions
been illegally seized and therefore the fruit of a poisonous tree.
found in his possession but he failed to do so. He was then taken to the Davao Metrodiscom office and
the prohibited articles recovered from him were indorsed to M/Sgt. Didoy the officer then on duty. He
Issue:
was prosecuted for illegal possession of firearms and ammunitions in the Regional Trial Court of Davao
City.
Whether or not the warrantless search and arrest was illegal.
Issue:
Held:
Whether or Not the warantless search is valid.
An evidence obtained as a result of an illegal search and seizure inadmissible in any proceeding for
any purpose as provided by Art. III sec 32 of the Constitution. Rule 113 sec.5 of the Rules of Court,
Held:
provides arrest without warrant lawful when: (a) the person to be arrested has committed, is actually
committing, or is attempting to commit an offense, (b) when the offense in fact has just been committed,
In justifying the warrantless search of the buri bag then carried by the petitioner, argues that under
and he has personal knowledge of the facts indicating the person arrested has committed it and (c) the
Section 12, Rule 136 of the Rules of Court a person lawfully arrested may be searched for dangerous
person to be arrested has escaped from a penal establishment or a place where he is serving final
weapons or anything used as proof of a commission of an offense without a search warrant. It is further
judgment or temporarily confined while his case is pending, or has escaped while being transferred
alleged that the arrest without a warrant of the petitioner was lawful under the circumstances.
from one confinement to another.
in the case at bar, there is no question that, indeed, it is reasonable considering that it was effected on
These requirements have not been established in the case at bar. At the time of the arrest in question,
the basis of a probable cause. The probable cause is that when the petitioner acted suspiciously and
the accused appellant was merely looking from side to side and holding his abdomen, according to the
attempted to flee with the buri bag there was a probable cause that he was concealing something illegal
arresting officers themselves. There was apparently no offense that has just been committed or was
in the bag and it was the right and duty of the police officers to inspect the same.
being actually committed or at least being attempt by Mengote in their presence. Moreover a person
may not be stopped and frisked in a broad daylight or on a busy street on unexplained suspicion.
It is too much indeed to require the police officers to search the bag in the possession of the petitioner
only after they shall have obtained a search warrant for the purpose. Such an exercise may prove to be
Judgment is reversed and set aside. Accused-appellant is acquitted.
useless, futile and much too late.

Page 57
Commanding Officer of NARCOM, that same morning that a Caucasian coming from Sagada had in his
possession prohibited drugs. The group composed of seven (7) NARCOM officers, in coordination with
Tublay Police Station, set up a checkpoint at the designated area at about 10:00 o'clock in the morning
PEOPLE VS. TANGLIBEN and inspected all vehicles coming from the Cordillera Region.
[184 SCRA 220; G.R. No.L-63630; 6 Apr 1990]
The two (2) NARCOM officers started their inspection from the front going towards the rear of the bus.
Facts: Accused who was the sole foreigner riding the bus was seated at the rear thereof.

Patrolmen Silverio and Romeo Punzalan were conducting surveillance at the San Fernando Victory During the inspection, CIC Galutan noticed a bulge on accused's waist. Suspecting the bulge on
Liner Terminal. At around 9:30pm they noticed a person, Medel Tangliben, carrying a traveling bag who accused's waist to be a gun, the officer asked for accused's passport and other identification papers.
acted suspiciously. They confronted him, inspected his bag, and there they found marijuana leaves. When accused failed to comply, the officer required him to bring out whatever it was that was bulging
The accused was then taken to the Police Headquarters for further investigations. The TC found on his waist. The bulging object turned out to be a pouch bag and when accused opened the same
Tangliben guilty of violating sec.4 art. 2 of the RA 6425 or the Dangerous Drugs Act of 1972. bag, as ordered, the officer noticed four (4) suspicious-looking objects wrapped in brown packing tape,
prompting the officer to open one of the wrapped objects. The wrapped objects turned out to contain
Issue: hashish, a derivative of marijuana.

Whether or Not there was an unlawful search due to lack of search warrant. Thereafter, accused was invited outside the bus for questioning. But before he alighted from the bus,
accused stopped to get two (2) travelling bags from the luggage carrier. Upon stepping out of the bus,
Held; the officers got the bags and opened them. A teddy bear was found in each bag. Feeling the teddy
bears, the officer noticed that there were bulges inside the same which did not feel like foam stuffing. It
No. Rule 113 sec. 5 provides the a peace officer or a private person may w/o a warrant arrest a person was only after the officers had opened the bags that accused finally presented his passport.
when in his presence the person to be arrested has committed, is committing, or is attempting to
commit an offense. Accused was then brought to the headquarters of the NARCOM at Camp Dangwa, La Trinidad,
Benguet for further investigation. At the investigation room, the officers opened the teddy bears and
In the present case, the accused was found to have been committing possession of marijuana and can they were found to also contain hashish. Representative samples were taken from the hashish found
be therefore searched lawfully even without a search warrant. Another reason is that this case poses among the personal effects of accused and the same were brought to the PC Crime Laboratory for
urgency on the part of the arresting police officers. It was found out that an informer pointed to the chemical analysis.
accused telling the policemen that the accused was carrying marijuana. The police officers had to act In the chemistry report, it was established that the objects examined were hashish. a prohibited drug
quickly and there was not enough time to secure a search warrant. which is a derivative of marijuana. Thus, an information was filed against accused for violation of the
Dangerous Drugs Act.
ACCUSED’S DEFENSE
PEOPLE VS. MALMSTEDT During the arraignment, accused entered a plea of "not guilty." For his defense, he raised the issue of
[198 SCRA 401; G.R. No. 91107; 19 Jun 1991] illegal search of his personal effects. He also claimed that the hashish was planted by the NARCOM
officers in his pouch bag and that the two (2) travelling bags were not owned by him, but were merely
Facts: entrusted to him by an Australian couple whom he met in Sagada. He further claimed that the
Australian couple intended to take the same bus with him but because there were no more seats
In an information filed against the accused- appellant Mikael Malmstead was charged before the RTC available in said bus, they decided to take the next ride and asked accused to take charge of the bags,
of La Trinidad, Benguet, for violation of Section 4, Art. II of Republic Act 6425, as amended, otherwise and that they would meet each other at the Dangwa Station.
known as the Dangerous Drugs Act of 1972, as amended.
Accused Mikael Malmstedt, a Swedish national, entered the Philippines for the third time in December The trial court found the guilt of the accused Mikael Malmstedt established beyond reasonable doubt.
1988 as a tourist. He had visited the country sometime in 1982 and 1985.

In the evening of 7 May 1989, accused left for Baguio City. Upon his arrival thereat in the morning of Seeking the reversal of the decision of the trial court finding him guilty of the crime charged, accused
the following day, he took a bus to Sagada and stayed in that place for two (2) days. Then in the 7 in argues that the search of his personal effects was illegal because it was made without a search warrant
the morning of May 11, 1989, the accused went to Nangonogan bus stop in Sagada. and, therefore, the prohibited drugs which were discovered during the illegal search are not admissible
as evidence against him.
At about 8: 00 o'clock in the morning of that same day (11 May 1989), Captain Alen Vasco, the
Commanding Officer of the First Regional Command (NARCOM) stationed at Camp Dangwa, ordered Issue:
his men to set up a temporary checkpoint at Kilometer 14, Acop, Tublay, Mountain Province, for the
purpose of checking all vehicles coming from the Cordillera Region. The order to establish a checkpoint Whether or Not the contention of the accused is valid, and therefore the RTC ruling be reversed.
in the said area was prompted by persistent reports that vehicles coming from Sagada were
transporting marijuana and other prohibited drugs. Moreover, information was received by the Held:
Page 58
fiscal filed a motion to dismiss the charge against Ali on the basis of a sworn statement of the arresting
The Constitution guarantees the right of the people to be secure in their persons, houses, papers and officers absolving her after a 'thorough investigation." The motion was granted, and trial proceeded only
effects against unreasonable searches and seizures. However, where the search is made pursuant to a against the accused-appellant, who was eventually convicted . In his defense, Aminnudin disclaimed
lawful arrest, there is no need to obtain a search warrant. A lawful arrest without a warrant may be the marijuana, averring that all he had in his bag was his clothing consisting of a jacket, two shirts and
made by a peace officer or a private person under the following circumstances. two pairs of pants. He alleged that he was arbitrarily arrested and immediately handcuffed. His bag was
confiscated without a search warrant. At the PC headquarters, he was manhandled to force him to
Sec. 5 Arrest without warrant; when lawful. –– A peace officer or a private person admit he was carrying the marijuana, the investigator hitting him with a piece of wood in the chest and
may, without a warrant, arrest a person: arms even as he parried the blows while he was still handcuffed. He insisted he did not even know what
marijuana looked like and that his business was selling watches and sometimes cigarettes. However
(a) When, in his presence, the person to be arrested has committed is the RTC rejected his allegations. Saying that he only has two watches during that time and that he did
actually committing, or is attempting to commit an offense; not sufficiently proved the injuries allegedly sustained.
(b) When an offense has in fact just been committed, and he has personal
knowledge of facts indicating that the person to be arrested has Issue:
committed it; and
(c) When the person to be arrested is a prisoner who has escaped from a Whether or not search of defendant’s bag is legal.
penal establishment or place where he is serving final judgment or
temporarily confined while his case is pending, or has escaped while Held:
being transferred from one confinement to another.
The search was illegal. Defendant was not caught in flagrante delicto, which could allow warrantless
Accused was searched and arrested while transporting prohibited drugs (hashish). A crime was actually arrest or search. At the moment of his arrest, he was not committing a crime. Nor was he about to do so
being committed by the accused and he was caught in flagrante delicto. Thus, the search made upon or had just done so. To all appearances, he was like any of the other passengers innocently
his personal effects falls squarely under paragraph (1) of the foregoing provisions of law, which allow a disembarking from the vessel. The said marijuana therefore could not be appreciated as evidence
warrantless search incident to a lawful arrest. While it is true that the NARCOM officers were not against the defendant, and furthermore he is acquitted of the crime as charged.
armed with a search warrant when the search was made over the personal effects of accused,
however, under the circumstances of the case, there was sufficient probable cause for said officers to
believe that accused was then and there committing a crime. PEOPLE VS. SAYCON
[236 SCRA 325; G.R. NO. 110995; 5 SEPT 1994]
Probable cause has been defined as such facts and circumstances which could lead a reasonable,
discreet and prudent man to believe that an offense has been committed, and that the objects sought in Facts:
connection with the offense are in the place sought to be searched. Warrantless search of the personal
effects of an accused has been declared by this Court as valid, because of existence of probable On or about 8 July 1992, at about 6:00 in the morning, the Coastguard personnel received information
cause, where the smell of marijuana emanated from a plastic bag owned by the accused, 10 or where from NARCOM agent Ruben Laddaran that a suspected "shabu" courier by the name of Alvaro Saycon
the accused was acting suspiciously, 11 and attempted to flee. was on board the MV Doña Virginia, which was arriving at that moment in Dumaguete City. Upon
receipt of the information, the Coastguard chief officer CPO Tolin, instructed them to intercept the
The appealed judgment of conviction by the trial court is hereby affirmed. Costs against the accused- suspect. A combined team of NARCOM agents and Philippine Coastguard personnel consisting of CPO
appellant. Tolin, a certain Miagme, and Senior Police Officers Ruben Laddaran and Winifredo Noble of NARCOM
posted themselves at the gate of Pier 1. The MV Doña Virginia docked at 6:00 a.m. that same morning
at Pier 1 in Dumaguete City. Alvaro Saycon alighted from the boat carrying a black bag and went
PEOPLE VS. AMMINUDIN through the checkpoint manned by the Philippine Coastguard where he was identified by police officer
[163 SCRA 402; G.R. L-74869; 6 Jul 1988] Winifredo Noble of NARCOM. Saycon was then invited to the Coastguard Headquarters at the Pier
area. He willingly went with them. At the headquarters, the coastguard asked Saycon to open his bag,
Facts: and the latter willingly obliged. In it were personal belongings and a maong wallet. Inside that maong
wallet, there was a Marlboro pack containing the suspected "shabu". When police officer Winifredo
Idel Aminnudin, accused-appellant was arrested on June 25, 1984, shortly after disembarking from the Noble asked Saycon whether the Marlboro pack containing the suspected "shabu" was his, Saycon
M/V Wilcon 9 at about 8:30 in the evening, in Iloilo City. The PC officers who were in fact waiting for him merely bowed his head. Then Saycon, his bag and the suspected "shabu" were brought to the
because of a tip from one their informers simply accosted him, inspected his bag and finding what NARCOM office for booking. When Alvaro Saycon was arrested, the NARCOM agents did not have a
looked liked marijuana leaves took him to their headquarters for investigation. The two bundles of warrant of arrest. The PNP's Forensic Analyst declared in court that she had conducted an examination
suspect articles were confiscated from him and later taken to the NBI laboratory for examination. It was of the specimens and found out that the specimens weighed 4.2 grams in total, consisted of
found to contain three kilos of what were later analyzed as marijuana leaves by an NBI forensic methamphetamine hydrochloride, more widely known as "shabu."
examiner. An information for violation of the Dangerous Drugs Act was filed against him. Later, the
information was amended to include Farida Ali y Hassen, who had also been arrested with him that Issue:
same evening and likewise investigated. Both were arraigned and pleaded not guilty. Subsequently, the
Page 59
Whether or Not the warrantless search was valid. In the case at bar, the plastic bag was not in the ‘plain view’ of the police. They arrested the accused in
the living room and moved into the kitchen in search for other evidences where they found the plastic
Held: bag. Furthermore, the marijuana inside the plastic bag was not immediately apparent from the ‘plain
view’ of said object.
The warrantless search was valid, as the accused was a passenger of a motor vehicle. There was
probable cause to believe that the accused was carrying prohibited drugs. Three weeks earlier, agents Therefore, the ‘plain view’ does not apply. The plastic bag was seized illegally and cannot be presented
of the Narcotics Command bought methamine hydrochloride from him. An agent of the Narcotics in evidence pursuant to Article III Section 3 (2) of the Constitution.
Command reported that the accused would be arriving on board the vessel and carrying
methamphetamine hydrochloride with him. Drug couriers do not go about their trade with some external PITA VS. COURT OF APPEALS
sign indicating that they are transporting prohibited drugs. This must be taken into account in [178 SCRA 362; G.R. NO. 80806; 5 OCT 1989]
determining probable cause.
Facts:

PEOPLE VS. MUSA On December 1 and 3, 1983, pursuing an Anti-Smut Campaign initiated by the Mayor of the City of
[217 SCRA 597; G.,R. NO. 96177; 27 JAN 1993] Manila, Ramon D. Bagatsing, elements of the Special Anti-Narcotics Group, Auxilliary Services Bureau,
Western Police District, INP of the Metropolitan Police Force of Manila, seized and confiscated from
Facts: dealers, distributors, newsstand owners and peddlers along Manila sidewalks, magazines, publications
and other reading materials believed to be obscene, pornographic and indecent and later burned the
A civilian informer gave the information that Mari Musa was engaged in selling marijuana in Suterville, seized materials in public at the University belt along C.M. Recto Avenue, Manila, in the presence of
Zamboanga City. Sgt. Ani was ordered by NARCOM leader T/Sgt. Belarga, to conduct a surveillance Mayor Bagatsing and several officers and members of various student organizations.
and test buy on Musa. The civilian informer guided Ani to Musa’s house and gave the description of
Musa. Ani was able to buy one newspaper-wrapped dried marijuana for P10.00. Among the publications seized, and later burned, was "Pinoy Playboy" magazines published and co-
edited by plaintiff Leo Pita.
The next day, a buy-bust was planned. Ani was to raise his right hand if he successfully buys
Plaintiff filed a case for injunction with prayer for issuance of the writ of preliminary injunction against
marijuana from Musa. As Ani proceeded to the house, the NARCOM team positioned themselves
Mayor Bagatsing and Narcisco Cabrera, as superintendent of Western Police District of the City of
about 90 to 100 meters away. From his position, Belarga could see what was going on. Musa came
Manila, seeking to enjoin said defendants and their agents from confiscating plaintiff’s magazines or
out of the house and asked Ani what he wanted. Ani said he wanted more marijuana and gave Musa
from preventing the sale or circulation thereof claiming that the magazine is a decent, artistic and
the P20.00 marked money. Musa went into the house and came back, giving Ani two newspaper
educational magazine which is not per se obscene, and that the publication is protected by the
wrappers containing dried marijuana. Ani opened and inspected it. He raised his right hand as a signal
Constitutional guarantees of freedom of speech and of the press. Plaintiff also filed an Urgent Motion
to the other NARCOM agents, and the latter moved in and arrested Musa inside the house. Belarga
for issuance of a temporary restraining order against indiscriminate seizure, confiscation and burning of
frisked Musa in the living room but did not find the marked money (gave it to his wife who slipped
plaintiff's "Pinoy Playboy" Magazines, pending hearing on the petition for preliminary injunction. The
away). T/Sgt. Belarga and Sgt. Lego went to the kitchen and found a ‘cellophane colored white and
Court granted the temporary restraining order. The case was set for trial upon the lapse of the TRO.
stripe hanging at the corner of the kitchen.’ They asked Musa about its contents but failed to get a
RTC ruled that the seizure was valid. This was affirmed by the CA.
response. So they opened it and found dried marijuana leaves inside. Musa was then placed under
arrest.
Issue:
Issue:
Whether or Not the seizure was illegal.
Whether or Not the seizure of the plastic bag and the marijuana inside it is unreasonable, hence,
Held:
inadmissible as evidence.
The Court ruled that the government authorities have not shown the required proof to justify a ban and
Held:
to warrant confiscation of the literature. First of all, they were not possessed of a lawful court order: (1)
finding the said materials to be pornography, and (2) authorizing them to carry out a search and
Yes. It constituted unreasonable search and seizure thus it may not be admitted as evidence. The
seizure, by way of a search warrant. The court provides the following guidelines to be observed:
warrantless search and seizure, as an incident to a suspect’s lawful arrest, may extend beyond the
person of the one arrested to include the premises or surroundings under his immediate control.
1. The authorities must apply for the issuance of a search warrant from a judge, if in their
Objects in the ‘plain view’ of an officer who has the right to be in the position to have that view are
opinion an obscenity seizure is in order;
subject to seizure and may be presented as evidence. The ‘plain view’ doctrine is usually applied where
2. The authorities must convince the court that the materials sought to be seized are obscene
a police officer is not searching for evidence against the accused, but nonetheless inadvertently comes
and pose a clear and present danger of an evil substantive enough to warrant State
across an incriminating object. It will not justify the seizure of the object where the incriminating nature
interference and action;
of the object is not apparent from the ‘plain view’ of the object.

Page 60
3. The judge must determine whether or not the same are indeed obscene. The question is to
be resolved on a case-to-case basis and on the judge’s sound discretion;
4. If in the opinion of the court, probable cause exists, it shall issue the search warrant prayed PEOPLE VS. ARUTA
for; [288 SCRA 626; G.R. NO. 120515; 13 APR 1998]
5. The proper suit is then brought in the court under Article 201 of the RPC (Obscene
publications). Facts:
6. Any conviction is subject to appeal. The appellate court may assess whether or not the
properties seized are indeed obscene. On Dec. 13, 1988, P/Lt. Abello was tipped off by his informant that a certain “Aling Rosa” will be arriving
from Baguio City with a large volume of marijuana and assembled a team. The next day, at the Victory
Liner Bus terminal they waited for the bus coming from Baguio, when the informer pointed out who
GUANZON VS. DE VILLA “Aling Rosa” was, the team approached her and introduced themselves as NARCOM agents. When
[181 SCRA 623; G.R. 80508; 30 JAN 1990] Abello asked “aling Rosa” about the contents of her bag, the latter handed it out to the police. They
found dried marijuana leaves packed in a plastic bag marked “cash katutak”.
Facts:
Instead of presenting its evidence, the defense filed a demurrer to evidence alleging the illegality of the
The 41 petitioners alleged that the "saturation drive" or "aerial target zoning" that were conducted in search and seizure of the items. In her testimony, the accused claimed that she had just come from
their place (Tondo Manila) were unconstitutional. They alleged that there is no specific target house to Choice theatre where she watched a movie “Balweg”. While about to cross the road an old woman
be search and that there is no search warrant or warrant of arrest served. Most of the policemen are in asked her for help in carrying a shoulder bag, when she was later on arrested by the police. She has no
their civilian clothes and without nameplates or identification cards. The residents were rudely rouse knowledge of the identity of the old woman and the woman was nowhere to be found. Also, no search
from their sleep by banging on the walls and windows of their houses. The residents were at the point warrant was presented.
of high-powered guns and herded like cows. Men were ordered to strip down to their briefs for the
police to examine their tattoo marks. The residents complained that they're homes were ransacked, The trial court convicted the accused in violation of the dangerous drugs of 1972
tossing their belongings and destroying their valuables. Some of their money and valuables had
disappeared after the operation. The residents also reported incidents of maulings, spot-beatings and Issue:
maltreatment. Those who were detained also suffered mental and physical torture to extract
confessions and tactical informations. The respondents said that such accusations were all lies. Whether or Not the police correctly searched and seized the drugs from the accused.
Respondents contends that the Constitution grants to government the power to seek and cripple
subversive movements for the maintenance of peace in the state. The aerial target zoning were Held:
intended to flush out subversives and criminal elements coddled by the communities were the said
drives were conducted. They said that they have intelligently and carefully planned months ahead for The following cases are specifically provided or allowed by law:
the actual operation and that local and foreign media joined the operation to witness and record such
event. 1. Warrantless search incidental to a lawful arrest recognized under Section 12, Rule 126
of the Rules of Court 8 and by prevailing jurisprudence
Issue: 2. Seizure of evidence in "plain view," the elements of which are: (a) a prior valid intrusion
based on the valid warrantless arrest in which the police are legally present in the
Whether or Not the saturation drive committed consisted of violation of human rights. pursuit of their official duties; (b) the evidence was inadvertently discovered by the
police who had the right to be where they are; (c) the evidence must be immediately
Held: apparent, and (d) "plain view" justified mere seizure of evidence without further search;
3. Search of a moving vehicle. Highly regulated by the government, the vehicle's inherent
It is not the police action per se which should be prohibited rather it is the procedure used or the mobility reduces expectation of privacy especially when its transit in public
methods which "offend even hardened sensibilities" .Based on the facts stated by the parties, it appears thoroughfares furnishes a highly reasonable suspicion amounting to probable cause that
to have been no impediment to securing search warrants or warrants of arrest before any houses were the occupant committed a criminal activity;
searched or individuals roused from sleep were arrested. There is no showing that the objectives 4. Consented warrantless search;
sought to be attained by the "aerial zoning" could not be achieved even as th rights of the squatters and 5. Customs search;
low income families are fully protected. However, the remedy should not be brought by a tazpaer suit 6. Stop and Frisk;
where not one victim complaints and not one violator is properly charged. In the circumstances of this 7. Exigent and Emergency Circumstances.
taxpayers' suit, there is no erring soldier or policeman whom the court can order prosecuted. In the
absence of clear facts no permanent relief can be given. The essential requisite of probable cause must still be satisfied before a warrantless search and seizure
can be lawfully conducted.
In the meantime where there is showing that some abuses were committed, the court temporary
restraint the alleged violations which are shocking to the senses. Petition is remanded to the RTC of The accused cannot be said to be committing a crime, she was merely crossing the street and was not
Manila. acting suspiciously for the Narcom agents to conclude that she was committing a crime. There was no
Page 61
legal basis to effect a warrantless arrest of the accused’s bag, there was no probable cause and the judge shall ascertain whether section 11 of this Rule has been complained with and shall
accused was not lawfully arrested. require that the property seized be delivered to him. The judge shall see to it that subsection
(a) hereof has been complied with.
The police had more than 24 hours to procure a search warrant and they did not do so. The seized (c) The return on the search warrant shall be filed and kept by the custodian of the log book on
marijuana was illegal and inadmissible evidence. search warrants who shall enter therein the date of the return, the result, and other actions of
the judge.

RULE 113, RULES OF COURT A violation of this section shall constitute contempt of court.

Section 5. Arrest without warrant; when lawful. — A peace officer or a private person may, without a
warrant, arrest a person:

(a) When, in his presence, the person to be arrested has committed, is actually committing,
or is attempting to commit an offense;
(b) When an offense has just been committed, and he has probable cause to believe based
on personal knowledge of facts or circumstances that the person to be arrested has
committed it; and
(c) When the person to be arrested is a prisoner who has escaped from a penal
establishment or place where he is serving final judgment or is temporarily confined
while his case is pending, or has escaped while being transferred from one confinement
to another.

In cases falling under paragraph (a) and (b) above, the person arrested without a warrant shall be
forthwith delivered to the nearest police station or jail and shall be proceeded against in accordance
with section 7 of Rule 112.

RULE 126, RULES OF COURT

Section 2. Court where application for search warrant shall be filed. — An application for search
warrant shall be filed with the following:

a) Any court within whose territorial jurisdiction a crime was committed.


b) For compelling reasons stated in the application, any court within the judicial region where the
crime was committed if the place of the commission of the crime is known, or any court within the
judicial region where the warrant shall be enforced.

However, if the criminal action has already been filed, the application shall only be made in the court
where the criminal action is pending.

Section 7. Right to break door or window to effect search. — The officer, if refused admittance to the
place of directed search after giving notice of his purpose and authority, may break open any outer or
inner door or window of a house or any part of a house or anything therein to execute the warrant or
liberate himself or any person lawfully aiding him when unlawfully detained therein.

Section 12. Delivery of property and inventory thereof to court; return and proceedings thereon.

(a) The officer must forthwith deliver the property seized to the judge who issued the warrant,
together with a true inventory thereof duly verified under oath.
(b) Ten (10) days after issuance of the search warrant, the issuing judge shall ascertain if the
return has been made, and if none, shall summon the person to whom the warrant was
issued and require him to explain why no return was made. If the return has been made, the
Page 62
Petitioner was charged with estafa. He posted bail. Petitioner filed before each of the trial courts a
LIBERTY OF ABODE AND OF TRAVEL motion entitled, "motion for permission to leave the country," stating as ground therefor his desire to go
to the United States, "relative to his business transactions and opportunities." The prosecution opposed
said motion and after due hearing, both trial judges denied the same. Petitioner thus filed a petition for
Art 3, Sec. 6. “The liberty of abode and of changing the same within the limits prescribed by law certiorari and mandamus before the then Court of Appeals seeking to annul the orders dated March 9
shall not be impaired except upon lawful order of the court. Neither shall the right to travel be impaired and 26, 1982, of Judges Camilon and Pronove, respectively, as well as the communication-request of
except in the interest of national security, public safety, or public health, as may be provided by law.” the Securities and Exchange Commission, denying his leave to travel abroad. He likewise prayed for
the issuance of the appropriate writ commanding the Immigration Commissioner and the Chief of the
Aviation Security Command (AVSECOM) to clear him for departure. The Court of Appeals denied the
CAUNCA VS. SALAZAR petition.
[82 PHIL 851; NO.L-2690; 1 JAN 1949]
Petitioner contends that having been admitted to bail as a matter of right, neither the courts which
Facts: granted him bail nor the Securities and Exchange Commission which has no jurisdiction over his liberty
could prevent him from exercising his constitutional right to travel.
This is an action for habeas corpus brought by Bartolome Caunca in behalf of his cousin Estelita Flores
who was employed by the Far Eastern Employment Bureau, owned by Julia Salazar, respondent Issue:
herein. An advanced payment has already been given to Estelita by the employment agency, for her to
work as a maid. However, Estelita wanted to transfer to another residence, which was disallowed by the Whether or Not his constitutional right to travel has been violated.
employment agency. Further she was detained and her liberty was restrained. The employment agency
wanted that the advance payment, which was applied to her transportation expense from the province Held:
should be paid by Estelita before she could be allowed to leave.
A court has the power to prohibit a person admitted to bail from leaving the Philippines. This is a
necessary consequence of the nature and function of a bail bond. The condition imposed upon
Issue: petitioner to make himself available at all times whenever the court requires his presence operates as a
valid restriction on his right to travel. Indeed, if the accused were allowed to leave the Philippines
Whether or Not an employment agency has the right to restrain and detain a maid without returning the without sufficient reason, he may be placed beyond the reach of the courts. Petitioner has not shown
advance payment it gave? the necessity for his travel abroad. There is no indication that the business transactions cannot be
undertaken by any other person in his behalf.
Held:

An employment agency, regardless of the amount it may advance to a prospective employee or maid, MARCOS VS. MANGLAPUS
has absolutely no power to curtail her freedom of movement. The fact that no physical force has been [177 SCRA 668; G.R. NO. 88211; 15 SEPT 1989]
exerted to keep her in the house of the respondent does not make less real the deprivation of her
personal freedom of movement, freedom to transfer from one place to another, freedom to choose Facts:
one’s residence. Freedom may be lost due to external moral compulsion, to founded or groundless
fear, to erroneous belief in the existence of an imaginary power of an impostor to cause harm if not This case involves a petition of mandamus and prohibition asking the court to order the respondents
blindly obeyed, to any other psychological element that may curtail the mental faculty of choice or the Secretary of Foreign Affairs, etc. To issue a travel documents to former Pres. Marcos and the
unhampered exercise of the will. If the actual effect of such psychological spell is to place a person at immediate members of his family and to enjoin the implementation of the President's decision to bar
the mercy of another, the victim is entitled to the protection of courts of justice as much as the individual their return to the Philippines. Petitioners assert that the right of the Marcoses to return in the
who is illegally deprived of liberty by duress or physical coercion. Philippines is guaranteed by the Bill of Rights, specifically Sections 1 and 6. They contended that Pres.
Aquino is without power to impair the liberty of abode of the Marcoses because only a court may do so
within the limits prescribed by law. Nor the President impair their right to travel because no law has
authorized her to do so.

They further assert that under international law, their right to return to the Philippines is guaranteed
particularly by the Universal Declaration of Human Rights and the International Covenant on Civil and
Political Rights, which has been ratified by the Philippines.
MANOTOC VS. COURT OF APPEALS
[142 SCRA 149; G.R. NO. L-62100; 30 MAY 1986] Issue:

Facts: Whether or not, in the exercise of the powers granted by the constitution, the President (Aquino) may
prohibit the Marcoses from returning to the Philippines.
Page 63
imposed that the accused must make himself available whenever the court requires his presence. A
Held: person facing criminal charges may be restrained by the Court from leaving the country or, if abroad,
compelled to return (Constitutional Law, Cruz, Isagani A., 1987 Edition, p. 138). So it is also that "An
"It must be emphasized that the individual right involved is not the right to travel from the Philippines to accused released on bail may be re-arrested without the necessity of a warrant if he attempts to depart
other countries or within the Philippines. These are what the right to travel would normally connote. from the Philippines without prior permission of the Court where the case is pending (ibid., Sec. 20 [2nd
Essentially, the right involved in this case at bar is the right to return to one's country, a distinct right par. ]).
under international law, independent from although related to the right to travel. Thus, the Universal
Declaration of Human Rights and the International Covenant on Civil and Political Rights treat the right Article III, Section 6 of the 1987 Constitution should be interpreted to mean that while the liberty of
to freedom of movement and abode within the territory of a state, the right to leave the country, and the travel may be impaired even without Court Order, the appropriate executive officers or administrative
right to enter one's country as separate and distinct rights. What the Declaration speaks of is the "right authorities are not armed with arbitrary discretion to impose limitations. They can impose limits only on
to freedom of movement and residence within the borders of each state". On the other hand, the the basis of "national security, public safety, or public health" and "as may be provided by law," a
Covenant guarantees the right to liberty of movement and freedom to choose his residence and the limitive phrase which did not appear in the 1973 text (The Constitution, Bernas, Joaquin G.,S.J., Vol. I,
right to be free to leave any country, including his own. Such rights may only be restricted by laws First Edition, 1987, p. 263). Apparently, the phraseology in the 1987 Constitution was a reaction to the
protecting the national security, public order, public health or morals or the separate rights of others. ban on international travel imposed under the previous regime when there was a Travel Processing
However, right to enter one's country cannot be arbitrarily deprived. It would be therefore inappropriate Center, which issued certificates of eligibility to travel upon application of an interested party (See
to construe the limitations to the right to return to ones country in the same context as those pertaining Salonga vs. Hermoso & Travel Processing Center, No. 53622, 25 April 1980, 97 SCRA 121).
to the liberty of abode and the right to travel.
Holding an accused in a criminal case within the reach of the Courts by preventing his departure from
The Bill of rights treats only the liberty of abode and the right to travel, but it is a well considered view the Philippines must be considered as a valid restriction on his right to travel so that he may be dealt
that the right to return may be considered, as a generally accepted principle of International Law and with in accordance with law. The offended party in any criminal proceeding is the People of the
under our Constitution as part of the law of the land. Philippines. It is to their best interest that criminal prosecutions should run their course and proceed to
finality without undue delay, with an accused holding himself amenable at all times to Court Orders and
The court held that President did not act arbitrarily or with grave abuse of discretion in determining that processes
the return of the Former Pres. Marcos and his family poses a serious threat to national interest and
welfare. President Aquino has determined that the destabilization caused by the return of the
Marcoses would wipe away the gains achieved during the past few years after the Marcos regime.
DEFENSOR-SANTIAGO VS. VASQUEZ
The return of the Marcoses poses a serious threat and therefore prohibiting their return to the [217 SCRA 633; G.R. NOS. 99289-90; 27 JAN 1993]
Philippines, the instant petition is hereby DISMISSED.
Facts:

An information was filed against petitioner with the Sandiganbayan for violation of the Anti Graft and
Corrupt Practices Act. The order of arrest was issued with bail for release fixed at Php. 15,000 so she
SILVERIO VS. COURT OF APPEALS filed a motion for acceptance of cash bail bond. On the same day the Sandiganbayan issued a
[195 SCRA 760 ; G.R. 94284; 8 APR 1991] resolution authorizing the petitioner to post cash bond which the later filed in the amount of Php.15,
000. Her arraignment was set, but petitioner asked for the cancellation of her bail bond and that she be
Facts: allowed provisional release on recognizance. The Sandiganbayan deferred it. The Sandiganbayan
issued a hold departure order against petitioner, by reason of the announcement she made that she
Petitioner was charged with violation of Section 2 (4) of the revised securities act. Respondent filed to would be leaving for the U.S. to accept a fellowship a Harvard. In the instant motion she submitted
cancel the passport of the petitioner and to issue a hold departure order. The RTC ordered the DFA to before the S.C. she argues that her right to travel is impaired.
cancel petitioner’s passport, based on the finding that the petitioner has not been arraigned and there
was evidence to show that the accused has left the country with out the knowledge and the permission Issue:
of the court.
Whether or Not the petitioner’s right to travel is impaired.
Issue:
Held:
Whether or Not the right to travel may be impaired by order of the court.
The petitioner does not deny and as a matter of fact even made a public statement, that she he every
intension of leaving the country to pursue higher studies abroad. The court upholds the course of action
Held:
of the Sandiganbayan in taking judicial notice of such fact of petitioners pal to go abroad and in
thereafter issuing a sua sponte the hold departure order is but an exercise of respondent court’s
The bail bond posted by petitioner has been cancelled and warrant of arrest has been issued by
reason that he failed to appear at his arraignments. There is a valid restriction on the right to travel, it is
Page 64
inherent power to preserve and to maintain effectiveness of its jurisdiction over the case and the person expert opinion because petitioner's motion was based on the advice of her physician. The court could
of the accused. not be expected to just accept the opinion of petitioner's physician in resolving her request for
permission to travel. What would be objectionable would be if respondent court obtained information
Also, the petitioner assumed obligations, when she posted bail bond. She holds herself amenable at all without disclosing its source to the parties and used it in deciding a case against them.
times to the orders and process of eth court. She may legally be prohibited from leaving the country
during the pendency of the case. (Manotoc v. C.A.) In disregarding the medical reports, the petitioner failed to prove the necessity for a trip abroad. It
should be emphasized that considering the fact that she is facing charges before the courts in several
cases, in two of which she was convicted although the decision is still pending reconsideration,
MARCOS VS. SANDIGANBAYAN petitioner did not have an absolute right to leave the country and the burden was on her to prove that
[247 SCRA 127; G.R. NO. 115132-34; 9 AUG 1995] because of danger to health if not to her life there was necessity to seek medical treatment in foreign
countries.
Facts:
On the third issue, the Court ordered petitioner to undergo several tests which summarily states that the
This is a petition for certiorari to set aside as arbitrary and in grave abuse of discretion resolutions of the required medical treatment was available here in the Philippines and that the expertise and facilities
Sandiganbayan's First Division denying petitioner's motion for leave to travel abroad for medical here were more than adequate to cater to her medical treatment. The heart ailments of the petitioner
treatment. were not as severe as that was reported by Dr. Anastacio.

The former first lady Imelda Marcos was found guilty by the First Division of the Sandiganbayan of Wherefore, the petitioner is Dismissed without prejudice to the filling of another motion for leave to
violating 3 of the Anti Graft and Corrupt Practices Act. After conviction she filed a "Motion for Leave to travel abroad, should petitioner still desire, based on her heart condition. In such an event the
Travel Abroad" to seek diagnostic tests and treatment by practitioners of oriental medicine in China determination of her medical condition should be made by joint panel of medical specialists
allegedly because of "a serious and life threatening medical condition" requiring facilities not available recommended by both the accused and the prosecution.
in the Philippines that was denied. Then she again filed an "Urgent Ex-Parte Motion for Permission to
Travel Abroad" to undergo diagnosis and treatment in China. This was supported by several medical
reports that were prepared by her doctor Roberto Anastacio. RUBI VS. PROVINCIAL BOARD OF MINDORO
[39 PHIL 660; NO. 14078; 7 MAR 1919]
Again another Motion to leave was filed by Mrs. Marcos to US and Europe for treatment of several
Facts:
Heart diseases alleging that the tests were not available here.
The provincial board of Mindoro adopted resolution No. 25 wherein non-Christian inhabitants
The presiding justice, Garchitorena, contacted Dr. Gregorio B. Patacsil, Officer-in-Charge of the (uncivilized tribes) will be directed to take up their habitation on sites on unoccupied public lands. It is
Philippine Heart Center, and later wrote him a letter, asking for "expert opinion on coronary medicine". resolved that under section 2077 of the Administrative Code, 800 hectares of public land in the sitio of
The court still found no merit to allow the petitioners motion to leave and denied all of the motions. Tigbao on Naujan Lake be selected as a site for the permanent settlement of Mangyanes in Mindoro.
Further, Mangyans may only solicit homesteads on this reservation providing that said homestead
Petitioner filed a motion for reconsideration and a "Motion to Admit Clinical Summary and to Resolve applications are previously recommended by the provincial governor.
Motion for Reconsideration." Attached was a recent medical report and letters of Vice President Joseph
E. Estrada offering to be guarantor for the return of petitioner and those of twenty four members of the In that case, pursuant to Section 2145 of the Revised Administrative Code, all the Mangyans in the
House of Representatives requesting the court to allow petitioner to travel abroad. This was also denied townships of Naujan and Pola and the Mangyans east of the Baco River including those in the districts
by the Court also stating their express disapproval of the involvement of the VP and the Cabinet of Dulangan and Rubi's place in Calapan, were ordered to take up their habitation on the site of Tigbao,
members so as to influence the resolutions, decisions or orders or any judicial action of respondent Naujan Lake. Also, that any Mangyan who shall refuse to comply with this order shall upon conviction
court. be imprisoned not exceed in sixty days, in accordance with section 2759 of the revised Administrative
Code.
Issue:
Said resolution of the provincial board of Mindoro were claimed as necessary measures for the
Whether or Not the Sandiganbayan erred in disallowing the Motion for Leave to Travel Abroad because protection of the Mangyanes of Mindoro as well as the protection of public forests in which they roam,
it (1) disregarded the medical findings (2) it motu propio contacted a third party asking the latter to give and to introduce civilized customs among them.
an opinion on petitioner's motion and medical findings (3) said that there was no necessity to get
medical treatment abroad. It appeared that Rubi and those living in his rancheria have not fixed their dwelling within the
reservation of Tigbao and are liable to be punished.
Held:
It is alleged that the Manguianes are being illegally deprived of their liberty by the provincial officials of
that province. Rubi and his companions are said to be held on the reservation established at Tigbao,
No. The contention of the petitioner that was invalid to contact a third party asking the latter to give an
opinion on petitioner's motion and medical findings was erroneous. Respondent court had to seek
Page 65
Mindoro, against their will, and one Dabalos is said to be held under the custody of the provincial sheriff
in the prison at Calapan for having run away form the reservation.

Issue:

Whether or Not Section 2145 of the Administrative Code deprive a person of his liberty pf abode. Thus,
WON Section 2145 of the Administrative Code of 1917 is constitutional.

Held:

The Court held that section 2145 of the Administrative Code does not deprive a person of his liberty of
abode and does not deny to him the equal protection of the laws, and that confinement in reservations
in accordance with said section does not constitute slavery and involuntary servitude. The Court is
further of the opinion that section 2145 of the Administrative Code is a legitimate exertion of the police
power. Section 2145 of the Administrative Code of 1917 is constitutional.
Assigned as reasons for the action: (1) attempts for the advancement of the non-Christian people of the
province; and (2) the only successfully method for educating the Manguianes was to oblige them to live
in a permanent settlement. The Solicitor-General adds the following; (3) The protection of the
Manguianes; (4) the protection of the public forests in which they roam; (5) the necessity of introducing
civilized customs among the Manguianes.

One cannot hold that the liberty of the citizen is unduly interfered without when the degree of civilization
of the Manguianes is considered. They are restrained for their own good and the general good of the
Philippines.

“Liberty regulated by law": Implied in the term is restraint by law for the good of the individual and for
the greater good of the peace and order of society and the general well-being. No man can do exactly
as he pleases.

None of the rights of the citizen can be taken away except by due process of law.

Therefore, petitioners are not unlawfully imprisoned or restrained of their liberty. Habeas corpus can,
therefore, not issue.

Page 66
FREEDOM OF RELIGION a. Resolution No. 5- Reviving the traditional socio-religious celebration every fifth of April.
This provided for the acquisition of the image of San Vicente Ferrer and the construction
of a waiting shed. Funds for the said projects will be obtained through the selling of
Art 3, Sec. 5. “No law shall be made respecting an establishment of religion, or prohibiting the tickets and cash donations.
free exercise thereof. The free exercise and enjoyment of religious profession and worship, without b. Resolution No. 6- The chairman or hermano mayor of the fiesta would be the caretaker
discrimination or preference, shall forever be allowed. No religious test shall be required for the of the image of San Vicente Ferrer and that the image would remain in his residence for
exercise of civil or political rights.” one year and until the election of his successor. The image would be made available to
the Catholic Church during the celebration of the saint’s feast day.

AGLIPAY VS. RUIZ These resolutions have been ratified by 272 voters, and said projects were implemented. The image
[64 PHIL 201; G.R. NO. 45459; 13 MAR 1937] was temporarily placed in the altar of the Catholic Church of the barangay. However, after a mass,
Father Sergio Marilao Osmeña refused to return the image to the barangay council, as it was the
Facts: church’s property since church funds were used in its acquisition.

Petitioner seeks the issuance of a writ of prohibition against respondent Director of Posts from issuing Resolution No. 10 was passed for the authorization of hiring a lawyer for the replevin case against the
and selling postage stamps commemorative of the 33 rd International Eucharistic Congress. Petitioner priest for the recovery of the image. Resolution No. 12 appointed Brgy. Captain Veloso as a
contends that such act is a violation of the Constitutional provision stating that no public funds shall be representative to the case. The priest, in his answer assailed the constitutionality of the said
appropriated or used in the benefit of any church, system of religion, etc. This provision is a result of resolutions. The priest with Andres Garces, a member of the Aglipayan Church, contends that Sec. 8
the principle of the separation of church and state, for the purpose of avoiding the occasion wherein the Article IV1 and Sec 18(2) Article VIII) 2 of the constitution was violated.
state will use the church, or vice versa, as a weapon to further their ends and aims. Respondent
contends that such issuance is in accordance to Act No. 4052, providing for the appropriation funds to Issue:
respondent for the production and issuance of postage stamps as would be advantageous to the
government. Whether or Not any freedom of religion clause in the Constitution violated.

Issue: Held:

Whether or Not there was a violation of the freedom to religion. No. As said by the Court this case is a petty quarrel over the custody of the image. The image was
purchased in connection with the celebration of the barrio fiesta and not for the purpose of favoring any
Held: religion nor interfering with religious matters or beliefs of the barrio residents. Any activity intended to
facilitate the worship of the patron saint(such as the acquisition) is not illegal. Practically, the image was
What is guaranteed by our Constitution is religious freedom and not mere religious toleration. It is placed in a layman’s custody so that it could easily be made available to any family desiring to borrow
however not an inhibition of profound reverence for religion and is not a denial of its influence in human the image in connection with prayers and novena. It was the council’s funds that were used to buy the
affairs. Religion as a profession of faith to an active power that binds and elevates man to his Creator image, therefore it is their property. Right of the determination of custody is their right, and even if they
is recognized. And in so far as it instills into the minds the purest principles of morality, its influence is decided to give it to the Church, there is no violation of the Constitution, since private funds were used.
deeply felt and highly appreciated. The phrase in Act No. 4052 “advantageous to the government” Not every government activity which involves the expenditure of public funds and which has some
does not authorize violation of the Constitution. The issuance of the stamps was not inspired by any religious tint is violative of the constitutional provisions regarding separation of church and state,
feeling to favor a particular church or religious denomination. They were not sold for the benefit of the freedom of worship and banning the use of public money or property.
Roman Catholic Church. The postage stamps, instead of showing a Catholic chalice as originally
planned, contains a map of the Philippines and the location of Manila, with the words “Seat XXXIII AMERICAN BIBLE SOCIETY VS. CITY OF MANILA
International Eucharistic Congress.” The focus of the stamps was not the Eucharistic Congress but the [101PHIL 386; G.R. NO. 9637; 30 APR 1957]
city of Manila, being the seat of that congress. This was to “to advertise the Philippines and attract
more tourists,” the officials merely took advantage of an event considered of international importance. Facts:
Although such issuance and sale may be inseparably linked with the Roman Catholic Church, any
benefit and propaganda incidentally resulting from it was no the aim or purpose of the Government. New York’s Education Law requires local public school authorities to lend textbooks free of charge to all
students in grade 7 to 12, including those in private schools. The Board of Education contended that
GARCES VS. ESTENZO said statute was invalid and violative of the State and Federal Constitutions. An order barring the
[104 SCRA 510; G.R. L-53487; 25 MAY 1981] Commissioner of Education (Allen) from removing appellant’s members from office for failure to comply
with the requirement and an order preventing the use of state funds for the purchase of textbooks to be
Facts: lent to parochial schools were sought for. The trial court held the statute unconstitutional. The Appellate
Division reversed the decision and dismissed the complaint since the appellant have no standing. The
Two resolutions of the Barangay Council of Valencia, Ormoc City were passed: New York Court of Appeals, ruled that the appellants have standing but the law is not unconstitutional.

Page 67
Issue: President and his family, government officials and diplomatic and foreign guests transacting business
with Malacanang. The restriction was also intended to secure the executive offices within the
Whether or Not the said ordinances are constitutional and valid (contention: it restrains the free Malacanang grounds from possible external attacks and disturbances. (Minority opinion) The sole
exercise and enjoyment of the religious profession and worship of appellant). justification for a prior restraint or limitation on the exercise of the freedom of religion is the existence of
a grave and imminent, of a serious evil to public safety, public morals, public health or any other
Held: legitimate public interest that the State has a right to prevent. The burden to show the existence of
grave and imminent danger lies on the officials who would restrain petitioners. Respondents were in full
Section 1, subsection (7) of Article III of the Constitution, provides that: control and had the capability to stop any untoward move. There was no clear and present danger of
any serious evil to public safety or the security of Malacanang.
(7) No law shall be made respecting an establishment of religion, or prohibiting the free exercise
thereof, and the free exercise and enjoyment of religious profession and worship, without
discrimination or preference, shall forever be allowed. No religion test shall be required for the EBRALINAG VS. DIVISION SUPERINTENDENT OF CEBU
exercise of civil or political rights. [219 SCRA 256 ; G.R. NO. 95770; 1 MAR 1993]

The provision aforequoted is a constitutional guaranty of the free exercise and enjoyment of religious Facts:
profession and worship, which carries with it the right to disseminate religious information.
Two special civil actions for certiorari, Mandamus and Prohibition were filed and consolidated for raising
It may be true that in the case at bar the price asked for the bibles and other religious pamphlets was in same issue. Petitioners allege that the public respondents acted without or in excess of their
some instances a little bit higher than the actual cost of the same but this cannot mean that appellant jurisdiction and with grave abuse of discretion. Respondents ordered expulsion of 68 HS and GS
was engaged in the business or occupation of selling said "merchandise" for profit. For this reason. The students of Bantayan, Pinamungajan, Caracar, Taburan and Asturias in Cebu. Public school
Court believe that the provisions of City of Manila Ordinance No. 2529, as amended, cannot be applied authorities expelled these students for refusing to salute the flag, sing the national anthem and recite
to appellant, for in doing so it would impair its free exercise and enjoyment of its religious profession the “Panatang Makabayan” required by RA1265. They are Jehovah’s Witnesses believing that by
and worship as well as its rights of dissemination of religious beliefs. doing these is religious worship/devotion akin to idolatry against their teachings. They contend that to
compel transcends constitutional limits and invades protection against official control and religious
With respect to Ordinance No. 3000, as amended, the Court do not find that it imposes any charge freedom. The respondents relied on the precedence of Gerona et al v. Secretary of Education.
upon the enjoyment of a right granted by the Constitution, nor tax the exercise of religious practices. Gerona doctrine provides that we are a system of separation of the church and state and the flag is
devoid of religious significance and it doesn’t involve any religious ceremony. The freedom of religious
It seems clear, therefore, that Ordinance No. 3000 cannot be considered unconstitutional, however belief guaranteed by the Constitution does not mean exception from non-discriminatory laws like the
inapplicable to said business, trade or occupation of the plaintiff. As to Ordinance No. 2529 of the City saluting of flag and singing national anthem. This exemption disrupts school discipline and demoralizes
of Manila, as amended, is also not applicable, so defendant is powerless to license or tax the business the teachings of civic consciousness and duties of citizenship.
of plaintiff Society.
WHEREFORE, defendant shall return to plaintiff the sum of P5,891.45 unduly collected from it. Issue:

Whether or Not religious freedom has been violated.


GERMAN VS. BARANGAN
[135 SCRA 514; G.R. NO. 68828; 27 MAR 1985] Held:

Facts: Religious freedom is a fundamental right of highest priority. The 2 fold aspect of right to religious
worship is: 1.) Freedom to believe which is an absolute act within the realm of thought. 2.) Freedom to
Petitioners converged at J.P. Laurel Street to hear Mass at the St. Jude Chapel, which adjoined act on one’s belief regulated and translated to external acts. The only limitation to religious freedom is
Malacañang. Respondent barred them for security reasons. Petitioners filed a petition for mandamus. the existence of grave and present danger to public safety, morals, health and interests where State
has right to prevent. The expulsion of the petitioners from the school is not justified.
Issue:
The 30 yr old previous GERONA decision of expelling and dismissing students and teachers who
Whether or Not there was a violation of the constitutional freedom. refuse to obey RA1265 is violates exercise of freedom of speech and religious profession and worship.
Jehovah’s Witnesses may be exempted from observing the flag ceremony but this right does not give
Held: them the right to disrupt such ceremonies. In the case at bar, the Students expelled were only standing
quietly during ceremonies. By observing the ceremonies quietly, it doesn’t present any danger so evil
Petitioners' intention was not really to perform an act of religious worship but to conduct an anti- and imminent to justify their expulsion. What the petitioner’s request is exemption from flag ceremonies
government demonstration since they wore yellow T-shirts, raised their clenched fists and shouted anti- and not exclusion from public schools. The expulsion of the students by reason of their religious beliefs
government slogans. While every citizen has the right to religious freedom, the exercise must be done is also a violation of a citizen’s right to free education. The non-observance of the flag ceremony does
in good faith. Besides, the restriction was reasonable as it was designed to protect the lives of the not totally constitute ignorance of patriotism and civic consciousness. Love for country and admiration
Page 68
for national heroes, civic consciousness and form of government are part of the school curricula. national funds, or contractors for public works of the municipality." The respondent Judge, in sustaiing
Therefore, expulsion due to religious beliefs is unjustified. Fr. Gonzaga’s right to the office, ruled that the provision had already been impliedly repealed by the
Election Code of 1971. Petitioner on the other hand argues that there was no implied repeal.
Petition for Certiorari and Prohibition is GRANTED. Expulsion is ANNULLED.
Issue:

FONACIER VS. COURT OF APPEALS Whether or Not Fr. Gonzaga is eligible for the position of municipal mayor, according to law.
[96 PHIL 417; G.R. L-5917; 28 JAN 1955]

Facts: Whether or Not the prohibition regarding elected or appointed ecclesiastics is constitutional.

Case was filed by Iglesia Filipina Independiente (IFI), represented by its supreme bishop Gerardo Held:
Bayaca, against Bishop Fonacier seeking to render an accounting of his administration of all the
temporal properties and to recover the same on the ground that he ceased to be the supreme bishop of The court was divided. Five voted that the prohibition was not unconstitutional. Seven others voted that
IFI. Isabelo De los Reyes Jr. had been elected as the Supreme Bishop. the provision was impliedly repealed. However, the minority vote overruled the seven. According to the
dissenting seven, there are three reasons for the said provision to be inoperative. First, the 1935
Petitioner claims that he was not properly removed as Supreme Bishop and his legal successor was Constitution stated, “No religious test shall be required for the exercise of civil or political rights.”
Juan Jamias. He claims that the there was an accounting of his administration and was turned over to Second, said section 2175 is superseded by the Constitution. Third, section 2175 has been repealed by
bishop Jamias. Also, that Isabelo De los Reyes and Bayaca have abandoned their faith and formally Sec. 23 of the Election Code (1971): “Appointive public office holders and active members of the Armed
joined the Prostestant Episcopal Church of America. Forces are no longer disqualified from running for an elective office”. Ecclesiastics were no longer
included in the enumeration of persons ineligible under the said Election Code. On the other hand, the
CFI rendered judgment declaring Isabelo De Los Reyes, Jr. as the sole and legitimate Supreme Bishop controlling five argued: Section 2175 of the Administrative Code deals with a matter different from that
of IFI and ordered Fonacier to render an accounting of his admistration of section 23 of the Election Code. Also, section 2175 of the Administrative Code did not violate the
CA affirmed the decision of the CFI right to freedom of religion because it did not give any requirement for a religious test.

Issue: The view of the dissenting seven failed to obtain a vote of eight members, so it was not controlling. The
provision of the Administrative Code remained operative.
Whether or not the petitioner should still be regarded as the legitimate supreme bishop of IFI.

Held: ESTRADA VS. ESCRITOR


[492 SCRA 1 ; AM NO P-02-1651; 22 JUN 2006]
Supreme Court affirmed CA’s decision. The legitimate Supreme Bishop of IFI is Isabelo De los Reyes,
Jr. The Supreme Court affirms the validity of the election of Bishop Delos Reyes as the Supreme Facts:
Bishop based on their internal laws
Escritor is a court interpreter since 1999 in the RTC of Las Pinas City. She has been living with
To finally dispose of the property issue, the Court, citing Watson v. Jones,368 declared that the rule in Quilapio, a man who is not her husband, for more than twenty five years and had a son with him as
property controversies within religious congregations strictly independent of any other superior well. Respondent’s husband died a year before she entered into the judiciary while Quilapio is still
ecclesiastical association (such as the Philippine Independent Church) is that the rules for resolving legally married to another woman.
such controversies should be those of any voluntary association. If the congregation adopts the
majority rule then the majority should prevail; if it adopts adherence to duly constituted authorities within Complainant Estrada requested the Judge of said RTC to investigate respondent. According to
the congregation, then that should be followed. complainant, respondent should not be allowed to remain employed therein for it will appear as if the
court allows such act.

PAMIL VS. TELECOM Respondent claims that their conjugal arrangement is permitted by her religion—the Jehovah’s
[86 SCRA 413; G.R. 34854; 20 NOV 1978] Witnesses and the Watch Tower and the Bible Trace Society. They allegedly have a ‘Declaration of
Pledging Faithfulness’ under the approval of their congregation. Such a declaration is effective when
Facts: legal impediments render it impossible for a couple to legalize their union.

Fr. Margarito Gonzaga was elected as Municipal Mayor in Alburquerque, Bohol. Petitioner, also an Issue:
aspirant for said office, then filed a suit for quo warranto for Gonzaga’s disqualification based on the
Administrative Code provision: “In no case shall there be elected or appointed to a municipal office Whether or Not the State could penalize respondent for such conjugal arrangement.
ecclesiastics, soldiers in active service, persons receiving salaries or compensation from provincial or
Page 69
Whether or Not EO violates the constitutional provision on the separation of Church and State.
Held:
Held:
No. The State could not penalize respondent for she is exercising her right to freedom of religion. The
free exercise of religion is specifically articulated as one of the fundamental rights in our Constitution. It is unconstitutional for the government to formulate policies and guidelines on the halal certification
As Jefferson put it, it is the most inalienable and sacred of human rights. The State’s interest in scheme because said scheme is a function only religious organizations, entity or scholars can lawfully
enforcing its prohibition cannot be merely abstract or symbolic in order to be sufficiently compelling to and validly perform for the Muslims. According to petitioner, a food product becomes halal only after the
outweigh a free exercise claim. In the case at bar, the State has not evinced any concrete interest in performance of Islamic religious ritual and prayer. Thus, only practicing Muslims are qualified to
enforcing the concubinage or bigamy charges against respondent or her partner. Thus the State’s slaughter animals for food. A government agency like herein respondent OMA cannot therefore perform
interest only amounts to the symbolic preservation of an unenforced prohibition. a religious function like certifying qualified food products as halal. Without doubt, classifying a food
product as halal is a religious function because the standards used are drawn from the Qur'an and
Furthermore, a distinction between public and secular morality and religious morality should be kept in Islamic beliefs. By giving OMA the exclusive power to classify food products as halal, EO 46
mind. The jurisdiction of the Court extends only to public and secular morality. encroached on the religious freedom of Muslim organizations like herein petitioner to interpret for
Filipino Muslims what food products are fit for Muslim consumption. Also, by arrogating to itself the task
The Court further states that our Constitution adheres the benevolent neutrality approach that gives of issuing halal certifications, the State has in effect forced Muslims to accept its own interpretation of
room for accommodation of religious exercises as required by the Free Exercise Clause. This the Qur'an and Sunnah on halal food.
benevolent neutrality could allow for accommodation of morality based on religion, provided it does not
offend compelling state interests. Assuming arguendo that the OSG has proved a compelling state In the case at bar, we find no compelling justification for the government to deprive Muslim
interest, it has to further demonstrate that the state has used the least intrusive means possible so that organizations, like herein petitioner, of their religious right to classify a product as halal, even on the
the free exercise is not infringed any more than necessary to achieve the legitimate goal of the state. premise that the health of Muslim Filipinos can be effectively protected by assigning to OMA the
Thus the conjugal arrangement cannot be penalized for it constitutes an exemption to the law based on exclusive power to issue halal certifications. The protection and promotion of the muslim Filipinos' right
her right to freedom of religion. to health are already provided for in existing laws and ministered to by government agencies charged
with ensuring that food products released in the market are fit for human consumption, properly labeled
and safe. Unlike EO 46, these laws do not encroach on the religious freedom of Muslims.
ISLAMIC DA'WAH COUNCIL OF THE PHILIPPINES VS. EXECUTIVE SECRETARY
[405 SCRA 497;GR 153888; 9 JUL 2003]

Facts:

Petitioner Islamic Da'wah Council of the Philippines, Inc. (IDCP) is a corporation that operates under
Department of Social Welfare and Development, a non-governmental organization that extends
voluntary services to the Filipino people, especially to Muslim communities. It claims to be a federation
of national Islamic organizations and an active member of international organizations such as the
Regional Islamic Da'wah Council of Southeast Asia and the Pacific (RISEAP) and The World Assembly
of Muslim Youth. The RISEAP accredited petitioner to issue halal certifications in the Philippines. Thus,
among the functions petitioner carries out is to conduct seminars, orient manufacturers on halal food
and issue halal certifications to qualified products and manufacturers.

Petitioner alleges that, the actual need to certify food products as halal and also due to halal food
producers' request, petitioner formulated in 1995 internal rules and procedures based on the Qur'an
and the Sunnah for the analysis of food, inspection thereof and issuance of halal certifications. In that
same year, petitioner began to issue, for a fee, certifications to qualified products and food
manufacturers. Petitioner even adopted for use on its halal certificates a distinct sign or logo registered
in the Philippine Patent Office.

On 2001, respondent Office of the Executive Secretary issued EO 465 creating the Philippine Halal
Certification Scheme and designating respondent OMA to oversee its implementation. Under the EO,
respondent OMA has the exclusive authority to issue halal certificates and perform other related
regulatory activities.

Issue:
Page 70
FREEDOM OF EXPRESSION The reason for the enactment, as the state court has said, is that prosecutions to enforce penal statutes
for libel do not result in 'efficient repression or suppression of the evils of scandal.' In the present
instance, the proof was that nine editions of the newspaper or periodical in question were published on
Art 3, Sec. 4. “No law shall be passed abridging the freedom of speech, of expression, or of the successive dates, and that they were chiefly devoted to charges against public officers and in relation
press, or the right of the people peaceably to assemble and petition the government for redress of to the prevalence and protection of crime. In such a case, these officers are not left to their ordinary
grievances.” remedy in a suit for libel, or the authorities to a prosecution for criminal libel. The statute not only
operates to suppress the offending newspaper or periodical, but to put the publisher under an effective
Art 3, Sec. 7. “The right of the people to information on matters of public concern shall be censorship.
recognized. Access to official records, and to documents, and papers pertaining to official acts,
transactions, or decisions, as well as to government research data used as basis for policy Every freeman has an undoubted right to lay what sentiments he pleases before the public; to forbid
development, shall be afforded the citizen, subject to such limitations as may be provided by law.” this, is to destroy the freedom of the press; but if he publishes what is improper, mischievous or illegal,
he must take the consequence of his own temerity.
Art 3, Sec. 8. “The right of the people, including those employed in the public and private
sectors, to form unions, associations, or societies for purposes not contrary to law shall not be The liberty of the press was to be unrestrained, but he who used it was to be responsible in case of its
abridged.” abuse.' Public officers, whose character and conduct remain open to debate and free discussion in the
press, find their remedies for false accusations in actions under libel laws providing for redress and
Art 3, Sec. 18. “(1) No person shall be detained solely by reason of his political beliefs and aspirations.” punishment, and not in proceedings to restrain the publication of newspapers and periodicals.

Characterizing the publication as a business, and the business as a nuisance, does not permit an
NEAR VS. MINNESOTA invasion of the constitutional immunity against restraint. Nor can it be said that the constitutional
[283 US 697] freedom from previous restraint is lost because charges are made of derelictions which constitute
crimes.
Facts:
The preliminary freedom, by virtue of the very reason for its existence, does not depend, as this court
A complaint alleged that the defendants, on September 24, 1927, and on eight subsequent dates in has said, on proof of truth.
October and November, 1927, published and circulated editions of “The Saturday Press”(published in
Minneapolis) which were 'largely devoted to malicious, scandalous and defamatory articles'(based on
Session Laws of Minnesota). The articles charged, in substance, provides that a Jewish gangster was GROSJEAN VS. AMERICAN PRESS CO.
in control of gambling, bootlegging, and racketeering in Minneapolis, and that law enforcing officers and [297 US 233]
agencies were not energetically performing their duties. Most of the charges were directed against the
chief of police; he was charged with gross neglect of duty, illicit relations with gangsters, and with Facts:
participation in graft. The county attorney was charged with knowing the existing conditions and with
failure to take adequate measures to remedy them. The mayor was accused of inefficiency and The nine publishers(corporations) who brought the suit publish thirteen newspapers and these thirteen
dereliction. One member of the grand jury was stated to be in sympathy with the gangsters. A special publications are the only ones within the state of Louisiana having each a circulation of more than
grand jury and a special prosecutor were demanded to deal with the situation in general, and, in 20,000 copies per week. The suit assailed Act No. 23 1 of the Louisiana Legislature, as their freedom of
particular, to investigate an attempt to assassinate one Guilford, one of the original defendants, who, it the press was abridged in contravention to the due process clause.
appears from the articles, was shot by gangsters after the first issue of the periodical had been
published. Now defendants challenged the Minnesota statute which provides for the abatement, as a Issue:
public nuisance, of a malicious, scandalous and defamatory news paper, magazine or other periodical.
The District Court ruled against defendants. Hence the appeal. Whether or Not Act 23 unconstitutional.

Issue: Held:

Whether or Not the proceeding authorized by the statute herein constitutes an infringement of the Yes. Freedom of speech and of the press are rights of the same fundamental character, safeguarded
freedom of the press. by the due process of law clause. The word 'liberty' contained in that amendment embraces not only the
right of a person to be free from physical restraint, but the right to be free in the enjoyment of all his
Held: faculties as well.
The Act operates as a restraint in a double sense. First, its effect is to curtail the amount of revenue
Yes. The insistence that the statute is designed to prevent the circulation of scandal which tends to realized from advertising; and, second, its direct tendency is to restrict circulation. This is plain enough
disturb the public peace and to provoke assaults and the commission of crime is unavailing. when we consider that, if it were increased to a high degree, as it could be if valid it well might result in
destroying both advertising and circulation.

Page 71
Judge Cooley has laid down the test to be applied: The evils to be prevented were not the censorship [27 SCRA 835; G.R. L-27833; 18 APR 1969]
of the press merely, but any action of the government by means of which it might prevent such free and
general discussion of public matters as seems absolutely essential to prepare the people for an Facts:
intelligent exercise of their rights as citizens.
RA 4880 which took effect on June 17, 1967, prohibiting the too early nomination of candidates and
The tax here involved is bad not because it takes money from the pockets of the appellees. It is bad limiting the period of election campaign or partisan political activity was challenged on constitutional
because, it is seen to be a deliberate and calculated device in the guise of a tax to limit the circulation grounds. More precisely, the basic liberties of free speech and free press, freedom of assembly and
of information to which the public is entitled in virtue of the constitutional guaranties. A free press freedom of association are invoked to nullify the act. Petitioner Cabigao was, at the time of the filing the
stands as one of the great interpreters between the government and the people. petition, an incumbent councilor in the 4th District of Manila and the Nacionalista Party official candidate
for Vice-Mayor of Manila to which he was subsequently elected on November 11, 1967; petitioner
The form in which the tax is imposed is in itself suspicious. It is not measured or limited by the volume Gonzales, on the other hand, is a private individual, a registered voter in the City of Manila and a
of advertisements. It is measured alone by the extent of the circulation of the publication in which the political leader of his co-petitioner. There was the further allegation that the nomination of a candidate
advertisements are carried, with the plain purpose of penalizing the publishers and curtailing the and the fixing of period of election campaign are matters of political expediency and convenience which
circulation of a selected group of newspapers. only political parties can regulate or curtail by and among themselves through self-restraint or mutual
understanding or agreement and that the regulation and limitation of these political matters invoking the
police power, in the absence of clear and present danger to the state, would render the constitutional
NEW YORK TIMES VS. UNITED STATES rights of petitioners meaningless and without effect. Senator Lorenzo M. Tañada was asked to appear
[403 US 713] as amicus curiae, and elucidated that Act No. 4880 could indeed be looked upon as a limitation on the
preferred rights of speech and press, of assembly and of association. He did justify its enactment
Facts: however under the clear and present danger doctrine, there being the substantive evil of elections,
whether for national or local officials, being debased and degraded by unrestricted campaigning,
The court granted certiorari in the cases in which the United States seeks to enjoin the New York Times excess of partisanship and undue concentration in politics with the loss not only of efficiency in
and the Washington Post from publishing the contents of a classified study entitled "History of U.S. government but of lives as well. The Philippine Bar Association, the Civil Liberties Union, the U.P. Law
Decision-Making Process on Viet Nam Policy." Said articles reveal the workings of government that led Center and the U.P. Women Lawyers' Circle were requested to give their opinions. Respondents
to the Vietnam war. The Government argues that "the authority of the Executive Department to protect contend that the act was based on the police power of the state.
the nation against publication of information whose disclosure would endanger the national security
stems from two interrelated sources: the constitutional power of the President over the conduct of
foreign affairs and his authority as Commander-in-Chief. In such case the Executive Branch seeks Issue:
judicial aid in preventing publication. The court ruled in favor of the newspaper companies hence the
appeal. Whether or Not RA 4880 unconstitutional.

Issue: Held:

Whether or not the freedom of the press was abridged. Yes. As held in Cabansag v. Fernandez there are two tests that may supply an acceptable criterion for
permissible restriction on freedom of speech. These are the “clear and present danger” rule and the
'dangerous tendency' rule. The first, means that the evil consequence of the comment or utterance
Held: must be extremely serious and the degree of imminence extremely high before the utterance can be
punished. The danger to be guarded against is the 'substantive evil' sought to be prevented. It has the
Yes. To find that the President has "inherent power" to halt the publication of news by resort to the advantage of establishing according to the above decision a definite rule in constitutional law. It
courts would wipe out the First Amendment (Bill of Rights) and destroy the fundamental liberty and provides the criterion as to what words may be publicly established. The "dangerous tendency rule" is
security of the very people the Government hopes to make "secure." such that “If the words uttered create a dangerous tendency which the state has a right to prevent, then
such words are punishable.” It is not necessary that some definite or immediate acts of force, violence,
No branch of government could abridge the people's rights granted by the Constitution including the or unlawfulness be advocated. It is sufficient that such acts be advocated in general terms. Nor is it
freedom of the press. The language of the First Amendment support the view that the press must be necessary that the language used be reasonably calculated to incite persons to acts of force, violence,
left free to publish news, whatever the source, without censorship, injunctions, or prior restraints. The or unlawfulness. It is sufficient if the natural tendency and probable effect of the utterance be to bring
press was protected so that it could bare the secrets of government and inform the people. Only a free about the substantive evil which the legislative body seeks to prevent.
and unrestrained press can effectively expose deception in government. And paramount among the
responsibilities of a free press is the duty to prevent any part of the government from deceiving the The challenged statute could have been more narrowly drawn and the practices prohibited more
people and sending them off to distant lands to die of foreign fevers and foreign shot and shell. precisely delineated to satisfy the constitutional requirements as to a valid limitation under the clear and
present danger doctrine. As the author Tañada clearly explained, such provisions were deemed by the
legislative body to be part and parcel of the necessary and appropriate response not merely to a clear
GONZALES VS. COMELEC and present danger but to the actual existence of a grave and substantive evil of excessive
Page 72
partisanship, dishonesty and corruption as well as violence that of late has invariably marred election deeply held dogmas and tenets of other religions. RTC’s ruling clearly suppresses petitioner's freedom
campaigns and partisan political activities in this country. of speech and interferes with its right to free exercise of religion. “attack” is different from “offend” any
race or religion. The respondent Board may disagree with the criticisms of other religions by petitioner
The very idea of a government, republican in form, implies a right on the part of its citizens to meet but that gives it no excuse to interdict such criticisms, however, unclean they may be. Under our
peaceably for consultation in respect to public affairs and to petition for redress of grievances. As in the constitutional scheme, it is not the task of the State to favor any religion by protecting it against an
case of freedom of expression, this right is not to be limited, much less denied, except on a showing of attack by another religion. Religious dogmas and beliefs are often at war and to preserve peace among
a clear and present danger of a substantive evil that Congress has a right to prevent. their followers, especially the fanatics, the establishment clause of freedom of religion prohibits the
State from leaning towards any religion. Respondent board cannot censor the speech of petitioner
The prohibition of any speeches, announcements or commentaries, or the holding of interviews for or Iglesia ni Cristo simply because it attacks other religions, even if said religion happens to be the most
against the election of any party or candidate for public office and the prohibition of the publication or numerous church in our country. The basis of freedom of religion is freedom of thought and it is best
distribution of campaign literature or materials, against the solicitation of votes whether directly or served by encouraging the marketplace of dueling ideas. It is only where it is unavoidably necessary to
indirectly, or the undertaking of any campaign literature or propaganda for or against any candidate or prevent an immediate and grave danger to the security and welfare of the community that infringement
party is repugnant to a constitutional command. of religious freedom may be justified, and only to the smallest extent necessary to avoid the danger.
There is no showing whatsoever of the type of harm the tapes will bring about especially the gravity and
imminence of the threatened harm. Prior restraint on speech, including religious speech, cannot be
justified by hypothetical fears but only by the showing of a substantive and imminent evil. It is
inappropriate to apply the clear and present danger test to the case at bar because the issue involves
IGLESIA NI CRISTO VS. COURT OF APPEALS the content of speech and not the time, place or manner of speech. Allegedly, unless the speech is first
[259 SCRA 529; G.R. NO. 119673; 26 JUL 1996] allowed, its impact cannot be measured, and the causal connection between the speech and the evil
apprehended cannot be established. The determination of the question as to whether or not such
Facts: vilification, exaggeration or fabrication falls within or lies outside the boundaries of protected speech or
expression is a judicial function which cannot be arrogated by an administrative body such as a Board
Petitioner has a television program entitled "Ang Iglesia ni Cristo" aired on Channel 2 every Saturday of Censors." A system of prior restraint may only be validly administered by judges and not left to
and on Channel 13 every Sunday. The program presents and propagates petitioner's religious beliefs, administrative agencies.
doctrines and practices often times in comparative studies with other religions. Petitioner submitted to
the respondent Board of Review for Moving Pictures and Television the VTR tapes of its TV program
Series Nos. 116, 119, 121 and 128. The Board classified the series as "X" or not for public viewing on ADIONG VS. COMELEC
the ground that they "offend and constitute an attack against other religions which is expressly [207 SCRA 712; G.R. NO. 103956; 31 MAR 1992]
prohibited by law." On November 28, 1992, it appealed to the Office of the President the classification
of its TV Series No. 128 which allowed it through a letter of former Executive Secretary Edelmiro A. Facts:
Amante, Sr., addressed for Henrietta S. Mendez reversing the decision of the respondent Board.
According to the letter the episode in is protected by the constitutional guarantee of free speech and COMELEC promulgated Resolution No. 2347 which provides that decals and stickers may be posted
expression and no indication that the episode poses any clear and present danger. Petitioner also filed only in any of the authorized posting areas, prohibiting posting in "mobile" places, public or private.
Civil Case. Petitioner alleged that the respondent Board acted without jurisdiction or with grave abuse Petitioner Blo Umpar Adiong, a senatorial candidate in the May 11, 1992 elections now assails the
of discretion in requiring petitioner to submit the VTR tapes of its TV program and in x-rating them. It Resolution. In addition, the petitioner believes that with the ban on radio, television and print political
cited its TV Program Series Nos. 115, 119, 121 and 128. In their Answer, respondent Board invoked its advertisements, he, being a neophyte in the field of politics stands to suffer grave and irreparable injury
power under PD No. 19861 in relation to Article 201 of the Revised Penal Code. The Iglesia ni Cristo with this prohibition.
insists on the literal translation of the bible and says that our (Catholic) veneration of the Virgin Mary is
not to be condoned because nowhere it is found in the bible. The board contended that it outrages Issue:
Catholic and Protestant's beliefs. RTC ruled in favor of petitioners. CA however reversed it hence this
petition. Whether or Not the COMELEC’s prohibition unconstitutional.

Issue: Held:

Whether or Not the "ang iglesia ni cristo" program is not constitutionally protected as a form of religious The prohibition unduly infringes on the citizen's fundamental right of free speech. The preferred
exercise and expression. freedom of expression calls all the more for the utmost respect when what may be curtailed is the
dissemination of information to make more meaningful the equally vital right of suffrage. The so-called
Held: balancing of interests — individual freedom on one hand and substantial public interests on the other —
is made even more difficult in election campaign cases because the Constitution also gives specific
Yes. Any act that restrains speech is accompanied with presumption of invalidity. It is the burden of the authority to the Commission on Elections to supervise the conduct of free, honest, and orderly
respondent Board to overthrow this presumption. If it fails to discharge this burden, its act of censorship elections. When faced with border line situations where freedom to speak by a candidate or party and
will be struck down. This is true in this case. So-called "attacks" are mere criticisms of some of the freedom to know on the part of the electorate are invoked against actions intended for maintaining
Page 73
clean and free elections, the police, local officials and COMELEC, should lean in favor of freedom. The
regulation of election campaign activity may not pass the test of validity if it is too general in its terms or Yes. It seems a modest proposition that the provision of the Bill of Rights which enshrines freedom of
not limited in time and scope in its application, if it restricts one's expression of belief in a candidate or speech, freedom of expression and freedom of the press has to be taken in conjunction with Article IX
one's opinion of his or her qualifications, if it cuts off the flow of media reporting, and if the regulatory (C) (4) which may be seen to be a special provision applicable during a specific limited period — i.e.,
measure bears no clear and reasonable nexus with the constitutionally sanctioned objective. "during the election period." In our own society, equality of opportunity to proffer oneself for public
office, without regard to the level of financial resources that one may have at one's disposal, is clearly
The posting of decals and stickers in mobile places like cars and other moving vehicles does not an important value. One of the basic state policies given constitutional rank by Article II, Section 26 of
endanger any substantial government interest. There is no clear public interest threatened by such the Constitution is the egalitarian demand that "the State shall guarantee equal access to opportunities
activity so as to justify the curtailment of the cherished citizen's right of free speech and expression. for public service and prohibit political dynasties as may be defined by law." The essential question is
Under the clear and present danger rule not only must the danger be patently clear and pressingly whether or not the assailed legislative or administrative provisions constitute a permissible exercise of
present but the evil sought to be avoided must be so substantive as to justify a clamp over one's mouth the power of supervision or regulation of the operations of communication and information enterprises
or a writing instrument to be stilled. The regulation strikes at the freedom of an individual to express his during an election period, or whether such act has gone beyond permissible supervision or regulation of
preference and, by displaying it on his car, to convince others to agree with him. A sticker may be media operations so as to constitute unconstitutional repression of freedom of speech and freedom of
furnished by a candidate but once the car owner agrees to have it placed on his private vehicle, the the press. The Court considers that Section 11 (b) has not gone outside the permissible bounds of
expression becomes a statement by the owner, primarily his own and not of anybody else. The supervision or regulation of media operations during election periods.
restriction as to where the decals and stickers should be posted is so broad that it encompasses even
the citizen's private property, which in this case is a privately-owned vehicle. In consequence of this Section 11 (b) is limited in the duration of its applicability and enforceability. By virtue of the operation of
prohibition, another cardinal rule prescribed by the Constitution would be violated. Section 1, Article III Article IX (C) (4) of the Constitution, Section 11 (b) is limited in its applicability in time to election
of the Bill of Rights provides that no person shall be deprived of his property without due process of periods. Section 11 (b) does not purport in any way to restrict the reporting by newspapers or radio or
law. television stations of news or news-worthy events relating to candidates, their qualifications, political
parties and programs of government. Moreover, Section 11 (b) does not reach commentaries and
The prohibition on posting of decals and stickers on "mobile" places whether public or private except in expressions of belief or opinion by reporters or broadcasters or editors or commentators or columnists
the authorized areas designated by the COMELEC becomes censorship. in respect of candidates, their qualifications, and programs and so forth, so long at least as such
comments, opinions and beliefs are not in fact advertisements for particular candidates covertly paid
for. In sum, Section 11 (b) is not to be read as reaching any report or commentary other coverage that,
NATIONAL PRESS CLUB VS. COMELEC in responsible media, is not paid for by candidates for political office. Section 11 (b) as designed to
[201 SCRA 1; G.R. NO. 1026653; 5 MAR 1992] cover only paid political advertisements of particular candidates.

Facts: The limiting impact of Section 11 (b) upon the right to free speech of the candidates themselves is not
unduly repressive or unreasonable.
Petitioners in these cases consist of representatives of the mass media which are prevented from
selling or donating space and time for political advertisements; two (2) individuals who are candidates
for office (one for national and the other for provincial office) in the coming May 1992 elections; and US VS. BUSTOS
taxpayers and voters who claim that their right to be informed of election Issue and of credentials of the [37 PHIL. 731; G.R. L-12592; 8 MAR 1918]
candidates is being curtailed. It is principally argued by petitioners that Section 11 (b) of Republic Act
No. 66461 invades and violates the constitutional guarantees comprising freedom of expression. Facts:
Petitioners maintain that the prohibition imposed by Section 11 (b) amounts to censorship, because it
selects and singles out for suppression and repression with criminal sanctions, only publications of a In the latter part of 1915, numerous citizens of the Province of Pampanga assembled, and prepared
particular content, namely, media-based election or political propaganda during the election period of and signed a petition to the Executive Secretary(privileged communication) through the law office of
1992. It is asserted that the prohibition is in derogation of media's role, function and duty to provide Crossfield and O'Brien, and five individuals signed affidavits, charging Roman Punsalan, justice of the
adequate channels of public information and public opinion relevant to election Issue. Further, peace of Macabebe and Masantol, Pampanga, with malfeasance in office and asking for his removal.
petitioners contend that Section 11 (b) abridges the freedom of speech of candidates, and that the The specific charges against the justice of the peace include the solicitation of money from persons
suppression of media-based campaign or political propaganda except those appearing in the Comelec who have pending cases before the judge. Now, Punsalan alleged that accused published a writing
space of the newspapers and on Comelec time of radio and television broadcasts, would bring about a which was false, scandalous, malicious, defamatory, and libelous against him.
substantial reduction in the quantity or volume of information concerning candidates and Issue in the
election thereby curtailing and limiting the right of voters to information and opinion.
Issue:
Issue:
Whether or Not accused is entitled to constitutional protection by virtue of his right to free speech and
free press.
Whether or Not Section 11 (b) of Republic Act No. 6646 constitutional.
Held:
Held:
Page 74
Yes. The guaranties of a free speech and a free press include the right to criticize judicial conduct. The Manila, seeking to enjoin said defendants and their agents from confiscating plaintiff’s magazines or
administration of the law is a matter of vital public concern. Whether the law is wisely or badly enforced from preventing the sale or circulation thereof claiming that the magazine is a decent, artistic and
is, therefore, a fit subject for proper comment. If the people cannot criticize a justice of the peace or a educational magazine which is not per se obscene, and that the publication is protected by the
judge the same as any other public officer, public opinion will be effectively suppressed. It is a duty Constitutional guarantees of freedom of speech and of the press. Plaintiff also filed an Urgent Motion
which every one owes to society or to the State to assist in the investigation of any alleged misconduct. for issuance of a temporary restraining order against indiscriminate seizure, confiscation and burning of
It is further the duty of all who know of any official dereliction on the part of a magistrate or the wrongful plaintiff's "Pinoy Playboy" Magazines, pending hearing on the petition for preliminary injunction. The
act of any public officer to bring the facts to the notice of those whose duty it is to inquire into and Court granted the temporary restraining order. The case was set for trial upon the lapse of the TRO.
punish them. RTC ruled that the seizure was valid. This was affirmed by the CA.

The right to assemble and petition is the necessary consequence of republican institutions and the Issue:
complement of the part of free speech. Assembly means a right on the part of citizens to meet
peaceably for consultation in respect to public affairs. Petition means that any person or group of Whether or Not the seizure violative of the freedom of expression of the petitioner.
persons can apply, without fear of penalty, to the appropriate branch or office of the government for a
redress of grievances. The persons assembling and petitioning must, of course, assume responsibility Held:
for the charges made. All persons have an interest in the pure and efficient administration of justice and
of public affairs. Freedom of the press is not without restraint as the state has the right to protect society from
pornographic literature that is offensive to public morals, as indeed we have laws punishing the author,
Public policy, the welfare of society, and the orderly administration of government have demanded publishers and sellers of obscene publications. However, It is easier said than done to say, that if the
protection for public opinion. The inevitable and incontestable result has been the development and pictures here in question were used not exactly for art's sake but rather for commercial purposes, the
adoption of the doctrine of privilege. All persons have an interest in the pure and efficient administration pictures are not entitled to any constitutional protection. Using the Kottinger rule: the test of obscenity is
of justice and of public affairs. The duty under which a party is privileged is sufficient if it is social or "whether the tendency of the matter charged as obscene, is to deprave or corrupt those whose minds
moral in its nature and this person in good faith believes he is acting in pursuance thereof although in are open to such immoral influences and into whose hands a publication or other article charged as
fact he is mistaken. Although the charges are probably not true as to the justice of the peace, they were being obscene may fall." Another is whether it shocks the ordinary and common sense of men as an
believed to be true by the petitioners. Good faith surrounded their action. Probable cause for them to indecency. Ultimately "whether a picture is obscene or indecent must depend upon the circumstances
think that malfeasance or misfeasance in office existed is apparent. The ends and the motives of these of the case and that the question is to be decided by the "judgment of the aggregate sense of the
citizens— to secure the removal from office of a person thought to be venal — were justifiable. In no community reached by it." The government authorities in the instant case have not shown the required
way did they abuse the privilege. proof to justify a ban and to warrant confiscation of the literature First of all, they were not possessed of
a lawful court order: (1) finding the said materials to be pornography, and (2) authorizing them to carry
In the usual case malice can be presumed from defamatory words. Privilege destroys that presumption. out a search and seizure, by way of a search warrant. The court provides that the authorities must
A privileged communication should not be subjected to microscopic examination to discover grounds of apply for the issuance of a search warrant from a judge, if in their opinion an obscenity seizure is in
malice or falsity. order and that;

1. The authorities must convince the court that the materials sought to be seized are obscene
PITA VS. COURT OF APPEALS and pose a clear and present danger of an evil substantive enough to warrant State
[178 SCRA 362; G.R. NO.80806; 5 OCT 1989] interference and action;
2. The judge must determine whether or not the same are indeed obscene. The question is to
Facts: be resolved on a case-to-case basis and on the judge’s sound discretion;

On December 1 and 3, 1983, pursuing an Anti-Smut Campaign initiated by the Mayor of the City of
Manila, Ramon D. Bagatsing, elements of the Special Anti-Narcotics Group, Auxilliary Services Bureau, AYER PRODUCTIONS VS. CAPULONG
Western Police District, INP of the Metropolitan Police Force of Manila, seized and confiscated from [160 SCRA 861; G.R. NO. L-82380; 29 APR 1988]
dealers, distributors, newsstand owners and peddlers along Manila sidewalks, magazines, publications
and other reading materials believed to be obscene, pornographic and indecent and later burned the Facts:
seized materials in public at the University belt along C.M. Recto Avenue, Manila, in the presence of
Mayor Bagatsing and several officers and members of various student organizations. Petitioner McElroy an Australian film maker, and his movie production company, Ayer Productions,
envisioned, sometime in 1987, for commercial viewing and for Philippine and international release, the
Among the publications seized, and later burned, was "Pinoy Playboy" magazines published and co- historic peaceful struggle of the Filipinos at EDSA. The proposed motion picture entitled "The Four Day
edited by plaintiff Leo Pita. Revolution" was endorsed by the MTRCB as and other government agencies consulted. Ramos also
signified his approval of the intended film production.

Plaintiff filed a case for injunction with prayer for issuance of the writ of preliminary injunction against It is designed to be viewed in a six-hour mini-series television play, presented in a "docu-drama" style,
Mayor Bagatsing and Narcisco Cabrera, as superintendent of Western Police District of the City of creating four fictional characters interwoven with real events, and utilizing actual documentary footage
Page 75
as background. David Williamson is Australia's leading playwright and Professor McCoy (University of series of killings committed since Christmas of 1955. Losing no time, the Philippines defense
New South Wales) is an American historian have developed a script. establishment rushed to the island a platoon of scout rangers. Upon arriving Major Encarnacion and his
men found, instead of the alleged killers, a man named Fidel Cruz who merely wanted transportation
Enrile declared that he will not approve the use, appropriation, reproduction and/or exhibition of his home to Manila. In view of this finding, Major Encarnacion branded as a "hoax," the report of
name, or picture, or that of any member of his family in any cinema or television production, film or respondent.
other medium for advertising or commercial exploitation. petitioners acceded to this demand and the
name of Enrile was deleted from the movie script, and petitioners proceeded to film the projected This Week Magazine of the Manila Chronicle, then edited by Gatbonton, devoted a pictorial article to it
motion picture. However, a complaint was filed by Enrile invoking his right to privacy. RTC ordered for in its issue of January 15, 1956. Mention was made that while Fidel Cruz’ story turned out to be false it
the desistance of the movie production and making of any reference to plaintiff or his family and from brought attention to the government that people in that most people in the area are sick sick, only two
creating any fictitious character in lieu of plaintiff which nevertheless is based on, or bears substantial or individuals able to read and write, food and clothing being scarce.
marked resemblance to Enrile. Hence the appeal.
The magazine carried photographs of the person purporting to be Fidel Cruz. Unfortunately, the
Issue: pictures that were published were that of private respondent Fidel G. Cruz, a businessman contractor
from Santa Maria, Bulacan. It turned out that the photographs of respondent Cruz and that of Fidel
Whether or Not freedom of expression was violated. Cruz, sanitary inspector, were on file in the library of the Manila Chronicle but when the news quiz
format was prepared, the two photographs were in advertently switched. However a correction was
published immediately.

Respondent sued petitioners in the Court of First Instance of Manila for the recovery of damages
Held: alleging the defamatory character of the above publication of his picture. Defense interposed that they
are beating the deadline. The court ruled in his favor. Hence the appeal.
Yes. Freedom of speech and of expression includes the freedom to film and produce motion pictures
and exhibit such motion pictures in theaters or to diffuse them through television. Furthermore the Issue:
circumstance that the production of motion picture films is a commercial activity expected to yield
monetary profit, is not a disqualification for availing of freedom of speech and of expression. Whether or Not petitioners abused the freedom of the press.

The projected motion picture was as yet uncompleted and hence not exhibited to any audience. Neither Held:
private respondent nor the respondent trial Judge knew what the completed film would precisely look
like. There was, in other words, no "clear and present danger" of any violation of any right to privacy. No. The SC, quoting Quisumbing v. Lopez, found for plaintiff, but with reduced damages, since the
Subject matter is one of public interest and concern. The subject thus relates to a highly critical stage in error in this case could have been checked considering that this was a weekly magazine and not a
the history of the country. daily. The ruling: "there is no evidence in the record to prove that the publication of the news item under
consideration was prompted by personal ill will or spite, or that there was intention to do harm,' and that
At all relevant times, during which the momentous events, clearly of public concern, that petitioners on the other hand there was 'an honest and high sense of duty to serve the best interests of the public,
propose to film were taking place, Enrile was a "public figure:" Such public figures were held to have without self-seeking motive and with malice towards none.' Every citizen of course has the right to enjoy
lost, to some extent at least, their right to privacy. a good name and reputation, but we do not consider that the respondents, under the circumstances of
this case, had violated said right or abused the freedom of the press. The newspapers should be given
The line of equilibrium in the specific context of the instant case between the constitutional freedom of such leeway and tolerance as to enable them to courageously and effectively perform their important
speech and of expression and the right of privacy, may be marked out in terms of a requirement that role in our democracy. In the preparation of stories, press reporters and editors usually have to race
the proposed motion picture must be fairly truthful and historical in its presentation of events. with their deadlines; and consistently with good faith and reasonable care, they should not be held to
account, to a point of suppression, for honest mistakes or imperfection in the choice of words.

LOPEZ VS. SANDIGANBAYAN “No inroads on press freedom should be allowed in the guise of punitive action visited on what
[34 SCRA 116; L-26549; 31 JUL 1970] otherwise could be characterized as libel whether in the form of printed words or a defamatory
imputation resulting from the publication of respondent's picture with the offensive caption as in the
Facts: case here complained of. This is merely to underscore the primacy that freedom of the press enjoys.”

In the early part of January, 1956, there appeared on the front page of The Manila Chronicle, of which
petitioner Lopez was the publisher, as well as on other dailies, a news story of a sanitary inspector PRIMICIAS VS. FUGOSO
assigned to the Babuyan Islands, Fidel Cruz, sending a distress signal to a passing United States [80 PHIL 71; L-1800; 27 JAN 1948]
Airforce plane which in turn relayed the message to Manila. An American Army plane dropping on the
beach of an island an emergency-sustenance kit containing, among other things, a two-way radio set. Facts:
He utilized it to inform authorities in Manila that the people in the place were living in terror, due to a
Page 76
An action was instituted by the petitioner for the refusal of the respondent to issue a permit to them to (b) That he "was approached and asked to refrain from investigating the COA report on illegal
hold a public meeting in Plaza Miranda for redress of grievances to the government. The reason disbursements in the Supreme Court because 'it will embarass the Court;" and (c) that in several
alleged by the respondent in his defense for refusing the permit is, "that there is a reasonable ground to instances, the undersigned respondent was called over the phone by a leading member of the Court
believe, basing upon previous utterances and upon the fact that passions, specially on the part of the and was asked to dismiss the cases against two Members of the Court." Statements of the respondent
losing groups, remains bitter and high, that similar speeches will be delivered tending to undermine the saying that the SC’s order '"heightens the people's apprehension over the justice system in this country,
faith and confidence of the people in their government, and in the duly constituted authorities, which especially because the people have been thinking that only the small fly can get it while big fishes go
might threaten breaches of the peace and a disruption of public order." Giving emphasis as well to the scot-free” was publicized in leading newspapers.
delegated police power to local government. Stating as well Revised Ordinances of 1927 prohibiting as
an offense against public peace, and penalizes as a misdemeanor, "any act, in any public place, Now, the Court Resolved to require respondent to explain in writing why he should not be punished for
meeting, or procession, tending to disturb the peace or excite a riot; or collect with other persons in a contempt of court for making such public statements reported in the media. Respondent then sought to
body or crowd for any unlawful purpose; or disturb or disquiet any congregation engaged in any lawful get some members of the Court to inhibit themselves in the resolution of the Zaldivar case for alleged
assembly." Included herein is Sec. 1119, Free use of Public Place. 1 bias and prejudice against him. A little later, he in effect asked the whole Court to inhibit itself from
passing upon the Issue involved in proceeding and to pass on responsibility for this matter to the
Issue: Integrated Bar of the Philippines, upon the ground that respondent cannot expect due process from this
Court, that the Court has become incapable of judging him impartially and fairly. The Court found
Whether or Not the freedom of speech was violated. respondent guilty of contempt of court and indefinitely suspended from the practice of law. Now, he
assails said conviction, invoking his freedom of speech. Counsel for respondent urges that it is error
Held: "for this Court to apply the "visible tendency" rule rather than the "clear and present danger" rule in
disciplinary and contempt charges."
Yes. Dealing with the ordinance, specifically, Sec. 1119, said section provides for two constructions: (1)
the Mayor of the City of Manila is vested with unregulated discretion to grant or refuse, to grant permit Issue:
for the holding of a lawful assembly or meeting, parade, or procession in the streets and other public
places of the City of Manila; (2) The right of the Mayor is subject to reasonable discretion to determine Whether or Not there was a violation of the freedom of speech/expression.
or specify the streets or public places to be used with the view to prevent confusion by overlapping, to
secure convenient use of the streets and public places by others, and to provide adequate and proper Held:
policing to minimize the risk of disorder. The court favored the second construction. First construction
tantamount to authorizing the Mayor to prohibit the use of the streets. Under our democratic system of There was no violation. The Court did not purport to announce a new doctrine of "visible tendency," it
government no such unlimited power may be validly granted to any officer of the government, except was simply paraphrasing Section 3 (d) of Rule 71 of the Revised Rules of Court which penalizes a
perhaps in cases of national emergency. variety of contumacious conduct including: "any improper conduct tending, directly or indirectly, to
impede, obstruct or degrade the administration of justice."
The Mayor’s first defense is untenable. Fear of serious injury cannot alone justify suppression of free
speech and assembly. It is the function of speech to free men from the bondage of irrational fears. To Under either the "clear and present danger" test or the "balancing-of-interest test," the Court held that
justify suppression of free speech there must be reasonable ground to fear that serious evil will result if the statements made by respondent Gonzalez are of such a nature and were made in such a manner
free speech is practiced. There must be reasonable ground to believe that the danger apprehended is and under such circumstances, as to transcend the permissible limits of free speech. What is here at
imminent. There must be reasonable ground to believe that the evil to be prevented is a serious one . stake is the authority of the Supreme Court to confront and prevent a "substantive evil" consisting not
The fact that speech is likely to result in some violence or in destruction of property is not enough to only of the obstruction of a free and fair hearing of a particular case but also the avoidance of the
justify its suppression. There must be the probability of serious injury to the state. broader evil of the degradation of the judicial system of a country and the destruction of the standards
of professional conduct required from members of the bar and officers of the courts, which has some
implications to the society.
ZALDIVAR VS. SANDIGANBAYAN REYES VS. BAGATSING
[170 SCRA 1; G.R. NO. 79690-707; 1 FEB 1989] [125 SCRA 553; L-65366; 9 NOV 1983]

Facts: Facts:

The case stemmed from the resolution of the Supreme Court stopping the respondent from Petitioner sought a permit from the City of Manila to hold a peaceful march and rally on October 26,
investigating graft cases involving Antique Gov. Enrique Zaldivar. The Court ruled that since the 1983 from 2:00 to 5:00 in the afternoon, starting from the Luneta to the gates of the United States
adoption of the 1987 Constitution, respondent’s powers as Tanodbayan have been superseded by the Embassy. Once there, and in an open space of public property, a short program would be held. The
creation of the Office of the Ombudsman, he however becomes the Special Prosecutor of the State, march would be attended by the local and foreign participants of such conference. That would be
and can only conduct an investigation and file cases only when so authorized by the Ombudsman. A followed by the handing over of a petition based on the resolution adopted at the closing session of the
motion for reconsideration was filed by the respondent wherein he included statements which were Anti-Bases Coalition. There was likewise an assurance in the petition that in the exercise of the
unrelated in the Issue raised in the Court. This include: (a)That he had been approached twice by a constitutional rights to free speech and assembly, all the necessary steps would be taken by it "to
leading member of the court and he was asked to 'go slow on Zaldivar and 'not to be too hard on him; ensure a peaceful march and rally. However the request was denied. Reference was made to
Page 77
persistent intelligence reports affirming the plans of subversive/criminal elements to infiltrate or disrupt along Morayta Street and prevented them from proceeding further. They were then forcibly dispersed,
any assembly or congregations where a large number of people is expected to attend. Respondent causing injuries on one of them. Three other rallyists were arrested.
suggested that a permit may be issued if it is to be held at the Rizal Coliseum or any other enclosed
area where the safety of the participants themselves and the general public may be ensured. An oral All petitioners assail Batas Pambansa No. 880 The Public Assembly Act of 1985, some of them in toto
argument was heard and the mandatory injunction was granted on the ground that there was no and others only Sections 4, 5, 6, 12, 13(a), and 14(a), as well as the policy of CPR. They seek to stop
showing of the existence of a clear and present danger of a substantive evil that could justify the denial violent dispersals of rallies under the “no permit, no rally” policy and the CPR policy announced on
of a permit. However Justice Aquino dissented that the rally is violative of Ordinance No. 7295 of the Sept. 21, 2005.
City of Manila prohibiting the holding of rallies within a radius of five hundred (500) feet from any foreign
mission or chancery and for other purposes. Hence the Court resolves. Petitioners Bayan, et al., contend that BP 880 is clearly a violation of the Constitution and the
International Covenant on Civil and Political Rights and other human rights treaties of which the
Issue: Philippines is a signatory.

Whether or Not the freedom of expression and the right to peaceably assemble violated. They argue that B.P. No. 880 requires a permit before one can stage a public assembly regardless of
the presence or absence of a clear and present danger. It also curtails the choice of venue and is thus
Held: repugnant to the freedom of expression clause as the time and place of a public assembly form part of
the message for which the expression is sought.
Yes. The invocation of the right to freedom of peaceable assembly carries with it the implication that the
right to free speech has likewise been disregarded. It is settled law that as to public places, especially Petitioners Jess del Prado, et al., in turn, argue that B.P. No. 880 is unconstitutional as it is a
so as to parks and streets, there is freedom of access. Nor is their use dependent on who is the curtailment of the right to peacefully assemble and petition for redress of grievances because it puts a
applicant for the permit, whether an individual or a group. There can be no legal objection, absent the condition for the valid exercise of that right. It also characterizes public assemblies without a permit as
existence of a clear and present danger of a substantive evil, on the choice of Luneta as the place illegal and penalizes them and allows their dispersal. Thus, its provisions are not mere regulations but
where the peace rally would start. Time immemorial Luneta has been used for purposes of assembly, are actually prohibitions. Regarding the CPR policy, it is void for being an ultra vires act that alters the
communicating thoughts between citizens, and discussing public questions. standard of maximum tolerance set forth in B.P. No. 880, aside from being void for being vague and for
Such use of the public places has from ancient times, been a part of the privileges, immunities, rights, lack of publication.
and liberties of citizens.
KMU, et al., argue that the Constitution sets no limits on the right to assembly and therefore B.P. No.
With regard to the ordinance, there was no showing that there was violation and even if it could be 880 cannot put the prior requirement of securing a permit. And even assuming that the legislature can
shown that such a condition is satisfied it does not follow that respondent could legally act the way he set limits to this right, the limits provided are unreasonable: First, allowing the Mayor to deny the permit
did. The validity of his denial of the permit sought could still be challenged. on clear and convincing evidence of a clear and present danger is too comprehensive. Second, the
five-day requirement to apply for a permit is too long as certain events require instant public assembly,
A summary of the application for permit for rally: The applicants for a permit to hold an assembly should otherwise interest on the issue would possibly wane.As to the CPR policy, they argue that it is
inform the licensing authority of the date, the public place where and the time when it will take place. If preemptive, that the government takes action even before the rallyists can perform their act, and that
it were a private place, only the consent of the owner or the one entitled to its legal possession is no law, ordinance or executive order supports the policy. Furthermore, it contravenes the maximum
required. Such application should be filed well ahead in time to enable the public official concerned to tolerance policy of B.P. No. 880 and violates the Constitution as it causes a chilling effect on the
appraise whether there may be valid objections to the grant of the permit or to its grant but at another exercise by the people of the right to peaceably assemble.
public place. It is an indispensable condition to such refusal or modification that the clear and present
danger test be the standard for the decision reached. Notice is given to applicants for the denial. Respondents argued that petitioners have no standing. BP 880 entails traffic re-routing to prevent grave
public inconvenience and serious or undue interference in the free flow of commerce and trade. It is
BAYAN VS. EXECUTIVE SECRETARY ERMITA content-neutral regulation of the time, place and manner of holding public assemblies. According to
[488 SCRA 226; G.R. NO. 169838; 25 APR 2006] Atienza RA. 7160 gives the Mayor power to deny a permit independently of B.P. No. 880. and that the
permit is for the use of a public place and not for the exercise of rights; and that B.P. No. 880 is not a
Facts: content-based regulation because it covers all rallies.

Rallies of September 20, October 4, 5 and 6, 2005 is at issue. BAYAN’s rally was violently dispersed. Issue:
26 petitioners were injured, arrested and detained when a peaceful mass action they was preempted
and violently dispersed by the police. KMU asserts that the right to peaceful assembly, are affected by Whether or Not BP 880 and the CPR Policy unconstitutional.
Batas Pambansa No. 880 and the policy of “Calibrated Preemptive Response” (CPR) being followed to
implement it. KMU, et al., claim that on October 4, 2005, a rally KMU co-sponsored was to be Held:
conducted at the Mendiola bridge but police blocked them along C.M. Recto and Lepanto Streets and
forcibly dispersed them, causing injuries to several of their members. They further allege that on No question as to standing. Their right as citizens to engage in peaceful assembly and exercise the
October 6, 2005, a multi-sectoral rally which KMU also co-sponsored was scheduled to proceed along right of petition, as guaranteed by the Constitution, is directly affected by B.P. No. 880. B.P. 880 is not
España Avenue in front of the UST and going towards Mendiola bridge. Police officers blocked them an absolute ban of public assemblies but a restriction that simply regulates the time, place and manner
Page 78
of the assemblies. It refers to all kinds of public assemblies that would use public places. The MALABANAN VS. RAMENTO
reference to “lawful cause” does not make it content-based because assemblies really have to be for [129 SCRA 359; G.R. NO.62270; 21 MAY 1984]
lawful causes, otherwise they would not be “peaceable” and entitled to protection. Maximum tolerance 1
is for the protection and benefit of all rallyists and is independent of the content of the expressions in Facts:
the rally. There is, likewise, no prior restraint, since the content of the speech is not relevant to the
regulation. Petitioners were officers of the Supreme Student Council of respondent University. They sought and
were granted by the school authorities a permit to hold a meeting from 8:00 A.M. to 12:00 P.M, on
The so-called calibrated preemptive response policy has no place in our legal firmament and must be August 27, 1982. Pursuant to such permit, along with other students, they held a general assembly at
struck down as a darkness that shrouds freedom. It merely confuses our people and is used by some the Veterinary Medicine and Animal Science basketball court (VMAS), the place indicated in such
police agents to justify abuses. Insofar as it would purport to differ from or be in lieu of maximum permit, not in the basketball court as therein stated but at the second floor lobby. At such gathering they
tolerance, this was declared null and void. manifested in vehement and vigorous language their opposition to the proposed merger of the Institute
of Animal Science with the Institute of Agriculture. The same day, they marched toward the Life Science
The Secretary of the Interior and Local Governments, are DIRECTED to take all necessary steps for Building and continued their rally. It was outside the area covered by their permit. Even they rallied
the immediate compliance with Section 15 of Batas Pambansa No. 880 through the establishment or beyond the period allowed. They were asked to explain on the same day why they should not be held
designation of at least one suitable freedom park or plaza in every city and municipality of the country. liable for holding an illegal assembly. Then on September 9, 1982, they were informed that they were
After thirty (30) days from the finality of this Decision, subject to the giving of advance notices, no prior under preventive suspension for their failure to explain the holding of an illegal assembly. The validity
permit shall be required to exercise the right to peaceably assemble and petition in the public parks or thereof was challenged by petitioners both before the Court of First Instance of Rizal against private
plazas of a city or municipality that has not yet complied with Section 15 of the law. respondents and before the Ministry of Education, Culture, and Sports. Respondent Ramento found
petitioners guilty of the charge of illegal assembly which was characterized by the violation of the permit
granted resulting in the disturbance of classes and oral defamation. The penalty was suspension for
FERNANDO VS. ESTORNINOS one academic year. Hence this petition.
[G.R. NO 159751; 6 DEC 2006]
Issue:
Facts:
Whether on the facts as disclosed resulting in the disciplinary action and the penalty imposed, there
Acting on reports of sale and distribution of pornographic materials, officers of the PNP Criminal was an infringement of the right to peaceable assembly and its cognate right of free speech.
Investigation and Detection Group in the National Capital Region conducted police surveillance on the
store Gaudencio E. Fernando Music Fair (Music Fair) in Quiapo. A Search Warrant for violation of Held:
Article 201 of RPC against petitioner and a certain Warren Tingchuy and the seizure of the following
items: Yes. Student leaders are likely to be assertive and dogmatic. They would be ineffective if during a rally
they speak in the guarded and judicious language of the academe. But with the activity taking place in
a. Copies of New Rave Magazines with nude obscene pictures; the school premises and during the daytime, no clear and present danger of public disorder is
b. Copies of IOU Penthouse Magazine with nude obscene pictures; discernible. This is without prejudice to the taking of disciplinary action for conduct, "materially disrupts
c. Copies of Hustler International Magazine with nude obscene pictures; and classwork or involves substantial disorder or invasion of the rights of others."
d. Copies of VHS tapes containing pornographic shows.
The rights to peaceable assembly and free speech are guaranteed students of educational institutions.
The police searched the premises and confiscated twenty-five VHS tapes(among of which is “Kahit sa Necessarily, their exercise to discuss matters affecting their welfare or involving public interest is not to
Pangarap Lang” with Myra Manibog as actress who is naked) and ten different magazines(Dalaga, be subjected to previous restraint or subsequent punishment unless there be a showing of a clear and
Penthouse, Swank, Erotic, Rave, Playhouse, Gallery, QUI), which they deemed pornographic. present danger to a substantive evil that the state, has a right to present. As a corollary, the utmost
Petitioners were charged and convicted. CA affirmed the decision hence this appeal. leeway and scope is accorded the content of the placards displayed or utterances made. The
peaceable character of an assembly could be lost, however, by an advocacy of disorder under the
Issue: name of dissent, whatever grievances that may be aired being susceptible to correction through the
ways of the law. If the assembly is to be held in school premises, permit must be sought from its school
Whether or Not the CA erred in affirming RTC’s decision. authorities, who are devoid of the power to deny such request arbitrarily or unreasonably. In granting
such permit, there may be conditions as to the time and place of the assembly to avoid disruption of
Held: classes or stoppage of work of the non-academic personnel. Even if, however, there be violations of its
terms, the penalty incurred should not be disproportionate to the offense.
No. As obscenity is an unprotected speech which the State has the right to regulate, the State in
pursuing its mandate to protect the public from obscene, immoral and indecent materials must justify
the regulation or limitation. (Kottinger Rule Applied). NON VS. DAMES
[185 SCRA 523; G.R. NO. 89317; 20 MAY 1990]

Page 79
Facts: Yes. At the time Tulfo wrote the article, the checkpoints case had not yet been decided upon, and the
Supreme Court was still acting on an MR filed from the CA. The power to punish is inherent as it is
Petitioners, students in private respondent Mabini Colleges, Inc. in Daet, Camarines Norte, were not essential for self-preservation. Contempt of court is defiance of the authority, justice and dignity of the
allowed to re-enroll by the school for the academic year 1988-1989 for leading or participating in courts. It brings disrepute to the court. There are two kinds of publications which can be punished for
student mass actions against the school in the preceding semester. The subject of the protests is not, contempt:
however, made clear in the pleadings.
a. those whose object is to affect the decision in a pending case.
Petitioners filed a petition in the court seeking their readmission or re-enrollment to the school, but the
b. those whose object is to bring courts to discredit. Tulfo's article constituted both.
trial court dismissed the petition. They now petition the court to reverse its ruling in Alcuaz vs. PSBA 1,
which was also applied in the case. The court said that petitioners waived their privilege to be admitted
It should have been okay to criticize if respectful language was used, but if its object is only to degrade
for re-enrollment with respondent college when they adopted, signed, and used its enrollment form for
and ridicule, then it is clearly an obstruction of justice. Nothing constructive can be gained from them.
the first semester of school year 1988-89, which states that: The Mabini College reserves the right to
Being emotional is no excuse for being insulting. Quoting is not an excuse also, because at the end of
deny admission of students whose scholarship and attendance are unsatisfactory and to require
his article, Tulfo said, "So you bobo justices, watch out!" Also, he said he was not sorry for having
withdrawal of students whose conduct discredits the institution and/or whose activities unduly disrupts
written the articles.
or interfere with the efficient operation of the college. Students, therefore, are required to behave in
accord with the Mabini College code of conduct and discipline.
Tulfo is found in contempt of court and is gravely censured.
Issue:
PBM EMPLOYEES VS. PBM
Whether or Not the students’ right to freedom of speech and assembly infringed.
[51 SCRA 189; G.R. NO. L-31195; 5 JUN 1993]
Held:
Facts:
Yes. The protection to the cognate rights of speech and assembly guaranteed by the Constitution is
The petitioner Philippine Blooming Mills Employees Organization (PBMEO) is a legitimate labor union
similarly available to students is well-settled in our jurisdiction. However there are limitations. The
composed of the employees of the respondent Philippine Blooming Mills Co., Inc., and petitioners.
permissible limitation on Student Exercise of Constitutional Rights within the school presupposes that
Benjamin Pagcu and Rodulfo Munsod are officers and members of the petitioner Union. Petitioners
conduct by the student, in class or out of it, which for any reason whether it stems from time, place, or
claim that on March 1, 1969, they decided to stage a mass demonstration at Malacañang on March 4,
type of behavior should not materially disrupt classwork or must not involve substantial disorder or
1969, in protest against alleged abuses of the Pasig police. PBMEO thru Pagcu confirmed the planned
invasion of the rights of others.
demonstration and stated that the demonstration or rally cannot be cancelled because it has already
been agreed upon in the meeting. Pagcu explained further that the demonstration has nothing to do
with the Company because the union has no quarrel or dispute with Management. The Management,
IN RE: TULFO
thru Atty. C.S. de Leon, Company personnel manager, informed PBMEO that the demonstration is an
[A.M. NO. 90-4-1545-0; 17 APR 1990]
inalienable right of the union guaranteed by the Constitution but emphasized that any demonstration for
that matter should not unduly prejudice the normal operation of the Company. Workers who without
Facts:
previous leave of absence approved by the Company, particularly , the officers present who are the
organizers of the demonstration, who shall fail to report for work the following morning shall be
In Oct. 13, 1989, Tulfo wrote an article in his column in PDI 'On Target' stating that the Supreme Court
dismissed, because such failure is a violation of the existing CBA and, therefore, would be amounting to
rendered an idiotic decision in legalizing checkpoints, and again on Oct. 16, 1989, where he called the
an illegal strike. Because the petitioners and their members numbering about 400 proceeded with the
Supreme Court stupid and "sangkatutak na mga bobo justices of the Philippine Supreme Court". Tulfo
demonstration despite the pleas of the respondent Company that the first shift workers should not be
was required to show cause why he should not be punished for contempt. Tulfo said that he was just
required to participate in the demonstration and that the workers in the second and third shifts should
reacting emotionally because he had been a victim of harassment in the checkpoints, and "idiotic"
be utilized for the demonstration from 6 A.M. to 2 P.M. on March 4, 1969, filed a charge against
meant illogical and unwise, and "bobo" was just quoted from other attorneys, and since the case had
petitioners and other employees who composed the first shift, for a violation of Republic Act No.
been decided and terminated, there was not contempts. Lastly, the article does not pose any clear and
875(Industrial Peace Act), and of the CBA providing for 'No Strike and No Lockout.' Petitioners were
present danger to the Supreme court.
held guilty in by CIR for bargaining in bad faith, hence this appeal.
Issue:
Issue:
Whether or Not Tulfo is in contempt.
Whether or Not the petitioners right to freedom of speech and to peaceable assemble violated.

Held:
Held:

Page 80
Yes. A constitutional or valid infringement of human rights requires a more stringent criterion, namely
existence of a grave and immediate danger of a substantive evil which the State has the right to
prevent. This is not present in the case. It was to the interest herein private respondent firm to rally to
the defense of, and take up the cudgels for, its employees, so that they can report to work free from
harassment, vexation or peril and as consequence perform more efficiently their respective tasks
enhance its productivity as well as profits. Herein respondent employer did not even offer to intercede
for its employees with the local police. In seeking sanctuary behind their freedom of expression well as
their right of assembly and of petition against alleged persecution of local officialdom, the employees
and laborers of herein private respondent firm were fighting for their very survival, utilizing only the
weapons afforded them by the Constitution — the untrammelled enjoyment of their basic human rights.
The pretension of their employer that it would suffer loss or damage by reason of the absence of its
employees from 6 o'clock in the morning to 2 o'clock in the afternoon, is a plea for the preservation
merely of their property rights. The employees' pathetic situation was a stark reality — abused,
harassment and persecuted as they believed they were by the peace officers of the municipality. As
above intimated, the condition in which the employees found themselves vis-a-vis the local police of
Pasig, was a matter that vitally affected their right to individual existence as well as that of their families.
Material loss can be repaired or adequately compensated. The debasement of the human being broken
in morale and brutalized in spirit-can never be fully evaluated in monetary terms. As heretofore stated,
the primacy of human rights — freedom of expression, of peaceful assembly and of petition for redress
of grievances — over property rights has been sustained. To regard the demonstration against police
officers, not against the employer, as evidence of bad faith in collective bargaining and hence a
violation of the collective bargaining agreement and a cause for the dismissal from employment of the
demonstrating employees, stretches unduly the compass of the collective bargaining agreement, is "a
potent means of inhibiting speech" and therefore inflicts a moral as well as mortal wound on the
constitutional guarantees of free expression, of peaceful assembly and of petition. Circulation is one of
the aspects of freedom of expression. If demonstrators are reduced by one-third, then by that much the
circulation of the Issue raised by the demonstration is diminished. The more the participants, the more
persons can be apprised of the purpose of the rally. Moreover, the absence of one-third of their
members will be regarded as a substantial indication of disunity in their ranks which will enervate their
position and abet continued alleged police persecution.

Page 81
THE IMPAIRMENT CLAUSE
Plaintiff is engaged in real estate business, developing and selling lots to the public, particularly the
Highway Hills Subdivision along EDSA. On March 4, 1952, plaintiff, as vendor, and Augusto Padilla
Art 3, Sec. 10. “No law impairing the obligation of contracts shall be passed.” and Natividad Angeles, as vendees, entered into separate agreements of sale on installments over two
parcels of land of the Subdivision. On July 19, 1962, the said vendees transferred their rights and
interests over the aforesaid lots in favor of one Emma Chavez. Upon completion of payment of the
RUTTER VS. ESTEBAN purchase price, the plaintiff executed the corresponding deeds of sale in favor of Emma Chavez. Both
[93 PHIL 68; NO.L-3708; 18 MAY 1953] the agreements (of sale on installment) and the deeds of sale contained the stipulations or restrictions
that:
Facts:
1. The parcel of land shall be used exclusively for residential purposes, and she shall not be
On August 20,1941 Rutter sold to Esteban two parcels of land situated in the Manila for P9,600 of entitled to take or remove soil, stones or gravel from it or any other lots belonging to the
which P4,800 were paid outright, and the balance was made payable as follows: P2,400 on or before Seller.
August 7, 1942, and P2,400 on or before August 27, 1943, with interest at the rate of 7 percent per 2. All buildings and other improvements (except the fence) which may be constructed at any
annum. To secure the payment of said balance of P4,800, a first mortgage has been constituted in time in said lot must be, (a) of strong materials and properly painted, (b) provided with
favor of the plaintiff. Esteban failed to pay the two installments as agreed upon, as well as the interest modern sanitary installations connected either to the public sewer or to an approved septic
that had accrued and so Rutter instituted an action to recover the balance due, the interest due and the tank, and (c) shall not be at a distance of less than two (2) meters from its boundary lines.
attorney's fees. The complaint also contains a prayer for sale of the properties mortgaged in
accordance with law. Esteban claims that this is a prewar obligation contracted and that he is a war Eventually said lots were bought by defendant. Lot 5 directly from Chavez and Lot 6 from Republic
sufferer, having filed his claim with the Philippine War Damage Commission for the losses he had Flour Mills by deed of exchange, with same restrictions. Plaintiff claims that restriction is for the
suffered as a consequence of the last war; and that under section 2 of RA 342(moratorium law), beautification of the subdivision. Defendant claimed of the commercialization of western part of EDSA.
payment of his obligation cannot be enforced until after the lapse of eight years. The complaint was Defendant began constructing a commercial bank building. Plaintiff demand to stop it, which forced him
dismissed. A motion for recon was made which assails the constitutionality of RA 342. to file a case, which was later dismissed, upholding police power. Motion for recon was denied, hence
the appeal.
Issue:
Issue:
Whether or Not RA 342 unconstitutional on non-impairment clause grounds.
Whether or Not non-impairment clause violated.
Held:
Held:
Yes. The moratorium is postponement of fulfillment of obligations decreed by the state through the
medium of the courts or the legislature. Its essence is the application of police power. The economic No. Resolution is a valid exercise of police power. EDSA, a main traffic artery which runs through
interests of the State may justify the exercise of its continuing and dominant protective power several cities and municipalities in the Metro Manila area, supports an endless stream of traffic and the
notwithstanding interference with contracts. The question is not whether the legislative action affects resulting activity, noise and pollution are hardly conducive to the health, safety or welfare of the
contracts incidentally, or directly or indirectly, but whether the legislation is addressed to a legitimate residents in its route. Health, safety, peace, good order and general welfare of the people in the locality
end and the measures taken are reasonable and appropriate to that end. are justifications for this. It should be stressed, that while non-impairment of contracts is constitutionally
guaranteed, the rule is not absolute, since it has to be reconciled with the legitimate exercise of police
However based on the President’s general SONA and consistent with what the Court believes to be as power.
the only course dictated by justice, fairness and righteousness, declared that the continued operation
and enforcement of RA 342 at the present time is unreasonable and oppressive, and should not be
prolonged should be declared null and void and without effect. This holds true as regards Executive
Orders Nos. 25 and 32, with greater force and reason considering that said Orders contain no limitation
whatsoever in point of time as regards the suspension of the enforcement and effectivity of monetary LOZANO VS. MARTINEZ
obligations. [146 SCRA 323; NO.L-63419; 18 DEC 1986]

Facts:

A motion to quash the charge against the petitioners for violation of the BP 22 was made, contending
ORTIGAS VS. FEATI BANK that no offense was committed, as the statute is unconstitutional. Such motion was denied by the RTC.
[94 SCRA 533; NO.L-24670; 14 DEC 1979] The petitioners thus elevate the case to the Supreme Court for relief. The Solicitor General, commented
that it was premature for the accused to elevate to the Supreme Court the orders denying their motions
Facts:
Page 82
to quash. However, the Supreme Court finds it justifiable to intervene for the review of lower court's
denial of a motion to quash.

Issue:

Whether or Not BP 22 impairs freedom of contract. Whether or not BP 22 transgresses the


constitutional inhibition against imprisonment for debt.

Held:

The freedom of contract which is constitutionally protected is freedom to enter into "lawful" contracts.
Contracts which contravene public policy are not lawful. Checks can not be categorized as mere
contracts. It is a commercial instrument which, in this modem day and age, has become a convenient
substitute for money; it forms part of the banking system and therefore not entirely free from the
regulatory power of the state.

The offense punished by BP 22 is the act of making and issuing a worthless check or a check that is
dishonored upon its presentation for payment. It is not the non-payment of an obligation which the law
punishes. The law is not intended or designed to coerce a debtor to pay his debt. The thrust of the law
is to prohibit, under pain of penal sanctions, the making of worthless checks and putting them in
circulation.

Page 83
In the case at bar, the statute simply declares the CPP as an organized conspiracy for the overthrow of
EX POST FACTO LAWS the Government for purposes of example of SECTION 4 of the Act. The Act applies not only to the
CPP but also to other organizations having the same purpose and their successors. The Act’s focus is
on the conduct not person.
Art 3, Sec. 22. “No ex post facto law or bill of attainder shall be enacted.” Membership to this organizations, to be UNLAWFUL, it must be shown that membership was acquired
with the intent to further the goals of the organization by overt acts. This is the element of
MEMBERSHIP with KNOWLEDGE that is punishable. This is the required proof of a member’s direct
PEOPLE VS. FERRER participation. Why is membership punished. Membership renders aid and encouragement to the
[48 SCRA 382; NOS.L-32613-14; 27 DEC 1972] organization. Membership makes himself party to its unlawful acts.

Facts: Furthermore, the statute is PROSPECTIVE in nature. Section 4 prohibits acts committed after approval
of the act. The members of the subversive organizations before the passing of this Act is given an
Hon. Judge Simeon Ferrer is the Tarlac trial court judge that declared RA1700 or the Anti-Subversive opportunity to escape liability by renouncing membership in accordance with Section 8. The statute
Act of 1957 as a bill of attainder. Thus, dismissing the information of subversion against the following: applies the principle of mutatis mutandis or that the necessary changes having been made.
1.) Feliciano Co for being an officer/leader of the Communist Party of the Philippines (CPP) aggravated
by circumstances of contempt and insult to public officers, subversion by a band and aid of armed men
The declaration of that the CPP is an organized conspiracy to overthrow the Philippine Government
to afford impunity. 2.) Nilo Tayag and 5 others, for being members/leaders of the NPA, inciting,
should not be the basis of guilt. This declaration is only a basis of Section 4 of the Act. The
instigating people to unite and overthrow the Philippine Government. Attended by Aggravating
EXISTENCE OF SUBSTANTIVE EVIL justifies the limitation to the exercise of “Freedom of Expression
Circumstances of Aid or Armed Men, Craft, and Fraud. The trial court is of opinion that 1.) The
and Association” in this matter. Before the enactment of the statute and statements in the preamble,
Congress usurped the powers of the judge 2.) Assumed judicial magistracy by pronouncing the guilt of
careful investigations by the Congress were done. The court further stresses that whatever interest in
the CPP without any forms of safeguard of a judicial trial. 3.) It created a presumption of organizational
freedom of speech and association is excluded in the prohibition of membership in the CPP are weak
guilt by being members of the CPP regardless of voluntariness.
considering NATIONAL SECURITY and PRESERVATION of DEMOCRACY.

The Anti Subversive Act of 1957 was approved 20June1957. It is an act to outlaw the CPP and similar The court set basic guidelines to be observed in the prosecution under RA1700. In addition to proving
associations penalizing membership therein, and for other purposes. It defined the Communist Party circumstances/ evidences of subversion, the following elements must also be established:
being although a political party is in fact an organized conspiracy to overthrow the Government, not
only by force and violence but also by deceit, subversion and other illegal means. It declares that the 1. Subversive Organizations besides the CPP, it must be proven that the organization
CPP is a clear and present danger to the security of the Philippines. Section 4 provided that affiliation purpose is to overthrow the present Government of the Philippines and establish a
with full knowledge of the illegal acts of the CPP is punishable. Section 5 states that due investigation domination of a FOREIGN POWER. Membership is willfully and knowingly done by
by a designated prosecutor by the Secretary of Justice be made prior to filing of information in court. overt acts.
Section 6 provides for penalty for furnishing false evidence. Section 7 provides for 2 witnesses in open 2. In case of CPP, the continued pursuance of its subversive purpose. Membership is
court for acts penalized by prision mayor to death. Section 8 allows the renunciation of membership to willfully and knowingly done by overt acts.
the CCP through writing under oath. Section 9 declares the constitutionality of the statute and its valid
exercise under freedom if thought, assembly and association. The court did not make any judgment on the crimes of the accused under the Act. The Supreme Court
set aside the resolution of the TRIAL COURT.
Issue:

Whether or not RA1700 is a bill of attainder/ ex post facto law.

Whether or Not RA1700 violates freedom of expression.

Held:

The court holds the VALIDITY Of the Anti-Subversion Act of 1957.

BAYOT VS. SANDIGANBAYAN


A bill of attainder is solely a legislative act. It punishes without the benefit of the trial. It is the
[128 SCRA 383; NO.L-61776 TO NO.L-61861; 23 MAR 1984]
substitution of judicial determination to a legislative determination of guilt. In order for a statute be
measured as a bill of attainder, the following requisites must be present: 1.) The statute specifies
Facts:
persons, groups. 2.) the statute is applied retroactively and reach past conduct. (A bill of attainder
relatively is also an ex post facto law.)
Bayot is one of the several persons who was accused in more than 100 counts of estafa thru
falsification of Public documents before the Sandiganbayan. The said charges started from his alleged
Page 84
involvement as a government auditor of the commission on audit assigned to the Ministry of education ex parte preliminary investigation. A motion to quash the information was filed by the private
and culture, with some other employees from the said ministry. The bureau of treasury and the respondent contending among others that he is charged for an offence which has prescribed. Said
teacher’s camp in Baguio City for the preparation and encashment of fictitious TCAA checks for the motion was granted. The crime was committed on January 21, 1976, period of prescription was 10
nom-existent obligations of the teacher’s camp resulting in damage to the government of several years, therefore it has prescribed in 1986. Now the motion to quash was being assailed.
millions. The 1st 32 cases were filed on july 25, 1987, while Bayot ran for municipal mayor of Amadeo
Cavite and was elected on January 1980. but on May 1980 Sandiganbayan promulgated a decision Issue:
convicting the accused together with his other co-accused in all but one of the thirty two cases filed
against them. Whether or Not the motion to quash validly granted.

On Mach 16, 1982 Batas Pambansa Blg 195 was passed amending RA 3019. Held:

Issue: Yes. RA 3019, being a special law the computation of the period for the prescription of the crime is
governed by Sec. 29 of Act No. 3326, which begins to run from the day of the commission of the crime
Whether or Not it would be violative of the constitutional guarantee against an ex post facto law. and not the discovery of it. Additionally, BP 195 which was approved on March 16, 1982, amending
Sec. 11 of RA 3019 by increasing ten to fifteen years of the period for the prescription or
Held: extinguishment of a violation of RA 3019 may not be given retroactive application to the crime which
was committed by Paredes, as it is prejudicial to the accused. To apply BP 195 to Paredes would make
The court finds no merit in the petitioner’s contention that RA 3019 as amended by Batas Pambansa it an ex post facto law1 for it would alter his situation to his disadvantage by making him criminally liable
Blg 195, which includes the crime of estafa through falsification of Public Documents as among crimes for a crime that had already been extinguished under the law existing when it was committed.
subjecting the public officer charged therewith with suspension from public office pending action in
court, is a penal provision which violates the constitutional prohibition against the enactment of ex post
facto law. Accdg to the RPC suspension from employment and public office during trial shall not be
considered as a penalty. It is not a penalty because it is not a result of a judicial proceeding. In fact, if
acquitted the official who is suspended shall be entitled to reinstatement and the salaries and benefits
which he failed to receive during suspension. And does not violate the constitutional provision against
ex post facto law.

The claim of the petitioner that he cannot be suspended because he is currently occupying a position
diffren tfrom that under which he is charged is untenable. The amendatory provision clearly states that
any incumbent public officer against whom any criminal prosecution under a valid information under RA
3019 for any offense involving fraud upon the government or public funds or property or whatever
stage of execution and mode of participation shall be suspended from office. The use of the word
“office” applies to any office which the officer charged may be holding and not only the particular office
under which he was charged.

PEOPLE VS. SANDIGANBAYAN


[211 SCRA 241; G.R. NO. 101724; 3 JUL 1992]

Facts:

Two letter complaints were filed with the Tanodbayan by Teofilo Gelacio on October 28,1986 and
December 9, 1986, a political leader of Governor Valentina Plaza, wife of Congressman Democrito
Plaza of Agusan del Sur, shortly after private respondent had replaced Mrs. Plaza as OIC/provincial
Governor of Agusan del Sur on March 1986 The complaint questioned the issuance to Governor
Paredes, when he was still the provincial attorney in 1976 of a free patent title for a lot in the Rosario
public land subdivision in San Francisco, Agusan del Sur. He misrepresented to a Lands Inspector of
the Bureau of Lands that the lands subject herein are disposable lands, thereby inducing said inspector
to recommend approval of his application for free patent. On August 10, 1989 an information for
violation of RA 3019 Anti-Graft and Corrupt Practices Act was then filed in the Sandiganbayan after an
Page 85
NON-IMPRISONMENT FOR DEBT Held:

The enactment of BP 22 a valid exercise of the police power and is not repugnant to the constitutional
Art 3, Sec. 20. “No person shall be imprisoned for debt or non-payment of a poll tax.” inhibition against imprisonment for debt.

The offense punished by BP 22 is the act of making and issuing a worthless check or a check that is
SERAFIN VS. LINDAYAG dishonored upon its presentation for payment. It is not the non-payment of an obligation which the law
[67 SCRA 166; ADM. MATTER. NO. 297-MJ; 30 SEPT 1975] punishes. The law is not intended or designed to coerce a debtor to pay his debt.

Facts: The law punishes the act not as an offense against property, but an offense against public order. The
thrust of the law is to prohibit, under pain of penal sanctions, the making of worthless checks and
Plaintiff failed to pay a simple indebtedness for P1500 Carmelito Mendoza, then municipal secretary putting them in circulation. An act may not be considered by society as inherently wrong, hence, not
and his wife Corazon Mendoza and therefore an estafa case was filed against her. Complainant malum in se but because of the harm that it inflicts on the community, it can be outlawed and criminally
admitted complaint. Now complainant filed a case against respondent Judge for not dismissing the case punished as malum prohibitum. The state can do this in the exercise of its police power.
and issuing a warrant of arrest as it falls on the category of a simple indebtedness, since elements of
estafa are not present. Further she contended that no person should be imprisoned for non-payment of
a loan of a sum of money. Two months after respondent dismissed plaintiff’s case. (Judge here
committed gross ignorance of law. Even if complainant desisted case was pursued.)

Issue:

Whether or Not there was a violation committed by the judge when it ordered the imprisonment of
plaintiff for non-payment of debt?

Held:

Yes. Since plaintiff did not commit any offense as, his debt is considered a simple loan granted by her
friends to her. There is no collateral or security because complainant was an old friend of the spouses
who lent the money and that when they wrote her a letter of demand she promised to pay them and
said that if she failed to keep her promise, they could get her valuable things at her home. Under the
Constitution she is protected. Judge therefore in admitting such a "criminal complaint" that was plainly
civil in aspects from the very face of the complaint and the "evidence" presented, and issuing on the
same day the warrant of arrest upon his utterly baseless finding "that the accused is probably guilty of
the crime charged," respondent grossly failed to perform his duties properly.

LOZANO VS. MARTINEZ


[146 SCRA 323; NO.L-63419; 18 DEC 1986]

Facts:

A motion to quash the charge against the petitioners for violation of the BP 22 was made, contending
that no offense was committed, as the statute is unconstitutional. Such motion was denied by the RTC.
The petitioners thus elevate the case to the Supreme Court for relief. The Solicitor General, commented
that it was premature for the accused to elevate to the Supreme Court the orders denying their motions
to quash. However, the Supreme Court finds it justifiable to intervene for the review of lower court's
denial of a motion to quash.

Issue:

Whether or not BP 22 is constitutional as it is a proper exercise of police power of the State.

Page 86
INVOLUNTARY SERVITUDE

Art 3, Sec. 18. “(2) No involuntary servitude in any form shall exist except as a punishment for a crime
whereof the party shall have been duly convicted.”

CAUNCA VS. SALAZAR


[82 PHIL 851; NO.L-2690; 1 JAN 1949]

Facts:

This is an action for habeas corpus brought by Bartolome Caunca in behalf of his cousin Estelita Flores
who was employed by the Far Eastern Employment Bureau, owned by Julia Salazar, respondent
herein. An advanced payment has already been given to Estelita by the employment agency, for her to
work as a maid. However, Estelita wanted to transfer to another residence, which was disallowed by the
employment agency. Further she was detained and her liberty was restrained. The employment agency
wanted that the advance payment, which was applied to her transportation expense from the province
should be paid by Estelita before she could be allowed to leave.

Issue:

Whether or Not an employment agency has the right to restrain and detain a maid without returning the
advance payment it gave?

Held:

An employment agency, regardless of the amount it may advance to a prospective employee or maid,
has absolutely no power to curtail her freedom of movement. The fact that no physical force has been
exerted to keep her in the house of the respondent does not make less real the deprivation of her
personal freedom of movement, freedom to transfer from one place to another, freedom to choose
one’s residence. Freedom may be lost due to external moral compulsion, to founded or groundless
fear, to erroneous belief in the existence of an imaginary power of an impostor to cause harm if not
blindly obeyed, to any other psychological element that may curtail the mental faculty of choice or the
unhampered exercise of the will. If the actual effect of such psychological spell is to place a person at
the mercy of another, the victim is entitled to the protection of courts of justice as much as the individual
who is illegally deprived of liberty by duress or physical coercion.

Page 87
THE WRIT OF HABEAS CORPUS The President has authority however it is subject to judicial review. Two conditions must concur for the
valid exercise of the authority to suspend the privilege to the writ (a) there must be "invasion,
insurrection, or rebellion" or "imminent danger thereof," and (b) "public safety" must require the
Art 3, Sec. 15. “The privilege of the writ of habeas corpus shall not be suspended except in cases of suspension of the privilege. President has three (3) courses of action: (a) to call out the armed forces;
invasion or rebellion when the public safety requires it.” (b) to suspend the privilege of the writ of habeas corpus; and (c) to place the Philippines or any part
thereof under martial law. He had, already, called out the armed forces, proved inadequate. Of the two
other alternatives, the suspension of the privilege is the least harsh.
LANSANG VS. GARCIA
[42 SCRA 448; L-33964; 11 Dec 1971] Petitioners contention that CPP-NPA has no ability, is negatived by the killing of 5 mayors, 20 barrio
captains and 3 chiefs of police; that there were fourteen (14) meaningful bombing incidents in the
Facts: Greater Manila Area in 1970. CPP has managed to infiltrate or establish and control nine major labor
organizations; has exploited the (11) major student or youth organizations; about thirty (30) mass
In the evening of August 21, 1971, at about 9 p.m., while the Liberal Party of the Philippines was organizations actively advancing the CPP.
holding a public meeting at Plaza Miranda, Manila, for the presentation of its candidates in the general
elections scheduled for November 8, 1971, two hand grenades were thrown at the platform where said
candidates and other persons were. Eight persons were killed and many more injured. Proclamation
889 was issued by the President suspending privilege of writ of habeas corpus stating that there is a
conspiracy of rebellion and insurrection in order to forcibly seize political power. Petitions for writ of
habeas corpus were filed by persons (13) who have been arrested without a warrant.

It was stated that one of the safeguards of the proclamation was that it is to be applied to persons
caught in flagrante delicto. Incidentally, Proc. 889-A was issued as an amendment, inserting the word
“actually staging”. Proc. 889-B was also issued lifting the suspension of privilege in 27 provinces, 3 sub-
provinces and 26 cities. Proc. 889-C was issued restoring the suspension in 13 provinces and
cities(mostly in Mindanao). Proc. 889-D further lifted the suspension in 7 provinces and 4 cities. Only 18
provinces and sub-provinces and 2 cities whose privilege was suspended. Petitioners maintained that
Proclamation No. 889 did not declare the existence of actual "invasion insurrection or rebellion or
imminent danger thereof, however it became moot and academic since it was amended. Petitioners
further contend that public safety did not require the issuance of proclamations stating: (a) that there is
no rebellion; (b) that, prior to and at the time of the suspension of the privilege, the Government was
functioning normally, as were the courts; (c) that no untoward incident, confirmatory of an alleged July-
August Plan, has actually taken place after August 21, 1971; (d) that the President's alleged
apprehension, because of said plan, is non-existent and unjustified; and (e) that the Communist forces
in the Philippines are too small and weak to jeopardize public safety to such extent as to require the
suspension of the privilege of the writ of habeas corpus.

A resolution was issued by majority of the Court having tentatively arrived at a consensus that it may
inquire in order to satisfy itself of the existence of the factual bases for the proclamations. Now the
Court resolves after conclusive decision reached by majority.

Issue:

Whether or Not the authority to decide whether the exigency has arisen requiring suspension (of the
privilege of the writ of habeas corpus) belongs to the President and his decision is final and conclusive
upon the courts and upon all other persons.

Whether or Not public safety require the suspension of the privilege of the writ of habeas corpus
decreed in Proclamation No. 889-A.

Held:

Page 88
charged. Petitioner filed a Motion to Acquit on the ground that the conduct of the line-up, without notice
RIGHTS OF THE ACCUSED and in the absence of his counsel violated his constitutional rights to counsel and to due process. The
court denied said motion. Hearing was set, hence the petition.

Art 3, Sec. 12. “(1) Any person under investigation for the commission of an offense shall have the Issue:
right to be informed of his right to remain silent and to have competent and independent counsel
preferably of his own choice. If the person cannot afford the services of counsel, he must be provided Whether or Not petitioner’s right to counsel and due process violated.
with one. These rights cannot be waived except in writing and in the presence of counsel.
(2) No torture, force, violence, threat, intimidation, or any other means which vitiate the free will Held:
shall be used against him. Secret detention places, solitary, incommunicado, or other similar forms of
detention are prohibited. No. The police line-up was not part of the custodial inquest, hence, petitioner was not yet entitled, at
(3) Any confession or admission obtained in violation of this or Section 17 hereof shall be such stage, to counsel. He had not been held yet to answer for a criminal offense. The moment there is
inadmissible in evidence against him. a move or even an urge of said investigators to elicit admissions or confessions or even plain
(4) The law shall provide for penal and civil sanctions for violations of this section as well as information which may appear innocent or innocuous at the time, from said suspect, he should then and
compensation to and rehabilitation of victims of torture or similar practices, and their families.” there be assisted by counsel, unless he waives the right, but the waiver shall be made in writing and in
the presence of counsel.
Art 3, Sec. 14. “(1) No person shall be held to answer for a criminal offense without due process of
law. On the right to due process, petitioner was not, in any way, deprived of this substantive and
(2) In all criminal prosecutions, the accused shall be presumed innocent until the contrary is proved, constitutional right, as he was duly represented by a counsel. He was accorded all the opportunities to
and shall enjoy the right to be heard by himself and counsel, to be informed of the nature and cause of be heard and to present evidence to substantiate his defense; only that he chose not to, and instead
the accusation against him, to have a speedy, impartial, and public trial, to meet the witnesses face to opted to file a Motion to Acquit after the prosecution had rested its case. What due process abhors is
face, and to have compulsory process to secure the attendance of witnesses and the production of the absolute lack of opportunity to be heard.
evidence in his behalf. However, after arraignment, trial may proceed notwithstanding the absence of
the accused provided that he has been duly notified and his failure to appear is unjustifiable.”
PEOPLE VS. JUDGE AYSON
Art 3, Sec. 11. “Free access to the courts and quasi-judicial bodies and adequate legal assistance shall [175 SCRA 216; G.R. NO. 85215; 7 JUL 1989]
not be denied to any person by reason of poverty.”
Facts:
Art 3, Sec. 16. “All persons shall have the right to a speedy disposition of their cases before all judicial,
quasi-judicial, or administrative bodies.” Felipe Ramos was a ticket freight clerk of the Philippine Airlines, assigned at its Baguio City station. It
was alleged that he was involved in irregularities in the sales of plane tickets, the PAL management
Art 3, Sec. 17. “No person shall be compelled to be a witness against himself.” notified him of an investigation to be conducted. That investigation was scheduled in accordance with
PAL's Code of Conduct and Discipline, and the Collective Bargaining Agreement signed by it with the
Art 3, Sec. 19. “(1) Excessive fines shall not be imposed, nor cruel, degrading or inhuman punishment Philippine Airlines Employees' Association (PALEA) to which Ramos pertained. A letter was sent by
inflicted. Neither shall death penalty be imposed, unless, for compelling reasons involving heinous Ramos stating his willingness to settle the amount of P76,000. The findings of the Audit team were
crimes, the Congress hereafter provides for it. Any death penalty already imposed shall be reduced to given to him, and he refuted that he misused proceeds of tickets also stating that he was prevented
reclusion perpetua. from settling said amounts. He proffered a compromise however this did not ensue. Two months after a
(2) The employment of physical, psychological, or degrading punishment against any prisoner or crime of estafa was charged against Ramos. Ramos pleaded not guilty. Evidence by the prosecution
detainee or the use of substandard or inadequate penal facilities under subhuman conditions shall be contained Ramos’ written admission and statement, to which defendants argued that the confession
dealt with by law.” was taken without the accused being represented by a lawyer. Respondent Judge did not admit those
stating that accused was not reminded of his constitutional rights to remain silent and to have counsel.
Art 3, Sec. 21. “No person shall be twice put in jeopardy of punishment for the same offense. If an act A motion for reconsideration filed by the prosecutors was denied. Hence this appeal.
is punished by a law and an ordinance, conviction or acquittal under either shall constitute a bar to
another prosecution for the same act.” Issue:

GAMBOA VS. CRUZ Whether or Not the respondent Judge correct in making inadmissible as evidence the admission and
[162 SCRA 642;L-56291; 27 JUN 1988] statement of accused.

Facts: Held:

Petitioner was arrested for vagrancy without a warrant. During a line-up of 5 detainees including
petitioner, he was identified by a complainant to be a companion in a robbery, thereafter he was
Page 89
No. Section 20 of the 1987 constitution provides that the right against self-incrimination (only to
witnesses other than accused, unless what is asked is relating to a different crime charged- not present No. The Sinumpaang Salaysay is inadmissible because it was in clear violation of the constitutional
in case at bar). rights of the accused. First, he was not informed of his right to remain silent and his right to counsel.
Second, he cannot be compelled to be a witness against himself. At the time of the confession, the
This is accorded to every person who gives evidence, whether voluntarily or under compulsion of accused was already facing charges in court. He no longer had the right to remain silent and to counsel
subpoena, in any civil, criminal, or administrative proceeding. The right is not to "be compelled to be a but he had the right to refuse to be a witness and not to have any prejudice whatsoever result to him by
witness against himself.” It prescribes an "option of refusal to answer incriminating questions and not a such refusal. And yet, despite his knowing fully well that a case had already been filed in court, he still
prohibition of inquiry." the right can be claimed only when the specific question, incriminatory in confessed when he did not have to do so.
character, is actually put to the witness. It cannot be claimed at any other time. It does not give a
witness the right to disregard a subpoena, to decline to appear before the court at the time appointed, The contention of the trial court that the accused is not entitled to such rights anymore because the
or to refuse to testify altogether. It is a right that a witness knows or should know. He must claim it and information has been filed and a warrant of arrest has been issued already, is untenable. The exercise
could be waived. of the rights to remain silent and to counsel and to be informed thereof under Section 12(1) of the Bill of
Rights are not confined to that period prior to the filing of a criminal complaint or information but are
Rights in custodial interrogation as laid down in miranda v. Arizona: the rights of the accused include: available at that stage when a person is "under investigation for the commission of an offense."

1) he shall have the right to remain silent and to counsel, and to be informed of such right. Pursuant to Section 12(3) of the Bill of Rights therefore, such extra-judicial admission is inadmissible as
2) nor force, violence, threat, intimidation, or any other means which vitiates the free will shall be evidence.
used against him.
3) any confession obtained in violation of these rights shall be inadmissible in evidence. As to the admissions made by Maqueda to Prosecutor Zarate and Ray Dean Salvosa, the trial court
admitted their testimony thereon only to prove the tenor of their conversation but not to prove the truth
The individual may knowingly and intelligently waive these rights and agree to answer or make a of the admission because such testimony was objected to as hearsay. Maqueda voluntarily and freely
statement. But unless and until such rights and waivers are demonstrated by the prosecution at the made them to Prosecutor Zarate not in the course of an investigation, but in connection with Maqueda's
trial, no evidence obtained as a result of interrogation can be used against him. plea to be utilized as a state witness; and as to the other admission (Salvosa), it was given to a private
person therefore admissible.

PEOPLE VS. MAQUEDA Note: a distinction between a confession and admission has been made by the SC:
[242 SCRA 565; G.R. NO.112983; 22 MAR 1994] Admission of a party. — The act, declaration or omission of party as to a relevant fact may be given in
evidence against him.
Facts:
Confession. — The declaration of an accused acknowledging his guilt of the offense charged, or of any
British Horace William Barker (consultant of WB) was slain inside his house in Tuba, Benguet while his offense necessarily included therein, may be given in evidence against him.
Filipino wife, Teresita Mendoza was badly battered with lead pipes on the occasion of a robbery. Two
household helpers of the victims identified Salvamante (a former houseboy of the victims) and
Maqueda as the robbers. Mike Tabayan and his friend also saw the two accused a kilometer away PEOPLE VS. BANDULA
from the house of the victims that same morning, when the two accused asked them for directions. [232 SCRA 566; G.R. NO. 89223; 27 MAY 1994]

Maqueda was then arrested in Guinyangan, Quezon. He was taken to Calauag, Quezon where he Facts:
signed a Sinumpaang Salaysay wherein he narrated his participation in the crime. According to SPO3
Molleno, he informed Maqueda of his constitutional rights before he signed such document. Afterwards Six armed men barged into the compound of Polo Coconut Plantation in Tanjay, Negros Oriental. The
he was brought to the Benguet Provincial Jail. While he was under detention, Maqueda filed a Motion armed men were identified by Security Guard, including accused. Salva and Pastrano, security guards
to Grant Bail. He stated therein that "he is willing and volunteering to be a State witness in the above were hogtied and accused proceeded to the Atty. Garay, counsel of plantation. They ransacked the
entitled case, it appearing that he is the least guilty among the accused in this case." place and took with them money and other valuables. Atty. Garay was killed. Accused-appellant is
charged with robbery with homicide along with 3 others who were acquitted for insufficiency of
Maqueda also admitted his involvement in the commission of the robbery to Prosecutor Zarate and to evidence. Appellant was convicted.
Salvosa.
Now, appellant argues that the extrajudicial confessions he and accused Dionanao executed suffer
Issue: from constitutional infirmities, hence, inadmissible in evidence considering that they were extracted
under duress and intimidation, and were merely countersigned later by the municipal attorney who, by
Whether or Not the trial court was correct in holding that the Sinumpaan Salaysay is admissible as the nature of his position, was not entirely an independent counsel nor counsel of their choice.
evidence. Consequently, without the extrajudicial confessions, the prosecution is left without sufficient evidence to
convict him of the crime charged.
Held:
Page 90
Issue:
The three accused denied complicity in the crime charged.
Whether or Not extrajudicial confessions of appellant is admissible as evidence against him.
Appellant Lucero's defense is alibi. He testified that he was at his house in Caloocan City.
Held:
He said he was surprised when several unidentified men accosted him while he was walking towards
No. When accused-appellant Bandula and accused Dionanao were investigated immediately after their his house. They chased him, handcuffed and blindfolded him and pushed him into a jeep. He was
arrest, they had no counsel present. If at all, counsel came in only a day after the custodial investigation blindfolded the whole night and did not know where he was taken. The men turned out to be police
with respect to accused Dionanao, and two weeks later with respect to appellant Bandula. And, counsel officers.
who supposedly assisted both accused was Atty. Ruben Zerna, the Municipal Attorney of Tanjay. On The next day, he learned he was in Camp Crame. He claimed that he was tortured. He was not
top of this, there are telltale signs that violence was used against the accused. Certainly, these are informed of the offense for which he was being investigated. Neither did they reveal the identity of the
blatant violations of the Constitution which mandates in complainant.
Sec. 12, Art. III. Irregularities present include:
Lucero denied knowing Dr. Madrid, the Echavez brothers and the other accused in this case. He said
1. The investigators did not inform the accused of their right to remain silent and to have he only met Dr. Madrid at the CIS Office during the police line-up. He was made to line-up four (4) times
competent and independent counsel, preferably of their own choice, even before attempting before Dr. Madrid finally identified him on the fourth time.
to elicit statements that would incriminate them.
2. Investigators continuously disregard the repeated requests of the accused for medical Lucero also claimed he signed the extrajudicial confession under duress. He denied engaging the
assistance. Reason for Accused Sedigo’s "black eye" which even services of Atty, Peralta. He likewise confirmed that Atty. Peralta was not present during his actual
Pat. Baldejera admitted is not established, as well as Bandula’s fractured rib. custodial interrogation.
3. Counsel must be independent. He cannot be a special counsel, public or private prosecutor,
counsel of the police, or a municipal attorney whose interest is admittedly adverse to the After trial, the court a quo acquitted the Echavez brothers for insufficient evidence. The trial court,
accused. however, convicted accused Lucero GUILTY as principal by direct participation of Robbery with
Homicide and sentenced to suffer an imprisonment term of RECLUSION PERPETUA.

PEOPLE VS. LUCERO Issue:


[244 SCRA 425; G.R. NO.97936; 29 MAY 1995]
Whether or Not the lower court erred in convicting accused-appellant.
Facts:
Held:
Alejandro Lucero, Bienvenido Echavez, Balbino Echavez, Peter Doe, Richard Doe and John Doe were
charged with the crime of robbery with homicide. Appellant's conviction cannot be based on his extrajudicial confession.

The prosecution: Constitution requires that a person under investigation for the commission of a crime should be
provided with counsel. The Court have constitutionalized the right to counsel because of hostility
Accused-appellant (alighted from a gray-reddish car), armed with handgun, blocked the way of the said against the use of duress and other undue influence in extracting confessions from a suspect. Force
complainant who was on board a Mercedes Benz passing along Road 14, Mindanao Avenue, Pag-asa, and fraud tarnish confessions can render them inadmissible.
QC, rob and carry away cash money; one gold necklace with cross pendant, 7 karat; one gold Rolex
watch; one 3 karat gold ring; one 2 karat gold ring, domino style; one solid gold bracelet; all worth The records show that Atty. Peralta, who was not the counsel of choice of appellant. Atty. Peralta
P363,600.00, belonging to DR. DEMETRIO Z. MADRID. Accused shot LORENZO BERNALES y himself admitted he received no reaction from appellant although his impression was that appellant
ALERIA, a driver of the said offended party, thus inflicting upon him mortal wounds, which resulted to understood him. More so, it was during his absence that appellant gave an uncounselled confession.
the instantaneous death of ALERIA.
Constitution requires the right to counsel, it did not mean any kind of counsel but effective and vigilant
Only the accused Echavez brothers and Alejandro Lucero were apprehended. counsel. The circumstances clearly demonstrate that appellant received no effective counseling from
Atty. Peralta.
When Lucero told him that he had no lawyer, in due time, Atty. Diosdado Peralta conferred with Lucero.
He apprised Lucero of his constitutional rights. He observed no reaction from Lucero. Nonetheless, Whereof, Decision convicting appellant Alejandro Lucero y Cortel is hereby reversed.
Atty. Peralta gathered the impression that Lucero understood his advice.

When the investigator started asking the preliminary questions, Atty. Peralta left to attend the wake of PEOPLE VS. AGUSTIN
his friend. The next morning, Lucero was accompanied by CIS agents to Atty. Peralta's house. The [240 SCRA 541; G.R. NO. 110290; 25 JAN 1995]
extrajudicial statement of Lucero was presented to Atty. Peralta. It was already signed by Lucero.
Page 91
Facts: immediately informed him that Atty. Cajucom was ready to assist him. Moreso said counsel is not
independent since he is an associate of the private prosecutor.
Dr. Bayquen, a dentist, together with his son, Anthony; Anthony's girlfriend, Anna Theresa; his
daughter, Dominic; and Danny, a family friend, were on their way aboard their Brasilia to the doctor's
residence at Malvar Street, Baguio City. While they were cruising along Malvar Street and nearing the PEOPLE VS. BOLANOS
Baptist church, a man came out from the right side of a car parked about two meters to the church. The [211 SCRA 262; G.R. NO. 101808; 3 JUL 1992]
man approached the Brasilia, aimed his armalite rifle through its window, and fired at the passengers.
The Brasilia swerved and hit a fence. The gunman immediately returned to the parked car which then Facts:
sped away. All those in the car were hit and Dr. Bayquen and Anna Theresa died on the spot. Dominic
was bale to get out of the Brasilia to run to the Alabanza store where she telephoned her mother. Later, Oscar Pagdalian was murdered in Marble Supply, Balagtas Bulacan. According to Pat. Rolando
she and her mother brought her father and Anthony to the hospital. Danny went home and was then Alcantara and Francisco Dayao, deceased was with two companions on the previous night, one of
brought to the Hospital for treatment. whom the accused who had a drinking spree with the deceased. When they apprehended the accused
they found the firearm of the deceased on the chair where the accused was allegedly seated. They
Accused Quiaño, an alleged former military agent who had been picked up by the police authorities, boarded accused along with Magtibay, other accused on the police vehicle and brought them to the
confessed during the investigation conducted by Baguio City Fiscal Erdolfo Balajadia in his office that police station. While in the vehicle Bolanos admitted that he killed the deceased. RTC convicted him
he was the triggerman. He implicated Abenoja, Jr., who engaged him to kill Dr. Bayquen for a fee, hence the appeal.
Cartel, who provided the armalite, and a certain "Jimmy." During the investigation, Wilfredo Quiaño was
assisted by Atty. Reynaldo Cajucom. Stenographic notes of the proceedings during the investigation as Issue:
transcribed with the sworn statement of Quiaño was signed, with the assistance of Atty. Cajucom, and
swore to before City Fiscal Balajadia. The following day, Agustin was apprehended, and was Whether or Not accused-appellant deprived of his constitutional right to counsel.
investigated and was afforded the privileges like that of Quijano. Agustin’s defense interpose that he
was forced to admit involvement at gunpoint in the Kennon Road. He further declared that although he Held:
was given a lawyer, Cajucom (a law partner of the private prosecutor), he nevertheless, asked for his
uncle Atty. Oliver Tabin, and that Atty. Cajucom interviewed him from only two minutes in English and Yes. Being already under custodial investigation while on board the police patrol jeep on the way to the
Tagalog but not in Ilocano, the dialect he understands. The promise that he would be discharged as a Police Station where formal investigation may have been conducted, appellant should have been
witness did not push through since Quijano escaped. However the RTC convicted him, since informed of his Constitutional rights under Article III, Section 12 of the 1987 Constitution, more
conspiracy was established. Hence the appeal. particularly par. 1 and par. 3.

Issue:
PEOPLE VS. MACAM
Whether or Not accused-appellant’s extrajudicial statements admissible as evidence. [238 SCRA 306; G.R. NOS. 91011-12; 24 NOV 1994]

Held: Facts:

No. Extrajudicial statement is not extrajudicial confession. In a confession, there is an acknowledgment


Prosecution’s version:
of guilt of the accused, while an admission is a statement direct or implied of facts pertinent to the
issue. The rule on inadmissibility, however expressly includes admissions, not just confessions.The
On Aug 18,1987, Eduardo Macam, Antonio Cedro, Eugenio Cawilan Jr., Danilo Roque and Ernesto
extrajudicial admission of the appellant, contained in twenty-two pages appear to be signed by him and
Roque went to the house of Benito Macam (uncle of Eduardo Macam) located at 43 Ferma Road QC.
Atty. Cajucom but for reasons not explained in the records, the transcript of the notes which consists of
Upon the arrival of the accused, Benito invited the former to have lunch. Benito asked his maid
twelve pages was not signed by the appellant. Since the court cannot even read or decipher the
Salvacion Enrera to call the companions of Eduardo who were waiting in a tricycle outside the house.
stenographic notes it cannot be expected that appellant, who is a farmer and who reached only the
A. Cedro, E. Cawilan and D. Roque entered the house while E. Roque remained in the tricycle. After all
fourth grade, to read or decipher its contents. The appellant, therefore was deprived of his rights under
the accused had taken their lunch, Eduardo Macam grabbed the clutch bag of Benito Macam and
Section 12(1), Article III of the Constitution. Firstly, he was not fully and properly informed of his rights.
pulled out his uncle’s gun then declared a hold-up. They tied up the wife (Leticia Macam), children,
The appellant was not explicitly told of his right to have a competent and independent counsel of his
maid (Salvacion) and Nilo Alcantara and brought them to the room upstairs. After a while Leticia was
choice, specifically asked if he had in mind any such counsel and, if so, whether he could afford to hire
brought to the bathroom and after she screamed she was stabbed and killed by A. Cedro. Benito, Nilo
his services, and, if he could not, whether he would agree to be assisted by one to be provided for him.
and Salvacion was also stabbed but survived. The total value of the items taken was P536, 700.00.
He was not categorically informed that he could waive his rights to remain silent and to counsel and that
this waiver must be in writing and in the presence of his counsel. He had, in fact, waived his right to
Defense’s version:
remain silent by agreeing to be investigated. Yet, no written waiver of such right appears in the
transcript and no other independent evidence was offered to prove its existence. In short, after the
Danilo Roque stated that he being a tricycle driver drove the 4 accused to Benito’s house for a fee of
appellant said that he wanted to be assisted by counsel, the City fiscal, through suggestive language,
P50.00. Instead of paying him, he was given a calling card by Eduardo Macam so that he can be paid
the following day. Upon arriving, he went with the accused inside the house to have lunch. Thereafter
Page 92
he washed the dishes and swept the floor. When Eugenio Cawilan pulled a gun and announced the charged with the Murder With the Use of Unlicensed firearms. Appellant alleges that he carried the
hold-up, he was asked to gather some things and which he abided out of fear. While putting the said victim to the shore to be brought to the hospital to save the latter, and who facilitated the surrender to
thins inside the car of Benito (victim) he heard the accused saying “kailangan patayin ang mga taong Pat. Padilla a gun which his helper found the following morning while cleaning the bar. Accused posted
yan dahil kilala ako ng mga yan”. Upon hearing such phrase he escaped and went home using his bail which was granted. The accused denied having made any oral confession alleging that he went to
tricycle. He also testified that his brother Ernesto Roque has just arrived from the province and in no Pat. Padilla not to report the incident but to state that a boy helper in the bar had found a gun on the
way can be involved in the case at bar. On the following day, together with his brother, they went to the sand floor while cleaning and that Pat. Padilla picked up the gun from the bar at his request. The
factory of the Zesto Juice (owned by the father of Eduardo Macam) for him to get his payment (50.00) . Accused argues that even if he did make such a confession, the same would be inadmissible in
He and his brother was suddenly apprehended by the security guards and brought to the police evidence. He was found guilty in the RTC. Hence the appeal.
headquarters in Q.C. They were also forced to admit certain things.
Issue:
After which, he together with all the accused, in handcuffs and bore contusions on their faces caused
by blows inflicted in their faces during investigation, was brought to the QC General Hospital before Whether or Not the lower court correct in saying that the constitutional procedure on custodial
each surviving victims and made to line-up for identification. Eugenio Cawilan was also charged with interrogation is not applicable in the instant case.
Anti-fencing Law but was acquitted in the said case.
Held:
Issue:
YES. Appellant's assertion that the gun he had surrendered was merely found by a boy helper while
Whether or Not their right to counsel has been violated. WON the arrest was valid. WON the evidence cleaning the bar deserves no credence for, if it were so, it would have been absurd for him to have
from the line-up is admissible. placed himself under police custody in the early morning after the incident. Sworn Complaint for
"Murder with Use of Unlicensed Firearm" signed by the Chief of Police also attests to Appellant's oral
Held: confession. That Complaint forms part of the record of the proceedings before the Municipal Circuit
Trial Court of Buruanga, Aklan, and is prima facie evidence of the facts therein stated. Appellant's
It is appropriate to extend the counsel guarantee to critical stages of prosecution even before trial. A voluntary surrender implies no violation as "no warrant of arrest is issued for the apprehension of the
police line-up is considered a “critical” stage of the proceedings. Any identification of an uncounseled accused for the reason that he is already under police custody before the filing of the complaint." What
accused made in a police line-up is inadmissible. HOWEVER, the prosecution did not present evidence was told by the Accused to Pat, Padilla was a spontaneous statement not elicited through questioning,
regarding appellant’s identification at the line-up. The witnesses identified the accused again in open but given in ordinary manner. No written confession was sought to be presented in evidence as a result
court. Also, accused did not object to the in-court identification as being tainted by illegal line-up. of formal custodial investigation.

The arrest of the appellants was without a warrant. HOWEVER, they are estopped from questioning the
legality of such arrest because they have not moved to quash the said information and therefore
voluntarily submitted themselves to the jurisdiction of the trial court by entering a plea of not guilty and NAVALLO VS. SANDIGANBAYAN
participating in trial. [234 SCRA 177; G.R. NO. 97214; 18 JUL 1994]

The court believed the version of the prosecution. Ernesto Roque, while remaining outside the house Facts:
served as a looked out.
Accused was the Collecting and Disbursing Officer of the Numancia National Vocational School, which
Wherefore, decision of lower court is Affirmed. Danilo Roque and Ernesto Roque is guilty of the crime school is also located at del Carmen, Surigao del Norte. His duties included the collection of tuition
of robbery with homicide as co-conspirators of the other accused to suffer reclusion perpetua. fees, preparation of vouchers for salaries of teachers and employees, and remittance of collections
exceeding P500.00 to the National Treasury. An information for malversation of public funds was filed.
Things taken: 2 toygun, airgun riffle, CO2 refiller, TV, betamax tapes, betamax rewinder, Samsonite A warrant of arrest was issued, but accused-petitioner could not be found. on 10 December 1978,
attache case, typewriter, chessboard, TOYOTA Crown Car Plate No. CAS-997, assorted jewelry. .22 Presidential Decree No. 1606 took effect creating the Sandiganbayan and conferring on it original and
gun and money. exclusive jurisdiction over crimes committed by public officers embraced in Title VII of the Revised
Penal Code. On 15 November 1984, Navallo was finally arrested. He was released on provisional
liberty upon the approval of his property bail bond. When arraigned by the RTC on 18 July 1985, he
PEOPLE VS. DY pleaded not guilty. Upon motion of the prosecution, the RTC transferred the case and transmitted its
[158 SCRA 111; G.R. 74517; 23 FEB 1988] records to the Sandiganbayan. Special Prosecutor Luz L. Quiñones-Marcos opined that since Navallo
had already been arraigned before the case was transferred to the Sandiganbayan, the RTC should
Facts: continue taking cognizance of the case. The matter was referred to the Office of the Ombudsman which
held otherwise. The information was then docketed with the Sandiganbayan. A new order for Navallo's
Pat. Padilla reported along with Benny Dy, with caliber .38 as suspect to the shooting incident at arrest was issued by the Sandiganbayan. The warrant was returned with a certification by the RTC
"Benny's Bar," at Sitio Angol, Manoc-Manoc Malay, Aklan (Boracay) situated on the Island which Clerk of Court that the accused had posted a bail bond. Navallo filed a motion to quash, contending (1)
caused the death of Christian Langel Philippe, tourist, 24 years old and a Swiss nationale. He was that the Sandiganbayan had no jurisdiction over the offense and the person of the accused and (2) that
Page 93
since the accused had already been arraigned by the RTC, the attempt to prosecute him before the No. The records do not reveal that the Information against the appellant was read in the language or
Sandiganbayan would constitute double jeopardy. However this was denied and trial ensued and he dialect known to him. The Information against the appellant is written in the English language. It is
was found guilty. unknown whether the appellant knows the English language. Neither is it known what dialect is
understood by the appellant. Nor is there any showing that the Information couched in English was
Issue: translated to the appellant in his own dialect before his plea of guilt. The RTC violated section 1(a) of
Rule 116, the rule implementing the constitutional right of the appellant to be informed of the nature and
Whether or Not the constitutional right against double jeopardy and in custodial investigations in favor cause of the accusation against him. It also denied appellant his constitutional right to due process of
of the accused violated. law. It is urged that we must presume that the arraignment of the appellant was regularly conducted.
When life is at stake, we cannot lean on this rebuttable presumption. There could be no presumption.
Held: The court must be sure.

No. Double jeopardy requires the existence of the following requisites: The trial court violated section 3 of Rule 116 when it accepted the plea of guilt of the appellant. Said
section requires that the court shall conduct a searching inquiry the voluntariness and full
(1) The previous complaint or information or other formal charge is sufficient in form and comprehension of the consequences of his plea and require the prosecution to prove his guilt and the
substance to sustain a conviction; precise degree of culpability. The accused may also present evidence in his behalf. The trial court
(2) The court has jurisdiction to try the case; simply inquired if appellant had physical marks of maltreatment. It did not ask the appellant when he
(3) The accused has been arraigned and has pleaded to the charge; and was arrested, who arrested him, how and where he was interrogated, whether he was medically
(4) The accused is convicted or acquitted or the case is dismissed without his express consent. examined before and after his interrogation, etc. It limited its efforts trying to discover late body marks
of maltreatment as if involuntariness is caused by physical abuse alone.
The RTC was devoid of jurisdiction when it conducted an arraignment of the accused which by then
had already been conferred on the Sandiganbayan. Moreover, neither did the case there terminate with Further, there are physical evidence to prove Khazie was raped. These consists of a pillow with
conviction or acquittal nor was it dismissed. bloodstains in its center 14 and the T-shirt 15 of the accused colored white with bloodstains on its
bottom. These physical evidence are evidence of the highest order. They strongly corroborate the
No. Appellant is not in custodial investigation. A person under a normal audit examination is not under testimony of Luisa Rebada that the victim was raped.These are inadmissible evidence for they were
custodial investigation. An audit examiner himself can hardly be deemed to be the law enforcement gathered by PO3 Danilo Tan of the Iloilo City PNP as a result of custodial interrogation where appellant
officer contemplated in the above rule. In any case, the allegation of his having been "pressured" to verbally confessed to the crime without the benefit of counsel.
sign the Examination Report prepared by Dulguime (examined cash, as ordered by Espino, the
provincial auditor) appears to be belied by his own testimony.
PEOPLE VS. DE GUZMAN
[224 SCRA 93; G.R. NOS. 98321-24; 30 JUN 1993]
PEOPLE VS. ALICANDO
[251 SCRA 293; G.R. NO. 117487; 2 DEC 1995] Facts:

Facts: All the accused were charged before the Regional Trial Court of Cebu with three counts of murder and
one count of frustrated murder in four Informations. The victim Jose Bantug was found with gunshots in
Appellant was charged with the crime of rape with homicide of Khazie Mae Penecilla, a minor, four the head, body, and skull. The other three informations charged them with the murder of Francisco
years of age, choking her with his right hand. The incident happened after appellant drank liquor. A Carteciano y Sorilla and Antonio S. Carteciano, and the frustrated murder of Lorna V. Carteciano. The
neighbor, Leopoldo Santiago found the victim’s body and the parents and police were informed. other 8 accused were acquitted on the ground of reasonable doubt, while Victor Nuñez was found
Appellant was living in his uncle's house some five arm's length from Penecilla's house. Appellant was guilty. The facts shown by evidence are: One morning, Major Antonio Carteciano was driving his private
arrested and interrogated by PO3 Danilo Tan. He verbally confessed his guilt without the assistance of jeep Camp General Arcadio Maxilom in Lahug, Cebu City where he was stationed as medical officer of
counsel. On the basis of his uncounselled verbal confession and follow up interrogations, the police the PC/INP Provincial Command. In the front seat with him is his wife Lorna, and at the backseat are
came to know and recovered from appellant's house, Khazie Mae's green slippers, a pair of gold his mother in law, son, brother Francisco, neighbor Bantug, and Bantug’s son. Near the intersection,
earrings, a buri mat, a stained pillow and a stained T-shirt all of which were presented as evidence for gunshots were heard from the left side of the street. Major Carteciano took his .45 cal pistol and fired.
the prosecution. He was arraigned with the assistance of Atty. Rogelio Antiquiera of the PAO. Appellant However, gunshots were fired in succession, and Major Carteciano, his brother Francisco, Jose
pleaded guilty. The RTC convicted him. Hence an automatic review for the imposition of death penalty. Bantug, and his wife Lorna were hit. When the jeep stopped, several gunmen approached them. Nuñez
demanded Lorna to give Nuñez her husband’s pistol. Lorna asked to take her valuables instead. Then,
Issue: Nuñez shot Major Carteciano’s head point blank. Then the gunmen hijacked another jeep and took off.
Lorna, her mother Juanita Ricaplaza, and her son Reiser Carteciano positively identified the accused.
Whether or Not the death penalty proper. Lorna identified Nuñez as the one who shot her husband. Nuñez claimed that his arrest was illegal and
that he was deprived of his right to counsel when he was subjected to a paraffin test without the
Held: assistance of counsel.

Page 94
Issue: 1. The accused has evaded the authorities for thirteen years and was an escapee from
detention when arrested; (Chairman of CPP-NPA)
Whether or not the accused Nuñez’s constitutional right was violated 2. He was not arrested at his residence as he had no known address;
3. He was using the false name "Manuel Mercado Castro" at the time of his arrest and
presented a Driver's License to substantiate his false identity;
Held: 4. The address he gave "Panamitan, Kawit, Cavite," turned out to be also a false address;
5. He and his companions were on board a private vehicle with a declared owner whose identity
No. Nuñez pleaded not guilty at the arraignment. Therefore, he is estopped from questioning the and address were also found to be false;
validity of his arrest. Furthermore, the illegal arrest of an accused is not sufficient cause for setting 6. Pursuant to Ministry Order No. 1-A dated 11 January 1982 , a reward of P250,000.00 was
aside a valid judgment rendered upon a sufficient complaint after trial free from error. The witnesses offered and paid for his arrest.
also positively identified the accused, so he cannot question the credibility of the witnesses. Regarding
his right to counsel, the Supreme Court held that-- the right to counsel attaches only upon the start of This however was denied. Hence the appeal.
an investigation, that is, when the investigating officer starts to ask questions to elicit information and/or
confessions or admissions from the accused. At such point or stage, the person being interrogated Issue:
must be assisted by counsel to avoid the pernicious practice of extorting false or coerced admissions or
confessions from the lips of the person undergoing interrogation. In the case at bar, when accused was Whether or Not the private respondent has the right to bail.
subjected to a paraffin test, he was not then under custodial investigation. Accused-appellant also
argued that since his co-accused were acquitted, then their acquittal negates conspiracy among them, Held:
and he should not be convicted with the charges filed. However, the Court held that conspiracy was still
proven by the evidence, and the other co-accused were acquitted only because there was reasonable Yes. Bail in the instant case is a matter of right. It is absolute since the crime is not a capital offense,
doubt. Therefore, accused-appellant is still convicted of the four charges against him. therefore prosecution has no right to present evidence. It is only when it is a capital offense that the
right becomes discretionary. However it was wrong for the Judge to change the amount of bail from
We, therefore, find that the conviction of accused-appellant for the crimes charged has been 30K to 50K without hearing the prosecution.
established beyond reasonable doubt and the penalty imposed is in accordance with law. However, the
civil indemnity imposed by the trial court should be increased to P50,000 in conformity with our recent Republic Act No. 6968 approved on 24 October 1990, providing a penalty of reclusion perpetua to the
rulings on the matter. crime of rebellion, is not applicable to the accused as it is not favorable to him.

WHEREFORE, except for the modification that the civil indemnity to be paid by accused-appellant
Accused validly waived his right to bail in another case(petition for habeas corpus). Agreements were
Victor Nuñez, Jr. to the heirs of each victim who died is hereby increased to P50,000, the appealed
made therein: accused to remain under custody, whereas his co-detainees Josefina Cruz and Jose Milo
decision is hereby affirmed in all other respects, with costs against accused-appellant
Concepcion will be released immediately, with a condition that they will submit themselves in the
jurisdiction of the court. Said petition for HC was dismissed. Bail is the security given for the release of
a person in custody of the law. Ergo, there was a waiver. We hereby rule that the right to bail is another
PEOPLE VS. JUDGE DONATO
of the constitutional rights which can be waived. It is a right which is personal to the accused and whose
[198 SCRA 130; G.R. NO.79269; 5 JUN 1991]
waiver would not be contrary to law, public order, public policy, morals, or good customs, or prejudicial
to a third person with a right recognized by law.
Facts:

Private respondent and his co-accused were charged of rebellion on October 2, 1986 for acts
CARPIO VS. MAGLALANG
committed before and after February 1986. Private respondent filed with a Motion to Quash alleging
[196 SCRA 41; G.R. NO. 78162; 19 APR 1991]
that: (a) the facts alleged do not constitute an offense; (b) the Court has no jurisdiction over the offense
charged; (c) the Court has no jurisdiction over the persons of the defendants; and (d) the criminal action
Facts:
or liability has been extinguished. This was denied. May 9, 1987 Respondent filed a petition for bail,
which was opposed that the respondent is not entitled to bail anymore since rebellion became a capital
On January 8, 1987, information for the murder of Mayor Jose Payumo of Dinalupihan Bataan was filed
offense under PD 1996, 942 and 1834 amending ART. 135 of RPC. On 5 June 1987 the President
against Escaño and ten other unindentified persons by the provincial fiscal in the RTC of Bataan at
issued Executive Order No. 187 repealing, among others, P.D. Nos. 1996, 942 and 1834 and restoring
Balanga. Four days later, the Acting Executive Judge of said court issued an order of arrest against
to full force and effect Article 135 of the Revised Penal Code as it existed before the amendatory
Escaño recommending no bail for his provisional liberty. Pat. Cesar Diego who acted on the warrant
decrees. Judge Donato now granted the bail, which was fixed at P30,000.00 and imposed a condition
returned to the court with a certification issued by NBI agent Gonzales, stating therein that accused was
that he shall report to the court once every two months within the first ten days of every period thereof.
still under investigation.
Petitioner filed a supplemental motion for reconsideration indirectly asking the court to deny bail to and
to allow it to present evidence in support thereof considering the "inevitable probability that the accused
Through counsel Rolando T. Cainoy, Escaño filed in court an urgent ex-parte motion for his
will not comply with this main condition of his bail. It was contended that:
commitment at the provincial jail of Bataan on the ground that he wanted to be where his family and

Page 95
counsel could have easy access to him. He alleged therein that his detention at the NBI headquarters to all accused. As the court itself acknowledged in its order of April 2, 1987 that "capital punishment" in
in Manila was irregular and in defiance of the warrant of arrest issued by the court. This was granted. Section 4, Rule 114 has been amended to reclusion perpetua, the court should have proceeded
accordingly: i.e., resolved the application for bail pursuant to Section 13, Article III of the Constitution. It
A motion for reconsideration was filed by Director Carpio stating that the NBI needed physical custody did not have to invoke the abolition of the death penalty and the lack of legislative enactment restoring it
of Escaño for the identification of the other accused in the case who were still the objects of a manhunt in justifying the grant of bail. All it had to do was to determine whether evidence of guilt is strong in the
by NBI agents; that in view of the finding of NBI agents that the other accused and suspects in the case light of the provision of Section 13, Article III.
were subversive elements or members of the New People's Army, it was for the best interest of Escaño
that he be detained at the NBI lock-up cell where security measures were adequate; and that the NBI The RTC has the discretion in the consideration of the strength of the evidence at hand. However, in
would produce the person of Escaño before the court whenever required and every time that there the exercise of said discretion, the court is controlled by the following: first, the applicable provisions of
would be a hearing on the case. However another motion was executed by Escaño stating that he now the Constitution and the statutes; second, by the rules which this Court may promulgate; and third, by
wants to be detained in the NBI, alleging that he did not authorize his counsel to execute the first those principles of equity and justice that are deemed to be part of the laws of the land. 27 The lower
motion. Also, Escaño's counsel Rolando T. Cainoy filed an application for bail stating that Escaño was court not only failed to properly apply the pertinent provisions of the Constitution and the Rules but it
arrested by NBI agents on December 7, 1986 without a warrant having been presented to him and that also disregarded equity and justice by its failure to take into account the factual milieu surrounding the
since then he had been detained in the lock-up cell of the NBI; that said agents, also without a warrant, detention of Escaño
searched his house when he was arrested; that he was subjected to inhuman torture and forced to
admit participation in the killing of Mayor Payumo and to implicate other persons, and that during the
custodial investigation, he was not represented by counsel. In opposing said application, the public PEOPLE VS. FORTES
prosecutor averred that the accused was charged with a capital offense for which no bail may be [223 SCRA 619; G.R. NO. 90643; 25 JUN 1993]
availed of, that the reasons advanced in said application would be overcome by strong and sufficient
evidence; and that during the custodial investigation, he was represented by counsel. The court granted Facts:
the application for bail fixing the same at P30,000, having found no sufficient evidence against accused.
Director Carpio was ordered to justify his actions and so as not to be considered in contempt. Agripino Gine of Barangay Naburacan, Municipality of Matnog, Province of Sorsogon, accompanied his
13-year old daughter, Merelyn, to the police station of the said municipality to report a rape committed
Issue: against the latter by the accused. Following this, the accused was apprehended and charged. A bond
of P25000 was granted for accused’s provisional release. The MCTC found him guilty. An appeal to
Whether or Not the order granting right to bail was proper. RTC was filed, the request for the fixing of bond was denied. Now accused assails denial of bail on the
ground that the same amounted to an undue denial of his constitutional right to bail.
Held:
Issue:
No. The order granting bail had been rendered moot not only by the fact that he had been released
from NBI custody, but also because Escaño jumped bail and did not appear on the date set for his Whether or Not the accused’s right to bail violated.
arraignment. Notwithstanding, the Court resolved the issue of the legality of the order granting bail to
Escaño. Although the right to bail is principally for the benefit of the accused, in the judicial Held:
determination of the availability of said right, the prosecution should be afforded procedural due
process. Thus, in the summary proceeding on a motion praying for admission to bail, the prosecution No. It is clear from Section 13, Article III of the 1987 Constitution and Section 3, Rule 114 of the
should be given the opportunity to present evidence and, thereafter, the court should spell out at least a Revised Rules of Court, as amended, that before conviction bail is either a matter of right or of
resume of the evidence on which its order granting or denying bail is based. Otherwise, the order is discretion. It is a matter of right when the offense charged is punishable by any penalty lower than
defective and voidable. In the case at bar the RTC erred in not summarizing the factual basis of its reclusion perpetua. To that extent the right is absolute. If the offense charged is punishable by reclusion
order granting bail, the court merely stated the number of prosecution witnesses but not their respective perpetua bail becomes a matter of discretion. It shall be denied if the evidence of guilt is strong. The
testimonies, and concluded that the evidence presented by the prosecution was not "sufficiently strong" court's discretion is limited to determining whether or not evidence of guilt is strong. But once it is
to deny bail to Escaño. determined that the evidence of guilt is not strong, bail also becomes a matter of right. If an accused
who is charged with a crime punishable by reclusion perpetua is convicted by the trial court and
The facts, however, that Mayor Payumo was killed on August 20, 1986 when the 1973 Constitution sentenced to suffer such a penalty, bail is neither a matter of right on the part of the accused nor of
allowing the death penalty was still in force and that the application for bail was made on March 5, 1987 discretion on the part of the court.
during the effectivity of the 1987 Constitution which abolished the death penalty, should not have gotten
in the way of resolving the application for bail in accordance with the Constitution and procedural rules.
Section 13, Article III of the Constitution explicitly provides that "(a)ll persons, except those charged COMMENDADOR VS. DE VILLA
with offenses punishable by reclusion perpetua when evidence of guilt is strong, shall before conviction, [200 SCRA 80; G.R. NO. 93177; 2 AUG 1991]
be bailable by sufficient sureties, or be released on recognizance as may be provided by law." As the
phrase "capital offenses" has been replaced by the phrase "offenses punishable by reclusion perpetua, Facts:
25 crimes punishable by reclusion perpetua instead of those punishable by the death penalty, when
evidence of guilt is strong, are the exceptions to the rule that the right to bail should be made available
Page 96
The petitioners in G.R. Nos. 93177 and 96948 who are officers of the AFP were directed to appear in peremptory challenge under article 18 of the articles of war. In G.R. Nos. 95020 and 97454, the
person before the Pre-Trial Investigating Officers for the alleged participation the failed coup on petitions are also granted, and the orders of the respondent courts for the release of the private
December 1 to 9, 1989. Petitioners now claim that there was no pre-trial investigation of the charges as respondents are hereby reversed and set aside. No costs.
mandated by Article of War 71. A motion for dismissal was denied. Now, their motion for
reconsideration. Alleging denial of due process.
MANOTOC VS. COURT OF APPEALS
In G.R. No. 95020, Ltc Jacinto Ligot applied for bail on June 5, 1990, but the application was denied by [142 SCRA 149; G.R. NO. L-62100; 30 MAY 1986]
GCM No.14. He filed with the RTC a petition for certiorari and mandamus with prayer for provisional
liberty and a writ of preliminary injunction. Judge of GCM then granted the provisional liberty. However Facts:
he was not released immediately. The RTC now declared that even military men facing court martial
proceedings can avail the right to bail. Petitioner was charged with estafa. He posted bail. Petitioner filed before each of the trial courts a
motion entitled, "motion for permission to leave the country," stating as ground therefor his desire to go
The private respondents in G.R. No. 97454 filed with SC a petition for habeas corpus on the ground to the United States, "relative to his business transactions and opportunities." The prosecution opposed
that they were being detained in Camp Crame without charges. The petition was referred to RTC. said motion and after due hearing, both trial judges denied the same. Petitioner thus filed a petition for
Finding after hearing that no formal charges had been filed against the petitioners after more than a certiorari and mandamus before the then Court of Appeals seeking to annul the orders dated March 9
year after their arrest, the trial court ordered their release. and 26, 1982, of Judges Camilon and Pronove, respectively, as well as the communication-request of
the Securities and Exchange Commission, denying his leave to travel abroad. He likewise prayed for
Issue: the issuance of the appropriate writ commanding the Immigration Commissioner and the Chief of the
Aviation Security Command (AVSECOM) to clear him for departure. The Court of Appeals denied the
Whether or Not there was a denial of due process. petition.
Petitioner contends that having been admitted to bail as a matter of right, neither the courts which
Whether or not there was a violation of the accused right to bail. granted him bail nor the Securities and Exchange Commission which has no jurisdiction over his liberty
could prevent him from exercising his constitutional right to travel.
Held:
Issue:
NO denial of due process. Petitioners were given several opportunities to present their side at the pre-
trial investigation, first at the scheduled hearing of February 12, 1990, and then again after the denial of Whether or Not the right to bail a matter of right.
their motion of February 21, 1990, when they were given until March 7, 1990, to submit their counter-
affidavits. On that date, they filed instead a verbal motion for reconsideration which they were again Held:
asked to submit in writing. They had been expressly warned in the subpoena that "failure to submit
counter-affidavits on the date specified shall be deemed a waiver of their right to submit controverting The right to bail is a matter of right if the charge is not a capital offense or punishable by reclusion
evidence." Petitioners have a right to pre-emptory challenge. (Right to challenge validity of members of perpetua to death.
G/SCM)
A court has the power to prohibit a person admitted to bail from leaving the Philippines. This is a
It is argued that since the private respondents are officers of the Armed Forces accused of violations of necessary consequence of the nature and function of a bail bond.
the Articles of War, the respondent courts have no authority to order their release and otherwise
interfere with the court-martial proceedings. This is without merit. * The Regional Trial Court has The condition imposed upon petitioner to make himself available at all times whenever the court
concurrent jurisdiction with the Court of Appeals and the Supreme Court over petitions for certiorari, requires his presence operates as a valid restriction on his right to travel.
prohibition or mandamus against inferior courts and other bodies and on petitions for habeas corpus
and quo warranto. Indeed, if the accused were allowed to leave the Philippines without sufficient reason, he may be
placed beyond the reach of the courts.
The right to bail invoked by the private respondents has traditionally not been recognized and is not
available in the military, as an exception to the general rule embodied in the Bill of Rights. The right to a Petitioner has not shown the necessity for his travel abroad. There is no indication that the business
speedy trial is given more emphasis in the military where the right to bail does not exist. transactions cannot be undertaken by any other person in his behalf.

On the contention that they had not been charged after more than one year from their arrest, there was
substantial compliance with the requirements of due process and the right to a speedy trial. The AFP CALLANTA VS. VILLANUEVA
Special Investigating Committee was able to complete the pre-charge investigation only after one year [77 SCRA 377; G.R. NOS. 24646 & L-24674; 20 JUN 1977]
because hundreds of officers and thousands of enlisted men were involved in the failed coup.
Facts:
Accordingly, in G.R. No. 93177, the petition is dismissed for lack of merit. In G.R. No. 96948, the
petition is granted, and the respondents are directed to allow the petitioners to exercise the right of
Page 97
Two complaints for grave oral defamation were filed against Faustina Callanta. The City Judge of Assets and Liabilities for the calendar years 1973, 1976 and 1978. A motion to quash the information
Dagupan City, Felipe Villanueva, denied the motions to quash the complaints. Thus, petitioner Callanta was made alleging that the prosecution deprived accused of due process of law and of the right to a
brought the suits for certiorari in the Supreme Court. Petitioner questions the validity of the issuance of speedy disposition of the cases filed against him. It was denied hence the appeal.
warrant of arrest by respondent, arguing that the City Fiscal should have conducted the preliminary
investigation. According to petitioner’s counsel, there was jurisdictional infirmity. After the issuance of Issue:
the warrants of arrest and the bail fixed at P600, petitioner posted the bail bond, thus obtaining her
provisional liberty. The City Fiscal in this case did not disagree with the judge’s investigation, and Whether or not petitioner was deprived of his rights as an accused.
agreed with the complaints filed.

Issue:
Held:
Whether or Not petitioner’s contentions are to be given merit.
YES. Due process (Procedural) and right to speedy disposition of trial were violated. Firstly, the
Held: complaint came to life, as it were, only after petitioner Tatad had a falling out with President Marcos.
Secondly, departing from established procedures prescribed by law for preliminary investigation, which
Based on many precedent cases of the Supreme Court, “where the accused has filed bail and waived require the submission of affidavits and counter-affidavits by the complainant and the respondent and
the preliminary investigation proper, he has waived whatever defect, if any, in the preliminary their witnesses, the Tanodbayan referred the complaint to the Presidential Security Command for
examination conducted prior to the issuance of the warrant of arrest”. In the case at bar, it is futile for finding investigation and report. The law (P.D. No. 911) prescribes a ten-day period for the prosecutor
the petitioner to question the validity of the issuance of the warrant of arrest, because she posted the to resolve a case under preliminary investigation by him from its termination. While we agree with the
bail bond. Petitioner also erred in arguing that only the City Fiscal can conduct a preliminary respondent court that this period fixed by law is merely "directory," yet, on the other hand, it can not be
investigation. According to the Charter of the City of Dagupan, “the City Court of Dagupan City may disregarded or ignored completely, with absolute impunity. A delay of close to three (3) years can not
also conduct preliminary investigation for any offense, without regard to the limits of punishment, and be deemed reasonable or justifiable in the light of the circumstance obtaining in the case at bar.
may release, or commit and bind over any person charged with such offense to secure his appearance
before the proper court”. Petition for certiorari is denied. Restraining order issued by the Court is lifted
and set aside. GALMAN VS. SANDIGANBAYAN
[144 SCRA 43; G.R. NO.72670; 12 SEP 1986]

TATAD VS. SANDIGANBAYAN Facts:


[159 SCRA 70; G.R. NOS. L-72335-39; 21 MAR 1988]
Assassination of former Senator Benigno "Ninoy" Aquino, Jr. He was killed from his plane that had just
Facts: landed at the Manila International Airport. His brain was smashed by a bullet fired point-blank into the
back of his head by an assassin. The military investigators reported within a span of three hours that
The complainant, Antonio de los Reyes, originally filed what he termed "a report" with the Legal Panel the man who shot Aquino (whose identity was then supposed to be unknown and was revealed only
of the Presidential Security Command (PSC) on October 1974, containing charges of alleged violations days later as Rolando Galman) was a communist-hired gunman, and that the military escorts gunned
of Rep. Act No. 3019 against then Secretary of Public Information Francisco S. Tatad. The "report" was him down in turn.
made to "sleep" in the office of the PSC until the end of 1979 when it became widely known that
Secretary (then Minister) Tatad had a falling out with President Marcos and had resigned from the President was constrained to create a Fact Finding Board to investigate due to large masses of people
Cabinet. On December 12, 1979, the 1974 complaint was resurrected in the form of a formal complaint who joined in the ten-day period of national mourning yearning for the truth, justice and freedom.
filed with the Tanodbayan. The Tanodbayan acted on the complaint on April 1, 1980 which was around
two months after petitioner Tatad's resignation was accepted by Pres. Marcos by referring the The fact is that both majority and minority reports were one in rejecting the military version stating that
complaint to the CIS, Presidential Security Command, for investigation and report. On June 16, 1980, "the evidence shows to the contrary that Rolando Galman had no subversive affiliations. Only the
the CIS report was submitted to the Tanodbayan, recommending the filing of charges for graft and soldiers in the staircase with Sen. Aquino could have shot him; that Ninoy's assassination was the
corrupt practices against former Minister Tatad and Antonio L. Cantero. By October 25, 1982, all product of a military conspiracy, not a communist plot. Only difference between the two reports is that
affidavits and counter-affidavits were in the case was already for disposition by the Tanodbayan. the majority report found all the twenty-six private respondents above-named in the title of the case
However, it was only on June 5, 1985 that a resolution was approved by the Tanodbayan. Five criminal involved in the military conspiracy; " while the chairman's minority report would exclude nineteen of
informations were filed with the Sandiganbayan on June 12, 1985, all against petitioner Tatad alone. (1) them.
Section 3, paragraph (e) of RA. 3019 for giving D' Group, a private corporation controlled by his
brother-in-law, unwarranted benefits, advantage or preference in the discharge of his official functions; Then Pres. Marcos stated that evidence shows that Galman was the killer.
(2) Violation of Section 3, paragraph (b) for receiving a check of P125,000.00 from Roberto Vallar,
President/General Manager of Amity Trading Corporation as consideration for the release of a check of Petitioners pray for issuance of a TRO enjoining respondent court from rendering a decision in the two
P588,000.00 to said corporation for printing services rendered for the Constitutional Convention criminal cases before it, the Court resolved by nine-to-two votes 11 to issue the restraining order prayed
Referendum in 1973; (3) Violation of Section 7 on three (3) counts for his failure to file his Statement of for. The Court also granted petitioners a five-day period to file a reply to respondents' separate
Page 98
comments and respondent Tanodbayan a three-day period to submit a copy of his 84-page The Court then said that the then President (code-named Olympus) had stage-managed in and from
memorandum for the prosecution. Malacañang Palace "a scripted and predetermined manner of handling and disposing of the Aquino-
Galman murder case;" and that "the prosecution in the Aquino-Galman case and the Justices who tried
But ten days later, the Court by the same nine-to-two-vote ratio in reverse, resolved to dismiss the and decided the same acted under the compulsion of some pressure which proved to be beyond their
petition and to lift the TRO issued ten days earlier enjoining the Sandiganbayan from rendering its capacity to resist. Also predetermined the final outcome of the case" of total absolution of the twenty-six
decision. The same Court majority denied petitioners' motion for a new 5-day period counted from respondents-accused of all criminal and civil liability. Pres. Marcos came up with a public statement
receipt of respondent Tanodbayan's memorandum for the prosecution (which apparently was not aired over television that Senator Aquino was killed not by his military escorts, but by a communist hired
served on them). gun. It was, therefore, not a source of wonder that President Marcos would want the case disposed of
in a manner consistent with his announced theory thereof which, at the same time, would clear his
Thus, petitioners filed a motion for reconsideration, alleging that the dismissal did not indicate the legal name and his administration of any suspected guilty participation in the assassination. such a
ground for such action and urging that the case be set for a full hearing on the merits that the people procedure would be a better arrangement because, if the accused are charged in court and
are entitled to due process. subsequently acquitted, they may claim the benefit of the doctrine of double jeopardy and thereby avoid
another prosecution if some other witnesses shall appear when President Marcos is no longer in office.
However, respondent Sandiganbayan issued its decision acquitting all the accused of the crime
charged, declaring them innocent and totally absolving them of any civil liability. Respondents More so was there suppression of vital evidence and harassment of witnesses. The disappearance of
submitted that with the Sandiganbayan's verdict of acquittal, the instant case had become moot and witnesses two weeks after Ninoy's assassination. According to J. Herrera, "nobody was looking for
academic. Thereafter, same Court majority denied petitioners' motion for reconsideration for lack of these persons because they said Marcos was in power. The assignment of the case to Presiding
merit. Justice Pamaran; no evidence at all that the assignment was indeed by virtue of a regular raffle, except
the uncorroborated testimony of Justice Pamaran himself. The custody of the accused and their
Hence, petitioners filed their motion to admit their second motion for reconsideration alleging that confinement in a military camp, instead of in a civilian jail. The monitoring of proceedings and
respondents committed serious irregularities constituting mistrial and resulting in miscarriage of justice developments from Malacañang and by Malacañang personnel. The partiality of Sandiganbayan
and gross violation of the constitutional rights of the petitioners and the sovereign people of the betrayed by its decision: That President Marcos had wanted all of the twenty-six accused to be
Philippines to due process of law. acquitted may not be denied. In rendering its decision, the Sandiganbayan overdid itself in favoring the
presidential directive. Its bias and partiality in favor of the accused was clearly obvious. The evidence
Issue: presented by the prosecution was totally ignored and disregarded.

Whether or not petitioner was deprived of his rights as an accused. The record shows that the then President misused the overwhelming resources of the government and
his authoritarian powers to corrupt and make a mockery of the judicial process in the Aquino-Galman
Whether or not there was a violation of the double jeopardy clause. murder cases. "This is the evil of one-man rule at its very worst." Our Penal Code penalizes "any
executive officer who shall address any order or suggestion to any judicial authority with respect to any
Held: case or business coming within the exclusive jurisdiction of the courts of justice."

Petitioners' second motion for reconsideration is granted and ordering a re-trial of the said cases which Impartial court is the very essence of due process of law. This criminal collusion as to the handling and
should be conducted with deliberate dispatch and with careful regard for the requirements of due treatment of the cases by public respondents at the secret Malacañang conference (and revealed only
process. after fifteen months by Justice Manuel Herrera) completely disqualified respondent Sandiganbayan and
voided ab initio its verdict. The courts would have no reason to exist if they were allowed to be used as
Deputy Tanodbayan Manuel Herrera (made his expose 15 months later when former Pres. was no mere tools of injustice, deception and duplicity to subvert and suppress the truth. More so, in the case
longer around) affirmed the allegations in the second motion for reconsideration that he revealed that at bar where the people and the world are entitled to know the truth, and the integrity of our judicial
the Sandiganbayan Justices and Tanodbayan prosecutors were ordered by Marcos to whitewash the system is at stake.
Aquino-Galman murder case. Malacañang wanted dismissal to the extent that a prepared resolution
was sent to the Investigating Panel. Malacañang Conference planned a scenario of trial where the There was no double jeopardy. Courts' Resolution of acquittal was a void judgment for having been
former President ordered then that the resolution be revised by categorizing the participation of each issued without jurisdiction. No double jeopardy attaches, therefore. A void judgment is, in legal effect,
respondent; decided that the presiding justice, Justice Pamaran, (First Division) would personally no judgment at all. By it no rights are divested. It neither binds nor bars anyone. All acts and all claims
handle the trial. A conference was held in an inner room of the Palace. Only the First Lady and flowing out of it are void.
Presidential Legal Assistant Justice Lazaro were with the President. The conferees were told to take
the back door in going to the room where the meeting was held, presumably to escape notice by the Motion to Disqualify/Inhibit should have been resolved ahead. In this case, petitioners' motion for
visitors in the reception hall waiting to see the President. During the conference, and after an reconsideration of the abrupt dismissal of their petition and lifting of the TRO enjoining the
agreement was reached, Pres. Marcos told them 'Okay, mag moro-moro na lamang kayo;' and that on Sandiganbayan from rendering its decision had been taken cognizance of by the Court which had
their way out of the room Pres. Marcos expressed his thanks to the group and uttered 'I know how to required the respondents', including the Sandiganbayan's, comments. Although no restraining order
reciprocate'. was issued anew, respondent Sandiganbayan should not have precipitately issued its decision of total
absolution of all the accused pending the final action of this Court. All of the acts of the respondent

Page 99
judge manifest grave abuse of discretion on his part amounting to lack of jurisdiction which
substantively prejudiced the petitioner. Facts:

With the declaration of nullity of the proceedings, the cases must now be tried before an impartial court Petitioner Dumlao questions the constitutionality of Sec. 4 of Batas Pambansa Blg 52 as discriminatory
with an unbiased prosecutor. Respondents accused must now face trial for the crimes charged against and contrary to equal protection and due process guarantees of the Constitution. Sec. 4 provides that
them before an impartial court with an unbiased prosecutor with all due process. any retired elective provicial or municipal official who has received payments of retirement benefits and
shall have been 65 years of age at the commencement of the term of office to which he seeks to be
The function of the appointing authority with the mandate of the people, under our system of elected, shall not be qualified to run for the same elective local office from which he has retired.
government, is to fill the public posts. Justices and judges must ever realize that they have no According to Dumlao, the provision amounts to class legislation. Petitioners Igot and Salapantan Jr.
constituency, serve no majority nor minority but serve only the public interest as they see it in also assail the validity of Sec. 4 of Batas Pambansa Blg 52, which states that any person who has
accordance with their oath of office, guided only the Constitution and their own conscience and honor. committed any act of disloyalty to the State, including those amounting to subversion, insurrection,
rebellion, or other similar crimes, shall not be qualified for any of the offices covered by the act, or to
participate in any partisan activity therein: provided that a judgment of conviction of those crimes shall
PEOPLE VS. DRAMAYO be conclusive evidence of such fact and the filing of charges for the commission of such crimes before
[42 SCRA 60; G.R. L-21325; 29 OCT 1971] a civil court or military tribunal after preliminary investigation shall be prima facie evidence of such fact.

Facts: Issue:

Dramayo brought up the idea of killing Estelito Nogaliza so that he could not testify in the robbery case Whether or not the aforementioned statutory provisions violate the Constitution and thus, should be
where he is an accused. The idea was for Dramayo and Ecubin to ambush Estelito, who was returning declared null and void
from Sapao. The others were to station themselves nearby. Only Dramayo and Ecubin were convicted
in the RTC for murder. Hence the appeal Held:

Issue: In regards to the unconstitutionality of the provisions, Sec. 4 of BP Blg 52 remains constitutional and
valid. The constitutional guarantee of equal protection of the laws is subject to rational classification.
Whether or not the accused’s criminal liability proved beyond reasonable doubt. One class can be treated differently from another class. In this case, employees 65 years of age are
classified differently from younger employees. The purpose of the provision is to satisfy the “need for
Held: new blood” in the workplace. In regards to the second paragraph of Sec. 4, it should be declared null
Yes. It is to be admitted that the starting point is the Presumption of innocence. So it must be, and void for being violative of the constitutional presumption of innocence guaranteed to an accused.
according to the Constitution. That is a right safeguarded both appellants. Accusation is not, according “Explicit is the constitutional provision that, in all criminal prosecutions, the accused shall be presumed
to the fundamental law, synonymous with guilt. It is incumbent on the prosecution demonstrate that innocent until the contrary is proved, and shall enjoy the right to be heard by himself and counsel
culpability lies. Appellants were not even called upon then to offer evidence on their behalf. Their (Article IV, section 19, 1973 Constitution). An accusation, according to the fundamental law, is not
freedom is forfeit only if the requisite quantum of proof necessary for conviction be in existence. Their synonymous with guilt. The challenged proviso contravenes the constitutional presumption of
guilt be shown beyond reasonable doubt. What is required then is moral certainty. "By reasonable innocence, as a candidate is disqualified from running for public office on the ground alone that charges
doubt is meant that which of possibility may arise, but it is doubt engendered by an investigation of the have been filed against him before a civil or military tribunal. It condemns before one is fully heard. In
whole proof and an inability, after such investigation, to let the mind rest easy upon the certainty of guilt. ultimate effect, except as to the degree of proof, no distinction is made between a person convicted of
Absolute certain of guilt is not demanded by the law to convict of any carnal charge but moral certainty acts of dislotalty and one against whom charges have been filed for such acts, as both of them would
is required, and this certainty is required as to every proposition of proof regular to constitute the be ineligible to run for public office. A person disqualified to run for public office on the ground that
offense." charges have been filed against him is virtually placed in the same category as a person already
convicted of a crime with the penalty of arresto, which carries with it the accessory penalty of
The judgment of conviction should not have occasioned any surprise on the part of the two appellants, suspension of the right to hold office during the term of the sentence (Art. 44, Revised Penal Code).”
as from the evidence deserving of the fullest credence, their guilt had been more than amply
demonstrated. The presumption of innocence could not come to their rescue as it was more than And although the filing of charges is considered as but prima facie evidence, and therefore, may be
sufficiently overcome by the proof that was offered by the prosecution. The principal contention raised rebutted, yet. there is "clear and present danger" that because of the proximity of the elections, time
is thus clearly untenable. It must be stated likewise that while squarely advanced for the first time, there constraints will prevent one charged with acts of disloyalty from offering contrary proof to overcome the
had been cases where this Court, notwithstanding a majority of the defendants being acquitted, the prima facie evidence against him.
element of conspiracy likewise being allegedly present, did hold the party or parties, responsible for the Additionally, it is best that evidence pro and con of acts of disloyalty be aired before the Courts rather
offense guilty of the crime charged, a moral certainty having arisen as to their capability. than before an administrative body such as the COMELEC. A highly possible conflict of findings
between two government bodies, to the extreme detriment of a person charged, will thereby be
avoided. Furthermore, a legislative/administrative determination of guilt should not be allowed to be
DUMLAO VS. COMELEC substituted for a judicial determination.
[95 SCRA 392; L-52245; 22 JAN 1980]
Page 100
Being infected with constitutional infirmity, a partial declaration of nullity of only that objectionable He was led to another room, where he was handcuffed and left until the following day. Later, he was
portion is mandated. It is separable from the first portion of the second paragraph of section 4 of Batas made to sign prepared statements containing his full confession.
Pambansa Big. 52 which can stand by itself.
Alcantara was arraigned under an information charging him and four others (at large) with the crime of
Wherefore, the first paragraph of section 4 of Batas pambansa Bilang 52 is hereby declared valid and robbery with Homicide and Frustrated Homicide.
that portion of the second paragraph of section 4 of Batas Pambansa Bilang 52 is hereby declared null
and void, for being violative of the constitutional presumption of innocence guaranteed to an accused. The trial court convicted the accused despite the following inconsistency between Venancio’s affidavit
and testimony:

Affidavit Testimony
1. mentioned 5 assailants - only Alcantara was identified
2. stabbing was preceded by a - only 3 assailants had a
3. conference by all assailants - conference
4. claimed to have allowed assailants - failed to identify Alcantara
5. to hitch a ride because Alcantara - at the hospital and in open
PEOPLE VS. ALCANTARA 6. was familiar to them - court (pointed to another person)
[240 SCRA 122; G.R. NO. 91283; 17 JAN 1995]
Issue:
Facts:
Whether or not the rights of the accused was violated.
On July 19, 1988, Venancio Patricio, accompanied by Larry Salvador, drove a ten-wheeler truck a
Coca-Cola plant in Antipolo to load cases of softdrinks. They were about to leave the plant at 10:00pm
when several men approached them to hitch for a ride. Ascertaining that Salvador knew appellant,
Venancio accommodated appellant's request. Appellant had four companions. At Ortigas Ave., one of Held:
them poked a gun at Venancio and grabbed the steering wheel. At the North Diversion Road, Venancio
and Salvador(helper) were brought down from the vehicle and tied to the fence of the expressway, YES. The people’s evidence failed to meet the quantum required to overcome the presumption. The
thereafter they were stabbed and left bleeding to death. Venancio survived but Salvador did second identification which correctly pointed to accused by Venancio should not be credited. There is
not.Appellant was arrested in the vicinity of Otis Street in Pandacan, Manila. A few days later, he was no reason for him to err as they know each other for 3 years. It was also incorrect to give too much
turned over to the Constabulary Highway Patrol Group. Sgt. Alberto Awanan brought the appellant to weight to Police Sgt. Awanan’s testimony as to the “previous identification” at the hospital. The
the MCU hospital and was presented to Venancio for identification. Appellant was brought to the testimony of Sgt. Awanan was not corroborated by Venancio.
Headquarters at Camp Crame where he confessed.
The identification procedure was irregular. Due process demands that the identification procedure of
Appelant’s Defense: Denial and alibi. He said that he was just applying to be a driver and stayed there criminal suspects must be free from impermissible suggestions as the influence of improper suggestion
even if he was told that no work was available, to confirm with the truck drivers. While he was applying probably accounts for more miscarriages of justice than any other single factor. Conviction must be
for CONCEPCION TRUCKING located across Otis street from the Coca-cola plant. He was arrested. based on the strength of the prosecution and not the weakness of the defense. There was blatant
He denied any knowledge of the "hit" on the Coca-cola delivery truck. He remained in the custody of the violation of the constitutional rights of appellant as an accused. Appellant belongs to the economically
police for two days and two nights. On the third day of his detention, he was turned over to the deprived in our society. He is nearly illiterate(third grade education). Our Constitution and our laws
Constabulary Highway Patrol Group. Appellant was the brought to the MCU hospital. He was made to strictly ordain their protection following the Magsaysay desideratum that those who have less in life
confront Venancio whom he saw for the first time. CHPG Sgt. Awanan asked Venancio twice if should have more in law.
appellant was among those who hijacked the truck he was driving. On both times, Venancio did not
respond. Undaunted, Sgt. Awanan, called to a photographer present, forced appellant to stand about a
foot from Venancio, and told the latter to just point at the suspect. "Basta ituro mo lang," Sgt. Awanan CORPUZ VS. REPUBLIC
directed. Venancio obeyed, and pictures of him pointing to the suspect were taken. From the hospital, [194 SCRA 73; G.R. NO. 74259; 14 FEB 1991]
appellant was brought to the Constabulary Highway Patrol Group headquarters at Camp Crame.
Without being apprised of his rights nor provided with counsel, he was interrogated and urged to Facts:
confess his guilt. He balked. At ten o'clock that night, hours after questioning began, appellant's
interrogators started boxing him and kicking him. He was also hit on the back with a chair, and Generoso Corpuz is the Supervising Accounting Clerk in the Office of the Provincial Treasurer of Nueva
electrocuted. Still, he refused to admit to the crime. In the midst of his ordeal, appellant heard someone Viscaya. He was designated Acting Supervising Cashier in the said office. In this capacity, he received
say, "Tubigan na iyan." He was then blindfolded and brought to another room where he was made to lie collections, disbursed funds and made bank deposits and withdrawals pertaining to government
down. Water was slowly and continuously poured on his face, over his mouth. Appellant could no accounts. On April 13, 1981 his designation as Acting Supervising Cashier was terminated and a
longer bear the pain caused by the water treatment. Finally, he confessed to being one of the hijackers. transfer of accountabilities was effected between him and his successor. The Certificate of turnover
revealed a shortage of P72,823.00. He was able to pay only P10,159.50. After a final demand letter for
Page 101
the total of P50,596.07 which was not met, a case of malversation was filed against him. Corpuz did for procuring attorney.” This was violated. Moreso the guarantees of our Constitution that "no person
not deny such facts but he insists that the shortage was malversed by other persons. He alleged that shall be held to answer for a criminal offense without due process of law", and that all accused "shall
Paymaster Diosdado Pineda through 1 of 4 separate checks (PNB) issued and encashed such checks enjoy the right to be heard by himself and counsel." In criminal cases there can be no fair hearing
while he was of leave. Also, Acting Deputy Provincial Treasurer Bernardo Aluning made to post the unless the accused be given the opportunity to be heard by counsel.
amount on his cashbook although he had not received the said amount. He was convicted in
Sandiganbayan. The trial court failed to inquire as to the true import of the qualified plea of accused. The record does
not show whether the supposed instructions of Mr. Ocampo was real and whether it had reference to
Issue: the commission of the offense or to the making of the plea guilty. No investigation was opened by the
court on this matter in the presence of the accused and there is now no way of determining whether the
Whether or Not Corpuz is guilty of malversation. supposed instruction is a good defense or may vitiate the voluntariness of the confession. Apparently
the court became satisfied with the fiscal's information that he had investigated Mr. Ocampo and found
Held: that the same had nothing to do with this case. Such attitude of the court was wrong for the simple
reason that a mere statement of the fiscal was not sufficient to overcome a qualified plea of the
It is a subtle way of camouflaging the embezzlement of the money equivalent when 1 of the 4 checks accused. But above all, the court should have seen to it that the accused be assisted by counsel
issued and encashed in the same day was entered in the accused’s cash book 3 months after such especially because of the qualified plea given by him and the seriousness of the offense found to be
encashments. Also, Corpuz claim that he was absent when Paymaster Diosdado Pineda through 1 of 4 capital by the court.
separate checks (PNB) issued and encashed such checks, was not proven.

Post-Audit is not a preliminary requirement to filing a malversation case. The failure of the public officer PEOPLE VS. MAGSI
to have duly forthcoming any public funds with which he is chargeable, upon demand by an authorized [124 SCRA 64; G.R. NO.L-32888; 12 AUG 1983]
officer shall be a prima facie evidence that he has put such missing funds to personal use.
Facts:
The equipoise rule(balancing test) which is the presumption of innocence is applicable only where the
evidence of the parties is evenly balance, in which case the scale of justice should be tilt in favor of the Soon after appellant was apprehended on August 20, 1970, his arraignment was scheduled before the
accused. There is no such balance in the case at bar. The evidence of the prosecution is overwhelming Criminal Circuit Court of San Fernando, La Union. The case was actually set and rescheduled for six
and has not been overcome by the petitioner with his claims. The presumed innocence must yield to (6) times, first of which was on August 1, 1970. On that date, despite appointment by the court of Atty.
the positive finding that he is guilty of malversation. Mario Rivera as de officio counsel for the accused, hearing was re-set to September 8, 1970 on motion
of Atty. Rivera, who was prompted to ask for it because of accused desire to be represented by a de
Wherefore his petition is denied. He is guilty as principal of Malversation of Public Funds. parte counsel. Prior to the next hearing, Atty. Rivera moved to withdraw as de officio counsel and it was
favorably acted on by the court on September 7, 1970. At the second hearing on September 8, 1970,
for failure of the de officio and de parte counsels to appear, despite a second call of the case, the
PEOPLE VS. HOLGADO hearing was re-set for the next day and the court appointed Atty. Dominador Cariaso de officio counsel
[85 PHIL 752; G.R.L-2809; 22 MAR 1950] for the accused. On the third hearing date, neither the de parte nor the de officio counsel was in Court,
so Atty. Rivera was reappointed that day as de officio counsel for arraignment purposes only. The
Facts: accused del Rosario entered a plea of guilty but qualified it with the allegation that he committed the
crime out of fear of his co-accused Eloy Magsi and the other coaccused. Appellant was found guilty of
Appellant Frisco Holgado was charged in the court of First Instance of Romblon with slight illegal murder and made to suffer the death penalty.
detention because according to the information, being a private person, he did "feloniously and without
justifiable motive, kidnap and detain one Artemia Fabreag in the house of Antero Holgado for about Issue:
eight hours thereby depriving said Artemia Fabreag of her personal liberty. He pleaded guilty (without a
counsel) and said that he was just instructed by Mr. Ocampo, which no evidence was presented to Whether or not there was a violation of the rights of the accused.
indict the latter.
Held:
Issue:
YES. The desire to speed up the disposition of cases should not be effected at the sacrifice of the basic
Whether or Not there was any irregularity in the proceedings in the trial court. rights of the accused. Citing People vs. Domingo (55 SCRA 243-244): the trial courts should exercise
solicitous care before sentencing the accused on a plea of guilty especially in capital offenses by first
Held: insuring that the accused fully understands the gravity of the offense, the severity of the consequences
attached thereto as well as the meaning and significance of his plea of guilty; and that the prudent and
Yes. Rule 112, section 3 of ROC that : “If the defendant appears without attorney, he must be informed proper thing to do in capital cases is to take testimony, to assure the court that the accused has not
by the court that it is his right to have attorney being arraigned., and must be asked if he desires the aid misunderstood the nature and effect of his plea of guilty. Mere pro-forma appointment of de officio
of attorney, the Court must assign attorney de oficio to defend him. A reasonable time must be allowed counsel, who fails to genuinely protect the interests of the accused, resetting of hearing by the court for
Page 102
alleged reception of evidence when in fact none was conducted, perfunctory queries addressed to the
accused whether he understands the charges and the gravity of the penalty, are not sufficient BORJA VS. MENDOZA
compliance. [77 SCRA 422; G.R. NO.L-45667; 20 JUN 1977]

Facts:

Borja was accused of slight physical injuries in the City of Cebu. However, he was not arraigned. That
SORIANO VS. SANDIGANBAYAN not withstanding, respondent Judge Senining proceeded with the trial in absentia and rendered a
[131 SCRA 184; G.R. NO.L-65952; 31 JUL 1984] decision finding petitioner guilty of the crime charged. The case was appealed to the Court o First
Instance in Cebu presided by respondent Judge Mendoza. It was alleged that the failure to arraign him
Facts: is a violation of his constitutional rights. It was also alleged that without any notice to petitioner and
without requiring him to submit his memorandum, a decision on the appealed case was rendered The
Tan was accused of qualified theft. The petitioner, who was an Asst. Fiscal, was assigned to Solicitor General commented that the decision should be annulled because there was no arraignment.
investigate. In the course of the investigation, petitioner demanded Php.4000 from Tan as price for
dismissing the case. Tan reported it to the NBI which set up an entrapment. Tan was given a Php.2000, Issue:
marked bill, and he had supplied the other half. The entrapment succeeded and an information was
filed with the Sandiganbayan. After trial, the Sandiganbayan rendered a decision finding the petitioner Whether or Not petitioner’s constitutional right was violated when he was not arraigned.
guilty as a principal in violating the Anti Graft and Corrupt Practices Act (R.A.3019). A motion for
reconsideration was denied by the Sandiganbayan, hence this instant petition. Held:

Issue: Yes. Procedural due process requires that the accused be arraigned so that he may be informed as to
why he was indicted and what penal offense he has to face, to be convicted only on a showing that his
Whether or Not the investigation conducted by the petitioner can be regarded as contract or transaction guilt is shown beyond reasonable doubt with full opportunity to disprove the evidence against him. It is
within the purview of .RA.3019. also not just due process that requires an arraignment. It is required in the Rules that an accused, for
the first time, is granted the opportunity to know the precise charge that confronts him. It is imperative
Held: that he is thus made fully aware of possible loss of freedom, even of his life, depending on the nature of
the crime imputed to him. At the very least then, he must be fully informed of why the prosecuting arm
R.A. 3019 Sec.3. Corrupt practices of public officers - In addition to acts or omissions of public officers of the state is mobilized against him. Being arraigned is thus a vital aspect of the constitutional rights
already penalized by existing laws, the following shall constitute corrupt practices of any public officer guaranteed him. Also, respondent Judge Senining convicted petitioner notwithstanding the absence of
and are hereby declared to be unlawful: xxx b. Directly or indirectly requesting or receiving any gift, an arraignment. With the violation of the constitutional right to be heard by himself and counsel being
present, share percentage or benefit, for himself or for other person, in connection with any contract or thus manifest, it is correct that the Solicitor General agreed with petitioner that the sentence imposed
transaction between the Govt. and any other party wherein the public officer in his official capacity has on him should be set aside for being null. The absence of an arraignment can be invoked at anytime in
to intervene under the law. view of the requirements of due process to ensure a fair and impartial trial.

The petitioner stated that the facts make out a case of direct bribery under Art.210 of the RPC and not Wherefore, the petition for certiorari is granted. The decision of respondent Judge Romulo R. Senining
a violation of R.A. 3019 sec.3 (b). The offense of direct bribery is not the offense charged and is not dated December 28, 1973, finding the accused guilty of the crime of slight physical injuries, is nullified
included in the offense charged which is violation of R.A.3019 sec.3 (b). and set aside. Likewise, the decision of respondent Judge Rafael T. Mendoza dated November 16,
1976, affirming the aforesaid decision of Judge Senining, is nullified and set aside. The case is
The respondent claimed that, transaction as used hereof, is not limited to commercial or business remanded to the City Court of Cebu for the prosecution of the offense of slight physical injuries, with
transaction, but includes all kinds of transaction whether commercial, civil, or administrative in nature. due respect and observance of the provisions of the Rules of Court, starting with the arraignment of
petitioner.
The court agrees with the petitioner. It is obvious that the investigation conducted by the petitioner was
neither a contract nor transaction. A transaction like a contract is one which involves some
consideration as in credit transactions. And this element is absent in the investigation conducted by the
petitioner.

Judgment modified. Petitioner is guilty of direct bribery under Art.210 of the RPC.

CONDE VS. RIVERA


[45 PHIL 650; G.R. NO. 21741; 25 JAN 1924]

Page 103
Facts:
In determining the right of an accused to speedy disposition of their case, courts should do more than a
Aurelia Conde, formerly a municipal midwife in Lucena, Tayabas, has been forced to respond to no less mathematical computation of the number of postponements of the scheduled hearings of the case.
the five information for various crimes and misdemeanors, has appeared with her witnesses and What are violative of the right of the accused to speedy trial are unjustified postponements which
counsel at hearings no less than on eight different occasions only to see the cause postponed, has prolong trial for an unreasonable length of time. In the facts above, there was no showing that there
twice been required to come to the Supreme Court for protection, and now, after the passage of more was an unjust delay caused by the prosecution, hence, the respondent judge should have given the
than one year from the time when the first information was filed, seems as far away from a definite prosecution a fair opportunity to prosecute its case.
resolution of her troubles as she was when originally charged.
The private respondents cannot invoke their right against double jeopardy. In several cases it was held
Issue: that dismissal on the grounds of failure to prosecute is equivalent to an acquittal that would bar another
prosecution for the same offense, but in this case, this does not apply, considering that the rights of the
Whether or Not petitioner has been denied her right to a speedy and impartial trial. accused to a speedy trial was not violated by the State. Therefore, the order of dismissal is annulled
and the case is remanded to the court of origin for further proceedings.
Held:

Philippine organic and statutory law expressly guarantee that in all criminal prosecutions the accused REPUBLIC ACT NO. 8493 “THE SPEEDY TRIAL ACT”
shall enjoy the right to have a speedy trial. Aurelia Conde, like all other accused persons, has a right to
a speedy trial in order that if innocent she may go free, and she has been deprived of that right in The arraignment of an accused shall be held within 30 days from filing of the information, or from the
defiance of law. We lay down the legal proposition that, where a prosecuting officer, without good date the accused has appeared before the justice, judge or court in which the charge is pending,
cause, secures postponements of the trial of a defendant against his protest beyond a reasonable whichever date last occurs. Thereafter, where a plea of not guilty is entered, the accused shall have at
period of time, as in this instance for more than a year, the accused is entitled to relief by a proceeding least 15 days to prepare for trial. Trial shall commence within 30 days from arraignment as fixed by the
in mandamus to compel a dismissal of the information, or if he be restrained of his liberty, by habeas court. In no case shall the entire trial period exceed 180 days from the 1 st day of trial, except as
corpus to obtain his freedom. otherwise authorized by the Chief Justice of the Supreme Court.

PEOPLE VS. TAMPAL RE: REQUEST FOR LIVE TV OF TRIAL OF JOSEPH ESTRADA
[244 SCRA 202; G.R. NO. 102485; 22 MAY 1995] [360 SCRA 248; A.M. NO 01-4-03-SC; 29 JUN 2001]

Facts: Facts:

Luis Tampal, Domingo Padumon, Arsenio Padumon, Samuel Padumon, Pablito Suco, Dario Suco and The Kapisanan ng mga Brodkaster ng Pilipinas (KBP) sent a letter requesting the Court to allow live
Galvino Cadling were charged of robbery with homicide and multiple serious physical injuries in the media coverage of the anticipated trial of the plunder and other criminal cases filed against former
Regional Trial Court of Zamboanga with Hon. Wilfredo Ochotorena as presiding judge. However, only President Joseph E. Estrada before the Sandiganbayan in order "to assure the public of full
private respondents, Luis Tampal, Domingo Padumon, Arsenio Padumon, and Samuel Padumon were transparency in the proceedings of an unprecedented case in our history." The request was seconded
arrested, while the others remained at large. by Mr. Cesar N. Sarino and, still later, by Senator Renato Cayetano and Attorney Ricardo Romulo.

The case was set for hearing on July 26, 1991, but Assistant Provincial Prosecutor Wilfredo Guantero Issue:
moved for postponement due to his failure to contact the material witnesses. The case was reset
without any objection from the defense counsel. The case was called on September 20, 1991 but the Whether or Not live media coverage of the trial of the plunder and other criminal cases filed against
prosecutor was not present. The respondent judge considered the absence of the prosecutor as former President Joseph E. Estrada should be permitted by the court.
unjustified, and dismissed the criminal case for failure to prosecute. The prosecution filed a motion for
reconsidereation, claiming that his absence was because such date was a Muslim holiday and the Held:
office of the Provincial prosecutor was closed on that day. The motion was denied by respondent judge.
The propriety of granting or denying the instant petition involve the weighing out of the constitutional
Issue: guarantees of freedom of the press and the right to public information, on the other hand, along with the
constitutional power of a court to control its proceedings in ensuring a fair and impartial trial. When
Whether or Not the postponement is a violation of the right of the accused to a speedy disposition of these rights race against one another, jurisprudence tells us that the right of the accused must be
their cases. preferred to win.

Whether or Not the dismissal serves as a bar to reinstatement of the case. Due process guarantees the accused a presumption of innocence until the contrary is proved in a trial
that is not lifted about its individual settings nor made an object of public’s attention and where the
Held:
Page 104
conclusions reached are induced not by any outside force or influence but only be evidence and Pampanga where he was ultimately apprehended by the Mexico Police on September 22, 1992 after
argument given in open court, where fitting dignity and calm ambiance is demanded. chancing on a radio message by the police of Arayat to their Provincial commander that a vehicular
incident occurred at Arayat, Pampanga where one Elmer Salas was the victim and was hospitalized at
An accused has a right to a public trial but it is a right that belongs to him, more than anyone else, the district hospital at Arayat, Pampanga where he used the name of Rommel Salas and not Elmer
where his life or liberty can be held critically in balance. A public trial aims to ensure that he is fairly Salas. The trial court rendered convicting Salas for Robbery with Homicide
dealt with and would not be unjustly condemned and that his rights are not compromised in secret
conclaves of long ago. A public trial is not synonymous with publicized trial, it only implies that the court Issue:
doors must be open to those who wish to come, sit in the available seats, conduct themselves with
decorum and observe the trial process. Whether or Not there is evidence sufficient to sustain a conviction of the appellant of the crime of
Robbery with Homicide.
The courts recognize the constitutionally embodied freedom of the press and the right to public
Whether or Not the appellant’s crime homicide or robbery with homicide.
information. It also approves of media's exalted power to provide the most accurate and comprehensive
means of conveying the proceedings to the public. Nevertheless, within the courthouse, the overriding
Held:
consideration is still the paramount right of the accused to due process which must never be allowed to
suffer diminution in its constitutional proportions.
There was no eyewitness or direct evidence; either to the robbery or to the homicide and none of the
things allegedly stolen were ever recovered. However, direct evidence is not the only matrix from which
the trial court may draw its findings and conclusion of culpability. Resort to circumstantial evidence is
PEOPLE VS. SALAS
essential when to insist on direct testimony would result in setting felons free.
[143 SCRA 163; G.R. NO. L-66469; 29 JUL 1986]
For circumstantial evidence to be sufficient to support a conviction, all the circumstances must be
Facts:
consistent with each other, consistent with the theory that the accused is guilty of the offense charged,
and at the same time inconsistent with the hypothesis that he is innocent and with every other possible,
At about 6:00 o'clock in the morning of March 6, 1992, a 60 year old woman, identified as Virginia
rational hypothesis excepting that of guilt. All the circumstances established must constitute an
Talens was found lying dead in a canal at Bo. San Nicolas, Mexico, Pampanga; she was last seen alive
unbroken chain which leads to one and fair and reasonable conclusion pointing solely to the accused,
at about 3:00 o'clock early morning of March 6, 1992 by Orlando Pangan and Richard Pangan who
to the exclusion of all other persons, as the author of the crime. The facts and circumstances consistent
were with her going home coming from the wake of one Leonardo Flores; both Orlando and Richard
with the guilt of the accused and inconsistent with his innocence can constitute evidence which, in
Pangan testified that accused was with them in going home at about 3:00 o'clock in the morning of
weight and probative value, may be deemed to surpass even direct evidence in its effect on the court.
March 6, 1992; Orlando and Richard Pangan reached first their house and left the two on the way and
that was the last time Virginia was seen alive; just a few minutes after reaching his house and while
The fatal stabbing of Virginia Talens occurred at around 3:00 a.m. of March 6, 1992. Appellant hastily
inside his house, Orlando Pangan heard a shout; another woman, one Serafia Gutierrez, testified that
abandoned his house in Barrio San Nicolas, Mexico, Pampanga, his residence since childhood, on that
she likewise was awakened by a shout at about 3:00 in the morning; Dr. Aguda who autopsied the
very date. Appellant was nowhere when his co-worker and barrio mate, Eduardo Bagtas, came to
victim found hematoma on the head and chest, an abrasion on the left chin and stabwound on the neck
appellant's house to fetch him for work at around 6:30 to 7:00 a.m. of March 6, 1992. Appellant also
which stabwound, the doctor claims, was the cause of death of the victim; Police Investigator Gonzales
abandoned his job as a painter in Sta. Ana, Pampanga, on March 6, 1992, the date of the crime,
who immediately responded upon report, recovered at the scene a pin, the victim's wristwatch, earring,
leaving behind an unfinished painting project. He was not seen again from said date. Police
a ring and P135.00 money; he likewise found on March 9, 1992 when he continued his investigation
investigators found human bloodstains on the front door of appellant's house, on his clothing, and on
bloodstain on the front door of the house of the accused which bloodstain when submitted for
his yellow slippers after the victim was killed. Despite efforts of the police to find appellant as the
examination was found to be of human blood; one Resultay was with Virginia Talens at about 5:00
principal suspect, a fact known to appellant's family and neighbors, appellant did not present himself to
afternoon of March 5, 1992 in going to the wake, who claims that Virginia had money on a purse as
the authorities. Appellant was apprehended only a full six months after the date of the crime, following
while they were on the way Virginia bet on a jueteng she saw Virginia got money from her purse a
his confinement in a hospital in Arayat, Pampanga because he was sideswiped by a Victory Liner bus
P500.00 bill but as she had no change she instead took P8.00 from her other pocket; one Ramil
in Arayat. When hospitalized, appellant used the alias Rommel Salas, instead of his true name Elmer
Talens, a son of the victim corroborated the claim of Resultay that Virginia had with her at that time
Salas. These circumstances denote flight, which when unexplained, has always been considered by the
money worth P2,000.00 as in the morning of March 5, 1992 he gave her mother for safekeeping the
courts as indicative of guilt.
sum of P1,500.00 which he claims his mother placed in her purse and claims further that at the wake,
he asked and was given P50.00 by his mother as he also participated in the gambling thereat, however,
Both appellant and victim gambled at the wake they attended. The victim was, in fact, enjoying a
the purse of Virginia containing about P2,000.00 was no longer to be found when she was found dead;
winning streak when her son, Ramil Talens, came to fetch her but which he failed to do because his
Orlando Pangan saw the accused gambled in the wake; Virginia likewise gambled at the wake;
mother was winning, and she refused to leave. The purse of Talens containing cash was gone when
accused had been working for three days before March 6 at Sta. Ana, Pampanga and up to March 5,
her corpse was found in the canal with a stab wound and bruises. What was left was a safety pin which
1992, but the following day, he did not anymore report for work at Sta. Ana, Pampanga, was no longer
victim used to fasten the missing purse to her clothes.
to be found and was last seen at about 3:00 morning together with Virginia Talens on their way home
coming from the wake; the parents of [the] accused were informed by Investigator Gonzales that their
Denial is an inherently weak defense which must be buttressed by strong evidence of non-culpability to
son was the suspect and adviced them to surrender him, but since March 6, 1992 when accused left
merit credibility. Denial is negative and self-serving and cannot be given greater evidentiary weight over
Mexico, Pampanga, he returned only on September 19, 1992 at Arayat, Pampanga, not at Mexico,
Page 105
the testimonies of credible witnesses who positively testified that appellant was at the locus criminis and Magpalao, and Magumnang were all held guilty as principal by direct participation of the crime of
was the last person seen with the victim alive. Robbery with Homicide.

The absence of evidence showing any improper motive on the part of the principal witness for the Issue:
prosecution to falsely testify against the appellant strongly tends to buttress the conclusion that no such
improper motive exists and that the testimony of said witnesses deserve full faith and credit. Whether or Not the lower court erred in failing to apply the Constitutional mandate on the presumption
of innocence and proof beyond reasonable doubt when it allowed the trial in absentia to push through
The essence of voluntary surrender is spontaneity and the intent of the accused to give himself up and on the part of defendant-appellant Magumnang.
submit himself unconditionally to the authorities either because he acknowledges his guilt or he wants
to save the State the trouble of having to effect his arrest. Spontaneity and intent to give one's self up Held:
are absent where the accused went into hiding for six months after the incident and had to resort to an
alias when he was involved in an accident being investigated by the police authorities. The Court affirmed the decision of the lower court. The reason is that the lower court has jurisdiction
over Magumnang the moment the latter was in custody. Jurisdiction once acquired is not lost upon the
Robbery with Homicide is a special complex crime against property. Homicide is incidental to the instance of parties but until the case is terminated. Since all the requisites of trial in absentia are
robbery which is the main purpose of the criminal. In charging Robbery with Homicide, the onus complete, the court has jurisdiction over Magumnang.
probandi is to establish: "(a) the taking of personal property with the use of violence or intimidation
against a person; (b) the property belongs to another; (c) the taking is characterized with animus In addition, Magumnang was presumed innocent during his trial in absentia. The prosecution had
lucrandi; and (d) on the occasion of the robbery or by reason thereof, the crime of homicide, which is strong evidence against him as proof beyond reasonable doubt that he is a principal by direct
used in the generic sense, was committed." Although there was no witness as to the actual robbing of participation in the crime of Robbery with Homicide. Thus, the Constitutional mandate was not violated.
the victim, there is testimony that the victim had more or less P2,000.00; and wore gold earrings valued
at P750.00. These were never recovered.
PEOPLE VS. ACABAL
While there is indeed no direct proof that Virginia Talens was robbed at the time she was killed, we may [226 SCRA 694 ; G.R. NO. 103604, 23 SEP 1993]
conclude from four circumstances that the robbery occasioned her killing: (1) Both appellant and victim
gambled at the wake. (2) The appellant knew that victim was winning. (3) The victim was last seen alive Facts:
with appellant. (4) The victim's purse containing her money and earrings were missing from her body
when found. The accusatory portion in the information for murder. Facts are as follows:

The decision of the regional trial court is affirmed. Costs against appellant. So ordered. "That sometime in the evening of the 28th of January, 1980, at Nagbinlod, Municipality of Sta. Catalina,
Province of Negros Oriental, Philippines, and within the jurisdiction of this Honorable Court, the
accused, including several 'John Does', conspiring and confederating with one another, with intent to
PEOPLE VS. MAGPALAO kill, and with treachery and evident premeditation and being then armed with bolos and 'pinuti', did then
[197 SCRA 79; G.R. NO. 92415; 14 MAY 1991] and there willfully, unlawfully and feloniously attack, assault and use personal violence on the person of
one Rizalina Apatan Silvano while the latter was about to leave her house and inflicting upon her
Facts: injuries, to wit: 'right leg amputated below the knee; left leg hacked behind the knee; abdomen hacked
with viscerae evacerated,' and did then and there set the house on fire while the aforementioned
Eleven (11) people rode in a Ford Fiera going to Baguio. Namely they are: Felizardo Galvez, Jimmy Rizalina Apatan Silvano was inside said house trying to escape therefrom, and allowing her to be
Jetwani, Simeon Calama, Rene Salonga, Eduardo Lopez, Adolfo Quiambao, Aliman Bara-akal, Anwar burned inside said house which was burned to the ground, thereby causing upon said Rizalina Apatan
Hadji Edris, Gumanak Ompa and defendant-appelants in this case, Omar Magpalao and Rex Silvano her death and burning her beyond recognition.
Magumnang.
But on 16 May 1987, a fire gutted the building where Branch 37 was located and the records of these
After an hour of driving, the car stopped so that one of the passengers could urinate. While the car was two cases were burned. The records were subsequently reconstituted upon petition of the prosecuting
stopped the Bara-akal, Edris, Ompa, Magpalao and Magumnang pointed guns and knives at the other fiscal. The testimonies of the witnesses were retaken, however, before it could commence, accused
passengers and divested them of their properties. Engracio Valeriano jumped bail and the warrant for his arrest issued on 16 November 1987 was
returned unserved because he could not be found. An alias warrant for his arrest was issued on 26
On of the robbers then ordered Galvez to drive the car towards the precipice (bangin). When the car June 1989, but he remains at large up to the present.
was near the precipice, Galvez then stepped to the brakes. The other passengers jumped out of the car
and went to different directions to escape. Galvez however, was left in side the car and was stabbed by After the completion of the re-taking of the testimonies of the witnesses in Branch 37, Criminal Cases
one of the robbers. The robbers then escaped. Quiambao, who owned the car helped Galvez to get to Nos. 4584 and 4585 were re-raffled to Branch 33 of the trial court, then presided over by Judge Pacifico
a hospital. Galvez died in the hospital. The robbers were then apprehended with the exception of Edris S. Bulado.
who remain at large. Mangumnang however escaped while being in detention and Bara-akal died
inside the jail. Since Mangumnang was not arrested, the trial in absentia continued as to him. Ompa,
Page 106
The decision of the trial court, per Judge Pacifico S. Bulado, dated 31 October 1991 but promulgated Accused-appellants contend that the trial court did not impose any sentence and so cannot cancel
on 20 December 1991, contained no specific dispositive portion. Its rulings are found in the last two anymore their bail bonds and direct their arrest and immediate commitment because it already lost
paragraphs which read as follows: jurisdiction over their persons when they perfected their appeal.

"The elements of murder in this case, Criminal Case No. 4585 for the killing of Rizalina Apatan-Silvano The decision did impose the penalty of reclusion perpetua. Since the order cancelling their bail bonds
having been proved by the prosecution beyond doubt, the accused JUANITO RISMUNDO, MACARIO and directing their arrest is contained in the decision itself, it is apparent that their abovementioned
ACABAL and ABUNDIO NAHID, considering the attendant qualifying aggravating circumstances of contention is highly illogical. At the time the order in question was made, the trial court still had
nighttime, use of fire by burning the house of victim Rizalina Apatan-Silvano in order to forcibly drive her jurisdiction over the persons of the accused-appellants.
out of her house and hack her to death, the abuse of superior strength, the penalty impossable [sic]
here will be in its maximum degree, that is reclusion perpetua taking into account Article 248 of the The trial court further erred in holding that no penalty could be imposed on accused Engracio Valeriano
Revised Penal Code, the penalty now for murder is Reclusion Temporal to Reclusion Perpetua, and for in Criminal Case No. 4584 because he "is nowhere to be found, hence, not brought to the bar of justice,
all the accused to indemnify the heirs of the victim the sum of Thirty Thousand (P30,000.00) Pesos he being a fugitive or at large." The court ignored the fact that Engracio jumped bail after he had been
since this case occurred [sic] in 1980. For the wounding of the victim Wilson A. Silvano, this Court arraigned, just before the retaking of evidence commenced. Paragraph (2), Section 14, Article III of the
believes that simple frustrated homicide only is committed by the accused Engracio Valeriano only. Constitution permits trial in absentia after the accused has been arraigned provided he has been duly
notified of the trial and his failure to appear thereat is unjustified. One who jumps bail can never offer a
But since the person who actually inflicted the injuries of victim Wilson Silvano, accused Engracio justifiable reason for his non-appearance during the trial.
Valeriano only is nowhere to be found, hence, not brought to the bar of justice, he being a fugitive or at
large, no penalty could be imposed on him since he is beyond the jurisdiction of this court to reach. All Accordingly, after the trial in absentia, the court can render judgment in the case and promulgation may
the other two (2) accused, JUANITO RISMUNDO and ABUNDIO NAHID are hereby ordered and be made by simply recording the judgment in the criminal docket with a copy thereof served upon his
declared absolved from any criminal responsibility from frustrated homicide. counsel, provided that the notice requiring him to be present at the promulgation is served through his
bondsmen or warden and counsel.
The bail bond put up by the three accused, namely: Juanito Rismundo, Macario Acabal and Abundio
Nahid are hereby ordered cancelled and let a warrant of arrest be issued for their immediate
In conclusion, because of reasonable doubt as to their guilt, the accused-appellants must be acquitted.
confinement."
Every accused is presumed innocent until the contrary is proved; that presumption is solemnly
guaranteed by the Bill of Rights. The contrary requires proof beyond reasonable doubt, or that degree
Issue:
of proof which produces conviction in an unprejudiced mind. Short of this, it is not only the right of the
accused to be freed; it is even the constitutional duty of the court to acquit him.
Whether or not the judgment complied with the Rules of Court.

Whether or not the cancellation of the bail bonds of the accused is valid.
US VS. TAN TENG
Whether or not the accused may be tried in absentia. [23 PHIL 145; G.R. NO. 7081; 7 SEP 1912]

Whether or not the accused is guilty of the crime of frustrated murder. Facts:

The defendant herein raped Oliva Pacomio, a seven-year-old girl. Tan Teng was gambling near the
Held:
house of the victim and it was alleged that he entered her home and threw the victim on the floor and
place his private parts over hers. Several days later, Pacomio was suffering from a disease called
We find that the decision substantially complies with the Rules of Court on judgments as it did sentence
gonorrhea. Pacomio told her sister about what had happened and reported it to the police.
the accused-appellants to reclusion perpetua. A judgment of conviction shall state (a) the legal
qualification of the offense constituted by the acts committed by the accused, and the aggravating or
Tan Teng was called to appear in a police line-up and the victim identified him. He was then stripped of
mitigating circumstances attending the commission, if there are any; (b) the participation of the accused
his clothing and was examined by a policeman. He was found to have the same symptoms of
in the commission of the offense, whether as principal, accomplice or accessory after the fact; (c) the
gonorrhea. The policeman took a portion of the substance emitting from the body of the defendant and
penalty imposed upon the accused; and (d) the civil liability or damages caused by the wrongful act to
turned it over to the Bureau of Science. The results showed that the defendant was suffering from
be recovered from the accused by the offended party, if there is any, unless the enforcement of the civil
gonorrhea.
liability by a separate action has been reserved or waived.
The lower court held that the results show that the disease that the victim had acquired came from the
It is obvious that they clearly understood that they were found guilty beyond reasonable doubt of the
defendant herein. Such disease was transferred by the unlawful act of carnal knowledge by the latter.
crime of murder and were sentenced to suffer the penalty of reclusion perpetua in Criminal Case No.
The defendant alleged that the said evidence should be inadmissible because it was taken in violation
4585. Were it otherwise, they would not have declared in open court their intention to appeal
of his right against self-incrimination.
immediately after the promulgation of the decision and would not have subsequently filed their written
notice of appeal.
Issue:
Page 107
Whether or Not the physical examination conducted was a violation of the defendant’s rights against Whether or not the writing from the fiscal's dictation by the petitioner for the purpose of comparing the
self-incrimination. latter's handwriting and determining whether he wrote certain documents supposed to be falsified,
constitutes evidence against himself within the scope and meaning of the constitutional provision under
Held: examination.

The court held that the taking of a substance from his body was not a violation of the said right. He was Held:
neither compelled to make any admissions or to answer any questions. The substance was taken from
his body without his objection and was examined by competent medical authority. The court ordered the respondents and those under their orders desist and abstain absolutely and
forever from compelling the petitioner to take down dictation in his handwriting for the purpose of
The prohibition of self-incrimination in the Bill of Rights is a prohibition of the use of physical or moral submitting the latter for comparison. Writing is something more than moving the body, or the hands, or
compulsion to extort communications from him, and not an exclusion of his body as evidence, when it the fingers; writing is not a purely mechanical act, because it requires the application of intelligence and
may be material. It would be the same as if the offender apprehended was a thief and the object stolen attention; and in the case at bar writing means that the petitioner herein is to furnish a means to
by him may be used as evidence against him. determine whether or not he is the falsifier, as the petition of the respondent fiscal clearly states. Except
that it is more serious, we believe the present case is similar to that of producing documents or chattels
in one's possession. We say that, for the purposes of the constitutional privilege, there is a similarity
VILLAFLOR VS. SUMMERS between one who is compelled to produce a document, and one who is compelled to furnish a
[41 PHIL 62; G.R. NO. 16444; 8 SEP 1920] specimen of his handwriting, for in both cases, the witness is required to furnish evidence against
himself. It cannot be contended in the present case that if permission to obtain a specimen of the
Facts: petitioner's handwriting is not granted, the crime would go unpunished. Considering the circumstance
that the petitioner is a municipal treasurer, it should not be a difficult matter for the fiscal to obtained
Petitioner Villaflor was charged with the crime of adultery. The trial judge ordered the petitioner to genuine specimens of his handwriting. But even supposing it is impossible to obtain specimen or
subject herself into physical examination to test whether or not she was pregnant to prove the specimens without resorting to the means complained herein, that is no reason for trampling upon a
determine the crime of adultery being charged to her. Herein petitioner refused to such physical personal right guaranteed by the constitution. It might be true that in some cases criminals may
examination interposing the defense that such examination was a violation of her constitutional rights succeed in evading the hand of justice, but such cases are accidental and do not constitute the raison
against self-incrimination. d' etre of the privilege. This constitutional privilege exists for the protection of innocent persons.

Issue:
PASCUAL VS. BME
Whether or Not the physical examination was a violation of the petitioner’s constitutional rights against [28 SCRA 345; G.R. NO. 25018; 26 MAY 1969]
self-incrimination.
Facts:
Held:
Petitioner Arsenio Pascual, Jr. filed an action for prohibition against the Board of Medical Examiners. It
No. It is not a violation of her constitutional rights. The rule that the constitutional guaranty, that no was alleged therein that at the initial hearing of an administrative case for alleged immorality, counsel
person shall be compelled in any criminal case to be a witness against himself, is limited to a prohibition for complainants announced that he would present as his first witness the petitioner. Thereupon,
against compulsory testimonial self-incrimination. The corollary to the proposition is that, an ocular petitioner, through counsel, made of record his objection, relying on the constitutional right to be
inspection of the body of the accused is permissible. exempt from being a witness against himself. Petitioner then alleged that to compel him to take the
witness stand, the Board of Examiners was guilty, at the very least, of grave abuse of discretion for
failure to respect the constitutional right against self-incrimination.
BELTRAN VS. SAMSON
[53 PHIL 570; G.R. NO. 32025; 23 SEPT 1929] The answer of respondent Board, while admitting the facts stressed that it could call petitioner to the
witness stand and interrogate him, the right against self-incrimination being available only when a
Facts: question calling for an incriminating answer is asked of a witness. They likewise alleged that the right
against self-incrimination cannot be availed of in an administrative hearing.
Beltran, as a defendant for the crime of Falsification, refused to write a sample of his handwriting as
ordered by the respondent Judge. The petitioner in this case contended that such order would be a Petitioner was sustained by the lower court in his plea that he could not be compelled to be the first
violation of his constitutional right against self-incrimination because such examination would give the witness of the complainants, he being the party proceeded against in an administrative charge for
prosecution evidence against him, which the latter should have gotten in the first place. He also argued malpractice. Hence, this appeal by respondent Board.
that such an act will make him furnish evidence against himself.  
Issue:
Issue:
Page 108
Whether or Not compelling petitioner to be the first witness of the complainants violates the Self-
Incrimination Clause.
  Facts:
Held:
The accused was charged with Robbery with Rape before the Municipal Court of Balungao,
The Supreme Court held that in an administrative hearing against a medical practitioner for alleged Pangasinan. He pleaded not guilty. His counsel moved for the dismissal of the charge for failure to
malpractice, respondent Board of Medical Examiners cannot, consistently with the self-incrimination allege vivid designs in the info. Said motion was granted. From this order of dismissal the prosecution
clause, compel the person proceeded against to take the witness stand without his consent. The Court appealed.
found for the petitioner in accordance with the well-settled principle that "the accused in a criminal case
may refuse, not only to answer incriminatory questions, but, also, to take the witness stand." If Issue:
petitioner would be compelled to testify against himself, he could suffer not the forfeiture of property but
the revocation of his license as a medical practitioner. The constitutional guarantee protects as well the Whether or Not the present appeal places the accused in Double Jeopardy.
right to silence: "The accused has a perfect right to remain silent and his silence cannot be used as a
presumption of his guilt." It is the right of a defendant "to forego testimony, to remain silent, unless he Held:
chooses to take the witness stand — with undiluted, unfettered exercise of his own free genuine will."
In order that the accused may invoke double jeopardy, the following requisites must have obtained in
The reason for this constitutional guarantee, along with other rights granted an accused, stands for a the original prosecution, a) valid complaint, b) competent court, c) the defendant had pleaded to the
belief that while crime should not go unpunished and that the truth must be revealed, such desirable charge, d) defendant was acquitted or convicted or the case against him was dismissed or otherwise
objectives should not be accomplished according to means or methods offensive to the high sense of terminated without his express consent.
respect accorded the human personality. More and more in line with the democratic creed, the
deference accorded an individual even those suspected of the most heinous crimes is given due In the case at bar, the converted dismissal was ordered by the Trial Judge upon the defendant's motion
weight. The constitutional foundation underlying the privilege is the respect a government ... must to dismiss. The “doctrine of double jeopardy” as enunciated in P.vs. Salico applies to wit when
accord to the dignity and integrity of its citizens. the case is dismissed with the express consent of the defendant, the dismissal will not be a bar to
another prosecution for the same offense because his action in having the case is dismissed
constitutes a waiver of his constitutional right/privilege for the reason that he thereby prevents the Court
PEOPLE VS. BALISACAN from proceeding to the trial on the merits and rendering a judgment of conviction against him.
[17 SCRA 1119; G.R. NO. L-26376; 31 AUG 1966]
In essence, where a criminal case is dismissed provisionally not only with the express consent of the
Facts: accused but even upon the urging of his counsel there can be no double jeopardy under Sect. 9 Rule
113, if the indictment against him is revived by the fiscal.
Aurelio Balisacan was charged with homicide in the CFI of Ilocos Norte. Upon being arraigned, he
entered into a plea of guilty. In doing so, he was assisted y counsel. At his counsel de officio, he was
allowed to present evidence and consequently testified that he stabbed the deceased in self-defense. PAULIN VS. GIMENEZ
In addition, he stated that he surrendered himself voluntarily to the police authorities. On the basis of [217 SCRA 386; G.R. NO. 103323; 21 JAN 1993]
the testimony of the accused, he was acquitted. Thus, the prosecution appealed.
Facts:
Issue:
Respondent and Brgy Capt. Mabuyo, while in a jeep, were smothered with dust when they were
Whether or Not the appeal placed the accused in double jeopardy. overtaken by the vehicle owned by Petitioner Spouses. Irked by such, Mabuyo followed the vehicle until
the latter entered the gate of an establishment. He inquired the nearby security guard for the identity of
Held: the owner of the vehicle. Later that day, while engaged in his duties, petitioners allegedly pointed their
guns at him. Thus, he immediately ordered his subordinate to call the police and block road to prevent
The Supreme Court held that it is settled that the existence of plea is an essential requisite to double the petitioners’ escape. Upon the arrival of the police, petitioners put their guns down and were
jeopardy. The accused had first entered a plea of guilty but however testified that he acted in complete immediately apprehended.
self-defense. Said testimony had the effect of vacating his plea of guilty and the court a quo should
have required him to plead a new charge, or at least direct that a new plea of not guilty be entered for A complaint “grave threats” was filed against the petitioners (Criminal Case No. 5204). It was dismissed
him. This was not done. Therefore, there has been no standing of plea during the judgment of acquittal, by the court acting on the motion of the petitioners. Mabuyo filed a MOR thus the dismissal was
so there can be no double jeopardy with respect to the appeal herein. reversed. Thereafter, petitioners filed for “certiorari, prohibition, damages, with relief of preliminary
injunction and the issuance of a TRO” (CEB-9207). Petition is dismissed for lack of merit and for being
a prohibited pleading and ordered to proceed with the trial of the case. Hence, this instant petition.
PEOPLE VS. OBSANIA
[23 SCRA 1249; G.R. L-24447; 29 JUN 1968] Issue:
Page 109
Yes the revival of the case will put the accused in double jeopardy for the very reason that the case has
Whether or Not the dismissal of 5204 was a judgment of acquittal. been dismissed earlier due to lack of merits. It is true that the criminal case of falsification was
dismissed on a motion of the accused however this was a motion filed after the prosecution had rested
Whether or Not the judge ignored petitioner’s right against double jeopardy by dismissing CEB-9207. its case, calling for the evidence beyond reasonable ground which the prosecution had not been able to
do which would be tantamount to acquittal therefore will bar the prosecution of another case. As it was
Held: stated on the requirements of a valid defense of double jeopardy it says: That there should be a valid
complaint, second would be that such complaint be filed before a competent court and to which the
For double jeopardy to attach, the dismissal of the case must be without the express consent of the accused has pleaded and that defendant was previously acquitted, convicted or dismissed or otherwise
accused. Where the dismissal was ordered upon motion or with the express assent of the accused, he terminated without express consent of the accused in which were all present in the case at bar. There
has deemed to have waived his protection against double jeopardy. In the case at bar, the dismissal was indeed a valid, legitimate complaint and concern against the accused Sensio, Millan and Jochico
was granted upon motion of the petitioners. Double jeopardy thus did not attach. which was filed at a competent court with jurisdiction on the said case. It was also mentioned that the
accused pleaded not guilty and during the time of trial, it was proven that the case used against the
Furthermore, such dismissal is not considered as an acquittal. The latter is always based on merit that accused were not sufficient to prove them guilty beyond reasonable doubt therefore dismissing the
shows that the defendant is beyond reasonable doubt not guilty. While the former, in the case at bar, case which translates to acquittal. It explained further that there are two instances when we can
terminated the proceedings because no finding was made as to the guilt or innocence of the petitioners. conclude that there is jeopardy when first is that the ground for the dismissal of the case was due to
insufficiency of evidence and second, when the proceedings have been reasonably prolonged as to
The lower court did not violate the rule when it set aside the order of dismissal for the reception of violate the right of the accused to a speedy trial. In the 2 requisites given, it was the first on that is very
further evidence by the prosecution because it merely corrected its error when it prematurely terminated much applicable to our case at bar where there was dismissal of the case due to insufficiency of
and dismissed the case without giving the prosecution the right to complete the presentation of its evidence which will bar the approval of the petition in the case at bar for it will constitute double
evidence. The rule on summary procedure was correctly applied. jeopardy on the part of the accused which the law despises.

PEOPLE VS. COURT OF SILAY PEOPLE VS. RELOVA


[74 SCRA 248; G.R. NO. L-43790; 9 DEC 1976] [149 SCRA 292; G.R. NO.L-45129; 6 MAR 1987]

Facts: FACTS: In this petition for certiorari and mandamus, People of the Philippines seeks to set aside the
orders of Respondent Judge Hon. Relova quashing an information for theft filed against Mr. Opulencia
That sometime on January 4,1974, accused Pacifico Sensio, Romeo Millan and Wilfredo Jochico who on the ground of double jeopardy and denying the petitioner’s motion for reconsideration.. On Feb.1
were then scalers at the Hawaiian-Philippine Company, weighed cane cars No.1743,1686 and 1022 1975, Batangas police together with personnel of Batangas Electric Light System, equipped with a
loaded with sugar canes which were placed in tarjetas (weight report cards), Apparently, it was proven search warrant issued by a city judge of Batangas to search and examine the premises of the
and shown that there was padding of the weight of the sugar canes and that the information on the Opulencia Carpena Ice Plant owned by one Manuel Opulencia. They discovered electric wiring devices
tarjetas were to be false making it appear to be heavier than its actual weight. The three accused then have been installed without authority from the city government and architecturally concealed inside the
were charged with “Falsification by private individuals and use of falsified document”. After the walls of the building. Said devices are designed purposely to lower or decrease the readings of electric
prosecution had presented, the respondent moved to dismiss the charge against them on the ground current consumption in the plant’s electric meter. The case was dismissed on the ground of prescription
that the evidences presented were not sufficient to establish their guilt beyond reasonable doubt. for the complaint was filed nine months prior to discovery when it should be 2months prior to discovery
Acting on the motion, respondent court issued its order dismissing the case on the ground that the acts that the act being a light felony and prescribed the right to file in court. On Nov 24, 1975, another case
committed by the accused do not constituted the crime of falsification as strictly enumerated in the was filed against Mr. Opulencia by the Assistant City Fiscal of Batangas for a violation of a Batangas
revised penal code defining the crime of falsification which was charged earlier and that their case be Ordinance regarding unauthorized electrical installations with resulting damage and prejudice to City of
dismissed. People asserts that the plea of double jeopardy is not tenable even if the case at bar was Batangas in the amount of P41,062.16. Before arraignment, Opulencia filed a motion to quash on the
dismissed because according to them, it was done with the consent of the accused therefore waiving ground of double jeopardy. The Assistant fiscal’s claim is that it is not double jeopardy because the first
there defense of double jeopardy. The accused on the other hand, reiterated the fact that the dismissal offense charged against the accused was unauthorized installation of electrical devices without the
was due to lack of merits of the prosecution which would have the same effect as an acquittal which will approval and necessary authority from the City Government which was punishable by an ordinance,
bar the prosecution from prosecuting the accused for it will be unjust and unconstitutional for the where in the case was dismissed, as opposed to the second offense which is theft of electricity which is
accused due to double jeopardy rule thus the appeal of the plaintiff. punishable by the Revised Penal Code making it a different crime charged against the 1 st complaint
against Mr.Opulencia.
Issue:
Issue:
Whether or Not the grant of petition by the court would place the accused Sensio, Millan and Jochico in
double jeopardy Whether or Not the accused Mr. Opulencia can invoke double jeopardy as defense to the second
offense charged against him by the assistant fiscal of Batangas on the ground of theft of electricity
Held: punishable by a statute against the Revised Penal Code.

Page 110
Held: conditions are present then the acquittal, conviction of the accused, and the dismissal or termination of
the case without his express consent constitutes res judicata and is a bar to another prosecution for the
Yes, Mr. Opulencia can invoke double jeopardy as defense for the second offense because as offense charged. In the case, it was evidently shown that the accused invoked their right to a speedy
tediously explained in the case of Yap vs Lutero, the bill of rights give two instances or kinds of double trial and asked for the trial of the case and not its termination which would mean that respondents had
jeopardy. The first would be that “No person shall be twice put in jeopardy of punishment for the same no expressed consent to the dismissal of the case which would make the case filed res judicata and
offense and the second sentence states that “If an act is punishable by a law or an ordinance, the has been dismissed by the competent court in order to protect the respondents as well for their right to
conviction or acquittal shall bar to another prosecution for the same act”. In the case at bar, it was very speedy trial which will be equivalent to acquittal of the respondents which would be a bar to further
evident that the charges filed against Mr. Opulencia will fall on the 2 nd kind or definition of double prosecution.
jeopardy wherein it contemplates double jeopardy of punishment for the same act. It further explains
that even if the offenses charged are not the same, owing that the first charge constitutes a violation of
an ordinance and the second charge was a violation against the revised penal code, the fact that the PEOPLE VS. DE LA TORRE
two charges sprung from one and the same act of conviction or acquittal under either the law or the [380 SCRA 586; G.R. NOS. 137953-58; 11 MAR 2002]
ordinance shall bar a prosecution under the other thus making it against the logic of double jeopardy.
The fact that Mr. Opulencia was acquitted on the first offense should bar the 2 nd complaint against him Facts:
coming from the same identity as that of the 1st offense charged against Mr.Opulencia.
Wilfredo dela Torre, appellee, has three children from a common-law relationship, the eldest of which is
Mary Rose. When Mary Rose was 7 yearsold, her mother left them together with her youngest brother
ESMENA VS. POGOY so she and her other brother were left to the care of her father.
[102 SCRA 861; G.R. NO. L-54110; 20 FEB 1981]
Mary Rose was the brightest in her class despite their poverty. However, in January 1997, a sudden
Facts: change in Mary Rose’s behavior behavior was noticed. She was twelve years old at that time. She
appeared sleepy, snobbish and she also urinated on her panty. When confronted by her head teacher,
Petitioners Esmeña and Alba were charged with grave coercion in the Court of Cebu City for allegedly Mary Rose admitted that she was abused repeatedly by her father. Her father, however, denied
forcing Fr. Thomas Tibudan to withdraw a sum of money worth P5000 from the bank to be given to vehemently the charges being imputed to him by her daughter.
them because the priest lost in a game of chance. During arraignment, petitioners pleaded “Not Guilty”.
No trial came in after the arraignment due to the priest’s request to move it on another date. Sometime The RTC convicted appellee of two counts of acts of lasciviousness and four counts of murder.
later Judge Pogoy issued an order setting the trial Aug.16,1979 but the fiscal informed the court that it However, the RTC refused to impose the supreme penalty of death on appellee. It maintained that
received a telegram stating that the complainant was sick. The accused invoked their right to speedy there were circumstances that mitigated the gravity of the offenses such as the absence of any actual
trial. Respondent judge dismissed the case because the trial was already dragging the accused and physical violence or intimidation on the commission of the acts, that after the mother of Mary Rose left
that the priest’s telegram did not have a medical certificate attached to it in order for the court to the conjugal home, for more than five years, Wilfredo, Mary Rose and her brother were living together
recognize the complainant’s reason to be valid in order to reschedule again another hearing. After 27 as a family and Mary Rose was never molested by her father.
days the fiscal filed a motion to revive the case and attached the medical certificate of the priest proving
the fact that the priest was indeed sick of influenza. On Oct.24,1979, accused Esmeña and Alba filed a The prosecution seeks to modify the RTC Decision by imposing the supreme penalty of death of the
motion to dismiss the case on the ground of double jeopardy. accused. It argues that it has proven that the victim is the daughter of the accused, and that she was
below eighteen years old when the rapes took place. As a consequence, the trial court should have
Issue: been imposed the penalty of death pursuant to Section 11 of R.A. 7659. .

Whether or Not the revival of grave coercion case, which was dismissed earlier due to complainant’s Issue:
failure to appear at the trial, would place the accused in double jeopardy
Whether or Not the Court erred in penalizing the appellee with reclusion perpetua in each of the four
Held: indictments of rape, instead of imposing the supreme penalty of death as mandated by R.A. 7659.

Yes, revival of the case will put the accused in double jeopardy for the very reason that the case has Held:
been dismissed already without the consent of the accused which would have an effect of an acquittal
on the case filed. The dismissal was due to complainant’s incapability to present its evidence due to Under Section 1, Rule 122 of the 2000 Rules of Criminal Procedure, any party may appeal from a
non appearance of the witnesses and complainant himself which would bar further prosecution of the judgment or final order unless the accused will be put in double jeopardy. In People vs. Leones, it
defendant for the same offense. For double jeopardy to exist these three requisites should be present, declared that:
that one, there is a valid complaint or information filed second, that it is done before a court of
competent jurisdiction and third, that the accused has been arraigned and has pleaded to the complaint “while it is true that this Court is the Court of last resort, there are allegations of
or information. In the case at bar, all three conditions were present, as the case filed was grave error committed by a lower court which we ought not to look into to uphold the right
coercion, filed in a court of competent jurisdiction as to where the coercion took place and last the of the accused. Such is the case in an appeal by the prosecution seeking to
accused were arraigned and has pleaded to the complaint or the information. When these three increase the penalty imposed upon the accused for this runs afoul of the right of
Page 111
the accused against double jeopardy…When the accused after conviction by the
trial court did not appeal his decision, an appeal by the government seeking to
increase the penalty imposed by the trial court places the accused in double
jeopardy and should therefore be dismissed.”

The ban on double jeopardy primarily prevents the State from using its criminal processes as an
instrument of harassment to wear out the accused by a multitude of cases with accumulated trials. It
also serves as a deterrent from successively retrying the defendant in the hope of securing a
conviction. And finally, it prevents the State, following conviction, from retrying the defendant again in
the hope of securing a greater penalty.

Being violative of the right against double jeopardy, the appeal of the prosecution cannot prosper.

Page 112
CITIZENSHIP

Page 113
CITIZENSHIP agreed that they were also asking for the termination of Frivaldo's incumbency as governor of Sorsogon
on the ground that he was not a Filipino.

Art. 4 Issue:

Sec. 1. The following are citizens of the Philippines: Whether or Not petitioner Juan G. Frivaldo was a citizen of the Philippines at the time of his election on
(1) Those who are citizens of the Philippines at the time of the adoption of this Constitution; January 18, 1988, as provincial governor of Sorsogon.
(2) Those whose fathers or mothers are citizens of the Philippines;
(3) Those born before January 17, 1973, of Filipino mothers, who elect Philippine Held:
citizenship upon reaching the age of majority; and
(4) Those who are naturalized in accordance with law. The reason for this inquiry is the provision in Article XI, Section 9, of the Constitution that all public
officials and employees owe the State and the Constitution "allegiance at all times" and the specific
Sec. 2. Natural-born citizens are those who are citizens of the Philippines from birth without having to requirement in Section 42 of the Local Government Code that a candidate for local elective office must
perform any act to acquire or perfect their Philippine citizenship. Those who elect Philippine citizenship be inter alia a citizen of the Philippines and a qualified voter of the constituency where he is running.
in accordance with paragraph (3), Section 1 hereof shall be deemed natural-born citizens. Section 117 of the Omnibus Election Code provides that a qualified voter must be, among other
qualifications, a citizen of the Philippines, this being an indispensable requirement for suffrage under
Sec. 3. Philippine citizenship may be lost or reacquired in the manner provided by law. Article V, Section 1, of the Constitution.

Sec. 4. Citizens of the Philippines who marry aliens shall retain their citizenship, unless by their act In the certificate of candidacy he filed on November 19, 1987, Frivaldo described himself as a "natural-
or omission they are deemed, under the law, to have renounced it. born" citizen of the Philippines, omitting mention of any subsequent loss of such status. The evidence
shows, however, that he was naturalized as a citizen of the United States in 1983 per the following
Sec. 5. Dual allegiance of citizens is inimical to the national interest and shall be dealt with by law. certification from the United States District Court, Northern District of California, as duly authenticated
by Vice Consul Amado P. Cortez of the Philippine Consulate General in San Francisco, California,
U.S.A.
FRIVALDO VS. COMELEC
[174 SCRA 245; G.R. NO. 87193; 23 JUN 1989] The Court sees no reason not to believe that the petitioner was one of the enemies of the Marcos
dictatorship. Even so, it cannot agree that as a consequence thereof he was coerced into embracing
Facts: American citizenship. His feeble suggestion that his naturalization was not the result of his own free and
voluntary choice is totally unacceptable and must be rejected outright.
Petitioner Juan G. Frivaldo was proclaimed governor-elect of the province of Sorsogon on January 22,
1988, and assumed office in due time. On October 27, 1988, the League of Municipalities, Sorsogon This Court will not permit the anomaly of a person sitting as provincial governor in this country while
Chapter, represented by its President, Estuye, who was also suing in his personal capacity, filed with owing exclusive allegiance to another country. The fact that he was elected by the people of Sorsogon
the COMELEC a petition for the annulment of Frivaldo; election and proclamation on the ground that he does not excuse this patent violation of the salutary rule limiting public office and employment only to
was not a Filipino citizen, having been naturalized in the United States on January 20, 1983. In his the citizens of this country. The qualifications prescribed for elective office cannot be erased by the
answer dated May 22, 1988, Frivaldo admitted that he was naturalized in the United States as alleged electorate alone. The will of the people as expressed through the ballot cannot cure the vice of
but pleaded the special and affirmative defenses that he had sought American citizenship only to ineligibility, especially if they mistakenly believed, as in this case, that the candidate was qualified.
protect himself against President Marcos. His naturalization, he said, was "merely forced upon himself Obviously, this rule requires strict application when the deficiency is lack of citizenship. If a person
as a means of survival against the unrelenting persecution by the Martial Law Dictator's agents seeks to serve in the Republic of the Philippines, he must owe his total loyalty to this country only,
abroad." He added that he had returned to the Philippines after the EDSA revolution to help in the abjuring and renouncing all fealty and fidelity to any other state.
restoration of democracy. In their Comment, the private respondents reiterated their assertion that
Frivaldo was a naturalized American citizen and had not reacquired Philippine citizenship on the day of It is true as the petitioner points out that the status of the natural-born citizen is favored by the
the election on January 18, 1988. He was therefore not qualified to run for and be elected governor. Constitution and our laws, which is all the more reason why it should be treasured like a pearl of great
They also argued that their petition in the Commission on Elections was not really for quo warranto price. But once it is surrendered and renounced, the gift is gone and cannot be lightly restored. This
under Section 253 of the Omnibus Election Code. The ultimate purpose was to prevent Frivaldo from country of ours, for all its difficulties and limitations, is like a jealous and possessive mother. Once
continuing as governor, his candidacy and election being null and void ab initio because of his alienage. rejected, it is not quick to welcome back with eager arms its prodigal if repentant children. The returning
Speaking for the public respondent, the Solicitor General supported the contention that Frivaldo was renegade must show, by an express and unequivocal act, the renewal of his loyalty and love.
not a citizen of the Philippines and had not repatriated himself after his naturalization as an American
citizen. As an alien, he was disqualified from public office in the Philippines. His election did not cure Petition Dismissed. Petitioner JUAN G. FRIVALDO is hereby declared not a citizen of the Philippines
this defect because the electorate of Sorsogon could not amend the Constitution, the Local and therefore disqualified from serving as Governor of the Province of Sorsogon. Accordingly, he is
Government Code, and the Omnibus Election Code. He also joined in the private respondent's ordered to vacate his office and surrender the same to the duly elected Vice-Governor of the said
argument that Section 253 of the Omnibus Election Code was not applicable because what the League province once this decision becomes final and executory.
and Estuye were seeking was not only the annulment of the proclamation and election of Frivaldo. He
Page 114
hand, private respondent’s oath of allegiance to the Philippine, when considered with the fact that he
MERCADO VS. MANZANO has spent his youth and adulthood, received his education, practiced his profession as an artist, and
[307 SCRA 630; G.R. NO. 135083; 26 MAY 1999] taken part in past elections in this country, leaves no doubt of his election of Philippine citizenship.

Facts:
TECSON VS. COMELEC
Petitioner Ernesto Mercado and Private respondent Eduardo Manzano are candidates for the position [424 SCRA 277; G.R. No. 161434; 3 Mar 2004]
of Vice-Mayor of Makati City in the May, 1998 elections. Private respondent was the winner of the said
election but the proclamation was suspended due to the petition of Ernesto Mamaril regarding the Facts:
citizenship of private respondent. Mamaril alleged that the private respondent is not a citizen of the
Philippines but of the United States. COMELEC granted the petition and disqualified the private Victorino X. Fornier, petitioner initiated a petition before the COMELEC to disqualify FPJ and to deny
respondent for being a dual citizen, pursuant to the Local Government code that provides that persons due course or to cancel his certificate of candidacy upon the thesis that FPJ made a material
who possess dual citizenship are disqualified from running any public position. Private respondent filed misrepresentation in his certificate of candidacy by claiming to be a natural-born Filipino citizen when in
a motion for reconsideration which remained pending until after election. Petitioner sought to intervene truth, according to Fornier, his parents were foreigners; his mother, Bessie Kelley Poe, was an
in the case for disqualification.  COMELEC reversed the decision and declared private respondent American, and his father, Allan Poe, was a Spanish national, being the son of Lorenzo Pou, a Spanish
qualified to run for the position.  Pursuant to the ruling of the COMELEC, the board of canvassers subject. Granting, petitioner asseverated, that Allan F. Poe was a Filipino citizen, he could not have
proclaimed private respondent as vice mayor.  This petition sought the reversal of the resolution of the transmitted his Filipino citizenship to FPJ, the latter being an illegitimate child of an alien mother.
COMELEC and to declare the private respondent disqualified to hold the office of the vice mayor of Petitioner based the allegation of the illegitimate birth of respondent on two assertions - first, Allan F.
Makati. Poe contracted a prior marriage to a certain Paulita Gomez before his marriage to Bessie Kelley and,
second, even if no such prior marriage had existed, Allan F. Poe, married Bessie Kelly only a year after
Issue: the birth of respondent.

Whether or Not private respondent is qualified to hold office as Vice-Mayor.

Held: Issue:

Dual citizenship is different from dual allegiance. The former arises when, as a result of the concurrent Whether or Not FPJ is a natural born Filipino citizen.
application of the different laws of two or more states, a person is simultaneously considered a national
by the said states. For instance, such a situation may arise when a person whose parents are citizens Held:
of a state which adheres to the principle of jus sanguinis is born in a state which follows the doctrine of
jus soli. Private respondent is considered as a dual citizen because he is born of Filipino parents but It is necessary to take on the matter of whether or not respondent FPJ is a natural-born citizen, which,
was born in San Francisco, USA. Such a person, ipso facto and without any voluntary act on his part, in turn, depended on whether or not the father of respondent, Allan F. Poe, would have himself been a
is concurrently considered a citizen of both states. Considering the citizenship clause (Art. IV) of our Filipino citizen and, in the affirmative, whether or not the alleged illegitimacy of respondent prevents him
Constitution, it is possible for the following classes of citizens of the Philippines to posses dual from taking after the Filipino citizenship of his putative father. Any conclusion on the Filipino citizenship
citizenship: (1) Those born of Filipino fathers and/or mothers in foreign countries which follow the of Lorenzo Pou could only be drawn from the presumption that having died in 1954 at 84 years old,
principle of jus soli; (2) Those born in the Philippines of Filipino mothers and alien fathers if by the laws Lorenzo would have been born sometime in the year 1870, when the Philippines was under Spanish
of their fathers’ country such children are citizens of that country; (3) Those who marry aliens if by the rule, and that San Carlos, Pangasinan, his place of residence upon his death in 1954, in the absence of
laws of the latter’s country the former are considered citizens, unless by their act or omission they are any other evidence, could have well been his place of residence before death, such that Lorenzo Pou
deemed to have renounced Philippine citizenship. Dual allegiance, on the other hand, refers to the would have benefited from the "en masse Filipinization" that the Philippine Bill had effected in 1902.
situation in which a person simultaneously owes, by some positive act, loyalty to two or more states. That citizenship (of Lorenzo Pou), if acquired, would thereby extend to his son, Allan F. Poe, father of
While dual citizenship is involuntary, dual allegiance is the result of an individual’s volition. respondent FPJ. The 1935 Constitution, during which regime respondent FPJ has seen first light,
confers citizenship to all persons whose fathers are Filipino citizens regardless of whether such children
By filing a certificate of candidacy when he ran for his present post, private respondent elected are legitimate or illegitimate.
Philippine citizenship and in effect renounced his American citizenship. The filing of such certificate of
candidacy sufficed to renounce his American citizenship, effectively removing any disqualification he But while the totality of the evidence may not establish conclusively that respondent FPJ is a natural-
might have as a dual citizen. born citizen of the Philippines, the evidence on hand still would preponderate in his favor enough to
hold that he cannot be held guilty of having made a material misrepresentation in his certificate of
By declaring in his certificate of candidacy that he is a Filipino citizen; that he is not a permanent candidacy in violation of Section 78, in relation to Section 74, of the Omnibus Election Code.
resident or immigrant of another country; that he will defend and support the Constitution of the
Philippines and bear true faith and allegiance thereto and that he does so without mental reservation,
private respondent has, as far as the laws of this country are concerned, effectively repudiated his BENGZON VS. HRET
American citizenship and anything which he may have said before as a dual citizen. On the other [357 SCRA 545; G. R. No. 142840; 7 May 2001]
Page 115
Facts:

Respondent Teodoro Cruz was a natural-born citizen of the Philippines. He was born in San Clemente,
Tarlac, on April 27, 1960, of Filipino parents. The fundamental law then applicable was the 1935
Constitution. On November 5, 1985, however, respondent Cruz enlisted in the United States Marine
Corps and without the consent of the Republic of the Philippines, took an oath of allegiance to the
United States. As a Consequence, he lost his Filipino citizenship for under Commonwealth Act No. 63,
section 1(4), a Filipino citizen may lose his citizenship by, among other, "rendering service to or
accepting commission in the armed forces of a foreign country.” He was naturalized in US in 1990. On
March 17, 1994, respondent Cruz reacquired his Philippine citizenship through repatriation under
Republic Act No. 2630. He ran for and was elected as the Representative of the Second District of
Pangasinan in the May 11, 1998 elections. He won over petitioner Antonio Bengson III, who was then
running for reelection.

Issue:

Whether or Not respondent Cruz is a natural born citizen of the Philippines in view of the constitutional
requirement that "no person shall be a Member of the House of Representative unless he is a natural-
born citizen.”

Held:

Respondent is a natural born citizen of the Philippines. As distinguished from the lengthy process of
naturalization, repatriation simply consists of the taking of an oath of allegiance to the Republic of the
Philippine and registering said oath in the Local Civil Registry of the place where the person concerned
resides or last resided. This means that a naturalized Filipino who lost his citizenship will be restored to
his prior status as a naturalized Filipino citizen. On the other hand, if he was originally a natural-born
citizen before he lost his Philippine citizenship, he will be restored to his former status as a natural-born
Filipino.

Page 116

Вам также может понравиться